Você está na página 1de 672

1d 1

Volume 3
Geometria Plana
&
Ví Carlos A. Gomes
I)•'!
•V
íó
Carlos A. Gomes

TÓPICOS DE MATEMÁTICA
Olimpíadas - ITA- IME

Volume 03

Geometria Plana

Editora VestSeller
Fortaleza/CE
1a Edição - Abril de 2016
É proibida a reprodução parcial ou total por quaisquer meios sem
autorização prévia do autor. Os transgressores serão punidos nos termos
da lei. Denuncie o plágio, cópias ilegais, pirataria pela internet, sites para
download pirata, comunidades piratas na internet anonimamente através
do correio eletrônico da editora:
adm@vestseller.com.br

Todos os direitos desta edição reservados a:


© 2016 Carlos A. Gomes

Editor responsável: Renato Brito Bastos Neto


Revisão Técnica: Renato Brito Bastos Neto
Editoração: Renato Brito Bastos Neto
Capa: Alex Souza

Esta obra pode ser adquirida diretamente na


LIVRARIA VESTSELLER
através de sua página eletrônica www.vestseller.com.br

FICHA CATALOGRÁFICA: Preparada por


Ruth Helena Linhares Leite e Luiza Helena de Jesus Barbosa.

M672 Gomes, Carlos A.


Tópicos de Matemática / IME - ITA - Olimpíadas /
Carlos A. Gomes - Fortaleza: Vestseller, 1a edição 2016. 669p; v.3.
I. Produtos Notáveis II. Indução III. Geometria Plana

ISBN: 978-85-60653-45-4
CDU 514
O autor

O professor Carlos A. Gomes, é Bacharel em Matemática pela UFRN, Mestre em


Matemática na Área de Probabilidade pela UFRN e atualmente doutorando em
Matemática na área de Álgebras e grupos de Lie pelo IME-USP. Tem um apreço
especial pelas Geometrias. É professor efetivo do Departamento de Matemática
da UFRN, é Coordenador Regional da OBM - Olimpíada Brasileira de Matemática e
Coordenador da OMRN - Olimpíada de Matemática do RN. Foi, por cerca de 20
anos, professor de diversos cursinhos Pré-Vestibulares das disciplinas de
Matemática e Física e tem uma larga experiência com treinamentos para
olimpíadas de Matemática e resolução de problemas.
Apresentação
Nos últimos anos, tem sido evidente, pelo Brasil afora, o crescimento do
número de jovens que almejam conseguir uma vaga nas excepcionais escolas
superiores militares do ITAe do IME. Adidonalmente, é fato o enorme crescimento do
movimento das olimpíadas de Matemática em todo o mundo e em particular no
Brasil.
A SBM - Sociedade Brasileira de Matemática organiza desde 1979 a OBM -
Olimpíada Brasileira de Matemática e mais recentemente o governo federal lançou
a OBMEP - Olimpíada Brasileira de Matemática das Escolas Públicas, programa
que teve na sua última versão a participação de mais de 17 milhões de alunos nos
quatro cantos do nosso pais. Neste contexto é bastante natural que surja a
necessidade da elaboração de materiais escritos na nossa lingua portuguesa que
sirvam de apoio para a preparação dos alunos para estas competições.
Nos últimos anos a revista EUREKA, publicada pela SBM, vem trazendo artigos,
provas anteriores e problemas propostos resolvidos. Além disso, foi colocado no ar
o excelente site da OBMEP, entre muitos outros, onde são encontrados bancos de
questões, livros, provas, enfim, muitos materiais de excelente qualidade que com
certeza têm auxiliado muitos alunos nas suas preparações para as competições
acima citadas.
Assim, vemos que o número de publicações direcionadas para esse tema vem
crescendo, apesar de ainda ser muito pequeno no nosso pais. Dentro deste
panorama, nós autores resolvemos criar a coleção "'TÓPICOS DE MATEMÁTICA
- OLIMPÍADAS - ITA - IME" que consiste numa coleção de livros com resumos
teóricos que apresentam um nivel adequado e muitos problemas resolvidos que
foram compilados ao longo de vários anos em revistas, provas, artigos e diversos
livros consultados pelos autores.
A ideia central do nosso trabalho é produzir uma obra que concentre num só lugar
vários problemas clássicos e interessantes e suas respectivas soluções
detalhadas, um material precioso ao qual um aluno iniciante de outra forma só teria
acesso caso consultasse várias fontes relacionadas. Nossa obra surge, portanto,
como uma excelente ferramenta que permite ao aluno iniciante obter um grande
salto de conhecimento num curto intervalo de tempo.

Nossa coleção se divide em 6 volumes, a saber:

Volume 01 - produtos notáveis, fatorações e desigualdades.


Volume 02 - indução matemática e teoria elementar dos números.
Volume 03 - geometria plana. I
Volume 04 - trigonometria.
Volume 05 - combinatória e probabilidade.
Volume 06 - números complexos, polinômios e equações algébricas.
Volume 07 - geometria espacial.
Volume 08 - geometria analítica.
Volume 09 - conjuntos, funções, sequências e séries.
Volume 10 - matrizes determinantes e sistemas lineares.

Por fim, gostaríamos de agradecer ao professor Renato Brito, diretor da editora


Vestseller, pelo acolhimento do nosso projeto e aproveitar a oportunidade de
parabenizá-lo tanto pela iniciativa de publicar novas obras quanto por reeditar
obras antigas cujo acesso estava cada vez mais raras para a presente e a futura
geração atual de alunos com aptidão natural para Ciências Exatas e aqueles que
procuram obter vagas para as respeitadas instituições militares onde se destacam
o IME e o ITA.

Os leitores que quiserem fazer contato com os autores para criticas, sugestões bem
para comunicar alguma errata eventualmente encontrada na presente obra, podem
fazê-lo pelo email cgomesmat@yahoo.com.br

Carlos A. Gomes.

Natal/RN, 04 de dezembro de 2015


Prefácio
A Editora VestSeller tem o prazer de lançar no mercado brasileiro mais
uma excelente coleção de Matemática para o segmento de preparação para os
vestibulares IME ITA, bem como para as Olimpíadas de Ciências exatas nacionais
e internacionais.

Com sua vasta experiência no ramo, os professores Carlos Gomes e José


Maria Gomes presenteiam os estudantes e professores brasileiros com uma obra
prima que permite a qualquer leitor obter um grande salto de conhecimento na
Matemática Elementar um curto intervalo de tempo.

Com didática admirável e notável capacidade de síntese, a presente obra


fornece aos alunos uma grande quantidade de problemas clássicos de Matemática
Elementar de alto nível, complementados, ao final do livro, com as resoluções
detalhadas de todos os problemas, o que é permite um estudo eficaz e produtivo
em especial para os leitores autodidatas.

Os autores fazem mágica com a Matemática e mostram na presente obra


todo um mundo de possibilidades para resolução de problemas aparentemente
terríveis fazendo uso de ferramentas elementares como fatoração, produtos
notáveis e desigualdade das médias. É de tirar o fôlego a cada página I

Com essa excelente obra, a VestSeller tem a certeza de estar mais uma
vez estar dando uma notável contribuição para a melhoria do nível e da qualidade
dos livros de Matemática disponíveis para estudantes e professores em todo Brasil.

Prof. Renato Brito

Fortaleza, 04 de Dezembro de 2015.


Dedicatória

Dedico este trabalho a dois grandes mestres que foram os primeiros a publicar no
Brasil dois magníficos livros de Geometria plana que ainda hoje influenciam
fortemente alunos e professores por todo o país; Os professores e amigos Augusto
César Morgado (In memoriam) e Eduardo Wagner, que tive o privilégio de
conhecê-lo pessoalmente e que me influenciaram fortemente pelo gosto da
Geometria.

Carlos A. Gomes
índice

Introdução............................ 13
Capitulo 1. Resumo Teórico 15

Capitulo 2. Questões ...................... . 67


TRIÂNGULOS................................... . 68
POLÍGONOS..................................... . 91
QUADRILÁTEROS........................... . 94
SEMELHANÇA.................................. .98
PITÁGORAS..................................... 105
LEI DOS SENOS E COSSENOS .... 111
QUADRILÁTEROS INSCRITÍVEIS.. 121
ÂNGULOS NA CIRCUNFERÊNCIA 126
ÁREAS................................................ 136

Capítulo 3 Resoluções. 181

Apêndice - Princípio da Boa Ordenação e Indução......................... .649


I. Pitágoras Trigonométrico................................................................. .650
II. Você sabia que lei dos cossenos vale para os senos?................. 651
Illl. Lei Dos Senos Para Os Cossenos.................................................. 652
IV. Uma Bela Demonstração (Sem Palavras) Da Fórmula De Heron 658
V. Heron para quadriláteros., brahmagupta!...................................... 661
VI. Uma Soma Incrivelmente Invariante............................................... 665

Referências Bibliográficas 670


INTRODUÇÃO

Este é o terceiro volume da coleção MATEMÁTICA - IME — ITA -


OLIMPÍADAS. Neste volume abordamos todo o conteúdo da GEOMETRIA
EUCLIDIANA PLANA e a TRIGONOMETRIA BÁSICA DOS TRIÂNGULOS. A
exemplo dos volumes anteriores, inicialmente temos um resumo teórico com os
principais resultados da Geometria Euclideana Plana e Trigonometria dos
triângulos, em seguida os enunciados dos cerca de 300 problemas propostos e por
fim as resoluções detalhadas de todos esses problemas propostos. As resoluções
foram minunciosamente produzidas e detalhadas para que o leitor possa ter uma
verdadeira aula sobre o assunto em cada uma delas. Em algumas soluções há
diversos comentários teóricos que ampliam o problema em questão ou descortinam
um novo horizonte sobre o assunto. Ao final do livro há alguns Apêndices onde o
leitor poderá ampliar seus conhecimentos sobre alguns temas tratados ao longo do
livro. O livro foi produzido tentando manter o mesmo espírito dos dois volumes
anteriores, a saber: ao estudar o livro o leitor terá um acesso rápido e direto a
teoremas, problemas e situações que o mesmo só teria após ler diversas obras
clássicas (e modernas) sobre o assunto aqui tratado, sendo pois, ao nosso ver,
uma ponte de acesso mais rápido a diversos problemas clássicos e outros mais
modernos sobre o tema aqui tratado. Convidamos então à todos para iniciar essa
leitura, que levará a um passeio sobre o encantado mundo da Geometria e
Trigonometria Clássica.
Carlos A Gomes
São Paulo, 19 de Outubro de 2015
Capítulo 1

Resumo Teórico
16 Tópicos de Matemática - Olimpíadas - IME - ITA

RESUMO TEÓRICO

ÂNGULO ENTRE RETAS PARALELAS


Observe a figura abaixo onde as retas paralelas (r) e (s) são cortadas pela reta
transversal (t).

(>')

(>•)//(«)

Alternos internos: Pi e aa; P2 e ou

Alternos externos: p< e 0.2i Pa e a.

Colaterais internos: P1 e ; P2 e aa

Colaterais externos: Pa ea;| p< eaj

Correspondentes: Pi e a, ; P2 e aa; Pa e aa I P« e cu

PROPRIEDADE: Os ângulos correspondentes e alternos são congruentes (e portanto


possuem a mesma medida) e os colaterais são suplementares.
Capitulo 1 - Resumo Teórico 17

LEI ANGULAR DE THALES


Num triângulo qualquer a soma das medidas dos ângulos interno s de um triângulo
qualquer é 180°

TEOREMA DO ÂNGULO EXTERNO


<P = 0 + 0
No triângulo ABC da figura abaixo: ■ y = a + 0
À. = a + 9

-
!
18 Tópicos de Matemática - Olimpíadas - IME - ITA

TEOREMA DA DESIGUALDADE TRIÀNGULAR

1. Condição de existência (Teorema da desigualdade triangular)


Dados três segmentos AB = c. AC = b e BC = a nem sempre podemos “fechar” um
triângulo ABC

b
a

Pode-se demonstrar que a condição necessária e suficiente para que o triângulo


ABC possa ser fabricado é a seguinte:
[diferença dos outros dois| < Qualquer lado < soma dos outros dois

2. Algumas classificações
a) Quanto aos lados:
Escaleno-» 3 lados com medidas diferentes (3 ângulos de medidas diferentes)
Isósceles -» 2 lados com medidas iguais (2 ângulos de medidas iguais - ângulos da
base)
Equilátero -» 3 lados com medidas iguais (3° ângulos com medidas iguais a 60°
cada um)

b) Quanto aos ângulos:


Acutângulo: 3 ângulos com medidas menores do que 90°.
Retângulo: 1 ângulo reto. (=90°)
Obtusângulo: 1 ângulo obtuso. (>90°)
Capítulo 1 - Resumo Teórico 19

Observações:
✓ Num triângulo o maior lado sempre está oposto ao maior ângulo (maior medida) e
reciprocamente, isto é, o ângulo de maior medida sempre está oposto ao lado de
maior medida.
■/ Se conhecermos as medidas dos lados a, b e c de um triângulo ABC. mesmo sem
conhecermos as medidas dos seus ângulos internos podemos identificar se ele é
acutângulo, retângulo ou obtusãngulo. Fazemos assim:
Seja a a medida do maior lado, (note que se não houver maior lado é porque os lados sâo
iguais sendo assim o triângulo é eqüilátero e portanto cada um dos seus ângulos internos
mede 60°, sendo portanto um triângulo acutângulo).
a —> medida do maior lado.
b, c medidas dos outros dois lados.

a2 < b2 + c2 <=> Triângulo acutângulo


a2 = b2 +c2 <=> Triângulo retângulo
a2 > b2 + c2 «=> Triângulo obtusãngulo

POLÍGONOS
3. Elementos de um polígono

ãnfpilo externo

ângulo interno vértice

diagonal

lado

Vértice: é o ponto de interseção de dois lados adjacentes.


Lado: é o segmento de reta que une dois vértices consecutivos.
Diagonal: é o segmento de reta que une dois vértices não consecutivos.
Ângulo Interno: é o ângulo da região interna formado por dois lados ad|acentes
20 Tópicos de Matemática - Olimpíadas - IME - ITA

Ângulo Externo: é o ângulo da região externa formado por um lado e pelo


prolongamento de um lado adjacente.

Observações:
v' Note que um ângulo interno e um ângulo externo adjacente a ele sâo
suplementares

Polígono Regular: é o polígono que possui lados iguais (congruentes) e ângulos


iguais (congruentes), isto é. um polígono regular é aquele que é eqüilátero e
eqüiângulo.

Nomenclatura dos Polígonos - Alguns polígonos recebem nomes de acordo com o


número de lados. Assim temos:

POLÍGONO ffi DE LADOS(GÊNERO)


Triângulo 3
Quadrilátero 4
Pentágono 5
Hexágono __________ 6__________
Heptágono 7
Octógono 8
Eneágono 9
Decágono 10
Undecágono 11
Dodecágono 12
Pentadecágono __________ 15_________
Icoságono 20

4. Número de Diagonais de um polígono convexo de n lados


Cada vértice do polígono não forma diagonal com ele próprio e com os dois vértices
adjacentes, formando diagonal com todos os outros (n - 3) vértices do polígono. Assim
sendo, de cada vértice de um polígono convexo de n lados partem n - 3 diagonais.
Capítulo 1 - Resumo Teórico 21

Cada diagonal é comum a dois vértices, então o total de diagonais de um polígono


n(n-3)
de n lados é dado por d =
2

A1

/
7 An

a2

ÍA5

(77. — 3) diagonais
partindo de cada vértice
A4

5. Soma das medidas dos ângulos internos de um polígono de n lados (Si)

A4

(n — 2) triângulos
22 Tópicos de Matemática - Olimpíadas - IME - ITA

Traçando-se todas as diagonais de um dos vértices o polígono de n lados fica


dividido em n - 2 triângulos. A soma das medidas dos ângulos internos do polígono é
igual a soma dos ângulos internos dos triângulos. Então,

S, = (n-2)-180°

6. Soma das medidas dos ângulos externos de um polígono de n lados (Se)

A
3e

■»
Se= 360°

Observação:
No caso dos polígonos equiângulos (todos os ângulos com medidas iguais) temos:

_ S, = (n-2)-180° = Se = 360°
a' n n 6 Se n n

CONGRUÊNCIA DE TRIÂNGULOS
TEOREMA DA BASE MÉDIA DE UM TRIÂNGULO
Dado um triângulo qualquer, o segmento que une os pontos médios de dois lados é
paralelo ao terceiro lado e tem um comprimento igual a metade do comprimento desse
terceiro lado.
Capitulo 1 - Resumo Teórico 23

TEOREMA DETHALES
Se duas retas são transversais de um feixe de retas paralelas, então a razão entre
as medidas de dois segmentos quaisquer de uma delas é igual ã razão entre as medidas
dos respectivos segmentos correspondentes da outra, ou seja,
A8 DE
(r)//(s)//(t)
BC " EF

A D (>•)

B E (»)

C F (t)

TEOREMA DAS BISSETRIZES


1. Teorema da bissetriz interna
Seja ABC um triângulo qualquer e AD a bissetriz do ângulo interno do vértice A.
conforme ilustra a figura abaixo:

. m n
Mostre que —
c b
24 Tópicos de Matemática - Olimpíadas - IME - ITA

2. Teorema da bissetriz externa


Seja ABC um triângulo qualquer e AE a bissetriz do ângulo interno do vértice A,
conforme ilustra a figura abaixo:

m
.. m n
Mostre que —
c b

CÍRCULO DE APOLÔNIO
Dados dois pontos distintos A e B o lugar geométrico dos pontos P de um plano tais
PA
que — = r, onde r é um número real positivo, é a mediatriz do segmento AB, se r = 1 e é
PB
uma circunferência, chamada de CIRCUNFERÊNCIA (OU CÍRCULO) DE APOLÔNIO, se
r é diferente de 1.
Capitulo 1 - Resumo Teórico 25

SEMELHANÇA DE TRIÂNGULOS

(possuem 3 ãnguloscongruentes
AABC-AA'B'C'<=.
[lados correspondentes proporcionais

b c
ou seja, AABC ~ aA'B'C’ <=> — = = k , onde k é um número real chamado de
a' b' c'
razão de semelhança.

OBSERVAÇÕES:
I) A razão de semelhança k é mantida para quaisquer segmentos correspondentes em
dois triângulos semelhantes.
II) Pode-se demonstrar que a razão entre as medidas das áreas de dois triângulos
semelhantes é igual ao quadrado da razão de semelhança entre eles, ou seja.
(ABC)
AABC~AA'B'C'<=> — = — — = k => = k2
a’ b' c' (A'B'C)

III) A propriedade anterior é válida para quaisquer figuras planas semelhantes.

RELAÇÕES MÉTRICAS NO TRIÂNGULO RETÂNGULO

ah = bc, b2 = a.n, c2 = am, a2 = b2 + c2 (Teorema de Pitágoras)


26 Tópicos de Matemática - Olimpíadas - IME - ITA

OBSERVAÇÃO:
Lembrando que a medida da área de uma quadrado de lado a é igual a a2. 0

teorema de Pitágoras, ou seja a afirmação de que num triângulo retângulo com hipotenusa
(maior lado) medindo a e catetos medindo b e c vale a relação a2 = b2 + c2 , isto equivale
a dizer que a medida da área do quadrado construído sobre a hipotenusa de um triângulo
retângulo é igual a soma das medidas das áreas dos quadrados construídos sobre os
catetos conforme ilustra a figura abaixo:

O Teorema de Pitágoras afirma que a área do quadrado construído sobre a


hipotenusa de um triângulo retângulo é igual à soma das áreas dos quadrados construídos
sobre os catetos Agora, imaginemos figuras semelhantes quaisquer, construídas sobre os
lados de um triângulo retângulo

Sejam então A, B e C as áreas de figuras semelhantes, construídas sobre a


hipotenusa a e sobre os catetos b e c de um triângulo retângulo, como mostra a figura
acima Sabemos que a razão entre as áreas de figuras semelhantes é igual ao quadrado
da razão de semelhança.
Capítulo 1 - Resumo Teórico 27

Então,

A C A
e —
B “IbJ =?

E portanto
A B C A
b2 + c2
4.
a2
s±£=a.b.c
a2

Isto quer dizer que, se figuras semelhantes são construídas sobre os lados de um
triângulo retângulo, a área da figura construída sobre a hípotenusa é igual â soma das
áreas das figuras construídas sobre os catetos. Esta é uma generalização do teorema de
Pitágoras.

UM TRIÂNGULO E SUAS CIRCUNFERÊNCIAS


28 Tópicos de Matemática - Olimpíadas - IME - ITA

MEDIDAS DOS RAIOS

,(p-a)(p-b)(p-c)
r=
P

R = — ____
abc - =■
4jp(p-a)(p-b)(p-c)

/p(p-b)(p-c) Jp(p-a)(p-c) fp(p-a)(p-b)


• rb = e rc =
M (P-a) 1 (P~b) (P~c)

A RELAÇÃO DOS CINCO RAIOS


Num triângulo ABC,
ra+rb+rc-r = 4R

Onde ra,rberc são os raios das circunferências ex-inscritas, r é o raio da


circunferência inscrita e R é o raio da circunferência circunscrita ao triângulo ABC.

A CIRCUNFERÊNCIA DE NOVE PONTOS


Dado um triângulo ABC sejam M,N e P os pontos médios dos seus lados; H,, H2
e H3 os pés das suas alturas e U, T e V os pontos médios dos segmentos HA, HB
e HC, onde H é o ortocentro do triângulo AB. Existe uma circunferência que
passa por esses nove pontos, chamada de CIRCUNFERÊNCIA DE NOVE
PONTOS (OU CIRCUNFERÊNCIA DE EULER -FEUERBACH)

T *2

/ p N
i

' h3/
i /I H

/u V

M
Capitulo 1 - Resumo Teórico 29

A medida do raio r da circunferência de nove pontos corresponde a metade


da medida do raio R da circunferência circunscrita ao triângulo ABC, ou seja,

R
r" 2 '

RELAÇÃO DE STEWART

ax2 + amn = b2m + c2m

PONTOS NOTÁVEIS DE UM TRIÂNGULO


Num triângulo existem muitos pontos que merecem destaque , alguns por sua
importância prática e muitos outros por mero capricho teórico de Matemáticos
profissionais. Apenas para que você tenha uma pequena noção apresentamos na figura a
seguir alguns dos muitos pontos notáveis de um triângulo

Alguns dos 500 primeiros pontos notáveis de um triângulo


(http://www.uff.br/trianglecenters/centers-from-0001-to-0500.html )
30 Tópicos de Matemática - Olimpíadas - IME - ITA

Hoje existem pessoas que se ocupam de identificar estes pontos e estudar as suas
mais variadas propriedades. No nosso caso introdutório, existem apenas alguns poucos
pontos que sâo explorados a nível do ensino médio e de questões olímpicas. Esses
pontos são:

1. BARICENTRO —> É o ponto de encontro das medianas de um triângulo

PROPRIEDADES:
I) É o centro de gravidade do triângulo, por isso costuma-se representá-lo por G.

AG = 2GD
II) Divide cada mediana em duas partes tal que uma é o dobro da outra; BG = 2 GF .
CG = 2-GE

2
AG = —AD
3
Podemos enxergar esta propriedade de outra forma, por exemplo,
GD=—AD
3
Capítulo 1 - Resumo Teórico 31

III) As três medianas de um triângulo qualquer o dividem em seis triângulos de mesma


área.

(BDG) = (CDG) = (CFG) = (AFG) = (AEG) = (BEG) = S

2. INCENTRO—» É o ponto de encontro das bissetrizes internas de um triângulo.

PROPRIEDADES:
I) É o centro da circunferência inscrita no triângulo.
II) É um ponto equidistante dos lados do triângulo.
32 Tópicos de Matemática - Olimpíadas - IME - ITA

3. CIRCUNCENTRO —> É o ponto de encontro das mediatrizes dos lados de um


triângulo.

PROPRIEDADES:
I) É o centro da circunferência circunscrita no triângulo.
II) É um ponto equidistante dos vértices do triângulo.

4. ORTOCENTRO É o ponto de encontro das retas suportes das alturas de um


triângulo.
Capitulo 1 - Resumo Teórico 33

PROPRIEDADES:
I) É o circuncentro do triângulo obtido quando traçamos pelos vértices do triângulo
ABC paralelas aos lados opostos.
H) É o incentro do triângulo cujos vértices são os pés das alturas (triângulo órtico ou
triângulo pedal do triângulo ABC).
III) O simétrico do ortocentro em relação aos lados do triângulo ABC pertence a
circunferência circunscrita ao triângulo.

OBSERVAÇÃO: No caso do triângulo equilátero estes 4 pontos acima citados são


coincidentes.

5. PONTO DE GERGONE -» Éo ponto de encontro das cevianas que ligam os vértices


de um triângulo ABC aos pontos em que a circunferência inscrita ao triângulo
intersecta cada um dos seus lados.

6. PONTO DE MIQUEL-> Considere um triângulo ABC e pontos X, Y e Z nos lados BC,


AC e AB, respectivamente tais que A, B, C, X, Y e Z são todos distintos entre si. As
circunferências circunscritas aos triângulos AYZ, BZX e CXY têm um ponto em
comum, que é chamado de ponto de Miquel do triângulo ABC.
34 Tópicos de Matemática - Olimpíadas - IME - ITA

7, PONTO DE NAGEL -> As cevianas que unem cada vértice de um triângulo ABC ao
ponto de contacto de cada circulo ex-inscrito com o lado oposto, intersectam-se num
mesmo ponto, chamado de ponto de Nagel do triângulo ABC, conforme ilustra a
figura abaixo:

8. PONTO DE FERMAT-TORRICELLI
Seja ABC um triângulo qualquer. Construindo externamente triângulos ABE, BCF e CAD
equiláteros sobre os lados AB, BC e CA do triângulo ABC, então:

i) As circunferências circunscritas aos triângulos equiláteros ABE. BCF e CAD tem um


ponto T em comum.

ü) Os segmentos AF, BD e CE são concorrentes no ponto T.

iii) AF = BD = CE = TA + TB + TC.

iv) Para qualquer ponto M do plano do triângulo ABC vale a desigualdade


MA+MB+MC>TA+TB+TC
Capitulo 1 - Resumo Teórico 35

Ou seja, o ponto T é o ponto que minimiza a soma das distâncias de um ponto do


plano do triângulo ABC até cada um dos seus vértices.

OBSERVAÇÃO: O ponto T acima descrito é chamado de Ponto de Fermat-Torricelli do


triângulo ABC.

MEDIDAS DAS MEDIANAS, BISSETRIZES INTERNAS, EXTERNAS E ALTURAS DE


UM TRIÂNGULO

1. Medianas

ma=^2(b2+c2)-a2

mb =^2(a2 + c2)-b2

mc=^2(a2 + b2)-c2
36 Tópicos de Matemática - Olimpíadas - IME - ITA

2. Bissetrizes internas
2 _________
sa=^Vbcp(p-a)
2 I----- 7----
sb = ^-^Vacp( p-b)

2 .________
Sc = i7b'/abp(p-c)

3. Bissetrizes externas
2 ______________
Pa =jKT^vbcp(p~b)(p~c)

2 ______________
p&=j^z^vacp(p-a)(p~c)
2 ._____________
Pc = Vabp (p - a) (p - b)

4. Alturas
2 _ _______________
ha = ~VP(p-a)(p_b)(P-C)

2 i___________________
hb =^Vp(p-a)(p-b)(p-c)

2 .________________
hc = — Vp(p_ a)(p-b)(p-C)

RAZÕES TRIGONOMÉTRICAS NO TRIÂNGULO RETÂNGULO

A c B
Capitulo 1 - Resumo Teórico 37

c b c b
sena = cosp = — , senp = cosa = — , tga = — e tgp =—
a abc

c2 + b2 a2
a2 = b2 +c2 sen2a + cos2 =1

LEI DOS SENOS

= 2R
sena senp

LEI DOS COSSENOS

a2 = b2 + c2 - 2bc cos a
b2 = a2+c2-2bc cos p
c2 = a2+b2-2bccos0
38 Tópicos de Matemática - Olimpíadas - IME - ITA

ALGUMAS IDENTIDADES TRIGONOMÉTRICAS


sen (a ±p) = senacosp isenpcosa

cos(a±p) = cosacosp+senpsena

tga + tgp
tg(a±p) =
1 + tga tgp

sen(2a) = 2senacosa

sen (3a) = 3sena - 4sen3a

cos(3a) = 4cos3a-3cosa

cos2a-sen2a
cos(2a) = ■ 2cos2a-1
1-2sen2a

senasenp = ^[cos(a-p)- cos(a + p)]

cos a cos p = — [cos (a + p) + cos (a-p)]

„ - a ++Pp) f a - p)
ía
sena + senp = 2sen[ cos[ J
2

„ _ (a-P'| fa + p')
sena - senp = 2sen^ Jcos|^ -J

„ _ (a + p) (a-p)
cosa + cosp = 2cos^ Jcos^ J

cosa-cosp = -2sen(a +íj sen^ a-p'


. 2 >
Capitulo 1 - Resumo Teórico 39

TEOREMA DE NAPOLEÃO
Dado um triângulo ABC, construimos sobre cada um dos seus lados um triângulo
equilátero conforme ilustra a figura abaixo:

o triângulo cujos vértices são os centros dos triângulos equiláteros que foram construídos
sobre os lados do triângulo original também é um triângulo equilátero.

TEOREMA DEMORLEY
Dado um triângulo ABC, mostre que as cevianas que trisecionam os ângulos internos
do triângulo ABC intersectam-se duas a duas formando três pontos que sâo os vértices de
um triângulo equilátero, conforme ilustra a figura abaixo:
40 Tópicos de Matemática - Olimpíadas - IME - ITA

LINHAS, TRIÂNGULOS E CONCORRÊNCIA


TEOREMA DE CEVA
Num triângulo ABC, três cevianas AP, BN e CM são concorrentes se, e somente se,

AM BP CN
1.
MB PC NA

TEOREMA DEMENELAUS
Se uma reta (r) intersecta as retas suportes dos lados BC, CA e AB de um triângulo
ABC nos pontos L, M e N, respectivamente, mostre que
BL CM AN 1
LCMANB“
Capítulo 1 - Resumo Teórico 41

E reciprocamente, ou seja, se L, M e N são pontos sobre as retas suportes dos lados

BC, CA e AB do triângulo ABC tais que = 1 então L, M e N são colineares.


LC MA NB

DIVISÃO HARMÔNICA DE UM SEGMENTO


Dado um segmento AB, com reta suporte (r), dizemos que os pontos C e D
. . . . . CA DA
(pertencentes a reta (r)), dividem harmonicamente o segmento AB quando —
- • CB- DB

(r)
A C B D

OBSERVAÇÃO: Quando os pontos C e D (da reta (r)) dividem harmonicamente o


segmento AB, dizemos que os pontos C e D são os pontos conjugados harmônicos em
relação ao segmento AB.

TEOREMA DE PASCAL
Seja ABCDEF um hexágono inscrito em um círculo e sejam H, K
e I os pontos de interseção de AB e DE, BC e EF e AF e CD, respectivamente. Então, H,
K e I são colineares.
42 Tópicos de Matemática - Olimpíadas - IME - ITA

RETA DE NEWTON
Seja ABCD um quadrilátero tal que BA e CD intersectam-se
em E, AD e BC intersectam-se em F e sejam N, L e M os pontos médios de EF,
AC e BD, respectivamente.

Os pontos N, L e M são colineares. (A reta que "passa" por esses três pontos é
chamada de Reta de Newton).

RETA DE SIMSON-WALLACE
Quando perpendiculares são traçadas a partir de um ponto Q sobre a circunferência
circunscrita de triângulo a seus lados, suas interseções com os lados do triângulo N. L e M
são colineares e pertencem à uma mesma reta que chamamos de Reta de Simson-
Wallace, conforme ilustra a figura abaixo:
Capitulo 1 - Resumo Teórico 43

TRIÂNGULO PEDAL
Dado um triângulo ABC e um ponto P do plano que contém o triângulo ABC (que não
pertença a circunferência circunscrita ao triângulo ABC). Chama-se triângulo pedal de P
associado ao triângulo ABC o triângulo cujos vértices são os pés das perpendiculares
traçadas de P às retas suportes dos lados do triângulo ABC, conforme ilustra a figura
abaixo:

M c

O triângulo MNQ é o triângulo pedal de P em relação ao triângulo ABC

OBSERVAÇÃO: Note que no caso em que o ponto P pertence a circunferência


circunscrita ao triângulo ABC o triângulo pedal não existe (ou dizemos que é degenerado),
pois os pés das perpendiculares traçadas por P âs retas suportes dos lados do triângulo
ABC são colineares -reta de Simson-Wallace).

TRIÂNGULO MEDIAL
Dado um triângulo ABC, chama-se triângulo mediai do triângulo ABC o triângulo
cujos vértices são os pontos médios dos lados do triângulo ABC, conforme ilustra a figura
abaixo:
44 Tópicos de Matemática - Olimpíadas - IME - ITA

M. N e P são os pontos médios dos lados AC, CB e BA do triângulo ABC.


OBSERVAÇÃO: Pode-se demonstrar que entre todos os triângulos com vértices nos
lados de um triângulo ABC. o triângulo mediai MNP é o de menor área e que o valor dessa
área corresponde a % da área do triângulo ABC.

RETA DE EULER
Num triângulo qualquer ABC, o ortocentro(H), o circuncentro(O) e o baricento(G) são
colineares (a reta que os contêm é chamada de Reta de Euler). Além disso mostre que
HG = 2 ■ GO.

B p M C

OBSERVAÇÃO: Euler demonstrou que num triângulo qualquer a distância entre o

incentro e o circuncentro é dada por d = vR2-2Rr , Ora, como em qualquer triângulo

d > 0. devemos ter R2 - 2Rr > 0 R2 > 2Rr => R á 2r. Mais ainda; ocorrendo a igualdade

se, e somente se, d=0. o que equivale a dizer que o triângulo ABC é equilátero.

CÍRCULOS E CIRCUNFERÊNCIAS
Circunferência - Dados um ponto O e uma distância R>0, chamamos de
circunferência de centro O e raio R ao conjunto de todos os pontos do plano que estão a
uma distância R de O.
Capitulo 1 - Resumo Teórico 45

Círculo ou disco - Dados um ponto O e uma distância R>0, chamamos de circulo


(ou disco) de centro O e raio R ao conjunto de todos os pontos do plano que estão a uma
distância menor do que ou igual a R de O. O circulo é a reunião da circunferência com seu
interior.

Circunferência (linha) Círculo ou disco (rcr/ião)

Elementos de uma circunferência

flecha
corda

diâmetro

Posições relativas de uma reta e uma circunferência


Reta secante: Uma reta é secante a uma circunferência se essa reta intersecta a
circunferência em dois pontos distintos quaisquer.
46 Tópicos de Matemática - Olimpíadas - IME - ITA

Reta tangente Uma reta tangente a uma circunferência é uma reta que intersecta a
circunferência em um único ponto P. Toda reta tangente a uma circunferência é
perpendicular ao raio no ponto de tangência.

Reta secante Reta tangente

ÂNGULOS E ARCOS NA CIRCUNFERÊNCIA


Arco - Chamamos de arco de circunferência o pedaço da linha da circunferência
compreendido entre dois dos seus pontos, como ilustra a figura abaixo:
Capítulo 1 - Resumo Teórico 47

ÂNGULOS NA CIRCUNFERÊNCIA
1. Ângulo Central - Ângulo que tem o vértice no centro da circunferência.

Por convenção a medida (em graus) de um ângulo central é igual a medida (em
graus) do arco que ele determina sobre a circunferência, isto é a = med(AB).

2. Ângulo Inscrito - Ângulo que tem o vértice na circunferência e os lados são


secantes a ela.

Pode-se demonstrar que a medida (em graus) de um ângulo inscrito é igual a


med( AB)
metade da medida (em graus) do arco que ele determina, na figura acima: p =
2
48 Tópicos de Matemática - Olimpíadas - IME - ITA

OBSERVAÇÕES:
I) Todo triângulo inscrito numa circunferência que tem como lado um diâmetro é um
triângulo retângulo.
II) A mediana relativa a hipotenusa de um triângulo retângulo o divide em dois
triângulos isósceles como ilustra a figura abaixo:

3. Ângulo excêntrico interior - Ângulo cujo vértice é um ponto interior à


circunferência que não seja o centro da circunferência.

med(AB) + med(CD)
Demonstra-se que 0 =------ *—
Capitulo 1 - Resumo Teórico 49

4. Ângulo excêntrico exterior — Ângulo de vértice num ponto exterior á circunferência


e lados sobre semirretas secantes ou tangentes à mesma.

ed(AD)-med (BC)
Demonstra-se que a =
2

5. Ângulo de segmento - Ângulo que tem vértice na circunferência, um lado tangente


e outro secante à circunferência.

med(AB)
Demonstra-se quep =
2
50 Tópicos de Matemática - Olimpíadas - IME - ITA

TEOREMA DE PITOT (OU DO BICO)


Se por um ponto P externo a uma circunferência conduzirmos PA e PB, ambos
tangentes à circunferência, então PA = PB.

POTÊNCIA DE UM PONTO EM RELAÇÃO A UMA CIRCUNFERÊNCIA


Dado uma circunferência O e um ponto P do mesmo plano da circunferência íi tal
que a distância desse ponto P ao centro de O seja d > 0, definimos a potência de P em

relação a Q, como sendo o número real Pot(P.Q) = d2 - R2 .

Considerando uma circunferência e várias secantes que passam pelo ponto P,


demonstra-se que:

PA.PB = PC PD = PE PF = ... = Pot(p.n) = d2 -R2 = constante


I
Capítulo 1 - Resumo Teórico 51

EIXO RADICAL DE DUAS CIRCUNFERÊNCIAS


Sejam Q, e n2 duas circunferências não concêntricas. O eixo radical de £1, e O2 é o

lugar geométrico dos pontos P tais que Pot(P,Q,) = Pot(P,02)

OBSERVAÇÃO: O eixo radical de duas circunferências é sempre uma reta perpendicular


ao segmento que une os centros das duas circunferências.

RELAÇÕES MÉTRICAS NA CIRCUNFERÊNCIA


1. Relação entre duas cordas

APPB = CPPD
52 Tópicos de Matemática - Olimpíadas - IME - ITA

Relação entre duas secantes

PAPB = PCPD

3. Relação entre secante e tangente

PT2 = PA PB
Capítulo 1 - Resumo Teórico 53

4. Quadrilátero Inscritivel (ou cíclico)

O quadrilátero ABCD é inscritivel (ou cíclico) se, e somente se. a + p = 180°. ou seja,
um quadrilátero é inscritivel se, e somente se, os ângulos opostos são suplementares

OBSERVAÇÃO: Há duas outras maneiras de reconhecer um quadrilátero cíclico que são


as seguintes:
a) Os ângulos interno e externo de dois vértices opostos são congruentes, como ilustra
a figura abaixo:

a.

A B
54 Tópicos de Matemática - Olimpíadas - IME - ITA

b) Existe um lado do quadrilátero que “olha" para dois ângulos retos conforme
ilustra a figura abaixo:

O lado AB “olha" para dois ângulos retos nos vértices que não pertencem a esse lado.

5. Quadrilátero circunscritivel

O quadrilátero ABCD é circunscritivel se. e somente se, AB + CD = BC + AD, ou


seja, um quadrilátero é circunscritivel se, e somente se, as somas das medidas dos lados
opostos são iguais
Capítulo 1 - Resumo Teórico 55

TEOREMA DE PTOLOMEU

Se ABCD é um quadrilátero convexo inscritivel em uma circunferência, então o


produto dos comprimentos das diagonais é igual à soma dos produtos dos comprimentos
dos lados opostos. Em outras palavras.
AC BD = AB CD + AD BC.
Reciprocamente, se um quadrilátero ABCD satisfaz a igualdade acima, então o
quadrilátero é inscritivel.

OBSERVAÇÃO: Existe uma generalização do Teorema de Ptolomeu que é a seguinte:


dado um quadrilátero convexo qualquer ABCD, conforme ilustra a figura abaixo:
56 Tópicos de Matemática - Olimpíadas - IME - ITA

Vale a seguinte desigualdade AC BD < AB CD +AD BC . Além disso, vale a


igualdade AC • BD = AB • CD + AD BC se, e somente se, o quadrilátero ABCD é inscritivel.

TEOREMA DE HIPARCO
Para qualquer quadrilátero inscritivel, a razão entre as diagonais é igual a razão da
soma dos produtos dos lados que concorrem com as respectivas diagonais".
ad + bc
Na figura abaixo. — =
n ab + cd

MEDIANA DE EULER DE UM QUADRILÁTERO


Seja ABCD um quadrilátero cujos lados medem a, b, c e d, conforme ilustra afigura
abaixo:
Capitulo 1 - Resumo Teórico 57

O Segmento PQ, que une os pontos médios das diagonais AC e BD é chamado de


mediana de Euler do quadrilátero. Para qualquer quadrilátero convexo é válida a relação:
a2+b2 + c2 + d2 = AC2 + BD2 + 4 ■ PQ2

O COMPRIMENTO DE UMA CIRCUNFERÊNCIA


O número n ■ É a razão entre o comprimento (ou perímetro) da circunferência e o
seu diâmetro.

p
— => C = 2nR
2R

ÁREA DAS PRINCIPAIS FIGURAS PLANAS


1. Introdução:
✓ O que significa medir uma grandeza?
Medir uma grandeza significa compará-la com outra de mesma espécie que é
tomada por unidade

O que é a área de uma figura plana?


É uma medida da porção do plano que é cercada (ocupada) pela figura

Como encontrar a medida da área de uma figura plana? Devemos comparar a sua
superfície (porção do plano que ela ocupa) com a de outra figura que é tomada como
unidade.
58 Tópicos de Matemática - Olimpíadas - IME - ITA

Qual uma boa sugestão para a unidade de área?

1
1 1 u.a.

2. Definição geral de área


Dado um polígono P associamos a esse polígono um número real nâo negativo,
chamado de área de P com as seguintes propriedades:
I. Polígonos congruentes têm áreas iguais.
II. Se P é um quadrado de lado 1, então (área de P) 1.
III. Se P pode ser decomposto como a reunião de n
Polígonos Pi.P2 Pn tais que dois quaisquer dele têm em comum no máximo alguns
lados, então

(área de P) = £Pk

3. ÁREA DO QUADRADO - Sendo A a área do quadrado de lado a, temos:


a
D
n
a a

E,
a B

(ABCD) = a2
Capítulo 1 - Resumo Teórico 59

4. RETÂNGULO
D C

E.
A b B

(ABCD) = bh

5. ÁREA DO TRIÂNGULO
EM FUNÇÃO DA BASE E DA ALTURA
C

A B
b

(ABC) = ^

6. EM FUNÇÃO DOS LADOS (FÓRMULA DE HERON)

A área A de um triângulo pode ser calculada pela fórmula:

A = 7p(P-a)(p-b)(p-c)

onde a, b, c são os comprimentos dos lados e p é o semi-perimetro.


60 Tópicos de Matemática - Olimpíadas - IME - ITA

EM FUNÇÃO DE DOIS LADOS E DO ÂNGULO ENTRE ELES

A = i-acsenp = - a-bseny = ^b-csena

8. EM FUNÇÃO DO RAIO DA CIRCUNFERÊNCIA INSCRITA

A = pr
a+b+c
Onde, p =
' 2
Capitulo 1 - Resumo Teórico 61

9. EM FUNÇÃO DO RAIO DA CIRCUNFERÊNCIA CIRCUNSCRITA

. a-bc
A =--------
4.R

10. EM FUNÇÃO DOS RAIOS DAS CIRCUNFERÊNCIAS EX-INSCRITAS

(ABC) = ra (p - a), (ABC) = rb(p-b) e (ABC) = rc (p-c)


62 Tópicos de Matemática - Olimpíadas - IME - ITA

PROPRIEDADES
•s Propriedade 1
A área de um triângulo nâo se altera quando sua base permanece fixa e o terceiro
vértice percorre uma reta paralela à base.

W/AB

Na figura acima, a reta r é paralela a AB. Os triângulos ABC e ABC têm mesma
área, pois possuem mesma base e mesma altura.

Propriedade 2
Em um triângulo, uma mediana divide sua área em partes iguais.

S, = Sa
De fato, os dois triângulos interiores possuem mesma base e mesma altura. Logo,
possuem mesma área. Quando duas figuras possuem mesma área, dizemos que elas são
equivalentes. Portanto, o enunciado desta propriedade pode ser: "Uma mediana divide o
triângulo em dois outros equivalentes."
Capitulo 1 - Resumo Teórico 63

v" Propriedade 3
Se dois triângulos têm mesma altura, então a razão entre suas áreas é igual à razão
entre suas bases. A afirmação acima tem comprovação imediata a partir da fórmula que
calcula a área do triângulo.

-?1 =
S2

11. TRAPÉZIO
D b
C
/
I
i
h !
I

£1 B
A
B

(abcd)=m±
12. PARALELOGRAMO
C

(ABCD) = b-h
FB
64 Tópicos de Matemática - Olimpíadas - IME - ITA

13. LOSANGO

(ABCD) =

14. ÁREA DOS POLÍGONOS CIRCUNSCRITÍVEIS

(ABCDEF) = pr
soma dos comprimentos dos lados
Onde, p =
2
Capitulo 1 - Resumo Teórico 65

15. ÁREA DE UM QUADRILÁTERO CONVEXO


Se ABCD é um quadrilátero convexo em que as suas diagonais AC e BD formam um

ângulo a . então (ABCD) = AC■ BD ■ sena .

16. ÁREA DO CÍRCULO


66 Tópicos de Matemática - Olimpíadas - IME - ITA

17. SETOR CIRCULAR

:R2 , com a em graus.


AsETOR“^nl
Capítulo 2

Questões
68 Tópicos de Matemática - Olimpíadas - IME - ITA

QUESTÕES

Triângulos

D Considerando todos os triângulos de perímetro 15, mostre que em nenhum


deles pode haver um cujo lado mede 8.

2) Se dois lados de um triângulo isósceles medem 38cm e 14cm, calcule seu


perímetro.

3) (Torneio das cidades) Sejam a, b e c as medidas dos lados de um triângulo


ABC. Mostre que

a3 + b3 + 3abc > c3 .

4) Se existe um triângulo de lados a,b e c mostre que existe um triângulo de


lados s/ã, Vbe Vc .

5) Seja ABC um triângulo. Se P é um ponto interior ao triângulo ABC, mostre


que PA + PB + PC < AB + AC + BC .

6) Dado um quadrilátero convexo ABCD prove que o ponto P do interior do


quadrilátero para o qual a soma PA + PB + PC + PD é mínima é o ponto de
interseção das diagonais do quadrilátero.
I
Capitulo 2 - Questões 69

7) (Olimpíada portuguesa-2010) Mostre que qualquer triângulo tem dois lados a


e b tais que:

75-1 a 75+1
2 b 2

8) Seja a, b e c as medidas dos lados de um triângulo ABC. Mostre que


a2 + b2 + c2 > ab + ac + bc

Além disso, mostre que a igualdade a2+b2 + c2 =ab + ac + bc ocorre se, e


somente se o triângulo ABC for equilátero.

9) Mostre que a soma das medidas das três medianas é menor do que o
perímetro e maior que o semi-perimetro do triângulo.

10) Sabendo que num triângulo ABC, a altura relativa ao vértice A mede 12cm e a
altura relativa ao vértice B mede 20cm, determine todos os possíveis valores
para a altura relativa ao vértice C.

11) Se duas das alturas de um triângulo tem lados 6 e 12 prove que a terceira
altura deve exceder 4.

12) Observe a figura

Com base nos dados dessa figura, pode-se afirmar que o maior segmento é:
70 Tópicos de Matemática - Olimpíadas - IME - ITA

13) Na figura abaixo determine a soma a + p + y+ 0 + X.

14) (UFPE)Na figura abaixo determine a medida do ângulo a

12T

122“

15) Na figura abaixo, qual a medida a?


Capítulo 2 - Questões 71

16) Na figura abaixo ZBAC = 12°.

A F 8

Quantos triângulos isósceles podem ser formados na sequência ADF, DFE.


...?

17) (OBM)O triângulo CDE pode ser obtido pela rotação do triângulo ABC de 90°
no sentido anti-horário ao redor de C, conforme mostrado no desenho abaixo.
Determine a medida do ângulo a .
72 Tópicos de Matemática - Olimpíadas - IME - ITA

18) No triângulo ABC temos AB = AC e os cinco segmentos marcados têm todos


a mesma medida.

B ~A

Qual é a medida do ângulo BÃC?

19) Na figura abaixo AB = BC = CD = DE = EF = FG = GA. Calcule ZDAE = a .

20) O triângulo ABC abaixo é isósceles de base BC. Determine x.


Capítulo 2 - Questões 73

21) Na figura abaixo. AB = AC, ZCBD = 20°, ZBCE = 50°. DCE = 30°. Determine
a medida do ângulo ZBDE.

22) Na figura abaixo, AB = AC, ZBAC = 20°, ZCDB = 60° e ZBCE = 50°. Calcule
a medida do ângulo ZBDE = a.
74 Tópicos de Matemática - Olimpíadas - IME - ITA

23) Suponha que o triângulo ABC seja isósceles e que ZBAC = 20° e que o
ponto P é um ponto do lado AB tal que AP = BC. Determine a medida do
ângulo ZACP = a .

24) (OBM) Considere um triângulo acutângulo ABC com ZBAC = 30°. Sejam Bi,

C2 os pés das alturas relativas aos lados AC, AB. respectivamente, e B2. C2
os pontos médios dos lados AC, AB, respectivamente. Mostre que os
segmentos B1C2 e BzC, são perpendiculares.
Capítulo 2 - Questões 75

25) Na figura abaixo, AB = AC, BAC = 100° e AD = BC. Determine x.

26) Na figura abaixo, A, C e E são pontos colineares, M e N são,


respectivamente, os pontos médios de AD e BE e os triângulos ABC e CDE
são equiláteros. Calcule a medida do ângulo ZCMN.
76 Tópicos de Matemática - Olimpíadas - IME - ITA

27) No triângulo ABC da figura abaixo AD é mediana relativa ao lado BC.


determine a medida do ângulo a.

28) Na figura abaixo ABCD é um quadrilátero onde AD = BC, ZDAB = 80° e


zCBA = 40°. Um ponto P é tal que o triângulo DPC é equilátero. Calcule o
perímetro do triângulo APB sabendo que AB = 6cm e CD = 3cm.

29) Num triângulo retângulo ABC de hipotenusa BC = 36cm, a altura AH e a


mediana AM dividem o ângulo reto em três partes iguais. Determine a medida
do segmento HM.
1
Capitulo 2 - Questões 77

30) (OBM) A figura mostra um pentágono regular ABCDE inscrito em um triângulo


equilátero MNP. Determine a medida do ângulo CMD.

31) (Austrália) Dez pontos P, Q, R, .... Y estão igualmente espaçados em torno


de uma circunferência de raio unitário, conforme figura abaixo.

Determine a diferença entre as medidas dos segmentos PS e PQ.

32) Demonstrar que é retângulo todo triângulo no qual o raio de um circulo ex-inscrito
é igual a soma dos raios dos outros dois ex-inscritos com o raio do inscrito.

33) Seja ABC um triângulo de lados 4, 5 e 6. Escolhemos um ponto D sobre um dos


lados de ABC e, em seguida, baixamos perpendiculares DP e DQ aos outros 2
lados de ABC. Calcule o menor comprimento possível do segmento PQ
78 Tópicos de Matemática - Olimpíadas - IME - ITA

34) Provar que num triângulo ABC retângulo em A , vale a relação:

(a - b)2 = c2 - 4absen2

35) Se uma reta (r) passa pelo baricentro de um triângulo ABC deixando o vértice
A de um lado e os vértices B e C do outro, mostre que a distância de A a (r) é
igual a somas das distâncias de B a (r) e de C a (r).

Mostre que AE = BD + CF

36) Na figura abaixo ABCD é um quadrado. Sabendo que AE 1 CF .

Qual a medida do ângulo ZBFG = o. ?


Capitulo 2 - Questões 79

37) (PROFMAT)Em um triângulo ABC, P é o pé da bissetriz interna relativa a A


os pontos M e N dos lados AB e AC, respectivamente, são tais que BM = BP
e CN = CP. Prove que MN é paralela a BC

38) Na figura abaixo o quadrado ABCD tem lado 9 e os pontos P e Q dividem o


lado CD em três segmentos congruentes. Determine a distância do bancentro
G do triângulo BPQ ao vértice A.

A B

D P
80 Tópicos de Matemática - Olimpíadas - IME - ITA

39) Na figura abaixo BCDE é um quadrado que possui um dos seus lados
coincidindo com a hipotenusa do triângulo retângulo ABC. Se AB = 4 e AC = 3,
determine a medida do segmento AO, onde O é o centro do quadrado BCDE.

40) Seja P um conjunto de 2008 pontos distintos de um plano tal que quaisquer 4
pontos distintos desse conjunto pertencem a uma certa circunferência deste
plano. Mostre que todos estes 2008 pontos pertencem a uma mesma
circunferência.

a^ b2 c2
41) Num triângulo ABC, mostre que r-ra < — r-rb<- e
4 ’ rr=sT-

42) Sem R e r as mediadas dos raios da circunferência circunscrita e da


circunferência inscrita num triângulo ABC, mostre que Râ2r e que a
igualdade ocorre se, e somente se, o triângulo ABC é equilátero.

43) (A relação dos cinco raios) Num triângulo ABC, mostre que:
ra+rb + rc-r = 4R

Onde ra, rb e rc são os raios das circunferências ex-inscritas, r é o raio da


circunferência inscrita e R é o raio da circunferência circunscrita ao triângulo
ABC
Capitulo 2 - Questões 81

44) (MIT-2014) Seja ABC um triângulo com AB = 5cm. BC = 4cm e CA = 3cm.


Inlcialmente existe uma formiga em cada um dos três vértices. Num dado
instante as três formigas começam a caminhar simultaneamente sobre os
lados do triângulo com velocidade de 1cm/s, seguindo o sentido A —> B —C
—> A. Para cada número real positivo t menor que 3, seja A(t) a medida da
área do triângulo cujos vértices são os pontos que definem as posições das
formigas após t segundos contados desde o instante em que as formigas
iniciaram os seus movimentos. Determine o valor de t que faz com que A(t)
seja a menor possivel.

45) Prove que se um triângulo tem duas bissetrizes internas iguais, então ele é
isósceles.

46) No triângulo ABC, suponha que D está em AB e E está em AC temos que


BD = DE = EC. Se A = 60°, prove que BE e CD se intersectam no
circuncentro O do triângulo ABC.

47) Qual deverá ser o número inteiro positivo que somado a cada um dos
números 6, 8 e 14, obtêm-se as medidas dos lados de um triângulo em que o
ortocentro está no seu interior.
82 Tópicos de Matemática - Olimpíadas - 1ME - ITA

48) O triângulo PQS da figura abaixo é tal que PQ = PR = QS e ZQPR = 20°.


Determine a medida do ângulo ZRQS

49) (OBM) O canto de um quadrado de cartolina foi cortado com uma tesoura. A
soma dos comprimentos dos catetos do triângulo recortado é igual ao
comprimento do lado do quadrado. Qual o valor da soma dos ângulos a e 0
marcados na figura abaixo?

50)
a) Mostre que as medianas de um triângulo são concorrentes.
b) Mostre que as alturas de um triângulo sâo concorrentes.
c) Mostre que as bissetrizes de um triângulo são concorrentes.
Capitulo 2 - Questões 83

d) Mostre que as cevianas que ligam os vértices aos pontos em que a


circunferência inscrita intersecta cada um dos lados de um triângulo são
concorrentes (esse ponto de concorrência é chamado de ponto de
Gergonne).
e) Mostre que as mediatrizes dos lados de um triângulo sâo concorrentes.

51) (OBM)No triângulo ABC isósceles abaixo, I é o encontro das bissetrizes e H é o


encontro das alturas. Sabe-se que ZHAI = ^HBC = a. Determine o ângulo a

52) Mostre que o ponto simétrico do ortocentro de um triângulo acutângulo ABC


pertence a circunferência circunscrita ao triângulo ABC.

53) (Teorema de Menelaus) Se uma reta (r) intersecta as retas suportes dos
lados BC, CA e AB de um triângulo ABC nos pontos l_ M e N.
respectivamente, mostre que
BL CM AN 1
LC MÃNB"
84 Tópicos de Matemática - Olimpíadas - IME - ITA

E reciprocamente, ou seja, se L, M e N são pontos sobre as retas suportes

dos lados BC, CA e AB do triângulo ABC tais que — ■ ■ — = 1, então L,


LC MA NB
M e N são colineares.

54) (Teorema de Pascal) Seja ABCDEF um hexágono inscrito em um circulo e


sejam H, K e I os pontos de interseção de AB e DE, BC e EF e AF e CD,
respectivamente Então, H, K e I são colineares.
Capitulo 2 - Questões 85

55) (Reta de Newton) Seja ABCD um quadrilátero tal que BA e CD intersectam-


se em E, AD e BC intersectam-se em F e sejam N, LeM os pontos médios
de EF, AC e BD, respectivamente.

Prove que N, L e M são colineares. (A reta que “passa" por esses três pontos
é chamada de Reta de Newton).

56) (Reta de Simson-Wallace) Mostre que se perpendiculares são traçadas a


partir de um ponto Q sobre a circunferência circunscrita de triângulo a seus
lados, suas interseções com os lados do triângulo N. L e M são colineares e
pertencem á uma mesma reta que chamamos de Reta de Simson-Wallace,
conforme ilustra a figura abaixo:
86 Tópicos de Matemática - Olimpíadas - IME - ITA

57) (A reta de Euler) Mostre que num triângulo qualquer ABC, o ortocentro(H), o
circuncentro(O) e o baricento(G) são colineares (a reta que os contêm é
chamada de Reta de Euler) Além disso mostre que HG = 2 ■ GO.

58) (O círculo de nove pontos) Mostre que em todo triângulo ABC, os pontos
médios dos segmentos que unem cada um dos vértices ao ortocentro, os
pontos médios dos lados do triângulo ABC e os pés das alturas do mesmo
triângulo ABC pertencem a uma mesma circunferência, que chamamos de
circulo de nove pontos. Além disso, mostre que se a circunferência
circunscrita ao triângulo ABC tem raio R. então o círculo de nove pontos do
p
referido triângulo tem raio —.

59) Demonstre o Teorema de Ceva: Num triângulo ABC, três cevianas AP, BN e
. AM BP CN
CM sao concorrentes se, e somente se,----------------- = 1.
MB PC NA

60) (Teorema da base média de um triângulo) Dado um triângulo qualquer,


demonstre que o segmento que une os pontos médios de dois lados é
paralelo ao terceiro lado e tem um comprimento igual a metade do
comprimento desse terceiro lado.
Capitulo 2 - Questões 87

61) Mostre que o baricentro de um triângulo divide cada mediana em duas partes
tal que uma é o dobro da outra.

62) Mostre que o ortocentro de um triângulo acutângulo ABC é o circuncentro do


triângulo MNP que é construído traçando-se por cada um dos vértices do
triângulo ABC uma paralela ao lado oposto, conforme ilustra a figura abaixo:

63) Dado um triângulo ABC, chamamos de triângulo órtico o triângulo cujos


vértices sâo os pés das alturas do triângulo ABC. Mostre que o ortocentro do
triângulo ABC é o incentro do seu triângulo órtico.

64) Os lados AB, BC e CA do triângulo ABC da figura abaixo estão divididos em


três partes iguais cada um pelos pontos D, E, F, G, H e I. Sabendo-se que a
soma das medidas dos comprimentos das cevianas Al, AH, BF, BG, CD e CE
é 200cm, determine o perímetro do hexágono hachurado na figura abaixo.
88 Tópicos de Matemática - Olimpíadas - IME - ITA

65) Um ponto A qualquer é considerado sobre o lado OX do ângulo ZXOY da


figura.
Traçamos então:
i) AB1OY
ü) AQ//OY
iii) tal que PQ = 2.0A.

o B Y

Se ZPOB = 26°, determine a medida do ângulo ZXOY.

66) (ITA) Se x, y e z são os ângulos internos de um triângulo ABC


seny + senz
e senx =-- ------ . prove que o triângulo ABC é retângulo.
cos y + cos z

67) Na figura abaixo, I é o baricentro do triângulo ABC. Calcule a distância d seu


baricentro G à reta r, sabendo que BB’ = 4cm, AA' = 11cm e CO = 3cm.

JZL
B’
XI ü
A' G-
ü
C
Capítulo 2 - Questões 89

68) Seja ABC um triângulo retângulo em A, CX a bissetriz do ângulo, sendo X um


ponto do lado AB. Se CX = 4cm e BC = 24cm, quanto mede AC?

69) (USAMO) ABCD é um quadrado e M e N são os pontos médios de BC e CD


respectivamente. Determine senB.
B M C

A D

70) Na figura abaixo ABC é um triângulo acutângulo e H é o seu ortocentro.

Mostre que a medida do raio da circunferência circunscrita ao triângulo BHC é


igual a medida do raio da circunferência circunscrita ao triângulo ABC.
90 Tópicos de Matemática - Olimpíadas - IME - ITA

71) Sendo m.ia, mb e mc as medidas das medianas relativas aos lados BC = a,


AC = b e AB = c de um triângulo ABC, mostre que:
ma+mj; + m2 =-|(a2+b2 + c2)

72) Seja ABC um triângulo equilátero. Se M é um ponto do plano do triângulo AC,


mostre que:
MA<MB+MC

73) Seja ABC um triângulo equilátero. Se M é um ponto do plano do triângulo AC


que não pertence a circunferência circunscrita ao triângulo ABC, mostre que
sempre existe um triângulo cujos lados sâo MA. MB e MC. (esse triângulo é
conhecido na literatura como triângulo de Pompeiü).

74) Com um segmento DE de tamanho 3, é possível dividir um triângulo ABC de


lados 3, 4 e 5, de modo que as extremidades do segmento DE fiquem sobre
os lados AB e BC do triângulo ABC?

POLÍGONOS

75) Mostre que nenhum polígono convexo pode ter mais que dois ângulos
internos de 30°.

76) Na figura abaixo ABCDE é um pentágono regular e ABPQ é um quadrado


interno ao pentágono. Calcule a medida do ângulo ZDBQ = a.
Capítulo 2 - Questões 91

77) Qual o número de polígonos regulares tais que quaisquer duas de suas
diagonais, que passam pelo centro, formam entre si um ângulo cuja medida
expressa em graus é dada por um número inteiro.

78) (OMRN)Um retângulo de ouro é um retângulo de dimensões 1xq> onde

V5 + 1 é a conhecida razão auréa. Este tipo de retângulo goza da


<P-------

propriedade de que ele pode ser dividido num quadrado e num retângulo
semelhante ao retângulo original. Este processo continua infinitamente
conforme ilustra a figura a abaixo:

1 1

<p 1 (p-1
s
Diante do exposto, mostre que:
.111
1 + ,p2 + ^+<Pe+-’<P

79) Mostre que a medida do comprimento da diagonal d de um pentágono regular


de lado 1 mede d = <p = ^+ - (medida da razão áurea).
92 Tópicos de Matemática - Olimpíadas - IME - ITA

80) ABCDE é um pentágono cujos ângulos B e E são iguais e no qual AB = AE,


BC=ED. Demonstrar que os ângulos C e D são iguais.

81) (OBM) Na figura abaixo o Hexágono ABCDEF é inscntível. Sabendo que


AB = 1, BC = 2. CD = 3, DE = 4 e EF = 5, determine o comprimento do lado
FA.

82) Prove que em qualquer pentágono convexo existem dois ângulos internos
consecutivos cuja soma é maior ou igual a 216°.

83) Se A, B. C e D sâo quatro consecutivos vértices de um polígono regular,


1 1 1
.. que — =
temos + . Quantos lados o polígono tem?
Ad AU AU

84) (OMERJ) Os lados de um hexágono convexo e equiângulo medem 10, 6, 12,


14, x e y, nessa ordem. Qual o perímetro desse hexágono?
Capitulo 2 - Questões 93

QUADRILÁTEROS

85) Se x, y e z são distâncias dos vértices B, C e D de um paralelogramo ABCD a


uma reta r que contém o vértice A e é exterior a ABCD. mostre que y = x+z.

86) Uma reta r pertencente ao plano de um paralelogramo ABCD é exterior a ele.


Se A. B e C distam x,y e z, respectivamente de r, determine, em função de x,
y e z a distância do vértice Dar.

87) As bases MQ e NP de um trapézio medem 42cm e 112cm, respectivamente.


Se a medida do ângulo ZMQP é o dobro da medida do ângulo ZPNM,
determine da medida do lado PQ.

88) No trapézio ABCD, AB = CD, X é o ponto médio do segmento AB, BX = 1 e


DXC = 90°. Qual é o perímetro do trapézio ABCD?
94 Tópicos de Matemática - Olimpíadas - IME - ITA

89) Um trapézio isósceles possui diagonais perpendiculares e área medindo


98cm2. Qual a medida da sua altura?

90) (UERJ) Observe a figura abaixo:


D c

8'

A
n B

ela representa um papel quadrado ABCD, com 10cm de lado, que foi dobrado
na linha AM, em que M é o ponto médio do lado BC. Se após a dobra, A. B.
C, D e M são coplanares, determine:
a) A distância entre o ponto B’ e o segmento CD.
b) O valor de tge.

91) (Mediana de Euler) Seja ABCD um quadrilátero cujos lados medem a,b,c e d,
conforme ilustra afigura abaixo:
Capítulo 2 - Questões 95

O Segmento PQ, que une os pontos médios das diagonais AC e BD é


chamado de mediana de Euler do quadrilátero. Mostre que
a2 + b2 + c2 +d2 = AC2 + BD2 +4 PQ2

92) Num trapézio ABCD, de bases AB = x e CD = y, mostre que o segmento PQ,


que une os pontos médios dos lados não paralelos é paralelo as bases AB e
_ . x+y
CD e sua medida e----- - .
2

93) Na figura seguinte, o perímetro do triângulo ABC vale 20cm, a base BC mede
8cm e o círculo está inscrito no quadrilátero BCED. Calcule o perímetro do
triângulo ADE.
96 Tópicos de Matemática - Olimpíadas - IME - ITA

94) O triângulo ABC da figura é equilátero e AD = 1, determine o tamanho da


medida do lado a do triângulo ABC.

95) Um retângulo é dividido em 9 retângulos menores conforme ilustra a figura


abaixo:

D C

12 4 6

A B

Os números indicados no interior dos retângulos menores correspondem aos


seus respectivos perímetros. Qual a medida do perímetro do retângulo maior?
Capítulo 2 - Questões 97

96) Dois círculos, de raios R e r, respectivamente, são inscritos num quadrado


ABCD de lado 1, conforme ilustra a figura a seguir
□ c

A B
Calcule a soma R + r.

97) Mostre que o perímetro de um quadrilátero convexo é maior que a soma de


suas diagonais e menor que o dobro da soma das suas diagonais.

98) Os ângulos A e B em um quadrilátero convexo ABCD são iguais e BC = 1,


AD = 3. Prove que a medida de CD é maior do que 2.

SEMELHANÇA

99) Na figura abaixo o ponto M é o ponto médio do lado BC, NA é bissetriz do


ângulo ZBAC e BN é perpendicular a NA Sabendo que AB e AC medem,
respectivamente, 14m e 19m, calcule o comprimento do segmento MN.
98 Tópicos de Matemática - Olimpíadas - IME - ITA

100) Em um triângulo ABC , retângulo em A. trace a altura AH . Mostre que a soma


das áreas dos círculos inscritos nos triângulos AHB e AH C é igual a área do
círculo inscrito em ABC .

B H

101) Duas retas paralelas r e s distam 5,0cm uma da outra. Entre elas está
marcado um ponto P, que dista 1,0cm de r. Pontos A e B sâo escolhidos
respectivamente sobre r e s, de modo que ZAPB = 90°.
a) Determine o comprimento de AP para que a área do triângulo APB seja
o menor possível.
b) Determine menor o valor possível para a área do triângulo APB.

102) Sejam ABC um triângulo retângulo em C, BCDE e ACFG dois quadrados


construídos externamente ao lados AC e BC do triângulo ABC. Conforme
ilustra a figura abaixo AE intersecta BC no ponto H, enquanto que BG
intersecta AC no ponto K. Determine a medida do ângulo ZDKH.
g a

D E
Capítulo 2 - Questões 99

103) Calcule o comprimento de DE na figura abaixo. Sabendo que DG = 7cm,


DF = 2cm, BDC é arco da circunferência tangente a AB e AC nos pontos B e
C respectivamente.

104) (UFPE) Os postes verticais AB e CD na figuram medem 12m e 8m


respectivamente Existem cabos de sustentação unindo a base de cada um
dos postes ao topo do outro poste.
A

Qual a distância, em metros, do ponto de interseção dos cabos á horizontal?


100 Tópicos de Matemática - Olimpíadas - IME - ITA

105) (UFRJ) Na figura a seguir, o círculo de raio 1cm rola da posição I para a
posição F, sempre tangenciando o cateto AC do triângulo retângulo ABC. Na
posição I o circulo também tangencia AB e na posição F ele é tangente a BC.
Os lados do triângulo valem AC = 6cm, AB = 8cm e BC = 10cm. Determine a
distância percorrida pelo centro do circulo.

A B

b) Na figura abaixo os segmentos AO e AB são diâmetros das semicircunferências


de centro E e O, respectivamente. Com centro no ponto B e raio BE é traçado o
arco de circunferência EF Sabendo-se que AE = EO = 1, determine a medida do
raio da circunferência menor que tangencia os três arcos de circunferência
indicados na figura.
Capítulo 2 - Questões 101

107) A circunferência de centro O é tangente ao lado AC no ponto E e aos


prolongamentos dos lados AB e BC, como mostra na figura. Sendo DE II BC.
AB = 18cm, AC = 9 cm e BC = 21 cm, então o segmento AD, em cm. mede

.O

D, E

B C G

108) (Teorema de Monge) Dadas três circunferências Ci, C2 e C3 de centros O,.

O2 e Oae raios r-i, ra e r3, respectivamente. Seja X a interseção das tangentes

comuns externas de Ci e Ca, Y a interseção das tangentes comuns externas

de C, e C3 e, finalmente, Z a interseção das tangentes comuns externas de

C2 e C3. Prove que X, Y e Z são colineares.


102 Tópicos de Matemática - Olimpíadas - IME - ITA

109) Na figura ABCD é um retângulo, M é o ponto médio de CD e o triângulo ABM


é equilátero. Se AB = 15m, calcule BS.

110) Sejam Q, e n2 duas circunferências de centros O e P, respectivamente, e

raios r e R tais que Q, e Qj não possuam pontos em comum. A partir de 0

são traçadas duas tangentes a e a partir de P são traçadas duas

tangentes a Q,, conforme ilustra a figura abaixo:

Sendo E, F G e H os pontos de interseção dos segmentos tangentes AO, OB,


PC e PD com Q e !i2 mostre que EF=GH.
Capitulo 2 - Questões 103

111) (OBMEP-2012 - Adaptada) A figura mostra um retângulo ABCD decomposto


em dois quadrados e um retângulo menor BCFE. Quando BCFE é
semelhante a ABCD, dizemos que ABCD é um retângulo de prata e a razão
AB . . . ,
— e chamada razao de prata.

D F C

A E B

Qual é o valor da razão de prata?

112) (QBMEP-2011 - Adaptada) Na figura, AEFD é um retângulo, ABCD é um


quadrado cujo lado mede 1 os segmentos BF e DE são
perpendiculares.

Qual é a medida, em centímetros, do segmento AE?


104 Tópicos de Matemática - Olimpíadas - IME - ITA

113) Em um quadrado ABCD, AC e BD se interceptam em E. O ponto F sobre BC


é tal que os ângulos ZCAF e ZFAB são iguais, se AF intersecta BD em G e
se EG = 24, determine CF.

PITÁGORAS

114) Calcule a medida do raio do círculo que passa pelo vértice C e é tangente aos
lados AB e AD do quadrado ABCD cujo lado mede 1m.

115) Um trapézio isósceles possui diagonais perpendiculares e área medindo


98cm2 Qual a medida da sua altura?
Capítulo 2 - Questões 105

116)
a) Mostre que se um quadrilátero tem diagonais perpendiculares então a soma
dos quadrados das medidas dos lados opostos é constante.
b) O quadrilátero ABCD representado figura abaixo tem diagonais
perpendiculares?

117) Traçamos o círculo T de centro O circunscrito a um triângulo ABC. retângulo


em A de catetos 3 cm e 4 cm. Encontre o raio do círculo W de centro O’,
tangente aos catetos de ABC e interiormente a T.

118) (UK Intermediate Challenge) Uma folha de papel retangular de lados 1 e Jz


é dobrada conforme ilustra a figura abaixo:

Qual o valor da medida d?


106 Tópicos de Matemática - Olimpíadas - IME - ITA

119) A figura abaixo mostra um retângulo ABCD com AB = 12cm e BC = 2cm, e


uma circunferência passando pelos pontos C e D e tangente ao lado AB.
Determine o raio dessa circunferência.

120) (CG) No triângulo ACB representado na figura abaixo as medidas x, y, z e w

cumprem a condição x2 +z2 = y2 +w2. Mostre que ZBDC = 90°.


Capítulo 2 - Questões 107

121) P é um ponto no interior do retângulo ABCD tal que PA = 3. PC = 5 e PD = 4.


determine a medida do segmento PB.

A B

122) P é um ponto interior ao quadrado ABCD tal que PA = 1, JPB = 2 e PC = 3.


Qual a medida do ângulo ZAPB ?
108 Tópicos de Matemática - Olimpíadas - IME - ITA

123) Num quadrante de circunferência cujo raio mede 1, está inscrito um quadrado
e circunferência menor, que tangencia o quadrado ABCD e a circunferência
maior. Qual a medida do raio da circunferência menor?

124) Dado que PQRS é um quadrado de lado a e que ABS é um triângulo


equilátero, determine, em função de a, a medida do lado do triângulo
equilátero ABS.

P A Q

S R
Capitulo 2 - Questões 109

125) (OBM)Em uma folha quadriculada em que cada quadrado tem lado 2cm, são
desenhados dois círculos como na figura ao lado. A distância mínima entre os
dois círculos mede:

126) Um octógono regular ABCDEFGH está inscrito numa circunferência de raio 1.


e P é um ponto arbitrário dessa circunferência. Calcule o valor de

PA2 + PB2 +... + PH2.


110 Tópicos de Matemática - Olimpíadas - IME - ITA

127) (OPM) No triângulo ABC as medianas dos lados AB e AC são


perpendiculares. Sabendo-se que AB = 6 e AC = 8, determine a medida do
segmento BC.

LEI DOS SENOS E COSSENOS

128) Os lados de um triângulo medem a, b e c centímetros. Se forem satisfeitas as


relações 3a = 7c e 3b = 8c, qual o valor em graus, do ângulo oposto ao lado
que mede a centímetros.

129) ABCD é um quadrado e ABE é um triângulo equilátero de lado AB = 6.


Determine as medidas x e y dos segmentos DE e CF.
Capitulo 2 - Questões 111

130) ABCD é um quadrado e BCE é um triângulo equilátero. Se AB = 6. determine


a medida x do segmento DF.

131) Seja ABC um triângulo qualquer e AD a bissetriz do ângulo interno do vértice


A, conforme ilustra a figura abaixo:

Mostre que —
c
112 Tópicos de Matemática - Olimpíadas - IME - ITA

132) Seja ABC um triângulo qualquer e AE a bissetriz do ângulo interno do vértice


A, conforme ilustra a figura abaixo:

. rn n
Mostre que —
c b

133)
a) Na figura abaixo o triângulo ABC é isósceles de base BC = 1. Se

ZBAC = 36°, mostre que AB = AC = <p = ———,

b) A partir do item anterior, conclua que:


1
cos36° = sen54° = — <p

cos 72° = sen18° = ~-


2<P
Capitulo 2 - Questões 113

134) No Triângulo ABC da figura abaixo o ponto P pode deslizar livremente sobre o
lado BC. A partir de P são traçados os segmentos PM e PN que sâo
perpendiculares aos lados AB e AC. respectivamente. Determine a posição do
ponto P para que o comprimento do segmento MN seja o menor possível.

135) (USA) Na figura abaixo, calcule a medida do lado do triângulo equilátero ABC,
sabendo que AD = AE = -J7cm , BD = CE = 2cm e DE = 1cm .
114 Tópicos de Matemática - Olimpíadas - IME - ITA

136) (CRUX-MATHEMATICORUM) Um quadrado cujo lado mede s está


simetricamente inscrito num setor circular de 60°, cujo raio mede r, conforme
ilustra a figura abaixo:

Calcule r/s.

137) Inscreve-se um polígono convexo de 12 lados num círculo de modo que ele
possui, em alguma ordem, seis lados de comprimento 72 e seis de

comprimento V24 . Qual a medida do raio dessa circunferência?

138) (OLIMPÍADA IBEROAMERICANA) Um ponto P dista 5, 7 e 8 dos vértices de


um triângulo equilátero. Qual a medida do lado desse triângulo?
Capítulo 2 - Questões 115

139) Num triângulo de lados A B e C temos (a + b-c)(a + b + c) = 3ab , determine o

ângulo interno C .

140) Determine a medida do lado AB do triângulo ABC representado na figura


abaixo:

135°
P

-.,30'-
ai?''
B C

141) Na figura, ABCD é um quadrado, ZABE = 15°, EF = AF. Mostre que o BDE é
isósceles.
116 Tópicos de Matemática - Olimpíadas - IME - ITA

142) Dois círculos se mtersectam nos ponto A e B. PQ é um segmento que passa


por A e termina em P e Q conforme a figura:

BP
Prove que----- é constante.
BQ

143) Prove que em um paralelogramo, a soma dos quadrados dos quatro lados é
igual à soma dos quadrados das diagonais.

144) No triângulo ABC da figura abaixo, determine a medida do lado BC.


Capitulo 2 - Questões 117

145) Um corredor A está sobre uma reta (r) e corre sobre ela no sentido Ax. Um
corredor B não está em (r) e, correndo em linha reta, pretende alcançar A,
conforme ilustra a figura abaixo:

(r)

Considerando que BÂX = 120°, VA = 8,0m/s e VB = 8 Jã m/s (módulos das

velocidades dos corredores A e B respectivamente). Determine a medida do


ângulo a que a trajetória de B deve fazer em uma reta para que o encontro
seja possível, supondo que A e B partem simultaneamente.

146) (OBM) As alturas de um triângulo medem 12, 15 e 20. Quanto mede o maior
ângulo interno desse triângulo?

147) Se A e C são ângulos agudos do triângulo ABC se A = 30° e a altura traçada


de A tem a mesma medida da mediana traçada de C, determine o ângulo
interno C.
118 Tópicos de Matemática - Olimpíadas - IME - ITA

148) Uma circunferência inscrita num triângulo ABC toca AB no ponto D de modo
que AD = 5 e DB = 3. Encontre a medida de BC, se  = 60°.

149) Sobre os lados de um triângulo ABC retângulo de lados AC = 6cm,


AB = 6x/3 cm e BC = 12cm construimos três quadrados externos. Calcule a
medida dos lados do triângulo determinado pelos centros desses quadrados.
E

D
K
H c

B
A

F G

150) Uma circunferência inscrita num triângulo ABC toca AB no ponto D de modo
que AD = 5 e DB = 3. Encontre a medida de BC, se  = 60°.

151) No triângulo ABC (obtusângulo) da figura abaixo, determine a medida b do


lado AC.
Capitulo 2 - Questões 119

152) Depois de uma pequena discussão, Paulinho Carlos e Ari seguiram cada um
o seu caminho, em direções de 120° uma com a outra. Suas velocidades
estavam na razão 1:2:4. Prove que, em qualquer instante, suas posições são
os vértices de um triângulo retângulo.

153) (Teorema de Napoleão) Dado um triângulo ABC, construimos sobre cada um


dos seus lados um triângulo equilátero conforme ilustra a figura abaixo:

Mostre que o triângulo cujos vértices são os centros dos triângulos equiláteros
que foram construídos sobre os lados do triângulo original também é um
triângulo equilátero.
120 Tópicos de Matemática - Olimpíadas - IME - ITA

QUADRILÁTEROS INSCRITÍVEIS

154) Na figura abaixo determine a medida do ângulo a sabendo que ABC é um


triângulo retângulo em A, AM é bissetriz interna do ângulo do vértice A e MN é
perpendicular à hipotenusa.

155) Mostre que um trapézio é inscritível em uma circunferência se, e somente se,
ele é isósceles.

156) Prove que todo paralelogramo inscritível em uma circunferência é uma


circunferência é um retângulo.

157) Na figura, o ponto C divide o arco ACB ao meio. Mostre que o quadrilátero
DEFG é inscritível em uma circunferência.
Capítulo 2 - Questões 121

158) Os lados BC e AD de um quadrilátero ABCD são paralelos. Um círculo


encontra o lado AB em E e o lado CD em F. Prove que o quadrilátero AEFD é
cíclico.

159) Seja P o centro do quadrado construído sobre a hipotenusa AC do triângulo


retângulo ABC. Prove que BP é bissetriz do ângulo ZABC.
A E

' X p.z

C D
122 Tópicos de Matemática - Olimpíadas - IME - ITA

160) Na figura abaixo, calcule a medida do ângulo o, sabendo que ABCDE é um


pentágono onde B = D = 90°, AB = BC = CD = DE e que M é o ponto médio
do lado AE.

161) Uma variante bem mais difícil do problema anterior é o seguinte: Na figura
abaixo, calcule a medida do ângulo a, sabendo que ABCDE é um pentágono
onde B = D = 90°, AB = BC, CD = DE e que M é o ponto médio do lado AE.
Capitulo 2 - Questões 123

162) ABCD é um quadrado cujas diagonais cortam-se no ponto I. Constrói-se


exteriormente um triângulo equilátero ABM. Calcule a medida do ângulo ZAIJ,
sabendo que J é o ponto médio do lado AM.

163) (IME)Quatro retas intersectam-se formando quatro triângulos conforme a


figura abaixo. Prove que as circunferências circunscritas aos quatro triângulos
possuem um ponto em comum.
124 Tópicos de Matemática - Olimpíadas - IME - ITA

164) Quatro circunferências de mesmo raio possuem um ponto O em comum,


conforme ilustra a figura abaixo:

Sabendo de os segmentos AB, BC, CD e DA são tangentes a cada duas


circunferências (conforme a figura acima), mostre que o quadrilátero ABCD é
cíclico (inscritivel).

165) Um quadrilátero convexo ABCD é formado pela justaposição de cinco


quadriláteros convexos menores conforme ilustra a figura abaixo:
D

B
A

Mostre que. se os cinco quadriláteros menores são cíclicos, isto é, cada um


deles pode ser inscrito num circulo, então o quadrilátero ABCD também é
cíclico.
Capitulo 2 - Questões 125

166) Um quadrilátero ABCD é dito bicêntrico quando este é inscritível e


circunscritível. Prove que a área máxima de um quadrilátero bicêntrico com
perímetro fixo ocorre quando as duas circunferências são concêntricas.

167) Sejam ABC um triângulo e M o ponto médio do lado BC. Se D, E são os pés
das alturas relativas aos lados AC, AB, respectivamente, prove que o
quadrilátero BCDE é cíclico e que M é o centro a circunferência circunscrita a
esse quadrilátero.

ÂNGULOS NA CIRCUNFERÊNCIA

168) Considere um triângulo ABC e pontos X, Y e Z nos lados BC, AC e AB


respectivamente tais que A, B, C, X, Y e Z são todos distintos entre si. Mostrfe
que as circunferências circunscritas aos triângulos AYZ, BZX e CXY têm um
ponto M em comum (esse ponto é chamado de ponto de Miquel do triângulo
ABC).
126 Tópicos de Matemática - Olimpíadas - IME - ITA

169) Na figura abaixo a reta r é tangente à circunferência e é perpendicular a reta s


no ponto P. determine a medida do ângulo ZPBC sabendo que o ângulo
ZACB mede 40°.

P c (r)

170) Demonstre o teorema de Hiparco.


Para qualquer quadrilátero inscritível, a razão entre as diagonais é igual a
razão da soma dos produtos dos lados que concorrem com as respectivas
diagonais".
.. _ . . m ad + bc
Na figura abaixo, — =
n ab + cd

171) Determine os comprimentos das diagonais de um quadrilátero inscritível


ABCD cujos lados medem a,b,c e d.
Capítulo 2 - Questões 127

172) No interior de um polígono regular de n lados de comprimento L, estão


situados n círculos, todos de raio r.cada um tangente a dois círculos vizinhos
e a dois lados consecutivos do polígono, Calcule, em função de L e e de n. o
valor de r

173) (CRUX-MATHEMATICORUM) No interior de um circulo de raio R são


construídos dois círculos menores de raios a e b. conforme ilustra a figura
abaixo:

Se os dois círculos menores são tangentes ao circulo maior nos pontos P e Q


e os dois círculos menores intersectem-se nos pontos S e T e além disso os
pontos P, S e Q são colineares, mostre que R = a + b.

174) Seja M o centro da semicircunferência abaixo, determine a .


128 Tópicos de Matemática - Olimpíadas - IME - ITA

175) Sejam AB. BC dois lados adjacentes de um polígono regular de 9 lados


inscrito num circulo de centro O, conforme ilustra a figura abaixo.

Sejam M o ponto médio de AB e N o ponto médio do raio OT perpendicular a


BC. Determine a medida, em graus, do ângulo zOMN = a .

176) Seja P um ponto sobre o arco BC do circulo circunscrito ao triângulo


equilátero ABC. Prove que PA = PB + PC.

177) (OMERJ) Na figura, a reta t é tangente ao circulo e paralela ao segmento DE.


SE AD = 6, AE = 5 e CE = 7, qual o valor da medida do segmento BD = x?
Capítulo 2 - Questões 129

178) Considere três circunferências Qi,Q2 e O3 tais que os seus centros O,, O2 e

Os não estão alinhados. Sejam r, s e t os eixos radicais de Q, e n2 ,

Q, e C3 e n2 e Q3 , respectivamente. Prove que r, s e t são concorrentes


em um ponto chamado centro radical.

179) (OCM) Duas tangentes OA e OB são traçadas a um circulo de um ponto


externo O. Uma corda AC é construída paralela a OB e uma secante OC é
desenhada intersectando o circulo em E. Se K é o ponto de interseção de OB
com o prolongamento de AE. prove que OK = KB

180) Seja P um ponto no interior de um circulo tal que existem três cordas que
passam
por P e tem o mesmo comprimento. Prove que P é o centro do circulo.

181) (SHORTLIST - IMO-85) Um circulo de centro O e raio R passa pelos pontos


A e C e intersecta os lados AB e BC do triângulo ABC nos pontos K e N.
respectivamente. Os círculos circunscritos aos triângulos ABC e KBN
intersectam-se em dois pontos distintos B e M. Mostre que ZOMB = 90° .
130 Tópicos de Matemática - Olimpíadas - IME - ITA

182) Na figura abaixo, estão desenhados dois círculos de raios 8cm e 6cm, cujos
centros estão situados a uma distância 12 cm um do outro. Por P, um dos
pontos de interseção dos círculos, passa um segmento de reta QR tal que as
cordas QP e PR possuem o mesmo comprimento. Ache o quadrado do
comprimento de QP

183) Mostre, na figura abaixo, que GE = HF.


Capítulo 2 - Questões 131

184) Na figura abaixo o círculo de raio R e o círculo de raio r são tangentes entre si
e também são tangentes a reta (s). O círculo menor tem raio r, e também é
tangente aos dois primeiros círculos e à reta (s).

.. X 111
Mostre que —=■ = —^ + —=•,
,./r, Vr ^R

185) (IBERO) Dado o triângulo escaleno ABC, sejam D,E e F os pontos de


tangência dos seus lados com a circunferência inscrita no triângulo ABC. Se
G e o ponto de Interseção do segmento AD com a circunferência inscrita no
triângulo ABCD. M é o ponto médio do segmento EF e I é o incentro do
triângulo ABCD, mostre que os pontos D, G. Mel sâo conciclicos, isto, é são
vértices de um quadrilátero inscritível.
132 Tópicos de Matemática - Olimpíadas - IME - ITA

186) Considere um triângulo ABC, a bissetriz interna AD do ângulo do vértice A e a


mediana AM. Considere ainda a circunferência Q , definida pelos pontos A,0
e M, conforme ilustra a figura abaixo:

Sendo E e F os pontos de interseção da circunferência Q com os lados AB e


AC do triângulo ABC, mostre que BE = CF.

187) Na figura abaixo a reta (t) é o eixo radical das circunferências O, e . Se o


segmento AB é tangente as duas circunferências e M é a interseção de AB
com a reta (t), mostre que o ponto M é o ponto médio do segmento AB.

(t)
Capítulo 2 - Questões 133

188) (O teorema da corda quebrada - Arquimedes) O ponto M é o ponto médio


do arco AB da circunferência Q . mostrada na figura abaixo:

Se o ponto C é um ponto arbitrário do arco MB e D é o pé da perpendicular de


M À corda AC, mostre que o ponto D bissecta a linha poligonal ACB. ou seja.
AD = DC + CB.

189) Na figura abaixo o triângulo ABC é equilátero e está inscrito no círculo Q . Se


M e N são pontos médios dos lados AB e AC e P é a interseção do
prolongamento do segmento MN com o circulo Q . mostre que o ponto N
divide o segmento MP em razão áurea.
134 Tópicos de Matemática - Olimpíadas - IME - ITA

190) (OMRN) Uma formiga tem de caminhar do ponto A ao ponto B, veja figura a
seguir.

Existem duas possibilidades: caminhar a partir de A ao longo do semi-círculo


C ou ir ao longo dos semi-circulos desenhados no diâmetro, AB, do circulo
maior
Sabendo que o comprimento de uma circunferência é proporcional ao seu
raio, qual é o caminho que a formiga deve escolher, se ela pretende percorrer
o menor percurso?

191) (OMRN) Na figura abaixo os dois círculos com centros PeQ são tangentes
externamente no ponto A.

Sabendo que o segmento BC é tangente a ambos os círculos. Determine a


medida do ângulo ZBAC
Capítulo 2 - Questões 135

192. Sejam a. b e c as medidas dos lados de um triângulo ABC e R a medida do


raio da circunferência circunscrita a esse triângulo. Mostre que:

1111
— +------ 1- — > —
ab ac bc R2

192) (RPM)Seja ABC um triângulo acutãngulo de circuncentro O e sejam D. E e F


as interseções das semirretas AO, BO e CO com os lados BC. AC e AB,
respectivamente. Sendo R o circunraio do triângulo ABC, mostre que:

1 1 16
OD + OE + OF“R

ÁREAS

193) Usando as figuras abaixo demonstre o teorema de Pitágoras.

b b

1
b /'
c

c
a
b b b

b b

194) Observe a figura a seguir. Por um ponto da diagonal do retângulo foram

traçadas paralelas a seus lados. Mostre que as áreas dos retângulos

sombreados são iguais.


136 Tópicos de Matemática - Olimpíadas - IME - ITA

195) (PROFMAT-2011)Na figura abaixo, as retas r e s são paralelas a uma


distância 2 uma da outra. AB é um segmento unitário contido em s, X é um
ponto de r com AX = 5ePéopéda perpendicular baixada de B sobre AX.
(’’)

Determine o comprimento de BR.

196) (PROFMAT-2011) Se espremermos um circulo de raio 10cm entre duas retas


paralelas que distam entre si 10cm, obteremos uma figura de área menor,
mas mesmo perímetro que o circulo original. Se as partes curvas desta figura
obtida são semicircunferências,

lOem

Determine a razão da área da figura espremida pela área do círculo inicial.

197) (OBM)Em algum momento, na primeira metade do século passado, uma


pessoa chamada Afrânio tinha um valioso terreno desocupado, perto do
centro da cidade do Rio de Janeiro. Com a urbanização da cidade, ruas novas
foram abertas e o terreno de Afrânio ficou reduzido a um triângulo ABC,
retângulo em B, ainda de grande valor, pois o lado AB media 156 metros. Pois
bem, Afrânio morreu e em seu testamento os advogados encontraram as
instruções para dividir o terreno “igualmenté’ entre seus dois filhos. Era assim:
‘um muro deve ser construído perpendicularmente ao lado AB, de forma que
os dois terrenos resultantes da divisão tenham mesmo valor; o que tem a
forma de um trapézio será do meu filho mais velho e o outro será do mais
Capitulo 2 - Questões 137

novo". Os advogados concluiram que os terrenos deviam ter mesma área,


pois o testamento dizia que deveriam ter mesmo valor. Mas não foram
capazes de decidir em que posição deveria ficar o muro. Conta meu avô que
o episódio ganhou as páginas dos jornais por vários dias, com leitores
opinando de diversas maneiras sobre a posição correta do muro. Ele falava e
se divertia muito com as opiniões absurdas mas, ao mesmo tempo, me
instigava a resolver o problema. E o problema retorna para vocês. Em que
posição, relativamente ao lado AB do terreno, o muro deve ser construído?

198) (TREINAMENTO OBMEP) A figura a seguir mostra um trapézio com bases


medindo 20 cm e 14 cm e com os outros dois lados medindo 5 cm cada um,
Duas circunferências com centros A e B são tangentes às bases, uma ao lado
esquerdo e outra ao lado direito. Pergunta-se qual é o comprimento do
segmento AB.
14

20
138 Tópicos de Matemática - Olimpíadas - IME - ITA

199) (OMRN)Na figura abaixo, suponha que todas os segmentos verticais são
paralelas, que todas os segmentos horizontais são igualmente espaçados e
que todos os ângulos são retos. Nessas condições, que fração da figura toda
representa a parte hachurada?

200) (OMRN)Na figura abaixo, todas as medidas mostradas são em centímetros.


Qual é a área da região hachurada?
4 3 6 3 2

4 4

3 3

G G

3 3
2 2
2 3 6 3 4

201) (UFPE)O retângulo ABCD ilustrado a seguir está dividido em seis quadrados
e tem perímetro 21. Qual a medida da sua área?
D C

A B
Capitulo 2 - Questões 139

202) Se o quadrilátero ADCD da figura abaixo é um trapézio de bases AB e CD.


mostre que as medidas S, e S2 das áreas dos triângulos hachurados são iguais.

203) Um trapézio ABCD é dividido em quatro triângulos conforme ilustra a figura


abaixo:

Sendo Sí e S2 as medidas das áreas dos triângulos hachurados, determine,


em função de Si e S2, a medida da área do trapézio ABCD.

204) As bases do AB e CD do trapézio isósceles ABCD representado na figura


abaixo medem a e b e a sua altura mede h. Determine, em função de a. b e
h, a medida da área do triângulo hachurado.
140 Tópicos de Matemática - Olimpíadas - 1ME - ITA

205) (Olimpíada de Maio) Um retângulo de papel 3cm x 9cm é dobrado ao longo


de uma reta, fazendo coincidir dois vértices opostos. Deste modo se forma um
pentágono. Calcular a medida da área desse pentágono.

D'

A G D
T

B F C

206) (OBM) Um terreno quadrangular foi dividido em quatro lotes menores por
duas cercas retas unindo os pontos médios dos lados do terreno. As áreas de
três dos lotes estão indicadas em metros quadrados no mapa a seguir.

Qual é a área do quarto lote, representado pela região escura no mapa?


Capitulo 2 - Questões 141

207) Na figura abaixo temos um quadrado dois semi-círculos e um quarto de

círculo. Determine a razão —, entre as medidas das áreas hachuradas.


b

208) Na figura abaixo a medida da área do quadrado maior é 1.

Sabendo que o quadrado maior foi subdividido em quadrados menores, qual é


a medida da área do quadrado pequeno hachurado?
142 Tópicos de Matemática - Olimpíadas - IME - ITA

209) (UFPE) O retângulo ao lado foi dividido em nove quadrados. Se o quadrado


menor (hachurado) tem área 1, qual a medida da área do retângulo?

210) (PROFMAT-2011) Considere um triângulo retângulo isósceles ABC com


hipotenusa BC. Tomando o ponto A como centro e AB como raio,
consideramos o arco de circunferência delimitado pela corda BC.
Consideremos ainda a semicircunferência de diâmetro BC, conforme a figura
abaixo:

B.

D C
A
Designando por T a área da região triangular ABC e por S e L as áreas das
outras duas regiões.
Prove que L = T.
Capítulo 2 - Questões 143

211) Na figura abaixo ABCD é um quadrado inscrito num círculo de lado 1. Sobre
cada um dos lados do quadrado foram construídos semi-círculos cujos
diâmetros são os lados do quadrado, conforme ilustra a figura abaixo:

Determine a medida da área da região hachurada.

212) Calcule a área do triângulo ABC abaixo, sendo dados BD = 4, DE = 2, EC = 6,


BF = FC = 3.
144 Tópicos de Matemática - Olimpíadas - IME - ITA

213) No quadrado ABCD da figura abaixo foram traçados quadro arcos de circulo
com centro nos vértices do quadrado e cujo raio coincide com o lado a do
quadrado ABCD. Determine a medida da área da região hachurada.
a D
A

a a

B a C

214) Seja ABC um triângulo tal que os lados BC e AC medem respectivamente a e

b. Sabendo que a área deste triângulo é dada por ^(a2+b2), calcule a

medida do ângulo ACB.

215. Prove a desigualdade de Weintzenbock: Num triângulo ABC,


(ABC)<^(a2+b2+c2)

Ocorrendo a igualdade se, e somente se o triângulo for equilátero.

216) São dados no plano dois quadrados de lado 1 tais que o centro de um deles
coincide com um vértice do outro. Determine os valores possíveis para a área
da porção comum aos dois quadrados.
Capitulo 2 - Questões 145

217) (QBMEP-2007) A figura abaixo foi feita com quatro quadrados de 10cm de
lada cada um. Os vértices A, B e C também são centros dos quadrados
correspondentes. Qual a medida da área da região sombreada?

218) (QBMEP-2013) Dois quadrados de papel se sobrepõem como na figura. A


região não sobreposta do quadrado menor corresponde a 52% de sua área e
a região não sobreposta do quadrado maior corresponde a 73% de sua área.
Qual é a razão entre o lado do quadrado menor e o lado do quadrado maior?

52% .

219) Demonstre a fórmula de Heron: A medida da área de um triângulo de lados a,


b ecé dada por:
(ABC) = ^p(p-a)(p-b)(p-c)

220) Para que valores de n é possivel particionar um triângulo equilátero de lado n


em trapézios de lados medindo 1, 1, 1 e 2?

221) ABCD é um trapézio de bases BC e AD e lados não paralelos AB e CD. Seja


E o ponto médio do lado CD e suponha que a área do triângulo AEB seja
igual a 360cm2. Calcule a área do trapézio.

222) Por um ponto P no interior de um triângulo ABC traçamos retas paralelas aos
lados de ABC. Tais retas particionam ABC em três triângulos e três
paralelogramos. Se as áreas dos triângulos são iguais a 1, 4 e 9, calcule a
área de ABC.
146 Tópicos de Matemática - Olimpíadas - IME - ITA

223) Um hexágono convexo tem 3 lados consecutivos medindo a e os outros 3


medindo b. Sabendo que tal hexágono está inscrito em um círculo de raio R,
pede-se:
a) Calcular R em função de a e b
b) Calcular a área do hexágono em função de a e b

224) Seja ABCD um trapézio de área 18 cm2 e a soma das diagonais 12 cm.

Mostre que as diagonais sâo perpendiculares e iguais,

225) Sendo r o raio do círculo inscrito a um triângulo de lados medindo a.b e c e


seja X,Y e Z as distâncias do incentro aos vértices A,B e C respectivamente,
. , XYZ abc . ...
demonstrar que: ----- - = ■—; onde p e o semiperimetro.
r P
226) Qual a medida da área do triângulo cujos lados são J3, 2 e >;5 9

227) (Austrália) Na figura abaixo, a distância do centro O do círculo C às cordas AB


e CD são 4 cm e 3 cm, respectivamente. Sendo S-i, S2, S3 e S4 as medidas
das áreas das regiões determinadas pelos segmentos e pela linha da
circunferência, verifique que:
(Si + S3)-(S2 + S4) = 48.
Capitulo 2 - Questões 147

228) O quadrilátero ABCD foi seccionado em quatro triângulos conforme ilustra a


figura abaixo:

Se os números que aparecem no interior de cada um dos triângulos


representam as respectivas áreas dos triângulos, determine a medida da
área indicada por x.

229) (OMRN) Na figura ao lado, ABCDE é um pentágono cujo lado DC está sobre
a reta ®. Traçamos EF//AD e BG//AC. Verifique que o triângulo AGF e o
pentágono ABCDE têm a mesma área.
148 Tópicos de Matemática - Olimpíadas - IME - ITA

230) (OMRN) Determine a medida da maior área que pode possuir um pentágono
equilátero de lado 1cm e que possui um dos seus ângulos internos retos.
B

C
A

D E

231) AB é um segmento de tamanho 7. P é um ponto tal que a distância de P à


linha suporte do segmento AB é 3. Nestas condições determine o menor valor
possível para AP BP

232) Se um triângulo ABC tem área S, mostre que a medida da área do triângulo
3
formado pelas medianas do triângulo ABC é — S .
Capitulo 2 - Questões 149

233) (CRUX-MATHEMATICORUM) Os segmentos DE, CE, BF e CF dividem o


retângulo ABC em regiões menores conforme ilustra a figura ao lado. Quatro
destas regiões, dois triângulos e dois quadriláteros estão hachuradas na
figura ao lado. As áreas destas quatro regiões hachuradas são 9, 35, 6 e x.
Determine o valor de x.

234) (OBM) Na figura, todas as circunferências menores têm o mesmo raio


r e os centros das circunferências que tocam a circunferência maior sâo
vértices de um quadrado. Sejam a e b as áreas cinzas indicadas na figura.
Então qual a razão a/b?
150 Tópicos de Matemática - Olimpíadas - IME - ITA

235) (CRUX-MATHEMATICORUM) A figura abaixo é constituída por seis


quadrados numerados por I, II, III, IV. V e VI. Mostre que a soma das medidas
das áreas dos quadrados I, II e III corresponde a três vezes a somas das
medidas das áreas dos quadrados IV, V e VI.

236) Num triângulo ABC ra,rb,rcer são, respectivamente, os raios das

circunferências ex-inscritas e da circunferência inscrita. Mostre que a área


(ABC) desse triângulo pode ser determinada por (ABC) = ^ra.rb ,rc.r.

237) Num triângulo ABC ra,rb,rcer são, respectivamente, os raios das

circunferências ex-inscritas e da circunferência inscrita. Mostre que


1
r’ra
1 2 2
rb rc

238) Mostre que as medianas de um triângulo dividem o triângulo em seis


triângulos menores de mesma área.
Capitulo 2 - Questões 151

239) (HUNGRIA) S é um ponto no interior do AABC tal que as áreas dos triângulos
ABS, BCS, CAS são todas iguais. Prove que S é o baricentro de ABC.

240) (IME/1990 - AIME/1985) Seja P um ponto no interior de um triângulo ABC.


dividindo-o em seis triângulos, quatro dos quais têm áreas 40, 30, 35 e 84,
como mostra a figura. Calcule a área do triângulo ABC.

F,
84 E
P
35

40 30

B D C
152 Tópicos de Matemática - Olimpíadas - IME - ITA

241) (TREINAMENTO DA OBMEP) Com os dados da figura abaixo, calcule a


razão entre as áreas x e y.

12) (OMRN) O professor Paulinho deixou desatentamente sobre uma mesa dois
exemplares do livro "OLIMPÍADAS DE MATEMÁTICA DO ESTADO DO RN”
de modo que um dos exemplares cobre parcialmente a capa do outro
conforme ilustra a figura da esquerda representada a seguir.

K>./
Considerando que os exemplares possuem o formato perfeitamente
retangular, descubra, justificando a sua resposta, se a medida da área da
região comum as capas, que está representada sombreada na figura da
direita, é maior, menor ou igual a metade da medida da área total da capa de
cada um dos exemplares.
Capítulo 2 - Questões 153

243) Mostre que o perímetro de um triângulo de área — é maior que 2.


71

244) (PERU) Determine a razão entre as medidas das áreas hachuradas no


triângulo ABC da figura abaixo.

245) (Teorema de Ceva para áreas) Mostre que se num triângulo ABC três
cevianas AE, BG e CD são concorrentes, então o produto alternado das áreas
dos seis triângulos menores formados é constante, ou seja, sendo x, y, z, u, v
e was medidas das áreas dos seis triângulos menores, entãox- yz = u v- w .
154 Tópicos de Matemática - Olimpíadas - IME - ITA

246) (PERU) O triângulo ABC da figura abaixo é retângulo e isósceles.

Se AS é um arco de circunferência de centro B e raio BA e QT é uma


semicircunferência de centro O. Sendo x,y e z as medidas das áreas das
regiões hachuradas. mostre que z = x + y .

247) Na figura abaixo ABCD é um quadrado de lado 1. Sabendo que o ponto E é


ponto médio do lado BC, determine a medida da área do triângulo BGE
Capítulo 2 - Questões 155

248) O triângulo ABC da figura abaixo é retângulo e tem catetos de medidas 9 e


12. As duas circunferências são tangentes entre si e a dois lados do triângulo
ABC. Sabendo que as duas circunferências têm o mesmo raio r. determine o
valor de r.

249) O triângulo ABC representado na figura abaixo é retângulo em A. O diâmetro


AE da semicircunferência se encontra sobre o lado AB do triângulo ABC. Os
lados AC e BC tangenciam a semicircunferência em A e D. respectivamente.
Se AC = 5 cm e BC = 12 cm. Determine a medida do raio da
semicircunferência inscrita no triângulo ABC.
156 Tópicos de Matemática - Olimpíadas - IME - ITA

250) (OBM) A circunferência abaixo tem raio 1, o arco AB mede 70° e o arco BC
mede 40°. A área da região limitada pelas cordas AB e AC e pelo arco BC
mede:

.51) (Teorema de Viviane) Demonstre que a soma das distâncias de um ponto


qualquer do interior de um triângulo equilátero aos lados do referido triângulo
é sempre igual a medida da altura do triângulo.

252) São dados' quadrado de lado a e um triângulo equilátero de lado a, como na


figura abaixo. Calcule a área hachurada.
I
Capitulo 2 - Questões 157

253) Na figura abaixo três quadrados foram construídos sobre os lados de um


triângulo retângulo ABC. Sendo x, y e z as medidas das áreas dos triângulos
hachurados, mostre que x

254) Seja ABCD um quadrado de lado a . Calcule a medida da área da figura


hachurada, sabendo-se que M e N são pontos médios dos lados AD e CD.
respectivamente.
158 Tópicos de Matemática - Olimpíadas - IME - ITA

255) (OPM) Na figura seguinte, AD mede um terço do comprimento de AB e os


pontos E e F dividem o lado BC em três partes iguais. Sabendo que a área do
triângulo ABC é 9cm2, qual é a área da região sombreada ADEF?

256) (OMRN) Um retângulo pequeno está inteiramente contido num retângulo


maior ABCD conforme ilustra a figura abaixo:
D C

A B

Usando somente linhas retas, construa justificando a sua construção), uma


linha que divide a região hachurada em duas partes de mesma área.
Capítulo 2 - Questões 159

257) Um retângulo é dividido em 9 retângulos menores conforme ilustra a figura


abaixo.

D C

12 4 6

A B

Os números indicados no interior dos retângulos menores correspondem as


suas respectivas áreas. Qual a medida da área do retângulo maior?

258) (OBMEP-Adaptada) A figura mostra um retângulo de área 42 cm2 com os


pontos médios dos lados em destaque. Qual é a área, em cm2, da região
cinza?
160 Tópicos de Matemática - Olimpíadas - IME - ITA

259) Dada uma folha retangular ABCD de lados a e b. com a > b, é dobrada ao
longo da sua diagonal, determine a medida da área da região que fica
sobreposta, indicada pela parte hachurada na figura abaixo.

b
D

r
A a

260) O pentágono ABCDE consiste de um quadrado ACDE cujo lado mede 8cm, e
um triângulo isósceles ABC, tal que AB=BC. Se a área do pentágono mede
90cm2 determine a medida da área do triângulo BEC.

C
A

E D

I
Capítulo 2 - Questões 161

261) Um certo número de quadrados unitários são dispostos conforme a figura ao


lado. Sendo O o vértice do canto esquerdo inferior do primeiro quadrado. P e
Q os vértices dos cantos superiores direitos dos quadrados 2011 e 2012
desta sequência de quadrados, determine a medida da área do triângulo
OXY.

P Q

261) A primeira quadratura rigorosa de uma área curvilinea. conhecida como


Lúnulas de Hipócrates, feita por volta de 430 a.c. pelo filósofo e geômetra
de mesmo nome, está descrita na construção abaixo.

Os arcos ACB, BFC e AEC são semicircunferèncias.


Se as medidas dos diâmetros AC e BC são. respectivamente, iguais a 3cm e
4cm, calcule a soma das áreas das regiões sombreadas.
162 Tópicos de Matemática - Olimpíadas - IME - ITA

262) Na figura abaixo o triângulo ABE é isósceles de base AB, ZBAC = 30°e
ZACB = ZAFC = 90° . Determine a razão entre as áreas do triângulo ESC e
do triângulo ABC.

263) Três quadrados ABGH. DEJI e BCDF sâo arranjados conforme ilustra a figura
abaixo

Mostre que os dois triângulos hachurados possuem áreas iguais.

264) Qual a medida da área de um pentágono regular de lada a?


Capitulo 2 - Questões 163

265) Dado um hexágono regular ABCDEF de lado 2, em que AD e CE intersectam-


se em G, determine a medida da área do quadrilátero ABCG.

266) Um dodecágono regular é um polígono regular é um polígono regular com 12


lados. Se a distância entre dois vértices opostos de um dodecágono é de
4cm, determine a medida da área deste dodecágono em cm2.
164 Tópicos de Matemática - Olimpíadas - IME - ITA

267) No diagrama ao lado os segmentos AB, CE e FH são paralelos. Além disso


os segmentos AF e BG são perpendiculares a AB. Suponha que a área do
retângulo ABCD é x, a área do retângulo CDGF é y, e área do triângulo BDE
é z Determine a área de DEHG em termos de x, y e z.
* B

C D E

F G H

268) (UFG) A "árvore pitagórica fundamental" é uma forma estudada pela


Geometria Fractal e sua aparência característica pode representar o formato
dos galhos de uma árvore, de uma couve-flor ou de um brócolis, dependendo
de sua variação A árvore pitagórica abaixo foi construída a partir de um
triângulo retângulo, ABC, de lados AB = 3, AC = 4 e CB = 5, e de quadrados
construídos sobre seus lados. A figura ramifica-se em quadrados e triângulos
retângulos menores, semelhantes aos iniciais, sendo que os ângulos C, F e
I sâo congruentes, seguindo um processo iterativo que pode se estender
infiniti vam ente.

Com base nessas informações, calcule a área do triângulo GHI, integrante


dessa árvore pitagórica.
Capitulo 2 - Questões 165

269) O triângulo ABC da figura a seguir tem área igual a 1. Cada um de seus lados
foi dividido em três partes iguais. Calcule a área do triângulo sombreado.

270) (OMPLP-2012) Um quadrilátero ABCD está inscrito numa circunferência de


centro O. Sabe-se que as diagonais AC e BD são perpendiculares. Sobre
cada um dos lados do quadrilátero ABCD construimos semicírculos,
externamente, como mostra a figura.

a) Mostre que os triângulos AOB e COD têm a mesma área.


b) Se AC = 8 cm e BD = 6 cm. determine a área da região sombreada
166 Tópicos de Matemática - Olimpíadas - IME - ITA

271) Determine as posições dos pontos E e F sobre os lados AB e BC do retângulo


ABCD para que os triângulos ADE, BEF e CDF possuam áreas iguais.

272) Dado um quadrilátero convexo ABCD em que as suas diagonais AC e BD

formam um ângulo a . Mostre que (ABCD) - i AC BD • sena .

273) (OBM) Uma sala quadrada com 81 m2 de área tem o seu piso inteiramente
coberto por dois tapetes retangulares A e B, que nâo se superpõem, conforme
mostrado na figura (1) abaixo. Em certo momento, o tapete B é deslocado, o
tapete A é girado de 90° e colocado sobre o tapete B, conforme indicado na
figura (2).

(1) (2)

B B
A

Sabendo que a área do tapete B é o dobro da área do tapete A, e que as


medidas dos seus lados sâo dadas por números inteiros, calcule a área da
parte do piso que ficou descoberta.
Capitulo 2 - Questões 167

274) (IME)Considere o quadrado XYZW lado a. Dividindo-se cada ângulo desse


quadrado em quatro partes iguais, obtém-se o octógono regular representado
na figura abaixo. Determine o lado e a área desse octógono em função de a.
As respostas finais não devem conter expressões trigonométricas.

w z

275) (IME) Sejam p o semi-perimetro de um triângulo, Ssua área. reRos raios de


suas circunferências inscrita e circunscrita, respectivamente. Demonstre que
vale a seguinte desigualdade:

9 27

276) Na figura abaixo temos um triângulo ABC e a sua circunferência ex-inscrita


tangente ao lado BC. Sendo P, Q e R os pontos de tangència a circunferência
com as retas suportes dos lados AB, BC e CA. respectivamente, sendo p o
semi-perimetro do triângulo ABC, mostre que:
168 Tópicos de Matemática - Olimpíadas - IME - ITA

a) AT = AQ = p - c e CQ = CP = p - a .

b) BP = BT = p

c) (ABC) = (p-b)-rb.

d) (ABC) = x/p(p-a)(p-b)(p-c) (FÓRMULA DE HERON).

277) Prove que a área de um triângulo nunca excede 1/6 da soma dos quadrados
das medidas dos seus lados, isto é para um triângulo ABC com lados a, b e c.

aABC < ^(a2 + b2 +c2j

.78) (OBM)Um quadrado de lado 12 foi dividido em sete regiões retangulares que
não se sobrepõem, conforme a figura. Uma delas é um quadrado de vértice
C, cuja área é metade da área de cada um dos dois retângulos vizinhos; outra
é um quadrado de vértice A, cuja área é metade da área de cada um dos dois
retângulos vizinhos

A B

D C

a) Mostre que o quadrilátero destacado é um quadrado.


b) Calcule a área do quadrado destacado.
Capitulo 2 - Questões 169

279) (OBMEP-TREINAMENTO)A figura abaixo mostra um triângulo de altura 1


dividido por duas retas paralelas à sua base em três partes de mesma área.
Qual é a altura do trapézio central?

1 x s

s
A 8

280) Dado um quadrilátero ABCD, prove que os pontos médios M, N, P, Q dos


lados AS, BC, CD, DA formam um paralelogramo cuja área é a metade da
área do quadrilátero ABCD.

281) Determine a medida da área do paralelogramo AEFG, sabendo-se que o


paralelogramo ABCD tem área 2.
170 Tópicos de Matemática - Olimpíadas - IME - ITA

282) Considere um triângulo genérico ABC de área S. Divida cada lado em três
partes congruentes, através de dois pontos. Uma esses pontos ao vértice
oposto, obtendo um hexágono, conforme ilustra a figura abaixo:

Determine a medida da área do hexágono, em função de S.

283) Mostre que para qualquer polígono convexo de área 1, existe um


paralelogramo de área menor que 2 que contém o polígono.

284) (CANADÀ-2013)Um quadrado WXYZ de lado 6 é desenhado no interior de


um quadrado EFGH de lado 10, de modo que os seus lados não se
intersectam, mas são dois a dois paralelos, como ilustra a figura abaixo:

E F

Z Y

H G
Capitulo 2 - Questões 171

Mostre que a soma das áreas dos trapézios EFXW e GHZY independe da
posição do quadrado WXYZ no interior do quadrado EFGH, desde que os
lados de um não intersectem os lados do outro e sejam dois a dois paralelos.

285) (OMRN-2015) Na Figura a seguir, temos quatro quadrados, com lados


medindo 11, 9, 7 e 5, respectivamente.

Quanto mede a área das regiões cinzas menos a área das regiões pretas?

286) Sendo ABCD um quadrado e Í2 um circulo inscrito nesse quadrado. Se


S, + S2 + S3 =8 cm2, determine a medida da área S.

D C

A B
172 Tópicos de Matemática - Olimpíadas - IME - ITA

287) (OMRN-2015) Na figura a seguir, temos um semicírculo de raio R = 10 cm. Os


pontos B e C dividem o semicírculo AD em três arcos de comprimentos iguais.

Calcular a área sombreada do triângulo curvilíneo ABC.

288) (CANADÁ-2014) Na figura abaixo ABCD é um retângulo de lados AB = 100 e


BC = 20. Os triângulos AFB e CDE são retângulos em E e F,
respectivamente.

Determine e medida da área da região hachurada


Capitulo 2 - Questões 173

289) (PERU) As medidas dos raios das três circunferências apresentadas na figura
abaixo são 6cm, 5cm e 4cm. Se A é a medida da área da região hachurada e
B é a medida da área da região branca, determine a razão A/B.

290) (PERU)Na figura abaixo os círculos Q-pQj e têm raios 8cm, 4cm e 6cm

respectivamente, determine a medida da área da região hachurada.

ííi
174 Tópicos de Matemática - Olimpíadas - IME - ITA

291) (PERU)Na figura abaixo o ponto B é o ponto médio do arco AC. Calcule a
medida da área da região hachurada, sabendo que AD=4cm e DC=3cm.

292) (PERU)O maior círculo da figura abaixo tem raio 10cm. Qual a medida do raio
do círculo menor da figura abaixo, de modo que a medida da sua área seja
numericamente igual a medida da área hachurada?
Capitulo 2 - Questões 175

293) (PERU)Calcule a medida da área da região hachurada, sabendo-se que


AB//CD. AB = 8, CD = 6 e que o arco BD mede 90°.

294) Sobre cada um dos lados de um triângulo retângulo ABC foram construídos
semicírculos conforme ilustra afigura abaixo:

Se Si e S2 são as medidas da áreas hachuradas, mostre que a medida da

área do triângulo ABC é igual a (ABC) = S, - S2.


176 Tópicos de Matemática - Olimpíadas - IME - ITA

295) Determine da medida da área hachurada. em função da área S do triângulo


ABC, sabendo-se que os pontos assinalados dividem cada lado em partes
iguais.

296) Sobre os lados AB e AC de um triângulo retângulo foram construídos os


quadrados ACDE e ABFG, conforme ilustra a figura abaixo.
D

A J B

G
1H
F

Mostre que as medidas da áreas do quadrado ACDE e do retângulo AGHJ


são iguais.
Capitulo 2 - Questões 177

297) (QBM-2015) João cortou os quatro cantos de uma folha retangular e obteve o
um octógono equiângulo ABCDEFGH, como mostra a figura abaixo Sabendo
que AB = 2>/2, BC = 7,CD = 3s/2, DE = 3, EF = 4^2 e GH = 5^2 ,

determine a área desse octógono.


G F

4v/2
5\/2
E

H
3

D
A
3\/2
2^/2
B C
7

298) (OBM-2014) O retângulo da figura foi repartido por meio de três segmentos
em várias regiões, algumas retangulares e outras triangulares. A linha nâo
paralela aos lados é uma diagonal e os números indicam as áreas em m2 das
regiões brancas em que se encontram. Qual é a do retângulo original?

24
18

8
178 Tópicos de Matemática - Olimpíadas - IME - ITA

299) (CANADÂ-2013)No diagrama abaixo ABC é um quarto de uma pizza circular


com centro A e raio 20cm. Esse pedaço de pizza foi colocado sobre um prato
circular, conforme ilustra a figura abaixo

Qual a fração da área do prato está sendo coberta pela fatia de pizza?

300) (OBM-2014) figura ao lado, temos AF = 12 cm, AE = 16 cm. Os vértices do


quadrado EFGH pertencem aos lados do quadrado ABCD e os pontos /, J, K,
L são pontos médios dos lados de EFGH.

a) Qual é a área do quadrado ABCD?


b) Qual é a área do quadrado EFGH?
c) Qual é a área do quadrado cinza no interior do quadrado EFGH?
Capitulo 3

Resoluções
180 Tópicos de Matemática - Olimpíadas - IME - ITA

TRIÂNGULOS

01) Considerando todos os triângulos de perímetro 15. mostre que em nenhum


deles pode haver um cujo lado mede 8.

Resolução:
De fato, suponha por absurdo que existisse um triângulo ABC cujas medidas
dos lados fossem a, b e c = 8 e que o seu perímetro fosse 15. Neste caso

teriamos, a + b + 8 = 15=sa + b = 7. Mas, pelo teorema da desigualdade


triangular, o comprimento de cada lado em um triângulo tem de ser menor
que a soma dos comprimentos dos outros dois. Assim, deveriamos ter
8 = c<a + b = 7, o que é uma contradição. Portanto nâo existe um triângulo
de perímetro 15 com um dos seus lados medindo 8.

02) Se dois lados de um triângulo isósceles medem 38cm e 14cm, calcule seu
perímetro.

Resolução:
Seja x o comprimento (em cm) do terceiro lado do triângulo considerado Ora,
como o triângulo é, por hipótese, isósceles temos duas possibilidades, a
saber:
1°caso: x = 38.
Neste caso as medidas dos lados do triângulo, em cm, seriam 38, 38 e 14, o
que é plenamente possível, pois 38 < 38 + 14 e 14 < 38 + 38, satisfazendo a
desigualdade triangular. Portando, supondo x = 38 o perímetro do triângulo
seria 2p = 38 + 38 + 14 = 90.

2°caso: x = 14.
Neste caso as medidas dos lados do triângulo, em cm, seriam 38, 14 e 14, o
que não é possível pois 14 + 14 < 38, não obedecendo a desigualdade
triangular.
Portanto se as medidas dos lados de um triângulo isósceles, em cm, sâo 38 e
14 o seu perímetro é de 90cm.
Capitulo 3 - Resoluções 181

03) (Torneio das cidades) Sejam a, b e c as medidas dos lados de um triângulo


ABC. Mostre que

a3 + b3 + 3abc > c3 .

Resolução:

Lembrando que a3 +b3 = (a + b)(a2 -ab + b2) e que a + b>c (pois num

triângulo qualquer lado é menor que a soma dos outros dois), segue que
a3 + b3 + 3abc = (a + b) (a2 - ab -i- b2 j + 3abc
>c
> c (a2 - ab + b2) + 3abc

= c (a2 - ab + b2 + 3ab)

= c(a2 + 2ab + b2)


.2
= c(a + b)'
>c
>cc2 = c3

Portanto a3 + b3 + 3abc > c3 , como queríamos demonstrar.

04) Se existe um triângulo de lados a,b e c mostre que existe um triângulo de


lados Vã, Vbe Vc .

Resolução:
Suponha que a < b < c . Neste caso, como a, b e c são as medidas dos lados
de um triângulo segue que c<a + b (o triângulo existe se o maior lado for
menor que a soma dos outros dois). Vamos então provar que Vc < Vã + Vb

(perceba que Vã < Vb < Vc ), o que garantirá, pelo teorema da desigualdade

triangular, a existência do triângulo de lados Vã, Vb e Vc .


De fato,

c<a + b=>Vc< Va + b < Vã + Vb


182 Tópicos de Matemática - Olimpíadas - IME - ITA

Note que usamos o fato de que 7a+ b < 7ã + 7b , quando a e b sâo números
reais positivos. Veja porque:
Se a e b são números reais positivos segue que

a + b < a+ b+27ãb =• a+b <(7ã +7b) => 7a + b < ^(7ã + 7b) =^7a + b<7ã + 7b

05) Seja ABC um triângulo. Se P é um ponto interior ao triângulo ABC, mostre


que PA + PB+PC < AB + AC + BC .

Resolução:
Inicialmente vamos mostrar que PB + PC< AB + AC Para isso, prolongue o
segmento BP até atingir o segmento AC em D, conforma ilustra a figura
abaixo.
Capítulo 3- Resoluções 183

Agora aplicando a desigualdade triangular no triângulo ABD, segue que:

BD < AB + AD => BP + PD < AB + AD

Agora aplicando a desigualdade triangular no triângulo CPD, segue que

PC<PD + DC. Por fim. adicionando membro a membro as desigualdades

BP+PD<AB+AD e PC < PD + DC , segue que

BP+PD<AB+AD
=. BP + PD + PC < AB + AD + DC + PD = PB + PC < AB + AC
PC<PD+DC ■ AC

Como queríamos demonstrar.

Por fim, de modo completamente análogo poderiamos demonstrar que

PA + PC<AB + BC e também PA + PB<AC + BC Portanto, adicionando as

três desigualdades abaixo, segue que.

PB+PC<AB+AC
• PC + PA < AB + BC => 2(PA + PB + PC) < 2(AB-r AC + BC) =>
PA + PB <AC + BC
=> PA + PB + PC < AB + AC + BC

06) Dado um quadrilátero convexo ABCD prove que o ponto P do interior do


quadrilátero para o qual a soma PA + PB + PC + PD é mínima é o ponto de
interseção das diagonais do quadrilátero.

Resolução:
Seja Pí Q um ponto qualquer no interior do quadrilátero ABCD e Q o ponto
de interseção das suas diagonais AC e BD. conforme ilustra a figura a seguir:
184 Tópicos de Matemática - Olimpíadas - IME - ITA

Aplicando o teorema da desigualdade triangular ao triângulo ACP, segue que:


AC < PA -r PC => QA + QC < PA + PC

Agora aplicando o teorema da desigualdade triangular ao triângulo BDP,


segue que.
BD < PB + PD =. QB + QD < PB + PC
Ou seja,

IQA +QC < PA + PC _ QA + QB + QC + QD < pA pB + pc + pD


[QB+QD<PB+PC

Portanto a soma PA + PB + PC + PD é mínima quando o ponto P é o ponto de


interseção das duas diagonais do quadrilátero ABCD.

07) (Olimpíada portuguesa-2010) Mostre que qualquer triângulo tem dois lados a
e b tais que:

75-1 a 75 + 1
2 b 2

Resolução:

a 75 + 1
Suponha que c < b < a . Temos duas possibilidades, a saber: — < ou
b 2
a > 75+1
b S 2 '

a 75 + 1
1°caso
b 2
Capítulo 3 - Resoluções 185

Ora, como estamos supondo que c<b<a. segue ique

b < a => § È 1 > - = 0,61. Portanto neste caso temos a 75 + 1


e
b 2 b 2
a 75-1 75-1 a 75+1
> —. ou seja. —— <
b b 2

2°Caso: - > ^±1.


b 2
Ora, como a,b e c são as medidas dos lados de um triângulo, pelo teorema da
desigualdade triangular devemos ter a < b + c . Assim,

a 75 + 1 ( 75 +1').
b 2 l 2 J

Portanto,
75 + 1
75 + 1 75 + 1
a < b + c => ------ -b s a < b + c a ——— b < b r c => -—— b - b < c +>
2 2 2 Í44<C
Por outro lado,

(75-11 n b 2 2(75 + 1) 75 + 1 b 75 + 1
l 2 ) c 75-1 ~ (75-i)(7õ +1) ” 2 c 2

Ora, como c<b<a, segue que c < b =>-> 1 > - = 0,61. Portanto.
C 2

b 75-1 b 75 + 1 . 75-1 b 75+1 ,


e - < —-— , o que implica que —-— < - <-------- . (neste caso
c> 2 c 2
só por uma questão de respeitar as letras que foram sugeridas pelo
enunciado poderiamos renomear o b por a, o c por b e o a por c ai teriamos
75-1 a 75 + 1
mais uma vez -------- < — < -- ------ 1
2 b 2 '
186 Tópicos de Matemática - Olimpíadas - IME - ITA

Portanto concluímos que em qualquer triângulo existem dois lados (que


J ..... >/5 -1 a
V5-1 V5 +1
chamaremos os seus comprimentos de a e b) tais que —-— < -
2 2

08) Seja a.b e c as medidas dos lados de um triângulo ABC. Mostre que

a2 +b2 + c2 s ab + ac + bc

Além disso, mostre que a igualdade a2 + b2 + c2 = ab + ac + bc ocorre se, e


somente se o triângulo ABC for equilátero.

Resolução:
Na verdade a desigualdade a2 + b2 + c2 > ab + ac + bc ocorre para quaisquer
número reais a, b e c, em particular ocorrerá para os comprimentos dos lados
de um triângulo. Vejamos:
Quaisquer- que sejam os números reais a, b e c temos que

(a -b)2 + (a -c)2 + (b-c)2 > 0, pois é a soma de três quadrados de números

reais e como sabemos o quadrado de um real nunca é negativo. Assim,

(a - b)2 + (a - c)2 + (b - c)2 > 0 c=>

a2-2ab+b2 + a2-2ac + c2 + b2-2bc + c2 > 0 <=>


2a2 + 2b2 + a2 + 2c2 > 2ab + 2ac + 2bc <=>
a2 + b2 + c2 > ab + ac + bc
Além disso, perceba que a igualdade O2 + b2 +C2 = ab + OC + bc ocorre

se. e somente se, cada uma das desigualdades acima é na verdade uma
igualdade, o que equivale a dizer que

(a - b)2 = 0 <=> a = b
(a - b)2 + (a - c)2 + (b - c)2 - 0 o (a - c)2 = 0 <=> a = c o a = b = c
(b-c)2 = 0 « b = c
Capitulo 3- Resoluções 187

Que no caso em que a, b e c são as medidas dos lados de um triângulo


significa que o referido triângulo é equilátero.

09) Mostre que a soma das medidas das três medianas é menor do que o
perímetro e maior que o semi-perímetro do triângulo.

Resolução:
Considere o triângulo ABC da figura abaixo e. por exemplo, a sua mediana m„

Usando o teorema da desigualdade triangular no triângulo ACD, segue que


a b c . .
b<ma+-, Analogamente, c<mb-s-- e a<mc+-. Adicionando-se,

membro a membro, essas três desigualdades, segue que

(a + b + c) < ma +m.'b + mc +^(a + b + c) =>

(a + b + c)-^-(a + b + c) < ma + mb + mc

l(a + b + c) < ma + mb +mc =>

—1 (2p), < ma+mb+mc =.

P < ma + Plb " mc (1)


188 Tópicos de Matemática - Olimpíadas - IME - ITA

O que prova uma das desigualdades desejadas.

Para provar a outra desigualdade, prolongue o segmento AD em linha reta até


um ponto E tal que DE = ma conforme ilustra a figura abaixo:

b
c
m0

..x 2

Note que o quadrilátero ABEC é um paralelogramo (pois as diagonais AE e


BC cruzam-se ao meio), o que releva que CE = AB = c. Agora aplicando o
teorema da desigualdade triangular ao triângulo ACE, temos 2ma<b+c.

Analogamente, podemos provar que 2mb<a + ce que 2mc<a + b.

Adicionando-se, membro a membro, essas três últimas desigualdades, segue


que
2ma + 2mb + 2mb < 2 (a + b + c) =>

mia-mbrmc < (a + b + c)

ma + mb + mc < 2p

O que. juntamente com (1), revela que p < ma +mb + mc < 2p , como
queríamos demonstrar.
Capitulo 3 - Resoluções 189

10) Sabendo que num triângulo ABC, a altura relativa ao vértice A mede 12cm e a
altura relativa ao vértice B mede 20cm, determine todos os possíveis valores
para a altura relativa ao vértice C.

Resolução:
Sejam a, b e c as medidas dos lados de um triângulo ABC e ha. hb e hc as
suas respectivas alturas. Ora, como o dobro da medida da área do triângulo é
o produto base x altura, segue que
a = hh
b hc
aha = bhb = chc
b = hç
c hb

O que revela que a proporção a: b: c é a mesma que hb : h_: -2-. pois


hc

h, = a e Jg- = — = -. Ora, se hb : h : é a mesma proporção que


ha b hb C hc
hc

a:b:c, significa que o existe o triângulo de lados hb,ha e—, pois é


%
semelhante ao triângulo de lados a, b e c devido a proporcionalidade

existente entre as medidas a, b e c com as medidas hb,ha c hahb Assim.


hc
utilizando o teorema da desigualdade triangular no triângulo de lados
. . h hb
hb,ha e-2-2-, segue que:
hc

ha<hb + !íA i ha hb
(1)
hc hc hahb

hb<ha + ÍA = _L hb-ha
(2)
hc hc hahb
^a^b 1
ha+hb ha + hb
(3)
hahb
190 Tópicos de Matemática - Olimpíadas - IME - ITA

|ha ~hb| 1 ha -I- hfa


As desigualdades (1), (2) e (3). revelam que Ora.
hc hahb

como no enunciado há uma altura de 12cm e a outra de 20cm, segue que a


medida h da terceira altura deve cumprir a condição
|20 -12| 2 20 + 12 8 2 32 240 .240 .
=>----- < h <------ => 7,5 < h < 30
20-12 h 20 12 240 h 240 32 8

11) Se duas das alturas de um triângulo tem lados 6 e 12 prove que a terceira
altura deve exceder 4

Resolução:
Pelo que foi exposto no problema anterior, a medida h da terceira altura desse
triângulo deve satisfazer a condição
|12-6| 2 12 + 6 22 2 lÊ=,Z2<h<Z2^4<h<12
12 6 < h < 12 6 72 h 72 18 6
O que revela que a terceira altura deve exceder 4.

12) Observe a figura

Com base nos dados dessa figura, pode-se afirmar que o maior segmento é.
Capítulo 3 - Resoluções 191

Resolução:
Olhando para o triângulo ACE, segue facilmente que o ângulo interno do
vértice A é de 40° e no triângulo ABC o ângulo interno do vértice A é de 60°
Por outro lado, num triângulo o maior lado sempre está oposto ao maior
ângulo e vice-versa. Assim, olhando para o triângulo CDE o maior lado é o
lado CE, pois o triângulo CDE é retângulo e o maior lado de um triângulo
retângulo é a hipotenusa, pois está oposto ao maior ângulo que é de 90°.
Agora olhando para o triângulo ACE perceba que o lado CE é o menor lado,
pois está oposto ao menor ângulo desse triângulo que é o ângulo do vértice A
que mede 40°. Além disso, perceba que AC = AE, pois o triângulo ACE
apresenta dois ângulo de 70°. Por fim olhando para o triângulo ABC, perceba
que AC < BC < AB, pois esses lados estão opostos, respectivamente aos
ângulos de 55°, 60° e 65°. Diante do exposto, segue que o maior segmento
da figura é o segmento AB.

13) Na figura abaixo determine a soma a + p + y+ 0 + X.


192 Tópicos de Matemática - Olimpíadas - IME - ITA

Resolução:
Observe a figura abaixo:

Note que ZIJD = a + 7 , pois é um ângulo externo ao triângulo ACJ e de modo


análogo, zJID = p+Ã. pois é um ângulo externo do triângulo BEI. Agora
observando o triângulo DIJ e utilizando o fato de que num triângulo a soma
das medidas dos seus ângulos internos é 180°, segue que
u+p+Y+e+À = e+(a+r)+(p+A.) = i80°.

14) (UFPE) Na figura abaixo determine a medida do ângulo a


Capitulo 3 - Resoluções 193

Resolução:
Inicialmente nomeie os vértices da figura como sendo A, B. C e D e perceba
que as medidas dos ângulos internos dos vértices A, B e C são
A->180°-121° = 59°
B-> 180°-122° = 58°
C-> 180°-123° = 57°

Prolongando-se o segmento CD até atingir o segmento AB no ponto E.


construimos o triângulo BCE e nesse triângulo.
ZBEC = 180°-58°-57° = 65°, conforme ilustra a figura abaixo:

Por fim. utilizando o teorema do ângulo externo no triângulo ADE. segue que
65° = 59° + a a = 6° .
194 Tópicos de Matemática - Olimpíadas - IME - ITA

15) Na figura abaixo, qual a medida a?

Resolução:
Fixando a atenção no triângulo ABD e lembrando que a soma das medidas
dos ângulos interno de um triângulo é 180°, segue que

60° + 60° + ZABD = 180° => ZABD = 60°

o que revela que o triângulo ABD é equilátero (pois têm três ângulos
congruentes medindo 60° cada um) e portanto, AD=AB=BD. Por outro lado,
no triângulo ACD temos que:

50° + (60° 4- 20°) + ZACD = 180° => ZACD = 50°

o que revela que o triângulo ACD é isósceles de base AC (pois possui dois
ângulos congruentes medindo 50° cada um) e portanto, AD = CD. Ora, como
AD = DB (no triângulo equilátero ABD), segue que CD = AD = DB, o que
revela que o triângulo BCD é isósceles de base BC e portanto
ZDCB = ZDBC = a, conforme ilustra a figura a seguir:
Capitulo 3 - Resoluções 195

Por fim, mais uma vez utilizando o fato de que a soma das medidas dos
ângulos de um triângulo é 180°, olhando para o triângulo BCD, segue que

a + a + 20° = 180° => 2a = 160° a =80°

16) Na figura abaixo ZBAC = 12°

Quantos triângulos isósceles podem ser formados na sequência ADF, DFE....?


196 Tópicos de Matemática - Olimpíadas - IME - ITA

Resolução:
Note que o triângulo ADF é isósceles da base AF, o que revela que

ZDFA = ZDAF = 12° Por outro lado o ângulo ZFDE é um ângulo externo

do triângulo ADF, o que revela que

ZFDE = ZDFA + ZDAF = 12° + 12° = 2-12°

O triângulo DEF também é isósceles de base DE, o que revela que

ZDEF = ZFDE = 2-12°. Note que o ângulo externo do vértice F do triângulo

AEF é

ZFAE + ZAEF = 12“ +2-12“ = 3-12°

Diante do exposto perceba que os ângulos das bases dos triângulos isósceles

da sequência ADF, DFE, ... sâo

Triângulo 1 —> 12°

Triângulo 2 2 12°

Triângulo 3 -> 3.12°

Triângulo n n.12°

Como num triângulo qualquer a soma das medidas dos ângulos internos é de

180°, segue que no n-ésimo triângulo a soma dos dois ângulos da base, que

medem n.12° cada um deve ser menor que 180°, ou seja,

180°
n -12° +n -12° < 180° => n 24° < 180° => n <------ =?n<7,5
24°

O que revela que o maior valor possível para n é n = 7. Assim na sequência

de triângulos isósceles ADF, DFE, .. existes apenas 7 triângulos.


Capítulo 3 - Resoluções 197

17) (OBM)O triângulo CDE pode ser obtido pela rotação do triângulo ABC de 90°
no sentido anti-horário ao redor de C, conforme mostrado no desenho abaixo.
Determine a medida do ângulo

Resolução:
Ora, como o triângulo DCE é obtido do triângulo ABC por uma rotação de 90'

no sentido anti-horário ao redor de C, segue que DC = CB e ZBCD = 90“.

Assim o triângulo BCD é retângulo e isósceles, o que nos permite concluir que

ZDBC = ZBDC = 45“. Por fim, como ZACE = 90“ (pois o segmento CE é

obtido do segmento AC por uma rotação de 90° no sentido anti-horário em

torno de C) e ZDCE = 180“- 60“ - 40“ = 80“, segue que

ZACD = ZACE - ZDCE = 90“ -80“ = 10“


198 Tópicos de Matemática - Olimpíadas - IME - ITA

a figura abaixo ilustra todos esses fatos:

/45‘
/
a
P/ '
C
A <45°
80°
D'
60°

,40°

E
Por fim, aplicando o teorema do ângulo externo ao triângulo CDP, segue que
a = 45° + 10° = 55°.

18) No triângulo ABC temos AB = AC e os cinco segmentos marcados tèm todos


a mesma medida.

B ~A

Qual é a medida do ângulo BÂC?

Resolução:
Chamando de a a medida do ângulo do vértice A, percebendo que os
triângulos AEF, DEF, CDE, BCD e ABC sâo isósceles, usando o fato de que
Capítulo 3 - Resoluções 199

os ângulos da base de um triângulo isósceles são congruentes e o teorema


do ângulo externo, podemos completar os ângulos na figura dada da seguinte
forma: (note que o ângulo ZCED = 3a, pois é ângulo esterno do triângulo
AEF e não do triângulo DEF, analogamente o ângulo ZBDC = 4a, pois é
ângulo externo do triângulo ACD e finalmente note queZCBD = ZBDC = 4a,
pois o triângulo BCD é isósceles de base BD e portanto
ZBCA = ZCBA = 4a, visto que o triângulo ABC é isósceles de base BC. Por

fim, ZBCD = 4a —3a = a ).

Portanto no triângulo BCD temos:


4a + 4a + a = 180° => 9a = 180° => a = 20°

19) Na figura abaixo AB = BC = CD = DE = EF = FG = GA. Calcule ZDAE = a .

Resolução:
Note que pelo fato de que GA = GF, o triângulo AGF é isósceles de base AF,
o que revela que ZAFG = ZGAF = a . Além disso, no mesmo triângulo GAF
temos que zFGE = 2a , pois é um ângulo externo a esse triângulo e portanto
ZFGE = ZGAF + ZAFG = a + a = 2a
200 Tópicos de Matemática - Olimpíadas - IME - ITA

Ora, como FG=EF, o triângulo GEF é isósceles de base GE. o que revela que
ZFEG = ZFGE = 2a , Além disso, ZDFE — 3a, pois
ZDFE = ZFAG + ZFEA = a + 2a — 3a
Como ED = EF, segue que o triângulo DEF é isósceles de base DF, o que
revela que ZEDF = ZDFE = 3a. Raciocinando de modo completamente
análogo e começando com o triângulo ABC podemos concluir que
ZCED = 3a e portanto,
ZDEF = ZCED - ZGEF = 3a - 2a = a
conforme ilustra a figura abaixo:

Por fim, note que no triângulo DEF temos:


180°
a + 3a + 3a = 180° => 7a = 180° => a =------ .
7

20) O triângulo ABC abaixo é isósceles de base BC Determine x.


Capítulo 3 - Resoluções 201

Resolução:
Ora, sendo o triângulo ABC isósceles de base BC. segue que
180°-40°
ZABC = ZACB = = 70°. Assim, ZEBD = 70°-35° = 35° e
2
ZECD = 70° -15° = 55°. Portanto,
ZBEC = 180°-70°-55° = 55° e ZBDC = 180°-35°-70° = 75°
Ou seja, completados os ângulos, o triângulo ABC se apresenta da seguinte
maneira:

Agora perceba que o triângulo BCE é isósceles de base CE, pois

ZBEC = ZBCE = 55° e portanto BE=BC. Portanto o ponto F é ponto médio

de CE (a bissetriz do vértice oposto â base de um triângulo isósceles também

é medianal). Além disso, ZBFC = 180°-35°-55° 90°. Assim

ZDFC = ZDFE = 90°, o que revela que os triângulos retângulos DEF e DCF

sâo congruentes pelo caso LAL (DF é lado comum e EF = CF). Portanto, pela

congruência dos triângulos DEF e DCF segue que x = ZEDF = ZCDF = 75°.
202 Tópicos de Matemática - Olimpíadas - IME - ITA

21) Na figura abaixo, AB = AC, ZCBD = 20°. ZBCE = 50°, DCE = 30° Determine
a medida do ângulo ZBDE.

Resolução:
Como o triângulo ABC base BC, segue que
180°-2x80°
ZABC = ACB = 80° o que revela que ZBAC = = 20°. Assim,
2
ZEBD = 80° - 20° = 60° . No triângulo BCD, temos:

20° + 80° + ZBDC = 180° ZBDC = 80°. Ora, como ZBDC = ZBCD = 80°,
segue que o triângulo BCD é isósceles de base CD, o que revela que BC=BD.
Por outro lado no triângulo BCE temos:

ZBEC + 80° + 50° = 180° => ZBEC = 50°

Portanto o triângulo BCE também é isósceles de base CE, o que releva que
BE = BC. Ora, como já havíamos concluído que BC = BD, segue que BC =
BD = BE. Ora, sendo BD = BE, segue que o triângulo BDE é isósceles de
base DE, o que revela que ZBED = BDE = a, conforme ilustra a figura a
seguir:
Capítulo 3 - Resoluções 203

Portanto, no triângulo BDE temos 60° + a + a = 180° =>2a = 120° =>a = 60°.

22) Na figura abaixo, AB = AC, ZBAC = 20°, ZCDB = 60° e ZBCE = 50°. Calcule
a medida do ângulo ZBDE = a.
204 Tópicos de Matemática - Olimpíadas - IME - ITA

Resolução:
Esse problema é conhecido como "desafio Russo" e é um problema bastante
folclórico em geometria euclideana. Todo aluno que estuda geometria plana
de um modo um pouquinho mais aprofundado acaba se deparando mais cedo
ou mais tarde com esse problema. Há muitas soluções para esse problema
(no livro Mathematical Gems - Ros Honsberger MAA há um um capitulo com
várias delas). Vamos apresentar aqui um solução que não estã no livro citado.
Como o triângulo ABC é isósceles de base BC. segue que
180°-20° e
ZABC - ZACB = = 80" . Portanto, ZDBE = 80°-50° = 30°
2
ZDCE = 80" - 60° = 20°. No triângulo BEC, temos:
ZBEC -r 80° + 50° = 180° => ZBEC = 50°

Portanto o triângulo BEC é isósceles de base BE, o que revela que CB=CE,
conforme ilustra a figura abaixo:
Capitulo 3 - Resoluções 205

Agora lembrando da figura que apresentamos na solução da questão 7 temos


que:

Como mostramos na solução da questão 7, sabemos que a = 20°. Assim


podemos redesenhar a figura acima da seguinte forma:
206 Tópicos de Matemática - Olimpíadas - IME - ITA

Desenhando o triângulo equilátero CDE na figura que estamos utilizando na


resolução deste questão e o chamando de triângulo CHE pois a letra D já está
sendo usada para um outro ponto, obtemos:

Perceba que como ZDCE = 20° e o triângulo CHE é equilátero, segue que

ZHCD=40° e ZHCB = 60°-40° = 20°. Ora, como ZHCD = ZCDH = 40° ,

segue que o triângulo CHD é isósceles de base CD, o que revela que

CH = HD. Por outro lado CH = HE, pois o triângulo CHE é equilátero. Assim.

HE = CH = HD, ou seja, HE = HD, o que revela que o triângulo DHE é

isósceles de base ED. Portanto ZHED = ZHDE= 40° + a . Por fim, perceba

que ZCHB = ZCBH = 80°, pois são ângulos da base do triângulo isósceles

CBH (note que BC = CH) e além disso, ZDHE = 180°-60°-80° = 40° .

Todos estes fatos estão resumidos na figura a seguir:


Capítulo 3 - Resoluções 207

!Oi

z y4°A
E j40/+a i
VL 80°
.607 H
20\
30°
V20° 5o;
c B

Agora, no triângulo DD-jE a soma das medidas dos ângulos internos é 180°.

Assim, (40° + a) + (40° + a) + 40° = 180° => 2a = 60° => a = 30°

23) Suponha que o triângulo ABC seja isósceles e que ZBAC = 20° e que o
ponto P é um ponto do lado AB tal que AP = BC. Determine a medida do
ângulo ZACP = a .
208 Tópicos de Matemática - Olimpíadas - IME - ITA

Resolução:
Ora. como o triângulo ABC é isósceles de base BC, segue que cada um dos
180°-20°
ângulos da base do triângulo ABC sâo = 80° Além disso
2
ZBPC = 20° t a , pois é um ângulo externo do triângulo APC, conforme ilustra
a figura abaixo:
A

!0t
a

20° +a
b

80°
B a C

Aplicando a lei dos senos nos triângulos BCP e ACP, segue que

b a sen (20° + a)
ABCP => ----- ---------- r
sen(20° +a) sen80° b sen80°

AACP =■—?— = b a sena


sena sen20° b sen20°

Igualando-se as duas últimas expressões, segue que:


sen (20° +a) sena
sen(20° + a)sen20° = sen80°sena
sen80° sen20°
Mas ocorre que sen80° = cos10°. Assim,
sen (20° + a) sen20° = sen80°sena => sen (20° + a) sen20° = cos 10°sena
Capitulo 3 - Resoluções 209

Além disso, sen20° = sen(2-10°) = 2sen10ocosí0o . Portanto,

sen(20° + a)sen20° = cos10°sena => sen(20° + a)2sen10°cos10° = cos10°sena

ou seja,

sen (20° + a) 2sen10° = sena => sen (20° + a) sen10° = sena

Lembrando que sen30° = -, segue que

1
sen (20° + a) sen10° = — sena => sen (20° + a) sen10° = sen30°sena

Que ainda podemos reescrever como:


sen (20° + a) sen10° = sen (20° +10°) sena

O que revela que a = 10° é claramente uma solução para a última equação
acima. Como no triângulo ABC temos 0 < a < 90° , segue que a = 10° é de
fato a solução do problema.

24) (OBM) Considere um triângulo acutângulo ABC com ZBAC - 30°. Sejam B,,

C2 os pés das alturas relativas aos lados AC, AB. respectivamente, e B2, C2
os pontos médios dos lados AC, AB, respectivamente. Mostre que os
segmentos B1C2 e B2C1 são perpendiculares.
210 Tópicos de Matemática - Olimpíadas - IME - ITA

Resolução:
Seja O a interseção entre B,C2e BjC,. Ora, como o ponto C2 é. por

hipótese, ponto médio do lado AB, segue que segmento 6,02 é uma

mediana do triângulo retângulo AB,B e portanto

AC2 = B,C2 e ZC2B,A = ZBAB, = 30°


De modo completamente análogo podemos mostrar que
ZAC,B2 = 30° Assim, aplicando 0 teorema do ângulo externo ao triângulo
AB1C2, segue que:
ZBC2B, = ZC2B,A + ZBAB, = 60°
e portanto
ZC,OC2 = 180o- ZB^B, - ZAC,B2 = 90°

25) Na figura abaixo, AB = AC, BAC = 100° e AD = BC. Determine x.

Resolução:
Como o triângulo ABC é isósceles da base BC, pois AB = AC. segue que os
180°-100°
ângulos da sua base medem = 40° cada um. Olhando para o
2
vértice C podemos ver que ZBCD = 180°-40° = 40°, conforme ilustra a
figura a seguir
Capítulo 3 - Resoluções 211

a 100°
a
C
40°
b
a+b
140° 40’
D, x -—/. B

Agora seja E um ponto do segmento BC tal que CE = b, o que obriga que


EB=a. Traçando-se os segmento DE e AE, segue que os triângulos CDE e
ABE sâo isósceles de bases DE e AE, respectivamente, o que revela que
180°-40°
ZBAE = ZBEA = = 70°
2
180°-140°
ZCDE = ZCED = = 20°
2
ZBED = 360°-110°-70°-20° = 160°

conforme ilustra a figura abaixo


A

a 307
70°
n
C 110°
40° b
b 70"
140° a
40°
D- 1 x B

20°
Chamando DE=c e usando a lei dos senos nos triângulos CDE e ACE. segue
que
c b c b
aCDE =>
sen140° sen20° sen40° sen20°
212 Tópicos de Matemática - Olimpíadas - IME - ITA

b a b a asen30°
AACE => — => b =------------
sen30° sen110° sen30° sen70'i° sen70°
Substituindo a segunda equação na primeira, segue que
c b c a
sen40° sen20° 2sen20°cos20° 2sen20°sen70°
c a c a
- => c = a
cos 20' sen70° sen70° sen70>»‘
Ora, sendo c=a, temos que o triângulo BDE é isósceles de base 80, o que
revela que ZBDE = zEBD = x e portanto no mesmo triângulo BDE temos
que
x + x + 160" = 180° => 2x = 20° => x = 10°

2* solução:

Sobre o lado AC do triângulo ABC construa o triângulo equilátero ABE.


Perceba que os triângulos BCE e ABD são congruentes, pois BE - BA.
ZCBE = ZDAB = 100° e BC = AD. Logo ZBEC = ZABD = 40° + x . Ora,
como o triângulo ACE é isósceles de base CE, pois AB=AC=AE, segue que
180°-100°-60°
ZAEC = ZACE = = 10° e, portanto. no vértice E.
2
60° - (40° + x) = 10° . o que revela que x = 10°.
Capítulo 3 - Resoluções 213

26) Na figura abaixo, A, C e E são pontos colineares, M e N são,


respectivamente, os pontos médios de AD e BE e os triângulos ABC e CDE
são equiláteros. Calcule a medida do ângulo ZCMN.

Resolução:
Sejam CD = DE = EC = a e AB = BC = CA = b as medidas dos lados dos

triângulos equiláteros CDE e ABC, respectivamente. Ora, como

ZECD = ZACB = 60° , pois são ângulos internos dos triângulos equiláteros

CDE e ABC, segue que ZMCD = 180° - 60 - 60° = 60° . Como ilustra a figura

a seguir:
214 Tópicos de Matemática - Olimpíadas - IME - ITA

Sendo AM = MD = c, perceba que os triângulos ACD e BCE são congruentes


pelo caso LAL (pois cada um deles possui lados de medidas a e b formando
um ângulo de 120° entre si). Ora, se os triângulos ACD e BCE são
congruentes as suas medianas CM e CN também são congruentes, pois as
medianas CM e CN atingem lados congruentes dos triângulos congruentes
ACD e BCE. Assim, CM = CN. Por fim, perceba que se fizermos uma rotação
no sentido anti-horário em torno do ponto C de 60° no triângulo ACD este
triângulo por ser congruente ao triângulo BCE irá se sobrepor exatamente
sobre o triângulo BCE e assim a mediada CM irá se sobrepor exatamente
sobre a mediana CN. o que revela que a = 60°, pois a é exatamente o
ângulo que a mediana CM deve girar no sentido anti-horário em torno do
vértice C para que coincida com a mediana CN.
Capítulo 3 - Resoluções 215

27) No triângulo ABC da figura abaixo AD é mediana relativa ao lado BC


determine a medida do ângulo a.

Resolução:
Como AD é mediana relativa ao lado BC segue que BD e DC possuem a
mesma medida, que representaremos por x. Traçando a altura CE relativa ao
lado AB, é fácil perceber que CE = x, visto que o triângulo BCE é retângulo de
hipotenusa 2x e CE é o cateto oposto ao ângulo de 30° e portanto mede
metade da hipotenusa, ou seja, x. Além disso, pelo teorema do ângulo
externo aplicado ao triângulo ADB segue que ZDAB = 45°-30° = 15° e no
triângulo retângulo BCE temos que ZBCE = 180°-90°-30° = 60° . Veja a
figura a seguir:
216 Tópicos de Matemática - Olimpíadas - IME - ITA

Traçando o segmento DE construímos o triângulo CDE que é ísósceles e


possui um ângulo de 60°. Assim, na verdade o triângulo CDE é equilátero
(pois é Ísósceles e possui um ângulo de 60°) e então DE = x. Além disso,
ZADE = 60°-45° = 15°, conforme ilustra a figura abaixo;

O que revela que o triângulo ADE é Ísósceles pois apresenta dois ângulos de
medida 15 o, portanto AE = x. conforme ilustra a figura abaixo:

Ora, como CE = x segue que o triângulo retângulo ACE é Ísósceles. Portanto,


a + 15° = 45°=>cr = 30°
Capítulo 3 - Resoluções 217

28) Na figura abaixo ABCD é um quadrilátero onde AD = BC, zDAB = 80° e


ZCBA = 40°. Um ponto P é tal que o triângulo DPC é equilátero. Calcule o
perímetro do triângulo APB sabendo que AB = 6cm e CD = 3cm.

Resolução:
Sejam AD = BC = a, ZADC = a, ZCDB = p, ZAPD = 0 , CD = DP = PC = 3
e AB = 6. Assim, temos a seguinte figura:
P

3 SP,
60° — ç;
3
D 60’
60“J
a 3
a
0
a
80°
40°
A B
6
No quadrilátero ABCD temos que a + p + 80° + 40° = 360° =■ p - 240°-a
Portanto olhando para o vértice C temos que:
ZBCP + (240° - a) + 60° = 360° => ZBCP = 60° + a
218 Tópicos de Matemática - Olimpíadas - IME - ITA

Agora observe que os triângulos ADP e BCP sâo congruentes pelo caso LAL,
o que revela que PA = PB e que, portanto, o triângulo ABP é isósceles de
base AB Além disso, ainda da congruência dos triângulos ADP e BCP temos
que ZCBP = ip, o que revela que ZAPB = (60°-<p) + <p = 60°. Assim os

180°-60°
ângulos da base do triângulo ABR medem = 60° cada um, o que
2
revela que, na verdade, o triângulo ABP é equilátero da lado 6. Portanto o
perímetro do triângulo ABP é 3x 6 = 18cm.

29) Num triângulo retângulo ABC de hipotenusa BC = 36cm, a altura AH e a


mediana AM dividem o ângulo reto em três partes iguais. Determine a medida
do segmento HM.

Resolução:
Ora. como o ponto M é ponto médio da hipotenusa do triângulo retângulo
ABC, segue que AM = MC = MB (pois é a medida do raio da
semicircunferência circunscrita a esse triângulo retângulo), conforme ilustra a
figura abaixo:
c
18 - x

H
x
M

18
18

2<30°
A B

Note que no triângulo retângulo AHM, temos AM = MB = BC/2 = 36/2 = 18cm.


Assim, no triângulo AHM podemos escrever:

sen30° = — => - x
— => x = 9cm
18 2 18
Capitulo 3 - Resoluções 219

30) (OBM) A figura mostra um pentágono regular ABCDE inscrito em um triângulo


equilátero MNP. Determine a medida do ângulo CMD

Resolução:

1a Solução:
Inicialmente trace o segmento BE. Ora, como o triângulo NPM é equilátero.
segue que o triângulo BEM também é equilátero, o que revela que cada um
dos seus ângulos internos é 60° e BE = EM = MB Note que no pentágono
ABCDE, as diagonais EC e EB sâo congruentes, conforme ilustra a figura
abaixo
220 Tópicos de Matemática - Olimpíadas - IME - ITA

Diante do exposto, temos que CE = EB = EM, Assim os pontos C, B e M


pertencem a uma mesma circunferência de centro E. Como o pentágono
ABCDE é regular, segue que

(5-2).180° 180°-108°
ZBAE = = 108° e ZABE = ZAEB = = 36°.
5 2

Além disso, ZCBE = 108° - 36° = 72°. Como o triângulo BCE é isósceles
de base BC. segue que ZBCE = ZCBE = 72° e
180°-2x72‘
ZCEB = = 36° conforme ilustra a figura abaixo:
2

36° B

* 108

Ora, como o ponto E é o centro da circunferência que passa pelos pontos C,


B e M. e ZCEB = 36°, segue que o arco BC mede 36° (visto que
corresponde ao ângulo central ZCEB = 36°). Por fim perceba que o ângulo
BMC é um ângulo inscrito na circunferência que “enxerga" o arco BC, o que
36°
revela que ZBMC = — = 18°. Racionando de modo completamente

análogo, podemos concluir que ZDME = ZBMC = 18°. Portanto,

60° = q> +18° +18° => <p = 24°


Capitulo 3 - Resoluções 221

2a Solução:
Seja 2a a medida do lado do pentágono e CN = DP = b Além disso, a medida
de cada um dos ângulos internos do pentágono ABCDE é
(n-2)180° (5-2)180°
a; = 4------ -------- = i--------= 108° , ZCNB = 60° (pois é ângulo interno do
n 5
triângulo equilátero MNP). Por fim traçamos os segmentos MQ (altura do
triângulo equilátero MNP) e CM, definindo então o ângulo zCMQ = <p,
conforme ilustra figura abaixo:

Perceba que o lado do triângulo equilátero MNP é, nesse caso. 2a+2b, o que

.... .. .. cV3 2(a + b)73


revela que a medida da sua altura é ----- = = (a + b)v3 . Assim.
2 2
a
no triângulo retângulo CMQ, temos tg<p =
(a + b)s/3 '

Por outro lado, aplicando a lei dos senos no triângulo CBN temos que. segue
que:

b 2a 2asen48° 4a\/3sen48°
---------- => b = => b =
sen48° sen60° V3 3
2
a
Substituindo essa informação em tg<p = , segue que:
(a + b) Vã
222 Tópicos de Matemática - Olimpíadas - IME - ITA

a a________ a 1
tgip =
(a + b)V3 f 4a73sen48°^ rr aV3 + 4asen48° 73 + 4sen48°
3 r3
1
Portanto, tgq> =
73 + 4sen48°

Mas ocorre que 73 + 4sen48° = cotg12° (vamos mostrar isso logo em

seguida), o que revela que


1 1
tgq> = = tg12° => <p = 12°
73+4sen48° cotg12°
Por fim, note que ZCMD = 2<p = 24°.

Para finalizar a questão mostremos que 73 + 4sen48° = cot g12°.


De fato,
73 + 4sen48° = cotg12° cs

V3 + 4sen48° = í^o
sen12°
\/3sen12° + 4sen48°sen12° = cos12° o

4sen48°sen12° = cos12° - Vãsenl2°

Por um lado,

senusen|J = ^[cos (a - (1) - cos (a + (5)] =>

4sen48°sen12° = 4 - ^[cos(48° -12°) - cos (48° +12°)] = 2 cos 36° -1


(1)

Por outro lado,


cos12°- V3sen12° = 1cos12°-%/3 sen12°

= 2-cos12°-2 — sen12°
2 2
= 2sen30°cos12°-2cos30°sen12° (2)
= 2(sen30°cos12°-cos30°sen12°)
= 2sen(30°-12°)
= 2sen18°
Capitulo 3 - Resoluções 223

Portanto as igualdades (1) e (2), revelam que

4sen48°sen12° = cos 12° - V3sen12° o 2 cos 36° -1 = 2sen18°

Vamos então demonstrar que 2 cos 36° -1 = 2sen18° . que como já

esclarecemos, é equivalente a demonstrar que

4sen48°sen12° = cos 12° - V3sen12° , o que resolve o problema.

1
De fato, lembrando que cos36° = -<p e que sen18° = —, onde <p é o
2<p

número de ouro da geometria clássica, ou seja. <p = 1 (que é raiz

positiva da equação quadrática <p2 - o -1 = 0 ), segue que

De(1), 2cos36°-1= 2^<p-1 = <p-1 (3)

De (2), 2sen18° = 2 —
2 (4)
2<p <P

1
Ora, como <p2 — <p — l = —<71 = 1=0 ——— = — =><p-j = —. Portando, segue
IJ! ç>

das igualdades (3) e (4), que 2cos36°-1 = 2sen18° e portanto.

4sen48°sen12° = cos! 2°- \/3sen12° => -75 +4sen48° = cotg12°

Como queríamos demonstrar.


224 Tópicos de Matemática - Olimpíadas - IME - ITA

31) (Austrália) Dez pontos P, Q, R Y estão igualmente espaçados em torno


de uma circunferência de raio unitário, conforme figura abaixo.

Determine a diferença entre as medidas dos segmentos PS e PQ.

Resolução:
Inicialmente traçamos os segmentos PS, PQ, OP, OQ, OS e em seguida
identificamos as medidas dos ângulos (lembrando que a circunferência de
centro O está dividida em 10 partes iguais) conforme fizemos na figura a
seguir:

Q R
72=
P ,36° Z s
72° 36°

36°\ 72°

Y T
O
Capitulo 3 - Resoluções 225

Agora perceba que o triângulo OZS é isósceles de base OZ e, portanto,


ZS = OS = 1 (medida do raio da circunferência). Além disso também perceba
o triângulo PQZ também é isósceles de base QZ, o que revela que PQ = PZ.
Portanto,
PS — PQ = PS — PZ = ZS =1

32) Demonstrar que é retângulo todo triângulo no qual o raio de um círculo ex-
inscrito é igual a soma dos raios dos outros dois ex-inscritos com o raio do
inscrito.

Resolução:
Sabemos que as medidas dos raios dos círculos ex-inscritos e do in-raio sâo:
q q q q
ra = ------- - , rb = ------— , rc = ------- r e r = - ,onde (ABC)=S
(P-a) (p-b) (p-c) p

Se ra = rb + rc + r então,

s s s s
(p-a) (p-b) (p-c) p

1______ 1 1
multiplicando por p(p — a)(p — b) (p — c),
(p-a) ~ (p-b) + (p-c) p

obtemos:
p(p-a)(p-b)(p-c) p(p-a)(p-b)(p-c) + p(p-a)(p-b)(p-c) + p(p-a)(p-b)(p-c)
(P-a) " (P-b) (p-c) P
p(p-b)(p-c) = p(p-a)(p-c) + p(p-a)(p-b) + (p-a)(p-b)(p-c)

Multiplicando ambos os membros por 8, segue que


8p(p-b)(p-c) = 8p(p-a)(p-c) + 8p(p-a)(p-b)+8(p-a)(p-b)(p-c)=>

2p(2p-2b)(2p-2c) = 2p(2p-2a)(2p-2c) + 2p(2p-2a)(2p-2b) + (2p-2a)(2p-2b)(2p-2c)=>

(a + b+c)(a + c-b)(a + b-c) = (a + b+c)(b+c-a)(a + b-c) +


(a+b+c)(b+c-a)(a+c-b) +
(b+c-a)(a + c-b)(a+b-c)
226 Tópicos de Matemática - Olimpíadas - IME - ITA

Usando um pouco de força e fazendo as devidas contas e as devidas


simplificações a última expressão acima nos leva a
4a3 = 4ac2 + 4ab2 => 4a a2 = 4a (b2 + c2) => a2 = b2 +c2

Que é justamente a expressão do teorema de Pitágoras, o que revela que o


triângulo ABC é retângulo.

33) Seja ABC um triângulo de lados 4,5 e 6 . Escolhemos um ponto D sobre um


dos lados de ABC e, em seguida, baixamos perpendiculares DP e DQ aos
outros 2 lados de ABC . Calcule o menor comprimento possível do segmento
PQ

Resolução:
O problema geral é o seguinte: (Já colocamos esse problema da prova da
olimpíada de Matemática do RN há alguns anos):
No Triângulo ABC da figura abaixo o ponto P pode deslizar livremente sobre
o lado BC. A partir de P são traçados os segmentos PM e PN que são
perpendiculares aos lados AB e AC , respectivamente. Determine a posição
do ponto P para que o comprimento do segmento MN seja o menor possível.

Inicialmente note que o quadrilátero AMPN é circunscritivel pois os ângulos


AMP e ANP são retos e portanto AMP + ANP = 180°, o que implica que
Capitulo 3 - Resoluções 227

MAN + MPN = 180°, visto que num quadrilátero a soma das medias dos
ângulos internos deve ser sempre 360°.

Na figura abaixo desenhamos a circunferência que passa pelos pontos A, M,


Pe N.

Note que AP é o diâmetro desta circunferência (o triângulo APM é retângulo


em M e portanto está inscrito na semidrcunferência!) A figura abaixo mostra a
referida circunferência com o seu diâmetro AP
228 Tópicos de Matemática - Olimpíadas - IME - ITA

Aplicando a lei dos senos no AAMN segue que


MN
------- = 2R => MN = 2R ■ senA => MN = AP senA
senA
Como senA é fixo pois o ângulo A é fixo , segue que MN será mínimo quando
AP for minimo. Ora, mas AP é a distância do vértice A do triângulo ABC ao
lado BC e assim essa distância será mínima quando AP for perpendicular ao
lado BC. Noutras palavras, MN será mínimo quando P for o pé da altura
traçada do vértice A ao lado BC.
Voltando ao problema...
... os valores mínimos do comprimento do segmento sâo as medidas das
respectivas alturas do triângulo ABC, dependendo do lado em que você
tomou inicialmente o ponto D, ou seja,
Tomando D no lado AC a comprimento mínimo do segmento PQ é a medida
da altura relativa ao lado AC, que pode ser determinada por
2 _________________
hb = ^VP(P-a)(P-b)(P-c) .°u seJa.

2 ------ w---- ---------- í 2 (157 53 15^7


(PQ)min = hb = -7p(p-a)(p-b)(p-c)=-^y---
8
Capitulo 3 - Resoluções 229

Analogamente,
Tomando D em BC,

= |7p(p-a)(p-b)(p-C)=|^||| 15V7
(PQ),'min
10
e tomando D em AB,

2 I—,------ w-----Cw------ 7 2 /15 7 5 3 15^7


(PQ),)min hc = — ^/p (p-a)(p-b)(p-c) = — —
12

34) Provar que num triângulo ABC retângulo em A , vale a relação:


(a - b)2 = c2 - 4absen2 gj

Resolução:

Como o triângulo ABC é retângulo segue que cos(C)g. Da trigonometria

fCA ,1-cosC
sabemos que senj—J = J------
. Assim,
-----

1->
fCA /1-cosC 1-cosC a a -b
Seíll?J “V 2 Sen2Ê) = 2 2 2a
Ou seja,
sen2g) = ^=>2asen2g| = a - b => 4asen2 gj =
2a-2b.
2a <2)
230 Tópicos de Matemática - Olimpíadas - IME - ITA

Multiplicando ambos os membros por b, segue que:


/Q'
4a sen2 = 2a - 2b => 4absen2 (y) = 2ab-2b2 2b2 - 2ab = -4absen2 —
k 2,

Por fim adicionando c2 em ambos os membros, obtemos.

4absen2^y^ => b2 + b2 + c2 - 2ab = c2 - 4absen2


2b2 - 2ab + c2 = c2 -

Portanto,

b2 + a2 - 2ab = c2 - 4absen2 j => (a - b)2 = c2-4absen2(2^


=(a-b)'

35) Se uma reta (r) passa pelo baricentro de um triângulo ABC deixando o vértice
A de um lado e os vértices B e C do outro, mostre que a distância de A a (r) é
igual a somas das distâncias de B a (r) e de C a (r).

Mostre que AE = BD + CF.


Capitulo 3 - Resoluções 231

Resolução:
De fato, Construa a mediana AM.

Sabemos que AG = 2GM. Assim marque o ponto H, médio de AG.

F
!
E
ÍG
D
!
B M C
232 Tópicos de Matemática - Olimpíadas - IME - ITA

Considere o ponto I sobre a reta de modo que Hl seja a distância de H â reta


(r).

Marque sobre a reta (r) o ponto J tal que MJ seja a distância de M â reta (r).

Agora perceba que os triângulos GIH e GJM são congruentes pois HG = GM


(pois H é ponto médio de AG e portanto HG = AG/2 e pela propriedade do
baricentro sabemos que GM = AG/2), os ângulos IGH e JGM são congruentes
(lembre-se que se dois triângulos retângulos tem um ângulo agudo em
comum e um lado em comum eles são congruentes). Agora perceba que
BCFD é um trapézio pois BD//CF. Como M é ponto médio de BC e
Capitulo 3- Resoluções 233

MJ//BD//CF, segue que MJ é a base média do trapézio BCFD, segue que


BD + CF „ U1 BD + CF
MJ = ------ ----- Como Hl = MJ segue que Hl =
= ----- ------ . Finalmente perceba

AE
que Hl é a base média do triângulo AGE e portanto Hl = — . Ora,

BD + CF
Hl =
2 AE BD + CF
=> AE = BD + CF
Hl = ^ 2 2
2

36) Na figura abaixo ABCD é um quadrado. Sabendo que AE ± CF .

D C

Qual a medida do ângulo ZBFG = a ?

Resolução:
Trace AC. note que ZCAB = 45°, que BC e AE são alturas do triângulo ACF e
que G é, portanto, o ortocentro do triângulo ACF. Assim, o prolongamento de
FG intersecta AC perpendicularmente no ponto H.
234 Tópicos de Matemática - Olimpíadas - IME - ITA

D c

a
E_ F
A B

Por fim, no triângulo AHF temos que 45° + 90° + a = 180° => a = 45°.

37) (PROFMAT)Em um triângulo ABC, P é o pé da bissetriz interna relativa a A


os pontos M e N dos lados AB e AC, respectivamente, são tais que BM = BP
e CN = CP. Prove que MN é paralela a BC

Resolução:
Pelo teorema das bissetnzes aplicado ao triângulo ABC. segue que
m ——, AB. = m Ora,
Ora, como
como BM = m,
BM = m, CN = n
CN = e ^5.
n e — . pela
ÃB AC AC n AC n
reciproca do teorema de Thales segue que MN//BC.
Capitulo 3 - Resoluções 235

38) Na figura abaixo o quadrado ABCD tem lado 9 e os pontos P e Q dividem o


lado CD em três segmentos congruentes. Determine a distância do baricentro
G do triângulo BPQ ao vértice A.
A B

Resolução:
Nesse caso, o uso de coordenadas, facilita bastante a solução. Vejamos:
Pondo um sistema de coordenadas cartesianas com origem no vértice D do
quadrado ABCD os pontos A, B, C, D, P e Q apresentarão as seguintes
coordenadas:

A B
9

c
D X
236 Tópicos de Matemática - Olimpíadas - IME - ITA

A = (0,9), B = (9,9), C = (9,0), D = (0,0), P = (3,0) e Q = (6,0)

Ora. como o ponto G = (xG,yG) é o baricentro do triângulo BPQ, segue que

as suas coordenadas são:


xB + Xp + Xq 9+3+6
XG = =6
3 3

yB + yp + yp 9+0+0
?G = =3
3 3

Portanto,

d(G. a) = 7(xG~xA)2+(yG-yA)2

= 7(6 - °)2 + (3 - 9)2


= 772
= 6^2

39) Na figura abaixo BCDE é um quadrado que possui um dos seus lados
coincidindo com a hipotenusa do triângulo retângulo ABC. Se AB = 4 e AC =
3, determine a medida do segmento AO, onde O é o centro do quadrado
BCDE.
Capitulo 3 - Resoluções 237

Resolução:
Construimos sobre os demais lados do quadrado BCDE triângulos
congruentes ao triângulo ABC conforme ilustra a figura abaixo:

Com isso fica fácil de perceber duas coisas: A primeira é que a figura AFGH é
um quadrado de lado AC + CF = 3 + 4 = 7 e a segunda é que o segmento AO
é a metade da diagonal do quadrado AFGH. Ora como a medida da diagonal
7V2
de um quadrado de lado o é a>/2 , segue que AO = ——

40) Seja P um conjunto de 2008 pontos distintos de um plano tal que quaisquer 4
pontos distintos desse conjunto pertencem a uma certa circunferência deste
plano. Mostre que todos estes 2008 pontos pertencem a uma mesma
circunferência.

Resolução:
Seja T = {x.|, x2, x3, x4) um subconjunto de P com quatro pontos distintos. De

acordo com o enunciado do problema existe uma circunferência X que


“passa" por estes quatro pontos. Suponha, por absurdo que existe um x eP

tal que x e x . Pela hipótese do enunciado deve existir uma circunferência 7


238 Tópicos de Matemática - Olimpíadas - IME - ITA

que “passa" pelos pontos x,x2,x3,x4. Ora, mas como sabemos três pontos
nâo colineares determinam uma única circunferência (aquela que é
circunscrita ao triângulo cujos vértices sâo os três pontos dados!). Assim X e

Y “passam" pelos pontos x2,x3,x4 , logo coincidem e portanto x e X , o que


é uma contradição!. Assim todos os 2008 pontos pertencem a uma mesma
circunferência.

a2 b2 c2
41) Num triângulo ABC, mostre que rra < — ,r rb < — e r rc < —.
4 ' 4 4

Resolução:
Consideremos a figura baixo onde / é o incentro do triângulo ABC e J é o
exincentro relativo ao vértice A.

Sabemos que CD = p-ce que AE = p.


Assim, CE = p - b, o que revela que
DE = DC + CE = (p - c) + (p - b) = 2p - (b + c) = a.
Capitulo 3 - Resoluções 239

No triângulo retângulo IJF temos r + ra <IJ, valendo a igualdade se. e


somente se AB = AC. Pois a igualdade ocorre se, e somente se, os pontos de
interseção das duas circunferências com os lados do triângulo ABC
coincidem, o que ocorre se, e somente se AB = AC). Além disso, aplicando o
teorema de Pitágoras ao triângulo IJF obtemos IJ2 = a2 + (ra - r)2. Portanto.

r + ra <IJ=>

(r + ra)2<IJ2=a2+(ra-r)2=>

r2 + 2rra + ra2 < a2 + r2 - 2rra + ra2 =.

2rra = a2-2rra =>

a2
r-r. < —
a 4

Note que a igualdade ocorre se, e somente se. o triângulo ABC é isósceles
com vértice A. De modo completamente análogo poderiamos demonstrar que

b2 c2
r.rb<-er.rc<T.

42) Sem R e r as mediadas dos raios da circunferência circunscrita e da


circunferência inscrita num triângulo ABC, mostre que R > 2r e que a
igualdade ocorre se, e somente se, o triângulo ABC é equilátero.

Resolução:
De fato. a partir do problema anterior. sabemos que

a2 b2 c2
r ra s—,r rb s —e r rc s —, multiplicando membro a membro essas très

desigualdades, obtemos:

r3-la’rb'rc S
a2b2c2
64
240 Tópicos de Matemática-Olimpíadas - IME - ITA

Por outro lado, sabemos que num triângulo ABC valem as igualdades

(ABC) = ^ e (ABC) = 7ra -rb -rc-r

Assim,

r’ ra rb rcí
a2b2c2 a2b2c2
r2 r ra rb rc <
64 64

[(ABC)4R]2
r2 -(ABC)2 <
64

r2£5Í R2 > 4r2 => R > 2r


4
Note que a igualdade ocorre se, e somente se, as três igualdades

a2 b2 c2
r ra = —.r rb =—e r-rc =—

Ocorrem, o que ocorre se, e somente se, o triângulo ABC for equilátero.

(Essa belíssima solução foi publicada na revista Eureka, da Sociedade


Brasileira de Matemática, no interessantíssimo artigo O TRIÂNGULO E SUAS
PRINCIPAIS CIRCUNFERÊNCIAS - Eduardo Wagner, Rio de Janeiro - RJ)

A demonstração clássica do resultado acima é a seguinte: Euler demonstrou


que num triângulo qualquer a distância entre o incentro e o circuncentro é

dada por d = vR2 - 2Rr . Ora, como em qualquer triângulo d > 0 , devemos ter

R2 -2Rr >0nRz >2Rr R >2r Mais ainda; ocorrendo a igualdade se, e


somente se, d = 0, o que equivale a dizer que o triângulo ABC é equilátero.

43) (A relação dos cinco raios) Num triângulo ABC, mostre que:
ra+rb + rc-r = 4R

Onde ra, rb e rc são os raios das circunferências ex-inscritas, r é o raio da


circunferência inscrita e R é o raio da circunferência circunscrita ao triângulo
ABC.
Capítulo 3 - Resoluções 241

Resolução:
Sabemos que:

abc
Tr"
pr
(ABC) = (P-a)<a
(p-b)rb
(P~c)rc

. ■ (ABC) (ABC)
Assim, rb =--------- - e rc = - -------- , que adicionadas revelam que
p-b p-c

(ABC) (ABC) a(ABC)


rb + rc =
p-b p-c (p-b)(p-c)

(ABC) (ABC)
Além disso, ra = e r= , que subtraídas revelam que
p-a P

r r _ (ABC) (ABC)
a P-a P

(ABC) (ABC)
Adicionando membro a membro as igualdades ra - r P-a p e

a(ABC)
rb+rc = segue que
(p-b)(p-c) '

(ABC) (ABC) a (ABC)


ra+rb+rc- p-a p T(p-b)(p-c)

~(ABC)[(p_a) p + (p-b)(p-c)J
/ao^Í p(p-b)(p-c)-(p-a)(p-b)(p-c) + ap(p-a))
'(A C)l p (p — a) (p — b) (p — c) j
Mas ocorre que

p(p-b)(p-c) = p3-(b + c) p2 + pbc

“ (P - a) (p-b)(p-c) = -p3 + (a -r- b + c) p2 -(ab + ac + bc)p-t-abc


242 Tópicos de Matemática - Olimpíadas - IME - ITA

ap(p-a) = ap2 -pa2

que adicionadas membro a membro revelam que:

p(p— b)(p —c) —(p - a)(p - b)(p-c) + ap(p-a) = a(2p2-(a + b+c)p + bc] í
= j[4pz-(a + b + c)2p + 2bcj

= |((2p)2-(a + b + c)2p + 2bcj

= -||(a + b + c)2 - (a + b + c)2 + 2bc)


2abc
2
= abc

Por fim, voltando a igualdade


r * r * r r - (ARCtí P(P ~ b)(P-c) ~ (P~ a) (P ~ b) (P ~c)+ap(p - afl
ra + rb+ rc-r - (ABC)^ p(p a)(p-b)(p-c) J

e lembrando que p(p-o)(p-b)(p-c) = (ABC)! segue que

abc
ra + rb+rc-r = (ABC)
(ABC)2
abc
" (ABC)
= 4R
O que demonstra a belíssima relação dos cinco raio, a saber:
ra+rb + rc-r = 4R

44) (MIT-2014) Seja ABC um triângulo com AB = 5cm, BC = 4cm e CA = 3cm.


Inicialmente existe uma formiga em cada um dos três vértices. Num dado
instante as três formigas começam a caminhar simultaneamente sobre os
lados do triângulo com velocidade de 1cm/s, seguindo o sentido A —> B ->C
—> A. Para cada número real positivo t menor que 3, seja A(t) a medida da
área do triângulo cujos vértices são os pontos que definem as posições das
formigas após t segundos contados desde o instante em que as formigas
Capitulo 3 - Resoluções 243

iniciaram os seus movimentos. Determine o valor de t que faz com que A(t)
seja a menor possível.

Resolução:
Para determinar o valor de t que faz minimizar a área do triângulo cujos
vértices sâo os pontos que definem as posições das três formigas no instante
t, vamos determinar o valor de t que maximiza a soma das medidas das áreas
dos três triângulos que são formados entre o triângulo original ABC e o
triângulo cujos vértices são os pontos em que as três formigas encontram-se
no instante t, conforme ilustra a figura abaixo:(como cada uma das três
formigas cominha com uma velocidade de 1cm/s, segue que após t segundos
cada uma delas percorreu uma distância de t cm.)

Lembrando que a medida da área de um triângulo que tem lados de medidas


x e y e um ângulo de medida <p entre eles é dada por Aysenrp segue que:

(BIJ) + (AHJ) + (CHI) = lt(5-t)| + it(34-t)| + ^t(4-t)1 = ^(-12t2-47t)

que é uma função quadrática de t e portanto tem ponto de máximo em

t- A- 47/10 47
2a ” 2(-12 /10) 24
2(-12/10)~
244 Tópicos de Matemática - Olimpíadas - IME - ITA

Então t=—s é o instante que maximiza a soma das áreas


24
(BIJ) + (AHJ) + (CHI) e portanto minimiza a área do triângulo HIJ.

45) Prove que se um triângulo tem duas bissetrizes internas iguais, então ele é
isósceles

Resolução:
Considere o triângulo ABC tal que ZABC = 2p e ZACB = 2<p . Trace as

bissetrizes BD e CE Construa o paralelogramo BDFE e trace CF. Ora. como


o quadrilátero BDFE é, por construção um paralelogramo, segue que EF=BD
e ZEFD = ZEBD = p Por fim, considere os ângulos ZDEC = 0 e
ZDCF = a , conforme ilustra a figura abaixo:

Diante do exposto, perceba que o triângulo CEF é isósceles de base CE, pois
EF = BD = EC, o que revela que os seus ângulos da base CF sâo
congruentes, ou seja, |) + 0 = <p + a .

Agora, suponha por absurdo, que os ângulos ZABC = 2p e ZACB = 2<p

sejam desiguais, por exemplo, 2(! >2<p (o caso contrário, ou seja, 2/1 <2</>,

pode ser tratado de modo completamente análogo!).


Capitulo 3 - Resoluções 245

De posse dessa suposição podemos argumentar da seguinte forma:


2p >2<p => p > tp

Ora. como p + 0 = <p + a , se p > <p , segue que 0 < a . Agora olhando para o
triângulo CDF, e usando o fato de que num triângulo o menor lado se opõe ao
menor ângulo, segue que:
0 < a => DC < DF = BE
o que revela que p < <p 2p < 2<p . que é uma contradição. Assim a

suposição inicial de que 2P >2<p é falsa. Raciocinando de modo

completamente análogo podemos mostrar que 2p < 2<p . o que revela que

2p = 2<p e portanto o triângulo ABC é isósceles.

46) No triângulo ABC, suponha que D está em AB e E está em AC temos que


BD = DE = EC. Se A = 60°, prove que BE e CD se intersectam no
circuncentro O do triângulo ABC.
246 Tópicos de Matemática - Olimpíadas - IME - ITA

Resolução:
Como sugere a figura do enunciado, o ponto O é a interseção dos segmentos
BE e CD. Ora, como BD = DE = EC, segue os triângulos BDE e DEC são
isósceles de bases BE e DC, respectivamente, o que revela que os ângulos
encostados nas respectivas bases são congruentes Sejam ZCBE = p e
ZBCD = 0 Note que pelo teorema do ângulo externo aplicado aos triângulos
DEO e BCO, segue, respectivamente, que ZCOE = a + <p e ZDOB = p + 0.

Ora. como os ângulos zCOE e ZDOB são opostos pelo vértice, segue que
eles sâo congruentes e, portanto p + 0 = a + cp , como ilustra a figura a seguir.

60°

D
E

/3+Ô
O

0"
B C
Além disso,
(<p + P) + (u + 0) = ZABC + ZACB

(P + 0)+(a + q>) = ZABC + ZACB

Ora, como p + 0 = a + q>, segue que

(P + 0) - (a + <p) = ZABC + ZACB

(P - U) -r (p + 0) = ZABC + ZACB

2(P-0) = ZABC + ZACB =>


Capitulo 3 - Resoluções 247

P + 0 = ^(ZABC + ZACB)

Mais uma vez lembrando que p + 0 = rx + <p , temos então.


1
a + <p = P + 0 = —(ZABC + ZACB)

Por fim, olhando para o triângulo ABC temos que:


1
60° + ZABC + ZACB = 180° =. ZABC + ZACB = 120° => -(ZABC + ZACB) = 60°
2
O que revela que a + <p = p + 0 = 60° .

Pondo essa informação na figura, obtemos:

60°

D
ri
E

rj 60°
O

•XI
O'
B C

O que revela que o quadrilátero ADOE (hachurado) é inscritivel (note que


ZEOC = 60° => ZDOE = 120° , o que implica que no quadrilátero ADOE os
ângulos opostos dos vértices A e O são suplementares, o que revela que o
quadrilátero ADOE é inscritivel). Traçando o segmento AO. perceba que
ZOAE=ZODE = a (pois “enxergam o mesmo arco OE na circunferência
pontilhada). Analogamente, ZDAO = ZDEO = <p (pois “enxergam o mesmo
248 Tópicos de Matemática - Olimpíadas - IME - ITA

arco DO na circunferência pontilhada), o que revela que os triângulos ABO e


ACO são isósceles de bases. AB e AC. respectivamente e portanto, AO = OB
= OC. o que revela que o ponto O é um ponto no interior do triângulo ABC
que é equidistante dos seus vértices A, B e C e portanto O é o circuncentro do
triângulo ABC, pois o circuncentro de um triângulo é justamente o único ponto
do seu interior que é equidistante dos seus três vértices e essa distância é,
pois a medida do raio da circunferência circunscrita ao triângulo ABC, como
ilustra a figura abaixo:

47) Qual deverá ser o número inteiro positivo que somado a cada um dos
números 6. 8 e 14, obtêm-se as medidas dos lados de um triângulo em que o
ortocentro está no seu interior?

Resolução:
O ortocentro de um triângulo é um ponto do seu interior se, e somente se, o
triângulo for acutângulo. Por outro lado, um triângulo é acutângulo se, e
somente se, o seu maior lado ao quadrado é menor que a soma dos
quadrados dos outros dois lados Assim sendo n o número inteiro procurado
devemos ter um triângulo de lados 6 + n, 8 + n e 14 + n, que pelo que já
Capítulo 3 - Resoluções 249

dissemos será acutângulo (e portanto terá o ortocentro no seu interior) se. e


somente se,

(14 + n)2 (6 + n)2 + (8 + n)2

196 + 28n + n2 < 36 + 12n + n2 + 64 + 16n + n2


n2-96>0

Ora, como n > 0, para que n2 - 96 > 0, basta que n > 9. Por outro lado, para
que exista o triângulo de lados 6 + n, 8 + ne14 + né suficiente que (teorema
da desigualdade triangular):
14 + n<(6 + n) + (8 + n)=>n<2n=>2n-n>0=>n>0

Portanto qualquer inteiro n tal que n > 9, faz com que o triângulo de lados
6 + n, 8 + ne14 + n seja acutângulo e portanto tenha o seu ortocentro no seu
interior.

48) O triângulo PQS da figura abaixo é tal que PQ = PR = QS e ZQPR = 20°


Determine a medida do ângulo ZRQS = a.
250 Tópicos de Matemática - Olimpíadas - IME - ITA

Resolução:
Ora, como o triângulo PQR é isósceles de base QR, segue que
180°-20°
ZPRQ = ZPQR = = 80“. Por outro lado o triângulo PQS também é
2
isósceles, mas agora de base OS, o que revela que ZQSP = ZQPS = 20“.
Conforme ilustra a figura abaixo:

Por fim, no triângulo PQS, segue que:


(80“ + a) + 20“ + 20“ = 180“ => a = 60“

49) (OBM) O canto de um quadrado de cartolina foi cortado com uma tesoura. A
soma dos comprimentos dos catetos do triângulo recortado é igual ao
comprimento do lado do quadrado. Qual o valor da soma dos ângulos a e p
marcados na figura abaixo?
Capítulo 3 - Resoluções 251

Resolução:
Na figura abaixo sejam BE = y e BF = x. Ora, como, por hipótese, BF + BE é

igual a medida do lado do quadrado, segue que a medida dos lado do

quadrado é x + y. Ora, como BE = y e BC = x + y. segue que EC = x. Além

disso sejam DE = z e AE = w. Note que CF = DE = z (pois os triângulos CDE

e CBF são triângulos retângulos com catetos iguais a x e x + y, o que revela

que eles são congruentes e portanto tem hipotenusas de mesmo tamanho) e ,

por fim DF = AE = w (pois os triângulos ABE e DAF sâo triângulos retângulos

com catetos iguais a x e x + y, o que revela que eles são congruentes e

portanto tem hipotenusas de mesmo tamanho). Todas essas informações

estão ilustradas na figura abaixo:

D ■t + ?/ C

27° T
z '.3
E
i
w
.T. + ?/ TO
Z
I
i
i

a
A y F X B

Ora, como os triângulos CDE e CBF sâo congruentes, segue que


ZCDE = ZBCF = p. Além disso, como os triângulos ABE e DAF sâo

congruentes, segue que ZADF = ZBAE = a, conforme ilustra a figura a


seguir:
252 Tópicos de Matemática - Olimpíadas - IME — ITA

D x+y C

-\27° x
a\ z
•E

w i

x+y W
z ii y
i
a
A y F X B

Por fim, como o ângulo interno do vértice D é de 90° (ângulo interno de um


quadrado), segue que a + 27° + 0 = 90° => a + p = 63°.

50)
a) Mostre que as medianas de um triângulo sâo concorrentes.
b) Mostre que as alturas de um triângulo sâo concorrentes.
c) Mostre que as bissetrizes de um triângulo sâo concorrentes.
d) Mostre que as cevianas que ligam os vértices aos pontos em que a
circunferência inscrita intersecta cada um dos lados de um triângulo sâo
concorrentes (esse ponto de concorrência é chamado de ponto de
Gergonne).
e) Mostre que as mediatrizes dos lados de um triângulo sâo concorrentes.

Resolução:
Há diversas maneiras de demonstrar esses fatos. Vamos optar pelo uso do
teorema de Ceva, pela facilidade e elegância.
Capitulo 3 - Resoluções 253

TEOREMA (Giovanni Ceva) Num triângulo ABC, três cevianas AP, BN e CM


AM BP CN A
são concorrentes se. e somente se,
sao se, —
------------------
— = 1.
MB PC NA

a) De fato, suponha que o triângulo ABC tenha lados AB - 2c, BC = 2a e


CA = 2b. Se AE, BG e CD forem medianas, teremos AM = MB = c,
BP - PC = a e CN = NA = b, conforme ilustra a figura a seguir:

Nesse caso,
AM BP CN c a b
=1
MB PC NÃ c a b
e portanto, pelo teorema de Ceva, as medianas do triângulo ABC são
concorrentes num ponto G que chamamos de BARICENTRO.
254 Tópicos de Matemática - Olimpíadas - IME - ITA

b) Sejam AE, BG e CD as alturas do triângulo ABC, cujos lados medem


AB = c, BC = a e CA = b, conforme ilustra a figura abaixo:

C
b.cosC
a.cosC
\p
N

c.cosB

c.cosA

A b.cosA M a.cosB B

Como está ilustrado na figura acima.

AACM -> cos A = => AM = b ■ cos A


b

ABCM -> cosB = MB = a • cosB


a
BP
ÁABP -> cos B = — => BP = c • cos B
c
PC
AACP cos C = — => PC = b ■ cos C
b

ABCN -> cosC = — CN = acosC


a
NA
AABN -> cos A = — => NA = c • cos A
c
Assim,
AM BP CN b cosA c cosB acosC - 1
-------------------------------------
MB PC NA a cosB b cosC c cosA

e portanto, pelo teorema de Ceva, as alturas do triângulo são concorrentes,


num ponto H que chamamos de ORTOCENTRO.
Capítulo 3 - Resoluções 255

c) Dado um triângulo ABC tracemos, por exemplo um pedaço da bissetriz


AP do ângulo A conforme ilustra a figura abaixo:

Ora, como os pontos da bissetriz de um ângulo são equidistantes dos lados


do ângulo, segue que PN = PM = x. Assim nos triângulos retângulos APN e
APM, temos:
X X
AAFM -> tga =-----=> AM =------ = xcotga
AM tga

x x
aAFN tga =---- => NA =------= xeotgcx
NA tga °
Procedendo de modo análogo para os demais ângulos do triângulo ABC
temos:

X Y
ACNF -> tg<p = —— => CN =----- = xcotgq)
CN tg<p
256 Tópicos de Matemática - Olimpíadas - IME - ITA

x x
aCPF —> tg<p = — => PC = — = xcotg<p
PC tg<p

X X
ABPF -> tgp = — => BP = — = xcotgp
BP tgP
x x
AMBF tgB = — => MB = — = xcotgp
aP MB tgp

Assim.
AM BP CN xcotga x cotgp x cotgrp
------------------------------------------------------ _ T
.
MB PC NA x cotgp x cotgrp x cotga

portanto, pelo teorema de Ceva, as bissetrizes dos ângulos internos do


triângulo sâo concorrentes, num ponto I que chamamos de INCENTRO.

d) Consideremos um triângulo ABC e sua circunferência inscrita, sendo M, N e P


os pontos de tangência da circunferência inscrita ao triângulo ABC com os
seus lados. Sejam AB = c, BC = a e CA = b, AM = NA = x, BM = BP = y e
CN = Cp = z. Sendo s o semi-perimetro do triângulo ABC, conforme ilustra a
figura a seguir

x+ y = c
Note que ■ y + z = a . Adicionando membro a membro essas igualdades.
z+x=b
Capítulo 3 - Resoluções 257

segue que:
a +b+c
2(x + y + z) = a + b + c=>x + y + z = =>x + y + z = p
2
Portanto,
c + z = p=>z = p-c
x + y + z = p=> y + b = p => z = p-b
x + a = p => x = p-a

Assim temos a seguinte configuração:


C
p—c
p—c P

N p—b

p—a

A p—a p—b 8

Assim,
AM = p-a, MB = p-b,BP = p-b, PC = p-c,CN = p-c, NA = p-a

Portanto,
AM BP CN (p-a) (p-b) (p-c) 1
MB PC NA (p-b) (p-c) (p-a)

e portanto, pelo teorema de Ceva, as cevianas que ligam os vértices do


triângulo ABC aos ponto em que a circunferência inscrita no triângulo ABC
tangenciam os lados do triângulo são concorrentes num ponto F, chamado de
PONTO DE GERGONE do triângulo ABC.
258 Tópicos de Matemática - Olimpíadas - IME - ITA

OBSERVAÇÃO: É comum confundir o ponto de Gergonne (F) de um


triângulo com o seu incentro (I), mas em geral esses dois pontos são distintos,
pois os pontos em que a circunferência inscrita num triângulo tangenciam os
seus lados nâo coincidem com os pontos em que as respectivas bissetrizes
dos ângulos internos intersectam os lados, como ilustra a figura abaixo:

Na figura acima AP', BN' e CM' sâo bissetrizes dos ângulos internos do
triângulo ABC (o ponto I é o incentro do triângulo ABC). As cevianas AP, BN e
CM (linhas pontilhadas) ligam cada um dos vértices do triângulo ABC aos
ponto em que a circunferência inscrita no triângulo ABC tangencia cada um
dos lados do triângulo ABC (o ponto F é o ponto de Gergonne) É claro que
esses (e os demais centros clássicos de um triângulo podem coincidir; o que
ocorre no caso do triângulo equilâtero).

e) Nesse caso não vamos aplicar o teorema de Ceva, pois as mediatrizes dos
lados de um triângulo não necessariamente passam pelos vértice do
triângulo. Mas, mesmo assim a demonstração é bem simples Vejamos:
Capitulo 3 - Resoluções 259

Consideremos um triângulo ABC e as mediatrizes dos lados AB e BC.

conforme ilustra a figura abaixo:

Ora, como todo ponto da mediatriz de um segmento é equidistante dos

extremos esse segmento, olhando para o mediatriz do lado AB, segue que

AO = OB. Agora olhando para a mediatriz do lado BC, temos que OB = OC.

Assim, AO = OB = C, o que revela que o ponto O é equidistante dos vértices

A, B e C e portanto o ponto O é o centro a circunferência circunscrita ao

triângulo ABC. Chamamos o ponto O de CIRCUNCENTRO do triângulo ABC.


260 Tópicos de Matemática - Olimpíadas - IME - ITA

51) (OBM)No triângulo ABC isósceles abaixo, / é o encontro das bissetrizes e Hé o


encontro das alturas. Sabe-se que ZHAI = ZHBC = a. Determine o ângulo a

Resolução:
Ora. como o triângulo é isósceles segue que, ZCBM = ZABM= a e ZACB =
90° - a, com isso, Z.CAQ = a, pois AQ é uma altura. Como Al é bissetriz,
então ^CAI - ZIAB = 2a, como ilustra a figura a seguir:
B

'2^90°-o,

A M C

Finalmente no triângulo AMB, segue que:


a + a +2a+ a = 90° => a = 18°
Capitulo 3 - Resoluções 261

52) Mostre que o ponto simétrico do ortocentro de um triângulo acutângulo ABC


pertence a circunferência circunscrita ao triângulo ABC.

Resolução:
Consideremos um triângulo acutângulo ABC, seu ortocentro H e a sua
circunferência circunscrita, como ilustra a figura abaixo:

Agora prolongue, em linha reta, o segmento AP até intersectar a


circunferência no ponto D, conforme ilustra figura abaixo:
262 Tópicos de Matemática - Olimpíadas - IME - ITA

Vamos mostrar que o ponto D é o simétrico do ponto H (ortocentro do


tnângulo ABC) em relação ao lado BC. Feito isso, como o ponto D pertence â
circunferência circunscrita ao triângulo ABC o problema estará terminado.
Traçando o segmento CD vamos mostrar que o triângulo CDP é congruente
ao triângulo CPH. De fato, no triângulo CPH sendo ZCHP = a , segue que
ZAHM=ZCHP = a (pois são ângulos opostos pelo vértice). Assim, se
ZMAH = p, segue que a + p = 90° e portanto no triângulo CPH temos
ZHCP = 90°- a = P. Além disso, como o ângulo ZBAD = p é um ângulo
inscrito na circunferência segue que o arco BD mede 2p (a medida de um
arco, em graus, é o dobro da medida do ângulo inscrito correspondente). Por
fim, note que ZBCD = p, pois é um ângulo inscrito na circunferência que
"enxerga" o arco BD que mede 2p e como já dissemos a medida do arco, em
graus é o dobro da medida do ângulo inscrito correspondente), conforme
ilustra a figura abaixo:

Diante do exposto, perceba que os triângulos CHP e CDP são congruentes


pelo caso ALA (pois ZHCP = ZDCP = p e o segmento CP é um lado comum

aos dois triângulos, o que revela que PH = PD. Como ZCPD = ZCPH = 90°,
Capitulo 3 - Resoluções 263

poderemos concluir que o ponto D é o simétrico do ponto H (ortocentro do


triângulo ABC) em relação ao segmento BC, como queríamos demonstrar.
OBSERVAÇÃO: Apesar do enunciado referir-se a um triângulo acutângulo,
essa propriedade é verdadeira num triângulo qualquer. A demonstração
desse fato pode ser vista no Apêndice 3 no final desse livro.

53) (Teorema de Menelaus) Se uma reta (r) intersecta as retas suportes dos
lados BC, CA e AB de um triângulo ABC nos pontos L, M e N,
respectivamente, mostre que
BL CM AN
LC'MA NB “

E reciprocamente, ou seja, se L, M e N sâo pontos sobre as retas suportes

dos lados BC, CA e AB do triângulo ABC tais que = 1 , então L,

M e N são colineares.
264 Tópicos de Matemática - Olimpíadas - 1ME - ITA

Resolução:
(=>) Suponhamos que uma reta (r) intersecta as retas suportes dos lados BC,

CA e AB de um triângulo ABC nos pontos L, M e N, conforme ilustra a figura a


seguir:

Note que na figura acima os pares de triângulos (BQL, CRL) , (PAN, QBN) e
(PAM, RCM) são semelhantes por apresentarem ângulos dois a dois
congruentes. Assim podemos estabelecer as seguintes proporções'

ABQL~a.^r,. BL
CRL=.— BQ
LC CR
AN AP
APAN - AQBN =. —
NB BQ
CR
APAM ~ ARCM => —
MA AP
Assim,
BL CM AN BQ CR AP 1
LC MA NB CRAPBQ’
O que demonstra o resultado desejado.
(<=) Suponhamos agora que L, M e N sejam os pontos sobre as retas

suportes dos lados BC, CA e AB do triângulo ABC tais que — ■ ■ — = 1.


LC MA NB
Capitulo 3 - Resoluções 265

Vamos mostrar que os pontos L. M e N são colineares. De fato, suponha, por

absurdo, que — — = 1, mas que os pontos L, M e N não são


LC MA NB
colineares e seja N' o ponto em que a reta LM intersecta o lado AB, conforme
ilustra a figura a seguir

Ora, como os pontos L, M e N' estão alinhados, segue, pelo que foi provado
na primeira parte da demonstração, que:
BL CM AN' _ 1
LC MA N'B “

„ BL CM AN „
Ora, como------------------ = 1, segue que:
LC MA NB b

BL CM AN BL CM AN' AN AN'
=----- => N = N
LC MA NB LC MA N'B NB N'B

e portanto como N = N', segue que os pontos L. M e N (que é igual a N') são
colineares, como queríamos demonstrar.
266 Tópicos de Matemática - Olimpíadas - IME - ITA

54) (Teorema de Pascal) Seja ABCDEF um hexágono inscrito em um circulo e


sejam H, K e I os pontos de interseção de AB e DE, BC e EF e AF e CD,
respectivamente. Então, H, K e I são colineares.

Resolução:
Considere o triângulo XYZ que é formado pela interseção das retas AB, CD e
EF, conforme ilustra a figura abaixo:
Capitulo 3 - Resoluções 267

Note que a reta AF intersecta as retas suportes dos lados do triângulo XYZ
nos pontos A, F e I, conforme ilustra a figura abaixo:

Aplicando o teorema de Menelaus ao triângulo XYZ (olhando para reta AF).


segue que:
ZF Yl XA 1
FY IX ÃZ ” (1)

Analogamente a reta DE intersecta as retas suportes dos lados do triângulo


XYZ nos pontos D. E e H. conforme ilustra a figura abaixo:
268 Tópicos de Matemática - Olimpíadas - IME - ITA

Aplicando o teorema de Menelaus ao triângulo XYZ (olhando para reta DE),


segue que:
YD XH ZE
=1 (2)
DX HZ EY

Por fim a reta BC intersecta as retas suportes dos lados do triângulo XYZ nos
pontos B, C e K, conforme ilustra a figura abaixo:

Aplicando o teorema de Menelaus ao triângulo XYZ (olhando para a reta BC),


segue que:
XB ZK YC
(3)
BZ KY CX ~

Multiplicando membro a membro as igualdades (1), (2) e (3), segue que:


ZF Yl XA YD XH ZE XB ZK YC 1
FY IX AZ DX HZ EY BZ KY CX ~

Que podemos apenas reescrever numa outra ordem (conveniente), a saber:


ZF ZE YD YC XB XA Yl XH ZK 1
ZA ZB YE YF XC XD IX HZ KY ”
Capitulo 3 - Resoluções 269

Mas ocorre que ZFZE = ZAZB, YD YC = YE-YF e XB XA = XC XD


(potência dos pontos X, Y e Z em relação á circunferência), o que revela que
ZF ZE YD YC XB XA , . .
---------- = 1,------------= 1 e------------ = 1. Assim,
ZA ZB------ YE YF---------- XC XD
ZF ZE YD YC XB XA Yl XH ZK _1=) Yl XH ZK _ 1
ZA ZB YE YF XC XD IX HZ KY ” ° IX HZ KY

Ora, como H, I e K são pontos que pertencem as retas suportes dos lados do
Yl XH 7K
triângulo XYZ tais que--------------- = 1 . segue pelo teorema de Menelaus (a
IX HZ KY
reciproca) que esses pontos são colineares, como queríamos demonstrar

55) (Reta de Newton) Seja ABCD um quadrilátero tal que BA e CD intersectam-


se em E, AD e BC intersectam-se em F e sejam N, Le Mos pontos médios
de EF, AC e BD, respectivamente.

Prove que N, L e M são colineares. (A reta que "passa” por esses três pontos
é chamada de Reta de Newton).
270 Tópicos de Matemática - Olimpíadas - IME - ITA

Resolução:
Sejam P, QeRos pontos médios dos lados do triângulo EBC.

Assim, pelo teorema da base média de um triângulo, aplicado aos triângulos


EA AB
LR//AB c
ACE e ABC segue que QL//AE,QL = —, LR.'.'.".R e LR -
= — . Assim,. os

pontos Q, L e R são colineares e além disso,


EA
QL QL EA
(1)
LR AB LR AB
2
Analogamente, aplicando o teorema da base média aplicado aos triângulos
DE CD
BED e BCD, podemos concluir PM//DE, PM = —, RM//CD e MR = —
—- .

Assim, os pontos P, M e R são colineares e além disso,


CD
RM 2 RM CD
(2)
MP " DE MP DE
2
mais uma vez aplicando o teorema da base média de um triângulo aos
___ ___ BF FC
triângulos EPQ e EPN, segue que PN//BF, PN = —2 •, NQ//FC
-e- NQ = —
2
Capítulo 3 - Resoluções 271

Assim, os pontos P, QeN são colineares e além disso,


BF
PN
PNPN BF
(3)
" * ” FC
NQ NQ FC
2
Multiplicando, membro a membro, (1), (2) e (3), segue que:
QL RM PN EA CD BF
LR MP NQ AB DE FC
Por outro lado, aplicando o teorema de Menelaus no triângulo EBC, olhando
para a reta que "passa" pelos pontos A, D e F. segue que
EA BF CD
AB FC DE “

Assim,
QL RM PN EA CD BF EA BF CD
—------- - 1
LR MP NQ AB DE FC AB FC DE

Ora, como L, M e N são pontos das retas suportes dos lados do triãngu.

PQR tais que = 1, seque do recíproco do teorema de Menelaus


LR MP NQ
que os pontos L. M e N são colineares, como ilustra a figura abaixo:
272 Tópicos de Matemática - Olimpíadas - IME - ITA

56) (Reta de Simson-Wallace) Mostre que se perpendiculares são traçadas a


partir de um ponto Q sobre a circunferência circunscrita de triângulo a seus
lados, suas interseções com os lados do triângulo N. L e M são colineares e
pertencem à uma mesma reta que chamamos de Reta de Simson-Wallace.
conforme ilustra a figura abaixo:

Resolução:
De fato, perceba que o quadrilátero CNLQ é inscritivel, pois o segmento CQ
“olha" para dois ângulos retos nos vértices N e L, conforme ilustra a figura
abaixo.
Capítulo 3 - Resoluções 273

perceba que ZNLC= ZNQC = a, pois esses ângulos “enxergam” o mesmo


arco CN na circunferência pontilhada.
Analogamente, perceba que o quadrilátero AMQL é inscritivel. pois os
ângulos ZAMQ e ZALQ medem 90° cada um e são ângulos opostos do
quadrilátero AMQL, conforme ilustra a figura abaixo:

perceba que ZALM = ZAQM = <p , pois esses ângulos "enxergam” o mesmo

arco AM na circunferência pontilhada menor.


Sendo ZABC = (5 e ZAQC = 0 pelo fato do quadrilátero ABCQ ser inscritivel.

segue que (5 + 0 +a = 180°. Além disso, ZBMQ = ZBNQ = 90°. o que revela
que a soma das medidas dos outros dois ângulos do quadrilátero BMNQ
somam 180° (pois a soma dos seus quatro ângulos mede 360°). Assim,
p + 9 + <p = 180°, o que revela que

P + O + <p = P + 0 + a=3-a = <p

e portanto os ângulos ZALM e ZCLN são opostos pelo vértice, o que nos
permite concluir que NLM é uma linha reta e que, portanto, os pontos N. L e M
são colineares, como queríamos demonstrar.
274 Tópicos de Matemática - Olimpíadas - IME - ITA

57) (A reta de Euler) Mostre que num triângulo qualquer ABC, o ortocentro(H), o
circuncentro(O) e o baricento(G) são colineares (a reta que os contêm é
chamada de Reta de Euler). Além disso mostre que HG = 2- GO.

Resolução:
Considere um triângulo ABC, com o seu ortocentro(H), circuncentro(O), M
ponto médio do lado BC, N ponto médio do lado AC e G' como sendo a
interseção do segmento HO com a mediana AM. conforme ilustra a figura
abaixo:

Vamos mostrar que G' é o baricentro do triângulo ABC. De fato, como N e M


são pontos médios dos lados AC e BC, respectivamente, segue que MN//AB e
MN = ^AB (teorema da base média de um triângulo). Ora, como MN//AB e

AD//OM, segue que ZOMN = ZBAH e ZONM = ZABH, o que revela que os
triângulos ABH e NOM sâo semelhantes (pois têm os ângulos dois a dois
iguais) Dessa semelhança nasce a seguinte proporção:
OM MN
AH “ AB
Capitulo 3- Resoluções 275

Mas. como jã dissemos, MN = ^AB , segue que.

OM MN 2AB 1
AH “ ÃB ” AB * 2
Por outro lado, como OM//AD, segue que os triângulos OMG’ e AHG' são
semelhantes (pois apresentam ângulos dois a dois congruentes). Dessa
semelhança nasce a seguinte proporção:
OG' MG' OM
HG7 “ ÃG7 “ AH

.. OM 1 A .
Mas ja provamos que — = — . Assim,

OG' MG' OM 1
HG' AG' AH ” 2

Ou seja, — = -aAG' = 2-G'l.' Ora, como AM é uma mediana do


AG' 2
triângulo ABC e o ponto G' a divide na razão de 2:1, segue que o ponto G' é,
de fato, o baricentro G do triângulo ABC, ou seja, G-G. Com isso provamos
que os pontos H, G e O são colineares e
OG' MG' OM 1 OG MG OM 2 — = 1=>HG = 2OG
HG' “ÃG^ÃH ~ 2 HG-ÃG-ÃH 2 HG 2
Como queríamos demonstrar.
OBSERVAÇÃO: O segmento HO, que liga o ortocentro(H) ao circuncentro(O)
do triângulo ABC é chamado de Mediana de Euler do triângulo ABC.

58) (O circulo de nove pontos) Mostre que em todo triângulo ABC, os pontos
médios dos segmentos que unem cada um dos vértices ao ortocentro. os
pontos médios dos lados do triângulo ABC e os pés das alturas do mesmo
triângulo ABC pertencem a uma mesma circunferência, que chamamos de
circulo de nove pontos. Além disso, mostre que se a circunferência
circunscrita ao triângulo ABC tem raio R, então o circulo de nove pontos do
p
referido triângulo tem raio —.
276 Tópicos de Matemática - Olimpíadas - IME - ITA

Resolução:
No triângulo ABC da figura abaixo o ponto O é o seu circuncentro, H seu
ortocentro, G o seu baricentro e M o ponto médio do lado BC.

Sabemos que HG = 2 ■ GO, como os triângulos AHG e MOG são semelhantes


(pois possuem ângulos congruentes dois a dois), segue que
HA HG HA 2 GO
-----=----- =>----- =--------- => HA = 2 ■ OM
OM GO OM GO
definindo A' como sendo o ponto médio do segmento HA, segue que
HA 2OM
= OM
A A =---- =---------
2 2
Assim, o quadrilátero A'AOM é um paralelogramo (pois os segmentos AA e
OM sâo paralelos e de mesmo tamanho), conforme ilustra a figura abaixo:
Capitulo 3 - Resoluções 277

Ora, como o quadrilátero A'AOM é um paralelogramo, segue que A M = AO.


Por outro lado, perceba que AO = R. onde R é a medida do raio da
circunferência circunscrita ao triângulo ABC. assim.
A'M = AO = R
Analogamente.
AH 2-OM
A H =---- =--------- = OM
2 2
Além disso, o quadrilátero HA’OM também é um paralelogramo (pois os
segmentos A’H e OM são paralelos e de mesmo tamanho), conforme ilustra a
figura abaixo:

Ora, como o quadrilátero A’HMO é um paralelogramo as suas diagonais HO e


A'M intersectam-se ao meio no ponto Q. conforme ilustra a figura abaixo.
278 Tópicos de Matemática - Olimpiadas - IME - ITA

Ora, como A'M = R, segue que

QA’ = QM = — =>QA' = QM = -
2 2
Por fim. como P é o pé da altura relativa ao lado BC do triângulo ABC, segue
que o triângulo A'PM é retângulo. Ora, como num triângulo retângulo a
medida da mediana relativa a hipotenusa mede a metade da hipotenusa,
segue que
D
QA' = QM = QP = —
2
p
Diante do exposto, segue que a circunferência de centro Q e raio — “passa"

pelos pontos A' (ponto médio do segmento HA), P (pé da altura relativa ao
lado BC) e M(ponto médio da lado BC).
Repetindo a mesma construção acima exposta, podemos demonstrar que o
mesmo ocorre para os lados CA e AB do triângulo ABC. Assim, dado um
triângulo ABC, existe uma circunferência que “passa" pelos pontos médios
dos segmentos que ligam o ortocentro a cada um dos vértices desse
triângulo, pelos pés das alturas e pelos pontos médios dos lados do triângulo
ABC. Essa circunferência é conhecida como a circunferência de nove
pontos. Como em inglês a circunferência é comumente chamado de “circulo",
(o nosso circulo, em inglês, costuma-se chamar de DISCO) o que justifica o
termo consagrado "CÍRCULO DE NOVE PONTOS”, CÍRCULO DE EULER
ou ainda CÍRCULO DE FEUERBACH. Conforme ilustra figura abaixo:
Capitulo 3 - Resoluções 279

OBSERVAÇÃO: Pelo que foi exposto acima podemos perceber que o círculo
de nove pontos é o círculo circunscrito ao triângulo órtico (triângulo cujos
vértices sâo os pés das alturas do triângulo ABC) do triângulo ABC e também
é o circulo circunscrito ao triângulo mediai (triângulo cujos vértices são os
pontos médios dos lados do triângulo ABC) do triângulo ABC, conforme ilustra
a figura abaixo:

59) Demonstre o Teorema de Ceva: Num triângulo ABC, três cevianas AP. BN e
. AM BP CN d
CM sao concorrentes se, e somente se,----------------- = 1
MB PC NA
280 Tópicos de Matemática - Olimpíadas - IME - ITA

Resolução:
Inicialmente vamos supor que as cevianas AP, BN e CM são concorrentes.
Lembrando que a razão entre as medidas das áreas de dois triângulos que
possuem a mesma altura é a razão entre as medidas das suas bases, segue que
AM (ACM) (AFM) (ACM)-(AFM) (ACF)
MB " (MCB) “ (MFB) " (MBC)- (MFB) “ (BCF)
(D

BP (ABP) (BFP) (ABP)-(BFP) (ABF)


PC ~ (ACP) " (CFP) = (ACP)-(CFP) “ (ACF) (2)

CN (CBN) (CFN) (CBN)-(CFN) (BCF)


NA “ (ABN) = (AFN) " (ABN)-(AFN) “ (ABF) (3)

Multiplicando membro a membro as três igualdades acima, segue que:


AM BP CN (ACF) (ABF) (BCF) 1
MB PC NA ~ (BCF)' (ACF) ’ (ABF)

Reciprocamente, ou seja, vamos supor que M, N e P são pontos sobre os


lados AB. AC e BC do triângulo ABC, respectivamente, tais que
AM BP CN , .
-----------------= 1. Vamos mostrar que as cevianas CM, BN e AP sao
MB PC NA
concorrentes. De fato, seja H o ponto de concorrência das cevianas AP e BN,
conforme ilustra a figura abaixo:

Seja CM' uma ceviana que sai de C, "passa" por H e que intersecta o lado AB
no ponto M'. Vamos mostrar que M = M’ e portanto as três cevianas AP, BN e
CM são. de fato, concorrentes e o ponto H é na verdade o ponto F da figura
do enunciado.
Capitulo 3 - Resoluções 281

De fato, sendo CM' a ceviana que passa por H e que atinge o lado AB no
ponto M', segue que as cevianas AP. BN e CM’ são concorrentes e então pelo
AM' BP CN ,
que ja demonstramos na primeira parte, segue que------------------- = 1. Mas
M'B PC NA
. . .. , AM BP CN , ,
por hipótese, estamos supondo que------------------ = 1. Assim,
MB PC NA
AM BP CN AM' BP CN AM AM' ,,, „
—- = => M = M
MB PC NA M’B PC NA MB M'B
r i AM BP CN
O que revela que se----------------- = 1, então as cevianas AP, BN e CM sao,
MB PC NA
de fato, concorrentes, como queríamos demonstrar.

60) (Teorema da base média de um triângulo) Dado um triângulo qualquer,


demonstre que o segmento que une os pontos médios de dois lados é
paralelo ao terceiro lado e tem um comprimento igual a metade do
comprimento desse terceiro lado.

Resolução:
Seja ABC um triângulo qualquer tal que AB = 2c, BC = 2a e CA = 2b. Além
disso sejam M. N e P os pontos médios dos lados AB. BC e CA,
respectivamente. Construa o segmento NP de comprimento x e o prolongue
até o ponto Q de modo que PQ = x, conforme ilustra a figura abaixo:
282 Tópicos de Matemática - Olimpíadas - IME - ITA

Note que ZNPC = ZBPQ (pois são opostos pelo vértice). Como PN = PQ =
x, PC = PB = a, segue que os triângulos NCP e PQB sâo congruentes pelo
caso LAL, o que revela que BQ=CN=b e ZNCP = zQBP = p e portanto
BQ//AC (os ângulos ZNCP e ZQBP são alternos internos), como ilustra a
figura a seguir:

O que revela que o quadrilátero ABQN é um paralelogramo Assim, NQ//AB e


2x = 2c, o que revela que x = c, o que nos permite concluir que NP//AB e
NP = AB/2, como queríamos demonstrar.

61) Mostre que o baricentro de um triângulo divide cada mediana em duas partes
tal que uma é o dobro da outra.

Resolução:
Considere um triângulo ABC e as suas medianas AP. BN e CM, conforme
ilustra a figura a seguir:
Capítulo 3 - Resoluções 283

Sejam R e S os pontos médios dos segmentos AG e BG, respectivamente.


Note que NP = c e RS = c, pois são bases médias dos triângulos ACB e AGB,
respectivamente. Além disso NP//AB e RS//AB (a base média de um triângulo
é paralela â base do triângulo). Assim o quadrilátero NPSR é um
paralelogramo. Assim, GN = GS e GP = GR, pois as diagonais de um
paralelogramo são cortam-se ao meio. Ora, como GS = SB, pois o ponto S é,
por construção o ponto médio do segmento BG, segue que GN = GS = BS =
x. Analogamente, GR = AR, pois por construção o ponto R é o ponto médio
do segmento AG. Assim, GP = GR = AR = y, conforme ilustra a figura abaixo:
284 Tópicos de Matemática - Olimpíadas - IME - ITA

Diante do exposto, segue que:


BG = 2x, GN = x =• BG = 2 GN

AG = 2y, GP = y => AG = 2 GP

De modo análogo podemos concluir que CG = 2-GM .

OBSERVAÇÃO: Essa propriedade do baricentro as vezes é apresentada de


outra maneira, a saber:

AG = — AP e GP = — AP
3 3

aq _ 2 /\p e
Pois AG _ 2y =>AG GP _ y gp 1 .p
P0,S1 AP'3y^A = -AP 3AP e6 AP’3y^G 3AP
3
E de modo completamente análogo, podemos provar que:
2 1
BG = —BN e GN = —BN
3 3
2 1
CG = — AM e GM = —CM
3 3

62) Mostre que o ortocentro de um triângulo acutângulo ABC é o circuncentro do


triângulo MNP que é construído traçando-se por cada um dos vértices do
triângulo ABC uma paralela ao lado oposto, conforme ilustra a figura abaixo:
i

c! P

/A
P B ''

M
Capitulo 3 - Resoluções 285

Resolução:
Sejam AB = c, BC = a e CA = b. Ora, como NP//AB e MB//AC, segue que o
quadrilátero ABPC é um paralelogramo e portanto, CP = AB = c e BP = AC =
b. De modo completamente análogo, ABCN também é um paralelogramo e
então CN = AB = c e NA = BC = a. Por fim, perceba que o quadrilátero AMBC
também é um paralelogramo, pois MN//BC e MP//AC, o que revela que MB =
AC = b e AM = BC = a, Além disso dos paralelismos também segue as retas
pontilhadas sâo perpendiculares aos lados do triângulo MNP, como
ilustramos na figura a seguir:

I
i
C
c c! P
N

a
b
a

/A Pc B

a ! b

Diante do exposto, segue que os pontos A.B e C são os pontos médios dos

lados do triângulo MNP, e portanto as retas pontilhadas são as mediatrizes

dos lados do triângulo MNP, sendo , pois o ponto H (ortocentro do triângulo

ABC), o circuncentro do triângulo MNP, isto é o centro a circunferência

circunscrita ao triângulo MNP, conforme ilustra a figura a seguir:


286 Tópicos de Matemática - Olimpíadas - IME - ITA

63) Dado um triângulo ABC, chamamos de triângulo órtico o triângulo cujos


vértices sâo os pés das alturas do triângulo ABC. Mostre que o ortocentro do
triângulo ABC é o incentro do seu triângulo órtico.

Resolução:
De fato, considere o triângulo ABC da figura abaixo, com suas três alturas AE, BF
e CD, seu ortocentro H e o triângulo órtico (ou pedal) DEF do triângulo ABC
Capitulo 3 - Resoluções 287

Como os ângulos ZAFH= ZADH= 90° , segue que o quadrilátero ADHF é


inscritivel (pois tem dois ângulos opostos suplementares), como revela a
figura abaixo:

Note que ZFAH= ZFDH = a , pois esses dois ângulos sâo ângulos inscritos
na circunferência circunscrita ao quadrilátero ADHF e que "enxergam" o
mesmo arco FH.
No triângulo ACD, perceba que a = 90°-C. No triângulo BCF temos que

ZCBF = 90° -C = a , conforme ilustra a figura abaixo:

Ct

A' - ------ D B
288 Tópicos de Matemática - Olimpíadas - IME - ITA

Agora perceba também que o quadrilátero BEHD também é inscritivel pois


tem dois ângulos opostos suplementares, visto que ZBEH= ZHDB = 90°,
como ilustra a figura abaixo:
C

E.

i
i
o

A"-------- ZB

Portanto ZHDE = ZHBE = a, pois esses ângulos são inscritos na


circunferência circunscrita ao quadrilátero BDHE e "exergam" o mesmo arco
HE, o que releva que DH está contido na bissetriz do ângulo interno do vértice
D do triângulo DEF.
De modo completamente análogo podemos demonstrar que o mesmo fato
ocorre nos vértices E e F do triângulo DEF, ou seja, EH e FH estão contidos
nas bissetrizes dos ângulos interno dos vértices E e F do triângulo DEF. Ora,
como as restas suportes do segmentos DH, RH e FH intersectam-se no
ponto H e as bissetrizes dos ângulos internos do triângulo DEF estão sobre
essas retas , segue que o ponto H é o ponto de encontro das bissetrizes dos
ângulos internos do triângulo DEF, ou seja, o ponto H é, pois, o incentro
(centro a circunferência inscrita) do triângulo DEF, como queríamos
demonstrar. Veja a figura abaixo, onde mostramos a circunferência inscrita no
triângulo DEF:
Capitulo 3 - Resoluções 289

64) Os lados AB, BC e CA do triângulo ABC da figura abaixo estão divididos em


três partes iguais cada um pelos pontos D, E, F, G, H e I. Sabendo-se que a
soma das medidas dos comprimentos das cevianas Al, AH, BF, BG, CD e CE
é 200cm, determine o perímetro do hexágono hachurado na figura abaixo.
290 Tópicos de Matemática - Olimpíadas - IME - ITA

Resolução:
Sejam 3a, 3b e 3c as medidas dos lados do triângulo ABC e além disso
nomeie os vértices do hexágono hachurado conforme a figura abaixo:

Seja ML = x. Aplicando o teorema de Menelaus ao triângulo BCE (com


transversal Al), segue que:
BI CL EA a CL 2c 1 _ CL _ 3
IC LE AB ~ =>2a LE '3c“ = LE “

mas ocorre que CL = CE - LE . Assim,


CE
Çt=3=> CE-LE = 3,=>------- CE LE
1 = 3 =>----- = 4 => — (1)
LE LE LE LE CE 4

Aplicando o teorema de Menelaus ao triângulo ACE (com transversal BG),


segue que:
AB EM ÇG 1=>3c EM b j EM 2 MC 3
BEMCGA” ^cMCa" MC “ 3 EM 2

mas ocorre que MC = CE - EM . Assim,


MC 3 CE-EM 3 CE d 3 CE 5 EM 2
-------- 1 = — => (2)
ÊM 2 EM 2 EM 2 EM 2 CE 5
Capitulo 3 - Resoluções 291

Por fim fazendo a diferença entre as igualdades (1) e (2), segue que
EM
““““ —
LE— “2— 1 EM-LE ~
3 “ P
Xx — 3 P X —~
3 X-P t
CE CE 5 4 CE 20 CÊ 20
CE 20

Procedendo de modo completamente análogo para os demais lados do


hexágono podemos mostrar que o comprimento de da um dos seus lados é
3
— da ceviana que o contém. Assim o perímetro do hexágono hachurado é

3
— da soma dos comprimentos das cevianas. Ora, como por hipótese, a

soma dos comprimentos das cevianas Al, AH, BF, BG, CD e CE é 200, segue
que o perímetro do hexágono hachurado é:
3
(2p)
V
.
^Hexágono
=—
20
-200 = 30 cm.

65) Um ponto A qualquer é considerado sobre o lado OX do ângulo ZXOY dl

figura.
Traçamos então:
i) AB ± OY
ü) AQ//OY
iii) tal que PQ = 2.0A.

X,

P
ZL
O B Y

Se ZPOB = 26°, determine a medida do ângulo ZXOY .


292 Tópicos de Matemática - Olimpíadas - IME - ITA

Resolução:
Sejam AO = x e M o ponto médio do segmento PQ. Ora, como PQ = 2.OA,
segue que PQ = 2x, o que revela que PM = MQ = x. Ora, como o triângulo
PAQ é retângulo (pois sendo AQ//OY o ângulo do vértice A é reto pois é
alterno interno com o ângulo do vértice B que é reto), segue que AM = MP =
MQ = x, pois como sabemos num triângulo retângulo o a mediana relativa a
hipotenusa tem a mesma medida da metade da hipotenusa. Por fim como
AQ//OU, segue que os ângulos ZAQM e ZMOY sâo alternos internos e
portanto têm a mesma medida. Assim, ZAQM = ZMOY = 26°. Como o
tnângulo AQM é isósceles, pois AM = MQ = x, segue que
ZMAQ = ZAQM = 26°. Por fim, o ângulo ZAMO é o ângulo externo do
vértice M do triângulo AQM. Assim, ZAMO = 26°+ 26° = 52° Ora. como o
triângulo OAM também é isósceles, pois AO = AM = x, segue que
ZAOP = ZAMO = 52°. conforme ilustra a figura abaixo:

X,

x
52’
x

x
5T . ' P

O B Y

Diante do exposto, segue que ZXOT 52°+ 26° = 78° .

66) (ITA) Se x, y e z são os ângulos internos de um triângulo ABC


seny + senz
e senx =-- ----- , prove que o triângulo ABC é retângulo.
cosy + cosz
Capitulo 3 - Resoluções 293

Resolução:
Note que
y + z> f y-z'}
2sen --- cos ---
seny + senz . 2 ) l 2 )
senx = (D
cosy + cosz f y + z> (
2008^^^-^—J cos|^ y-z
2 .
Ora. como x, y e z são as medidas dos ângulos internos do triângulo ABC,
segue que

x + y + z = 180° => y + z = 180° - x => = 90° “ ~

sen^+ = sen ^90° x

Assim, = 90° - — =>


,90°-â = COS —
2
2 2 = cos ( 90° X
COS
90°-â = sen —
2

Substituindo essas informações em (1), segue que

sen x
cos2 xx
senx =----- =-- — => Senx • sen — = cos — (2)
sen-x 2 2
2
x x
Mas ocorre que senx = 2sen-cos- . Assim, a partir de (2), segue que:
2 2
X X „ Xx x
X Xx xX
senx sen— = cos— =>2sen —cos —sen— = cos— =>
2 2 ~2 2 2 2
X o
cos— = 0
cos |^2sen2 = 2
0=>
X
sen — = ± —
2 2
Lembrando que x é tal que 0 < x <180°, pois é a medida de um dos ângulos
internos de um triângulo, segue que
X X
COS— = 0 =■ — = 90° => x = 180°
2 2
294 Tópicos de Matemática - Olimpíadas - IME - ITA

que não convém ao problema, pois como já dissemos 0 < x < 180“.
A outra opção é
x y y
sen — — => - = 45° ou - = 135“ => x = 90“ ou X = 270“
2 2 2 2

Ora, como 0 < x < 180“, segue que x = 90°.

x 72 os valores de x não cumprem a condição


Por fim, se sen- = -
. ........... ' ' ”"2 2 ’

0<x<180“, pois o seno não é negativo para valores de x tais que


0<x<180“.

Diante do exposto, a única possibilidade é x = 90°, o que revela que o


triângulo ABC é um triângulo retângulo, como queríamos demonstrar.

- 7) Na figura abaixo, I é o bancentro do triângulo ABC Calcule a distância d seu


bancento G à reta r, sabendo que BB' = 4cm, AA' = 11cm e CC = 3cm.

11
B'
n
A
ri
G'
n.
c
Capitulo 3 - Resoluções 295

Resolução:
Sejam M. N e P os pontos médios dos lados do triângulo ABC. Trace os
segmentos NN7/BB’, GQ//B'C’ e CT//B'C, conforme ilustra a figura a seguir:
A

N
p
G.

B ' 11

i tr ts" C

4 ! 3 i

h. h-tl
8' N' A'
__□_ G'
XI
c
Note que x = GG' = QN' = NN'- NQ .
Perceba que o segmento NN' é a base média do trapézio B’BAA' Assim.
BB'+AA' 4 + 11 15
NN' =
2 2 “ 2

Ora, como G é o baricentro do triângulo ABC. segue que GN = ^CN Como

os triângulos NGQ e NCT são semelhantesjpois apresentam ângulos dois a


1
dois congruentes) e GN = -CN, segue que a razão de semelhança entre

eles é e portanto. NQ = ^NT. Mas por outro lado,

NT = NN'-TN'= NN’-CC = — - 3 =—
2 2
1 19 3
Assim, NQ = —NT = —• — —. Ora, como x = GG'= QN'= NN'-NQ . segue
3 3 2
que:

X = GG' = QN' = NN'- NQ = — — = 6 cm.


2 2
296 Tópicos de Matemática - Olimpíadas - IME - ITA

68) Seja ABC um triângulo retângulo em A, CX a bissetriz do ângulo, sendo X um


ponto do lado AB. Se CX = 4cm e BC = 24cm, quanto mede AC?

Resolução:
Seja AC = b, conforme ilustra a figura abaixo:

No triângulo retângulo AXC, temos cosa = - e no triângulo retângulo ABC,


4
cos 2ct = ^. Mas ocorre que

b' = 3
= 2^ -1=5 3b2-b-24 = 0=>
cos2u = 2cos2a-1 =• —
24
r-5
Como AC = b > 0. segue que AC = b = 3.
Capitulo 3 - Resoluções 297

69) (USAMO) ABCD é um quadrado e M e N são os pontos médios de BC e CD


respectivamente. Determine sen0.

A D

Resolução:
Seja 2a a medida do lado do quadrado ABCD. Como M e N são os ponto,
médios dos lados BC e CD, respectivamente, segue que BM = MC = CN = ND
= a, o que revela que os triângulos ADN e ABM são congruentes pois são
triângulos retângulos de catetos congruentes, o que revela que
ZBAM = ZDAN = a , como ilustra a figura abaixo:
298 Tópicos de Matemática - Olimpíadas - IME - ITA

Por Pitágoras aplicado no triângulo ADN, segue que

AN2 = (2a)2 + a2 => AN = aVã. Por outro lado, olhando para o vértice A do

quadrado ABCD. segue que


2« + ü = 90° => 0 = 90° - 2a => senO = sen (90° - 2a) => senO = sen2a

Mas ocorre que sen2a = 2sentxcosa e no triângulo retângulo ADN, temos:


2a 2 a 1
cosa = —e sena = —7= = -7=
av5 v5 aV5 <5
2 _l__ 4
Por fim, senO = sen2a = 2 ■ •
■J5 \Í5 ~ 5 '

70) Na figura abaixo ABC é um triângulo acutãngulo e H é o seu ortocentro.

Mostre que a medida do raio da circunferência circunscrita ao triângulo BHC


é igual a medida do raio da circunferência circunscrita ao triângulo ABC.

Resolução:
De fato, sabemos que o ponto H' simétrico do ortocentro H do triângulo ABC
em relação ao lado BC pertence a circunferência circunscrita ao triângulo
ABC, conforme ilustra a figura a seguir:
Capitulo 3 - Resoluções 299

Traçando os segmentos BH' e CH', o triângulo BH’C é congruente ao


triângulo BHC (caso LLL, pois BC é lado comum aos dois e facilmente
podemos verificar que CH = CH' e BH = BH'). Por fim perceba que a
circunferência circunscrita ao triângulo BH'C é a mesma que a circunferência
circunscrita ao triângulo ABC. Ora, como os triângulos BH'C e BHC são
congruentes segue que as suas circunferências circunscritas também sâo
congruentes e portanto tem o mesmo raio, que é igual ao raio da
circunferência circunscrita ao triângulo ABC, pois como já dissemos é a
mesma que a circunferência circunscrita ao triângulo BH’C, conforme ilustra a
figura abaixo:
300 Tópicos de Matemática - Olimpíadas - IME - ITA

71) Sendo ma,mb e mc as medidas das medianas relativas aos lados BC - a,

AC = b e AB = c de um triângulo ABC. mostre que:


ml+ml + ml = ^(a2+b2+c2)

Resolução:
Sabemos que:
ma=^2(b2 + c2)-a2

mb = ^2(a2 +c2)-b2

mc =^2(a2+b2)-c2

Assim.

ma = J ^2 (b2 + c2) - a2 => ml = -1 [2 (b2 + c2) - a2 ]

n>b = 7^2(a2 + c2) - b2 =■ mb = ^[2 (a2 + c2) - b2]

mc = \\/2(a2+b2)-c2 ml = ±[2 (a2 +b2) -c2]

Adicionando membro a membro as três últimas igualdades acima, segue que

ml +mb + ml = ^[2(b2 + c2)-a2 + 2 (a2 + c2)-b2 +2 (a2 +b2)-c2]

= i[3a2 + 3b2 + 3c2]

OBSERVAÇÃO: Deixamos como desafio provar que também é válida a


seguinte relação:
ml+mj + mj =^(a4+b4+c4)
Capitulo 3 - Resoluções 301

72) Seja ABC um triângulo equilátero. Se M é um ponto do plano do triângulo AC.


mostre que:
MA <MB+MC

Resolução:
De fato, se M é um ponto exterior ao triângulo ABC. temos um quadrilátero
ABMC, conforme ilustra a figura abaixo.

Utilizando a desigualdade de Ptolomeu, segue que:

MA.MC < AC ■ MB + AB ■ MC =>

MA a < a MB + a MC => MA < MB + MC

Como queríamos demonstrar.

OBSERVAÇÃO: Lembre-se que na desigualdade de Ptolomeu acima vale a


igualdade se, e somente se, o ponto M pertencer a circunferência circunscrita
ao triângulo ABC.

Agora analisemos o caso em que o ponto M está no interior do triângulo ABC.


conforme ilustra a figura a seguir:
302 Tópicos de Matemática - Oiimpiadas - IME - ITA

Aplicando o teorema da desigualdade triangular no triângulo BCM, segue que:


BC < MB + MC => a < MB + MC (1)
Por outro lado, perceba que MA < a (2), pois a medida que o ponto M se
aproxima de um dos vértices B ou C o segmento comprimento do MA tende a
medida a, mas por valores inferiores a a. Assim, de (1) e (2), segue que:
MA < a < MB + MC =. MA < MB + MC
como queríamos demonstrar, (note que na verdade mostramos que
MA < MB + MC, mas é claro que podemos escrever MA < MB + MC, pois o
sinal '<” significa que pode ser menor ou pode ser igual). Além disso, de
modo completamente análogo podemos demonstrar que
MB < MA + MC e MC < MA + MB .

73) Seja ABC um triângulo equilátero. Se M é um ponto do plano do triângulo AC


que não pertence a circunferência circunscrita ao triângulo ABC, mostre que
sempre existe um triângulo cujos lados são MA. MB e MC. (esse triângulo é
conhecido na literatura como triângulo de Pompeiu).

Resolução:
De fato, pelo problema anterior, se o ponto M é um ponto do plano do
triângulo equilátero ABC, que não pertence a circunferência circunscrita ao
triângulo ABC, segue que
Capítulo 3- Resoluções 303

MA <MB+MC
I MB < MA + MC
MC < MA + MB
O que revela que a medida do maior lado (que pode ser MA, ou MB ou MC) é
menor que a soma das medidas dos outros dois lados, o que. pelo teorema
I
da desigualdade triangular, garante a existência do triângulo cujos lados são
MA, MB e MC, como queríamos demonstrar.

74) Com um segmento DE de tamanho 3, é possível dividir um triângulo ABC de


lados 3, 4 e 5, de modo que as extremidades do segmento DE fiquem sobre
os lados AB e BC do triângulo ABC?

Resolução:
Para facilitar suponhamos que cada palito tenha um comprimento igual a 1.
Assim com os três palitos que sobram podemos montar um segmento de
tamanho 3. Vamos mostrar que podemos "encaixar" este segmento de
tamanho 3 numa determinada posição no interior do triângulo de modo que o
triângulo retângulo de lados 3, 4 e 5 fique subdividido em duas regiões de
mesma área. Vejamos a seguir.

Para determinarmos a posição exata do segmento DE (de comprimento 3)


vamos determinar as medidas dos segmentos BE e CD de modo que o
segmento DE tenha medida 3, e as medidas das áreas dos polígonos BDE e
304 Tópicos de Matemática - Olimpíadas - IME - ITA

ACDE seja iguais e portanto a metade da ares do triângulo ABC. Ora como
3-4
(ABC) =-^-= 6u.a. segue que (BDE) = (ACDE) = 3u.a.

Na figura a seguir traçamos algumas linhas auxiliares:

4-x -y

No triângulo CDF,

sena = — =>
z 5 z 4

Por outro lado no triângulo BDG,


„ H h 3 h 3
senp =------ =------ - — => — =>h = —(20-5x)
5-Z 5- ,55 20-5x
—x
4 4
Por outro lado sabemos que (BDE) = (ACDE) = 3u.a. e daí

3 = (BDE) = l(4-x-y)(s-|x) senp => 10 = (4


-x-y)(20-5x)
=3/5

3 = (ACDE) = (3^h)x+^.hy (3 + h)x + hy = 6

Agora temos o seguinte sistema de equações

(4 - x - y)(20 - 5x) = 40
(3 + h)x + hy = 6

h = è(2°‘5X)
Capitulo 3 - Resoluções 305

3
Isolando o y na segunda equação e pondo h = —(20-5x) obtemos

5x2
y= substituindo este resultado na primeira equação obtemos
20-5x '

(4 - x - y)(20 - 5x) = 40 =>^4-x- 5x2 "1


_____ '
-l(20-5x) = 40=sx = 1
20-5x,

5 5 5
Ora, como z = — x , segue que z = —-1 = —
4 4 4

5x2
Finalmente como y= e x = 1, segue que
20-5X

y=
5x2 512 2
20-5x 20-5-1 3

Com os valores de x, y e z temos então a localização exata do segmento DE

(de tamanho 3). Veja:


306 Tópicos de Matemática - Olimpíadas - IME - ITA

POLÍGONOS

75) Mostre que nenhum polígono convexo pode ter mais que dois ângulos
internos de 30°.

Resolução:
Num polígono convexo os ângulos interno e externo em cada vértice são
suplementares. Assim para cada ângulo interno de 30° está associado um
ângulo externo de 180° - 30° = 150°. Suponha, por absurdo, que existisse um
polígono convexo com mais que dois ângulos internos de 30°, isso implicaria
em pelo menos três ângulos internos de 30°, que por sua vez, implicaria em
pelo menos três ângulos externos de 150° cada um, o que é uma contradição,
pois, como sabemos a soma das medidas dos ângulos externos de um
polígono convexo é 360° e três ângulos de 150° já somariam 450° Assim a
suposição inicial de que existisse um polígono convexo com mais que dois
ângulos internos de 30° e falsa e portanto não pode existir um polígono
convexo com mais de dois ângulos internos medindo 30° cada um.

76) Na figura abaixo ABCDE é um pentágono regular e ABPQ é um quadrado


interno ao pentágono. Calcule a medida do ângulo ZDBQ = a.
Capítulo 3 - Resoluções 307

Resolução:
Ora, como ABPQ é um quadrado e BQ é uma das duas diagonais, segue que
ZABQ = 45° (as diagonais de um quadrado também são bissetrizes dos
seus ângulos internos). Por outro lado como ABCDE é um pentágono regular,
segue que cada um dos seus ângulos internos medem
(n-2)180° (5-2)180°
ai = = 108° . Como o triângulo BCD é isósceles. pois
n 5
BC=CD (pois o pentágono ABCDE sendo regular todos os seus lados são
congruentes), segue que
180°-108°
ZCBD = ZCDB = = 36°
2

Nesse momento temos a seguinte configuração:

Ora, como o ângulo do vértice B do pentágono ABCDE mede 108°, segue


que:
45° + a + 36° = 108° => a = 27»
308 Tópicos de Matemática - Olimpíadas - IME - ITA

77) Qual o número de polígonos regulares tais que quaisquer duas de suas
diagonais, que passam pelo centro, formam entre si um ângulo cuja medida
expressa em graus é dada por um número inteiro.

Resolução:
Antes de qualquer coisa, note que para que um polígono regular possua
diagonais passando pelo centro é preciso que o seu número de lados seja par
(polígonos regulares com um número ímpar de lados não tém diagonais
passando pelo centro). Além disso perceba que para que a medida do ângulo
entre quaisquer duas diagonais que passam pelo centro seja dada por um
número inteiro de graus basta que a medida do ângulo entre duas diagonais
"vizinhas" seja dada por um número inteiro de graus Por outro lado todo
polígono regular é inscritivel, ou seja, existe uma circunferência passando por
todos os seus vértices. Supondo que o polígono regular em questão possua
2n lados, se traçarmos as diagonais que ligam os vértices diametralmente
opostos formaremos 2n triângulos isósceles (com dois lados iguais ao raio da
circunferência circunscrita ao polígono) e cujo ângulo oposto a base é de
360° 180°
------ =------- , que corresponde justamente a medida do ângulo entre duas
2n n
180°
diagonais "vizinhas”. Ora, como queremos que ------ seja inteiro, basta
n

procurarmos os divisores de 180. Como 180 = 22-32 5, segue que a


quantidade de divisores naturais de 180 é dada por
QN (180) = (2 + 1)(2 +1)(1 + 1) = 18

Note que n = 1 não convém ao problema, pois nesse caso teriamos 2n=2, o
como sabemos não existe um polígono com apenas 2 lados. Diante do
exposto, existem 18 — 1 = 17 polígonos regulares em que quaisquer duas de
suas diagonais, que passam pelo centro, formam entre si um ângulo cuja
medida expressa em graus é dada por um número inteiro.
Capítulo 3 - Resoluções 309

78) (OMRN)Um retângulo de ouro é um retângulo de dimensões 1xq> onde

Võ+1
-p = -y- é a conhecida razão auréa. Este tipo de retângulo goza da

propriedade de que ele pode ser dividido num quadrado e num retângulo
semelhante ao retângulo original. Este processo continua infinitamente
conforme ilustra a figura a abaixo:

1 1

>p i <p-i
H
Diante do exposto, mostre que:
. 1 1 1
<p2 <p4 q>6

Resolução:
1 1 1
Note que a corresponde a soma de uma
soma1WV+-
progressão geométrica infinita de primeiro termo 1 e razão 0<q = -X<1.
<P
Ora, como sabemos a soma dos infinitos termos de uma progressão

geométrica infinita com razão q tal que |q| < 1 é S = . Assim,

.1 d----1— d----1~r d- —1x" + ... 1 ■p2


<p2 <p4 <p6 <p2-1
1V
Mas ocorre que, pelo enunciado o retângulo hachurado da figura do meio é
semelhante ao retângulo da figura da esquerda, o que revela que os seus
lados são proporcionais, ou seja,
<p-1 _ 1
cp2 -<p =1 => <p2 -1 = (p
1 <p
310 Tópicos de Matemática - Olimpíadas - IME - ITA

Portanto,
<p2
1+_L+J_+J_+ p2 — = <P
<p2 <p4 <p6 ■ ■ <p2-i <P
Como queríamos demonstrar.

79) Mostre que a medida do comprimento da diagonal d de um pentágono regular

de lado 1 mede d = <p =


=-------- (medida da razão áurea).

Resolução:
De fato, como o pentágono ABCDE é regular, segue que ele é inscritivel,
como ilustra a figura abaixo:
Capítulo 3 - Resoluções 311

Note que o quadrilátero BCDE é inscritível e então pelo teorema de Ptolomeu


(num quadrilátero inscritível o produto das medidas das diagonais é igual a
soma dos produtos das medidas dos lados opostos), segue que:
BDCE = BECD + BCDE =>
dd = d-1 + 1-1=>d2-d-1 = 0=>d = ^^ = <p , pois d > 0.

80) ABCDE é um pentágono cujos ângulos B e E são iguais e no qual AB = AE,


BC = ED. Demonstrar que os ângulos C e D sâo iguais.

Resolução:
Seja ABCDE um pentágono cujos ângulos B e E sâo iguais de medida a e no
qual AB = AE, BC = ED, conforme ilustra a figura abaixo:

Vamos mostrar que 0 = cp . De fato, traçando-se o segmento BE, note que o


triângulo ABE é isósceles de base BE, o que revela que seus ângulos da base
sâo iguais, conforme ilustra a figura abaixo:
312 Tópicos de Matemática - Olimpíadas - IME - ITA

Por fim observe que o quadrilátero BCDE é um trapézio pois as distâncias dos

vértices C e D ao segmento BE são iguais ah = bsen(a -y), pois traçando

perpendiculares ao segmento BE pelos pontos C e D formamos triângulos

retângulos de hipotenusas b e um ângulo agudo a — y na base, o que

revela que no triângulo DEF, sen (a -y) = h, =bsen(a-y), e de modo

completamente análogo, no triângulo CBG. temos que

sen(a-y) = ^2. h2 = bsen(a-y), conforme ilustra a figura a seguir:

Por fim perceba que pelo fato de h, = h2 = h = bsen(a-y), segue que o

segmento CD é paralelo ao segmento BE e portanto o quadrilátero BCDE é

um trapézio isósceles, o que revela que os ângulos das suas bases sâo

congruentes, em particular, olhando para a base CD, segue que p = <p , como

queríamos demonstrar.
Capitulo 3 - Resoluções 313

81) (OBM) Na figura abaixo o Hexágono ABCDEF é inscritivel. Sabendo que


AB = 1, BC = 2, CD = 3, DE = 4 e EF = 5, determine o comprimento do lado FA.

Resolução:
Sabemos que se por um ponto externo a uma circunferência traçamos dois
segmentos tangentes á essa circunferência esses dois segmentos tem
comprimentos iguais (teorema de Pitot). Sendo G, H, I, J, L e M os pontos de
tangência dos lados do hexágono com a circunferência, conforme ilustra a
figura abaixo,

Sendo GB = x , segue que


AG = AB-GB = 1-x
314 Tópicos de Matemática - Olimpíadas - IME - ITA

BH = BG = x
HC = BC - BH = 2-x
CI = CH = 2-X

ID = CD-CI = 3-(2-x) = 1 + X

DJ = DI = 1 + x

EJ = DE-DJ = 4-(1 +x) = 3-X

EL = EJ = 3-x
LF = EF-EL = 5-(3-x) = 2 +X

FM = FL = 2 + x

Por fim, temos também que AM = AG = 1 - x . Assim,

FA = FM + MA = (2+x) +(1-x) = 3

OBSERVAÇÃO:
Um outro modo de resolver essa questão é usar o teorema que afirma que
num polígono circunscritível a soma alternada dos comprimentos dos seus
lados é constante (esse teorema é a generalização do teorema que afirma
que um quadrilátero é circunscritível se, e somente se, a soma dos
comprimentos dos seus lados opostos é constante). Usando esse para para o
hexágono do nosso problema, segue que
AB + CD + EF => BC + DE + FA

1 +3 + 5 = 2 +4 +FA => FA = 3

A demonstração desse fato é simplesmente repetir o raciocínio que expomos


na nossa primeira solução para um n-ágono circunscritível. Convidamos o
leitor à fazê-la.

82) Prove que em qualquer pentágono convexo existem dois ângulos internos
consecutivos cuja soma é maior ou igual a 216°
Capitulo 3 - Resoluções 315

Resolução:
Sejam a,p,y,0 e X as medidas dos ângulos internos e um pentágono
convexo (nessa ordem). Suponha, por absurdo, que nâo existissem dois
ângulos internos consecutivos cuja soma é maior ou igual a 216°. Nesse
caso, todas as somas das medidas de dois ângulos consecutivos seria menor
que 216°, ou seja,

a + P <216°
P + y <216°
y + 0<216°
6 + X<216°
X + a<216°
Adicionando membro a membro as cinco desigualdades acima, temos que:
2 (a + p + y + 0 + X) < 1080° => a + P + y + 0 + X < 540°

O que é uma contradição, pois a + p + y + 0 + X = 540°, pois a soma das

medidas dos ângulos internos de um pentágono é S5 = (5 - 2) • 180° = 540°.

Portanto a suposição inicial de que não existissem dois ângulos internos


consecutivos cuja soma é maior do que ou igual a 216° é falsa. Assim, segue
que em qualquer pentágono convexo sempre existem dois ângulos internos
consecutivos tais que a soma das suas medidas é maior do que ou igual a
216°.

83) Se A, B, C e D são quatro consecutivos vértices de um polígono regular,


, 1 1 1
temos que —- = — + — . Quantos lados o polígono tem?
que —
AB AC AD

Resolução:
Vamos começar a solução investigando alguns polígonos com poucos lados
para "ativar o desconfiõmetro" e então conjecturar o caminho que vai levar a
solução da questão. Não esqueça que o polígono que estamos procurando é
regular (lados congruentes e ângulos congruentes). Ora, como o pelo
enunciado existem pelo menos quatro vértices, A,B C e D, segue que o
316 Tópicos de Matemática - Olimpíadas - IME - ITA

polígono procurado não pode ser um triângulo, pois só possui três vértices.
Então comecemos a nossa análise pelo quadrado:

Nesse caso. AB = a, AC = a>/2 e AD = a , que claramente não cumprem a

1 1 1
condição — = + —. Assim, o polígono procurado possui pelo menos
AB AC
mais um vértice E. Ora, como o polígono que estamos procurando é regular,
segue que ele é inscritivel, Como não sabemos quantos lado possui o
polígono, desenhamos apenas uma parte dele com os vértices A, B, C, D e E.
conforme ilustra a figura abaixo:
Capítulo 3 - Resoluções 317

Considerando que AB = BC = CD = DE =...= a, AC = b e AD = c, segue que

1 1 1 1-1■ 1 => bc = ac + ab
AB - AC + AD a ~ b +—
c
(1)

Agora, notando que CE=AC=b aplicando o teorema de Ptolomeu no

quadrilátero ACDE (que é inscritível), segue que


AD CE = AC ■ DE + CD• AE => bc = ba + a • AE (2)

Assim, de (1) e (2), segue que:


ac + ab = ba + a ■ AE => AE = c
Portanto, AE = AD = c , o que revela que o vértice A é equidistante dos
vértices D e E e portanto pertence a mediatriz do segmento DE, que passa
pelo centro da circunferência circunscrita ao polígono ABCDE...
Diante do exposto, mediatriz do segmento DE é um eixo de simetria do
polígono, como à direita da mediatriz temos três vértices do polígono
procurado (B, C e D), segue que pela simetria, à esquerda do polígono
também devemos ter outros três vértices do polígono procurado, o que revela
que o polígono procurado é um heptágono, pois possui o vértice A mais seis
outros vértices, três à direita e três á esquerda da mediatriz do segmento DE.
conforme lustra a figura abaixo:
318 Tópicos de Matemática - Olimpíadas - IME - ITA

84) (OMERJ) Os lados de um hexágono convexo e equiãngulo medem 10, 6, 12.


14. x e y, nessa ordem. Qual o perímetro desse hexágono?

Resolução:
Seja ABCDEF o referido hexágono, conforme ilustra a figura abaixo:

Ora, a soma das medidas dos ângulos externos de um polígono convexo é


Se = 360° Como o hexágono é. por hipótese, equiãngulo, segue que a

360°
medida de cada um dos seus ângulos externos é ae = = 60°. Dessa

forma, prolongando-se os seus lados, obtemos o triângulo PQS. mostrado na


figura abaixo:
S

60° 60°
E D
14
X 12

60° y 6 '60°
60° .60°
P A B Q
10
Capítulo 3- Resoluções 319

Assim,
No triângulo AFP, temos: ZAPF = 180° - 60°-60° = 60°, o que revela que o
triângulo AFP é equilátero e, portanto, AP = PF = FA = y.
Analogamente,
No triângulo BCQ, temos; ZBQC = 180° - 60° - 60° = 60°, o que revela que o
triângulo BCQ é equilátero e, portanto, CQ = QB = BC = 6.

E, por fim, no triângulo DES, temos: ZDSE = 180°-60°-60° = 60°, o que


revela que o triângulo DES é equilátero e, portanto, DS = SE = ED = 14, como
ilustra a figura abaixo:

14 60° 14

60° 60°
E D
14
T. 12

F
,C
y G0° y 6 6Õ°\ 6
.60° 60' 10° 6(1X
P A B 6 Q
V 10

Note também que o triângulo PQS é equilátero (pois seus ângulos internos
medem 60° cada um). Assim,
PS = PQ=>y + x + 14 = y + 10 + 6=>x = 2
QS = QP => y +10 + 6 = 6 +12 +14 => y = 16

Portanto o perímetro do hexágono ABCDEF é 2p = 3x32 = 96, pois cada


lado do triângulo PQS mede 32.
320 Tópicos de Matemática - Olimpíadas - IME - ITA

QUADRILÁTEROS

85) Se x, y e z são distâncias dos vértices B, C e D de um paralelogramo ABCD a


uma reta r que contém o vértice A e é exterior a ABCD, mostre que y = x+z.

Resolução:
Inicialmente trace as diagonais o paralelogramo ABCD. Conforme ilustra a
figura abaixo:
C

H
y
z

B
x

A E
n
i F
_ZL
G

Sendo H o ponto de encontro das diagonais do paralelogramo ABCD, segue


que DH = HB (H é o ponto médio das diagonais). Assim o segmento Hl é a

base média do trapézio BDEF e portanto Hl = ■ Além disso AH = HC

(pois H também é o ponto médio da diagonal AC), o que revela que o


Capitulo 3- Resoluções 321

segmento Hl é a base média do triângulo ACG, logo Hl =—


2
X+Z V z+xv
Assim, Hl = —— e Hl = ^, o que implica que —— = ^=>y = x + z.

86) Uma reta r pertencente ao plano de um paralelogramo ABCD é exterior a ele.


Se A, B e C distam x, y e z, respectivamente de r, determine, em função de x,
y e z a distância do vértice Dar.

Resolução:
Denotando a distância do vértice D â reta (r) por w e sendo o ponto E o ponto
de encontro das diagonais AC e BD do paralelogramo, podemos esboçar a
figura a seguir:

Agora perceba que o segmento EF é a base média dos trapézios AGJC e


GIHD, pois o ponto E é o ponto médio das diagonais AC e BD. Portanto,

EF = ^Í^
2 w+y x+z
------ =>w = x + z- y
2
2
322 Tópicos de Matemática - Olimpíadas - IME - ITA

87) As bases MQ e NP de um trapézio medem 42cm e 112cm, respectivamente


Se a medida do ângulo ZMQP é o dobro da medida do ângulo ZPNM,
determine da medida do lado PQ.

Resolução:
O enunciado nos leva a construção da seguinte figura:

M 42
Q

2a

N 112 P

Traçando o segmento QS paralelo ao lado MN, segue que NS = MQ = 42,


SP = NP - NS = 112 - 42 = 70, ZQSP = ZMNS = a (pois QS // MN) e
ZQSP=ZMQS=a (ângulos alternos internos pois MQ//NP, visto que o
quadrilátero MNPQ é um trapézio e portanto as suas bases sâo paralelas).
Por fim note que ZSQP=ZMQP-ZMQS=2a-a=a, conforme ilustra a
figura abaixo:

M 42 Q

N 42 S 70 P

Diante do exposto, no triângulo PQS temos ZPQS = ZPSQ = a . o que revela


que o triângulo PQS é isósceles de base QS e portanto, PQ = SP = 70cm.
Capitulo 3 - Resoluções 323

88) No trapézio ABCD, AB = CD, X é o ponto médio do segmento AB. BX = 1 e


DXC = 90°, Qual é o perímetro do trapézio ABCD?

Resolução:
Sejam BC = x e AD = y. Ora, como o triângulo CDX é retângulo ele pode ser
inscrito nume semicircunferência como ilustra a figura abaixo:

Sendo E o centro dessa semicircunferência. segue que EC = ED = EX = 1.


Além disso note que o segmento EX é a base média do trapézio ABCD.
Assim,
ADtBC . x+y
EX = =>1 = —yf- => x + y = 2
2
324 Tópicos de Matemática - Olimpíadas - IME - ITA

O que revela que o perímetro do trapézio ABCD é


2p(ABCD) = AS + BC + CD + DA
=2+y+2+x
= 4+x +y
=4+2
=6

89) Um trapézio isósceles possui diagonais perpendiculares e área medindo


98cm2 Qual a medida da sua altura?

Resolução:

A área do trapézio acima corresponda a somas das áreas dos quatro


triângulos no qual ele está dividido pelas suas diagonais. Ora, como a área do
trapézio é, por hipótese, 98cm2, segue que
a a bb a-b a b „„
2 2 2 2
a2+ b2 + 2ab = 196

(a + b)2 = 196 => a+.b = 14 , pois a + b > 0.

Por outro lado olhando para os triângulos retângulos isósceles de catetos


iguais a a e também o outro de catetos iguais a b, segue, por Pitágoras, que
x = b J2 e y = aV2 . Por fim, a área do trapézio também pode ser calculada

(x + y)h
por , onde h é a medida da sua altura.
2
Capitulo 3 - Resoluções 325

Assim,
(x + y)h
= 98 => (bv/2 + aVJjh = 196=.
2
(a + b)V2h = 196 14V2h = 196=>

196
h= = 7 72 cm.
14^2

90) (UERJ) Observe a figura abaixo


D

ela representa um papel quadrado ABCD, com 10cm de lado, que foi dobrado
na linha AM. em que M é o ponto médio do lado BC. Se após a dobra. A. B.
C. D e M são coplanares. determine:
a) A distância entre o ponto B' e o segmento CD.
b) O valor de tgO.

Resolução:
Por B' trace um segmento perpendicular ao segmento AB. Além disso note
que os triângulos ABM e AB'M são congruentes, pois um é obtido a partir do
outro por um movimento rigido (rotação/translação), o que revela que MB' =
MB = 5, AB' = AB = 10 e que ZMAB = ZMAB' = a , conforme ilustra a figura a
seguir:
326 Tópicos de Matemática - Olimpíadas - IME - ITA

D c

8'
5
5

10 M

5
8/
-yo

A 10 E B

Utilizando 0 teorema de Pitágoras no triângulo ABM, segue que

AM = V52 + 102 = 5^5 e portanto no mesmo triângulo ABM , segue que

5 1 10 2
sena = —t= = -7= e cos a = —7= = —=■ , o que revela que
5v5 v5 5^5 VÕ
„ o 1 2 4
sen(2a) = 2senacosa = =—.

Por outro lado no triângulo ABE, segue que

B'E 4 B’E
sen (2a) = B'E = 8cm. Assim a distância do ponto B' ao
10 5 10
segmento CD é 10cm-8cm=2cm.

b) Olhando para o vértice A do quadrado ABCD é fácil perceber que

2a + 0 = 90°. Assim,

1
0 = 90° - 2a => tg0 = tg(90° - 2a) =
tg(2a)

Ora, já sabemos que B'E = 8 e que AB' = 10. Usando o teorema de Pitágoras

no triângulo AB'E temos que AE = V102 -82 = 6. Portanto, no mesmo

. . , B'E_8 4 A .
triângulo AB'E, temos que tg(2a) = = —. Assim,
-- ' AE' “ 6

1 1 3
0 = 9O°-2a=>tg0 =
tg(2a) “ 4 " 4
3
Capítulo 3 - Resoluções 327

91) (Mediana de Euler) Seja ABCD um quadrilátero cujos lados medem a. b. c e


d, conforme ilustra afigura abaixo.

O Segmento PQ, que une os pontos médios das diagonais AC e BO é


chamado de mediana de Euler do quadrilátero. Mostre que
a2 + b2 + c2 + d2 = AC2 + BD2 + 4 -PQ2

Resolução:
Pelo ponto B trace uma reta paralela ao segmento AD e pelo ponto D trace
uma reta paralela ao segmento AB. Seja E o ponto de interseção dessas duas
retas. Note que o quadrilátero ABED é um paralelogramo e portanto o ponto
Q, que é o ponto médio da diagonal BD também será o ponto médio da
diagonal AC (pois as diagonais de um paralelogramo cortam-se ao meio). Por
fim trace o segmento CE, conforme ilustra a figura abaixo:
C

c
D, P

a a

A b B
328 Tópicos de Matemática - Olimpíadas - IME - ITA

Perceba que o segmento PQ é a base média do triângulo ACE, pois os


pontos P e Q são pontos médios do segmentos AC e AE, respectivamente.
Assim, pelo teorema da base média de um triângulo, segue que

PQ = -CE=>CE = 2PQ.
2
Agora por B trace uma reta paralela ao segmento CD e por A trace uma reta
paralela ao segmento CE e seja F o ponto se interseção dessas duas retas.
Trace ainda os segmentos DF, CF e EF, conforme ilustra a figura abaixo:
C
!

'v E

Perceba que os quadriláteros BCDF e ACEF são paralelogramos sujas


diagonais são BD. CF e AE, CF, respectivamente. Assim,

No paralelogramo BCDF, temos:

BD2 + CF2 = 2(DC2 + CB2) CF2 = 2d2 + 2c2 - BD2 (1)


Capitulo 3 - Resoluções 329

No paralelogramo ACER, temos:


AE2 + CF2 = 2(aC2 +CE2) => CF2 = 2AC2 + 2CE2 - AE2 (2)

de (1) e (2), segue que:

2d2 + 2c2 -BD2 = 2AC2 + 2CE2 - AE2 (3)


Mas no paralelogramo ABDE, temos que:
AE2 + BD2 = 2(a2 + b2) => 2a2 + 2b2 = AE2 + BD2 (4)

Adicionando (3) e (4), membro a membro, segue que:

2a2 + 2b2 + 2c2 + 2d2 - BD2 = 2AC2 + 2CE2 + BD2 =>

2a2 + 2b2 + 2c2 + 2d2 = 2AC2 + 2CE2 + 2BD2 =>

a2 + b2 + c2 + d2 = AC2 + CE2 + BD2 =>

a2 + b2 + c2 + d2 = AC2 + BD2 + CE2

Por fim, lembrando que CE = 2■ PQ. segue que

a2 + b2 + c2 + d2 = AC2 + BD2 + CE2 =>

a2 + b2 + c2 + d2 = AC2 + BD2 + (2 ■ PQ)2 =>

a2 + b2 + c2+d2 = AC2 + BD2 +4 PQ2


como queríamos demonstrar.

92) Num trapézio ABCD, de bases AB = x e CD = y. mostre que o segmento PQ.


que une os pontos médios dos lados não paralelos é paralelo as bases AB e
... . x + y
CD e sua medida e •—- .
2

Resolução:
Consideremos a figura abaixo onde ABCD é um trapézio de bases AB = x e
CD = y, assim como os ponto P e Q (pontos médios dos lados não paralelos)
330 Tópicos de Matemática - Olimpíadas - IME - ITA

D y C

í
Q

A x B

Traçando a diagonal AC e sendo T o ponto de interseção de AC e PQ,


conforme ilustra afigura abaixo

segue pelo teorema da base média de um triângulo que T é o ponto médio de

AC, PT//DC e PT= Analogamente, QT//AB e QT = ^ . Ora, como AB//CD,

visto que AB e CD são as bases do trapézio ABCD, que por definição são

paralelas, segue que PQ=PT+TQ // AB (ou CD) e além disso,

x x+y
PQ = PT+TQ = —
2 2 " 2
Capítulo 3 - Resoluções 331

93) Na figura seguinte, o perímetro do triângulo ABC vale 20cm, a base BC mede
8cm e o círculo está inscrito no quadrilátero BCED. Calcule o perímetro do
triângulo ADE,

Resolução:
Sejam AD = u, AE = v, Dl = x, IE = y, BF = z e CG = w. Pelo teorema de Pitot.
segue que DF = x, EG = y, BH = z e HC = w, conforme ilustra a figura abaixo:
332 Tópicos de Matemática - Olimpíadas - IME - ITA

Mas, =
2pAAac = 20 => (u + x + z) + (z + w) + (w + y + v) = 20 => =
2Paabc - 20 u + (x + y) + v + 2(z + w) = 20
BC = 8=>z + w = 8
Assim,
u + (x + y) + 2(z + w) - 20 => 2pAADE +2.8 - 20 => 2pAADE - 4 cm.
=B

94) O triângulo ABC da figura é equilátero e AD = 1, determine o tamanho da


medida do lado a do triângulo ABC.

Resolução:
No triângulo ADE, segue que o ângulo do vértice A mede 60°, pois coincide
com o ângulo do vértice A do triângulo equilátero ABC. Assim,

tg60° = — =>73= — =>DE = 73


1 1

cos 60° = — => - = — => AE = 2


AE 2 AE
Capitulo 3 - Resoluções 333

Sendo a a medida do lado do triângulo equilátero ABC, segue que


EB = AB - AE = a - 2 , BC = a , DC = AC - AD = a -1 e DE = 75
Ora, o quadrilátero BCDE é drcunscritível e. portanto. BC + DE = EB + DC.
Assim,
BC + DE = EB +DC => a + 75 = (a-2)+ (a-1) =>a = 3+73

95) Um retângulo é dividido em 9 retângulos menores conforme ilustra a figura


abaixo:
D C

12 4 6

A B
Os números indicados no interior dos retângulos menores correspondem c
seus respectivos perímetros. Qual a medida do perímetro do retângulo maic

Resolução:
Inicialmente nomeie os comprimentos dos lados dos retângulos menores
como ilustra a figura abaixo:

D
a b c C

d. 6 d

e 12 4 6

8
f f

A a B
b
334 Tópicos de Matemática - Olimpíadas - IME - ITA

Ora. como os números indicados no interior de alguns retângulos menores


indicam as medidas dos seus perímetros, segue que:

2(a + e) = 12 a+e = 6
2(b + d) = 6 b+d = 3
■ 2(c + e) = 6 => c + e — 3
2(b + e) = 4 b+e = 2
2(b + f) = 8 b+f=4

Adicionando membro a membro as últimas igualdades acima, segue que


3(b + e) + (a + d + c + f) = 18=53-2 + (a + d + c + f) = 18=> a + c + d + f = 12

Por outro lado, o perimetro do retângulo maior (exteno) é

2(a-rb + c + d + e + f). Ora, como

[b + e = 2
ia + c + d + f = 12

Adicionando membro a membro as duas últimas igualdades acima, segue que


a + b + c + d + e + f = 14
Portanto, o perimetro do retângulo maior (externo) é
2. (a + b + c + d + e + f) — 2-14 = 28

96) Dois círculos, de raios R e r, respectivamente, são inscritos num quadrado


ABCD de lado 1, conforme ilustra a figura a seguir
D c

A B

Calcule a soma R + r.
Capitulo 3 - Resoluções 335

Resolução:
Pelo Teorema de Pitágoras, sabemos que a diagonal do quadrado de lado
medindo 1 tem comprimento 75 . Os centros dos círculos e o ponto onde eles
se tocam dividem a diagonal em quatro partes de comprimentos
R75,R,r e r75 , conforme ilustra a figura abaixo:

fix/2

Por outro lado, o comprimento da diagonal do quadrado é igual a -J2 . Assim.

72 = R^2 + R + r + rV2 = (R + r)(V2 +1) =•

/õ 75(72-1)
R + r = -—=>R + r => R = 2-72
75 + 1 = (72 + l)(V2-l)

97) Mostre que o perímetro de um quadrilátero convexo é maior que a soma de


suas diagonais e menor que o dobro da soma das suas diagonais.

Resolução:
Sejam a, b, c e d as medidas dos lados de um quadrilátero ABCD de
diagonais AC e BD, conforme ilustra a figura a seguir

I
336 Tópicos de Matemática - Olimpíadas - IME - ITA

Aplicando o teorema da desigualdade triangular aos triângulos ABC,


ACD.ABD e BCD, segue que
aABC -> AC < a + b
AACD -> AC < d + c
AABD -» BD < a + d
ABCD -> BD < b + c
Adicionando, membro a membro, as quatro últimas desigualdades acima.
segue que:
2(AC + BD) <2(a + b + c + d)=>AC + BD<a + b + c + d

Como 2p=a+b+c+déo perímetro do quadrilátero ABDC, segue que


AC + BD < 2p (1)
que é uma parte da desigualdade pedida no enunciado.
Agora, aplicando o teorema da desigualdade triangular nos triângulos ABE,
BCE, CDE E ADE, segue que:
AABE—>a<EA+EB
bBCE—>b<EB+EC
ACDE-» c< EC + ED
AADE -> d < ED + EA
Adicionando, membro a membro as quatro últimas desigualdades acima,
segue que
a + b + c + d < 2(ED + EB) + 2(EA + EC) =>
-BD =AC

a+b+c+d< 2(AC + BD)


Capitulo 3 - Resoluções 337

2p<2(AC + BD) (2)


Portanto, de (1) e (2), podemos concluir que
AC + BD < 2p < 2(AC + BD)

que é justamente a desigualdade pedida pelo enunciado.

98) Os ângulos A e B em um quadrilátero convexo ABCD são iguais e BC = 1,


AD = 3. Prove que a medida de CD é maior do que 2.

Resolução:
A figura a seguir resume as informações do enunciado:

Prolongando em linha reta os segmento AD e BC esses prolongamentos se


intersectarão no ponto E, formando um triângulo isósceles ABE, de base AB,
conforme ilustra a figura a seguir:
338 Tópicos de Matemática - Olimpíadas - IME - ITA

Ora, como EA = EB. segue que 3 + x = 1 + y=>y-x = 2. Por fim, aplicando o

teorema da desigualdade triangular ao triângulo CDE, segue que ( a medida

de qualquer lado de um triângulo é maior que o módulo da diferença das

medidas dos outros dois):

y - x < CD => 2 < CD => CD > 2

que é o resultado desejado.

SEMELHANÇA

99) Na figura abaixo o ponto M é o ponto médio do lado BC, NA é bissetriz do

ângulo ZBAC e BN é perpendicular a NA. Sabendo que AB e AC medem,

respectivamente, 14m e 19m, calcule o comprimento do segmento MN.


Capitulo 3 - Resoluções 339

Resolução:
Prolongue (em linha reta) o segmento BN até o ponto D do lado AC, conforme
ilustra figura abaixo:

Como M é o ponto médio do segmento BC, sejam BM = CM = a . No triângulo

ABD perceba que NA é bissetriz interna do ângulo do vértice A e além disso

também é altura em relação ao lado BD, o que revela que o triângulo ABD é

isósceles de base BD Ed portanto AD = AB = 14, o que implica que CD = AC

- AD = 19 - 14 = 5. Por fim, note que sendo o triângulo ABD isósceles de

base BE e NA altura referente a essa base, segue que NA também é

mediana, ou seja. BN = ND. Assim os pontos M e N são pontos médios dos

lados do triângulo BCD e portanto MN é base média desse triângulo, que

como sabemos, tem um comprimento igual a metade do comprimento da

5
base CD desse triângulo. Diante do exposto, temos que x = —.
340 Tópicos de Matemática - Olimpíadas - IME - ITA

100) Em um triângulo ABC, retângulo em A. trace a altura AH Mostre que a soma


das áreas dos círculos inscritos nos triângulos AHB e AH C é igual a área do
círculo inscrito em ABC .

Resolução:
Sejam R, resas medidas dos raios dos três círculos e a,b e c as medidas
dos lados do triângulo ABC conforme ilustra a figura abaixo:

Ora, como os triângulos ABH, ACH e ABC são semelhantes (pois possuem
ângulos congruentes) segue que: (aqui estamos utilizando o fato de que a
razão de semelhança também é mentida para as medidas dos raios das
circunferências inscritas em triângulos semelhantes).

r c r2 c2 s_
s b s2 b2
R2 a2 e6 R a R2 - a2
R a R
Capítulo 3- Resoluções 341

r2 c2
Adicionando membro a membro as igualdades —- = — e e
R 3 R2 a2 '

usando o fato de que no triângulo retângulo ABC temos a2 = b2 + c2

(teorema de Pitágoras)segue que:

s2+r2 c2+b2 a2
= 1 => s2 + r2 = R2
R2 a2 a2

Finalmente multiplicando os dois membros da igualdade s2 + r2=R2 por n ,

e lembrando que As = ns2, Ar = nr2 e que AR = nR2 , obtemos:

ns2 + nr2 = nR2 => Ar + As = Ar

101) Duas retas paralelas r e s distam 5,0cm uma da outra. Entre elas está
marcado um ponto P, que dista 1,0cm de r. Pontos A e B são escolhidos
respectivamente sobre r e s, de modo que ZAPB = 90°.
a) Determine o comprimento de AP para que a área do triângulo APB seja o
menor possível.
b) Determine menor o valor possível para a área do triângulo APB.

Resolução:
a) Os dados do problema nos levam a figura a seguir:

(r) A C

3/ V- Icm
P
a

x 4cnt

B ' D
(s) y/x2 - 16
342 Tópicos de Matemática - Olimpíadas - IME - ITA

Note que os triângulos ACP e BDP são semelhantes (pois apresentam


ângulos dois a dois congruentes). Portanto,

x.
1
X
=> y =
X

Vx2-16 Vx2-16
Por outro lado a área do triângulo APB pode ser calculada por:

1
(APB) = lxy = -x x x2
— X ,------ = =----- ,
Z 42 5/x2-16 2'/x2-16

Assim para encontrarmos o valor mínimo da área do triângulo APB basta

x2
encontrarmos o valor mínimo da função f (x) = —.
2vx2-16
Calculando a derivada e igualando-se a zero para achar o ponto critico,
obtemos:

2x 2\/x2-16-x2 2x
4x(x2-1ô)-2x3
Vx2-16 2x3-64x
f (x) =
^Vx2-16)2 4(x2-16)^x2-16 4 (x2 -16) Vx2-16

Assim,
f’(x) = 0 o 2x3 - 64x = 0ox = 0 ou x = 4\Í2 (x = 0 não pertence ao domínio de f).

Lembrando que y = , x , temos então:


Vx2-16
X 4^2 í—?■ = -J2 cm, que é o comprimento do
y=
Vx2-16 Í(4V2)2-16 4

segmento AP.

b) o valor mínimo da área do triângulo APB é:

(APB) = f (x) =
x2
=>f(4^) =
M2 ,g=4cm2.
2>/x2-16 2^(4V2)2-16 8
Capítulo 3 - Resoluções 343

102) Sejam ABC um triângulo retângulo em C, BCDE e ACFG dois quadrados


construídos externamente ao lados AC e BC do triângulo ABC. Conforme
ilustra a figura abaixo AE intersecta BC no ponto H, enquanto que BG
intersecta AC no ponto K. Determine a medida do ângulo ZDKH.
g a

C B
H

D E

Resolução:
Sejam BC = a e AC = b. Lembrando que AGFC e BCDE são quadrados,
podemos colocar os seguintes dados na figura:

G b A

b K
a

B
F b C H a

a
a

D a E
344 Tópicos de Matemática - Olimpíadas - IME - ITA

Note que os triângulos BGF e BKC são semelhantes por apresentarem


ângulos correspondentes com medidas iguais. Analogamente, que os
triângulos DAE e CAH são semelhantes por apresentarem ângulos
correspondentes com medidas iguais. Assim,

ABGF ~ ABKC => — = 3 => KC = '


b a+b a+b
CH b ab
ADAE — ACAH => =------ => CH =-------
a a+b a+b

O que revela que KC = CH e portanto o triângulo KCH é um triângulo


retângulo (em C) isósceles, o que revela que ZCKH = ZCHK = a. Assim, no
triângulo KHC temos que
90° + a + a = 180° => 2a = 90° => a = 45°

Por fim, perceba que ZDKH = ZCHK = a = 45°

103) Calcule o comprimento de DE na figura abaixo. Sabendo que DG = 7cm,


DF = 2cm, BDC é arco da circunferência tangente a AB e AC nos pontos B e
C respectivamente.
Capitulo 3 - Resoluções 345

Resolução:
Inicialmente trace os segmentos BD e CD. Note que os ângulos
ZDBE e ZCDF "enxergam" o mesmo arco CD da linha pontilhada de

circunferência, portanto têm medidas iguais. Seja ZDBE = ZFCD = a.


Analogamente os ângulos ZDCE eZDBG "enxergam" o mesmo arco BD na
linha da circunferência pontilhada, portanto têm medidas iguais.
Seja ZDCE = ZDBG = <p , conforme ilustra a figura abaixo:

G
7
F
/

<p&X

B
"'A
¥> A.C
E
Além do que já foi revelado, note também que ZGDB = ZCDE = 90°-q> = B ,

assim como ZBDE = ZCDF = 90°-a = p . Diante do exposto, podemos então


concluir que existem na figura acima dois pares de triângulos semelhantes, a
saber: ABDE - ACDF e aBDG ~ ACDE, donde podemos montar as
seguintes proporções:

ABDE-ACDF=>- = — (D
2 CD
BD
aBDG - aCDE => - (2)
x CD
Igualando-se as expressões (1) e (2) acima, segue que:
7
— = — =>X2=14=>X = -714 .
2 x
346 Tópicos de Matemática - Olimpíadas - IME - ITA

104) (UFPE) Os postes verticais AB e CD na figuram medem 12m e 8m


respectivamente. Existem cabos de sustentação unindo a base de cada um
dos postes ao topo do outro poste.
A

c
£
CN

E
CO

B D

Qual a distância, em metros, do ponto de interseção dos cabos à horizontal?

Resolução:
Com os dados da questão podemos montar a seguinte figura:

B rn F 71 D

Note que temos dois pares de triângulos semelhantes (pois em cada par os
triângulos apresentam ângulos congruentes); (BEF , BCD) e (DEF, DAB) .
Assim,
¥ m
ABEF - ABCD => - =
8 m +n
Y n
adef - aDAB => — =
12 m + n
Capitulo 3 - Resoluções 347

Adicionando membro a membro as duas últimas igualdades acima, segue que:


x x m n x x m+n x x . 24
— + — = 1 => x = —m
8 12 m+n m+n 8 12 m+n 8 12 5

105) (UFRJ) Na figura a seguir, o circulo de raio 1cm rola da posição I para a
posição F, sempre tangenciando o cateto AC do triângulo retângulo ABC. Na
posição I o circulo também tangencia AB e na posição F ele é tangente a BC.
Os lados do triângulo valem AC = 6cm, AB = 8cm e BC = 10cm. Determine a
distância percorrida pelo centro do circulo.

Resolução:
Considere o circulo desenhado no interior do triângulo ABC nas suas
posições inicial e final, conforme ilustra a figura do enunciado. Além disso
trace o s raios nos pontos de tangência com os lados do triângulo e o
segmento XY que liga as posições inicial e final do centro do círculo no
processo sugerido pelo enunciado, conforme ilustra a figura abaixo:

A d F 7—4 B
1 E
348 Tópicos de Matemática - Olimpíadas - IME - ITA

Note que BF = BA - FA = 8 - (d +1) = 7 - d e que BY é bissetriz do ângulo B

do triângulo ABC, pois o ponto Y é equidistante dos lados BA e BC (lembre-se


nesse ponto que a bissetriz de um ângulo pode ser visto como o lugar
geométrico dos pontos do plano que contém o ângulo que são equidistantes
AC _ 6 _ 3
dos lados do ângulo). Assim no triângulo ABC, senB = BC = 10 ~ 5 e

n AB 8_ 4 _ . , , , í'B'1 /1-cosB Í1-4/5


cos B = — -. Por outro lado, tg — =,/----------- ” 3
BC 10 5 V1 + cosB 1 + 4/5

Por fim no triângulo BYF, temos que

,9® = 1
5-d
11
— - ------- => d ■ 4, cm.
3 7-d

OBERVAÇÃO: Há uma outra saída que é utilizar simplesmente semelhança


de triângulos. Vejamos'

Trace o segmento DJ paralelo ao segmento AB, conforme ilustra a figura


abaixo:
Capítulo 3 - Resoluções 349

Perceba que os triângulos ABC e DJC são semelhantes por apresentarem


ângulos dois a dois congruentes e portanto os seus lados são proporcionais:
DJ DC DJ 5 _ . 20
'— —— —• —DJ — ■
AB AC 8 6 3

Por outro lado os triângulos YGJ e ABC também são semelhantes, por
apresentarem ângulos dois a dois congruentes e portanto os seus lados
também sâo proporcionais:
YJ YG YJ 1 w, 5
— => YJ = —
BC “ AC 10 6 3

Para finalizar, basta perceber que


on 5
DJ = DX+ XY + YJ => —— = 1 + d + —=> d = 4 cm.
3 3

106) (PERU) Na figura abaixo os segmentos AO e AB são diâmetros das


semicircunferências de centro E e O, respectivamente. Com centro no ponto
B e raio BE é traçado o arco de circunferência EF. Sabendo-se que AE = EO
= 1, determine a medida do raio da circunferência menor que tangencia os
três arcos de circunferência indicados na figura.
350 Tópicos de Matemática - Olimpíadas - IME - ITA

Resolução:
Seja D o centro da circunferência menor. Inicialmente note que o semicírculo

menor tem raio 1, o maior tem raio 3 e o arco EF tem raio 3 Agora trace os

segmentos DE, DO e DB e note que DE = 1+x, DO = 2-x e DB = 3 + x,

conforme ilustra a figura abaixo:

Aplicando a relação de Stewart ao triângulo BDE, segue que:

DO2 • BE = BD2 • EO + DE2 • BO - OE • OB ■ BE


(2 - x]2 ■ 3 = (3 + x)2 ■ 1 + (1 + x)2-2 — 1 ■ 2 ■ 3 =>

12-12x + 3x2 = 9 + 6x + x2 + 2 + 4x + 2x2 -6 =>

22x = 7 => x = —
22
Capítulo 3 - Resoluções 351

107) A circunferência de centro O é tangente ao lado AC no ponto E e aos


prolongamentos dos lados AB e BC, como mostra na figura. Sendo DE // BC.
AB = 18 cm, AC = 9 cm e BC = 21 cm, então o segmento AD, em cm, mede

Resolução:
Seja AD = x, como AB = 18, segue que DB = 18 - x. Por outro lado, supondo
que CG = y, segue pelo teorema de Pitot, que CE = CG = y. Ora. como AC =
9, segue que AE = 9 - y e mais uma vez pelo teorema de Pitot, segue que AF
= AE = 9 - y, como ilustra a figura a seguir:

18 -T
5.

B 21 C w G
Por hipótese, temos que DE//BC, o que revela que os triângulos ADE e ABC
são semelhantes e portanto têm lados proporcionais, o que nos permite
montar a seguinte igualdade:
AD AE x 9-y
= —--=>x = 18-2y
AB “ AC 18 9
352 Tópicos de Matemática - Olimpíadas - IME - ITA

Por outro lado, mais uma vez pelo teorema de Pitot, segue que
BF = BG =. 18 + (9 - y) = 21 + y => 2y = 6 =• y = 3

Assim,
x = 18-2y => x = 18-2-3 x = 12 cm.

108) (Teorema de Monge) Dadas três circunferências C-i, C2 e C3 de centros O,,

O2 e O3 e raios r,. r2 e ra, respectivamente. Seja X a interseção das tangentes

comuns externas de C1 e C2. Y a interseção das tangentes comuns externas

de C-i e C3 e, finalmente, Z a interseção das tangentes comuns externas de

C2 e C3. Prove que X, Y e Z são colineares.

•C3

.C2

2t

z
Y
X

Resolução:

Inicialmente perceba que os pontos X, O-, e O2 são colineares pois os pontos

O-, e O2 são equidistantes (distâncias iguais aos raios) dos lados do ângulo

formado pelas duas tangentes comuns aos dois círculos C-, e C2, o que revela
Capítulo 3 - Resoluções 353

que os pontos O, e O2 pertencem á bissetriz do ângulo, que evidentemente


também contêm o ponto X, que é o vértice do ângulo como ilustra a figura
abaixo:

Note que os triângulos XP,O, e XP2O2 são semelhantes (pois apresentan


ângulos dois a dois congruentes). Assim existe a seguinte proporcionalidade
XO, O1P1 . h
XO2 ^2P2 ^2
de modo completamente análogo podemos mostrar que:

YO3 _ °3P3 _ r3 e Z02 _ °2P2 _ f2


YO, O,P, q ZO3 O3P3 r3
Multiplicando membro a membro as três expressões que acabados de
deduzir, segue que:
XO, YOg ZO2 q r3 r2 .
XO2 YO, ZO3 r2 q r2
Assim os pontos X, Y e Z são pontos que pertencem as retas suportes dos
XO, YO3 ZO2
lados do triângulo de vértices O1, O2 e O3 tais que—q3- ——- =1
XO2 YO^ ZO3

Portanto pelo teorema de Menelaus (a reciproca) segue que os pontos X. Y e


Z sâo colineares, como queríamos demonstrar.
354 Tópicos de Matemática - Olimpíadas - IME - ITA

109) Na figura ABCD é um retângulo, M è o ponto médio de CD e o triângulo ABM


é equilâtero. Se AB = 15m, calcule BS.

Resolução:
Seja BS = x. Ora, como o triângulo ABM é equilâtero, segue que BM = MA =
AB = 15. Como M é, por hipótese, o ponto médio de CD e CD = AB = 15,
segue que CM = MD = 15/2 Sendo BS = x, segue que SM = 15-x. Note que
os triângulos ABS e CMS sâo semelhantes (pois possuem ângulos dois a dois
congruentes), conforme ilustra a figura abaixo:
D A

15
2

M 15
15 - X

15 S x
2

C B
Capitulo 3 - Resoluções 355

Dessa semelhança temos:


BS AB x ^x = 10
MS ~ CM 15-x
2

110) Sejam Q, e Qj duas circunferências de centros O e P, respectivamente, e

raios r e R tais que Q, e não possuam pontos em comum A partir de O

são traçadas duas tangentes a d? e a partir de P são traçadas duas

tangentes a O,, conforme ilustra a figura abaixo:

Sendo E, FG eH os pontos de interseção dos segmentos tangentes AO. OB


PC e PD com Q, e Oj ■ mostre que EF = GH.

Resolução:
Trace os raios nos pontos de tangência e o segmento, marque os pontos M e
N (pontos de interseção dos segmentos tangentes às circunferências) e o
segmento MN. conforme ilustra a figura abaixo:
356 Tópicos de Matemática - Olimpíadas - IME - ITA

Perceba que os triângulos COM e PAM são semelhantes, pois apresentam os


ângulos dois a dois congruentes. Dessa semelhança temos a seguinte
proporção:
OC PA
(1)
OM PM
Por outro lado perceba que OC=OE e PA=PG, assim podemos reescrever (1)
da seguinte forma:
OC PA OE PG
(2)
OM PM OM PM
Perceba que os triângulos OEF e OMN também são semelhantes (pois os
pontos M e N possuem a mesma potência em relação a O,, visto que estão a

mesma distância do cento O (pela própria simetria da figura) e portanto


MC2 = OM OE e ND2 = ON OF . Ora, como as potências dos pontos M e N

em relação a Q, são iguais (e iguais a MC2 ou ND2), segue que

OM
OM.OE = ONOF=> — e como o ângulo ZMON é comum aos
OF ON
triângulos OEF e OMN, concluímos que eles são semelhantes).
Assim, da semelhança dos triângulos OEF e ONM, temos a seguinte
proporção
OE EF
(3)
OM " MN
E por fim, note que os triângulos PGH e PMN também são semelhantes (pelo
mesmo motivo dos triângulos OEF e OMN), e portanto temos a seguinte
proporção:
PG GH
PM “ MN
(4)

Finalmente, por (2), (3) e (4), segue que:


OE PG EF GH
^4=>EF = GH
OM PM MN ~ MN

como queríamos demonstrar.


Capítulo 3 - Resoluções 357

111) (OBMEP-2012 - Adaptada) A figura mostra um retângulo ABCD decomposto


em dois quadrados e um retângulo menor BCFE. Quando BCFE é
semelhante a ABCD, dizemos que ABCD é um retângulo de prata e a razão
AB . ,
— e chamada razao de prata.

D F C

A E B

Qual é o valor da razão de prata?

Resolução:
Sejam a e b as medidas indicadas na figura abaixo
a b
D a F C

a a a a

A a a E b B

Com essas medidas, segue que AB = 2a + b e AD = a. Mas, por hipótese, o


retângulo BCFE é semelhante ao retângulo ABCD, o que revela que:

AB AD 2a+ b a
2ab + b2 =a2-2ab-b2 =0 (1)
BC CF a b
358 Tópicos de Matemática - Olimpíadas - IME - ITA

Dividindo os dois membros por b2. segue que


2
a2-2ab-b= = 0^g) -2g)-1 = 0, que é uma equação quadrática em

a p- a
- . Fazendo - = x temos
b b
2
x' = 1+72
(3 -<3-i=o^x2-2x-i=o^ x" = 1->/2

Ora, como x = - > 0 , segue que - = x = 1 + 72 . Por fim, como AB = 2a + b e


b b

.r. AB 2a + b 2a + b a .
AD = a, segue que — . Mas, por (1), ocorre que-------- = - . Assim,
a a b
AB 2a+ b
— = X = 1 + 72
AD a b

112) (OBMEP-2011- Adaptada) Na figura, AEFD é um retângulo, ABCD é um


quadrado cujo lado mede 1 cm e os segmentos BF e DE são
perpendiculares.

A B E

Qual é a medida, em centímetros, do segmento AE?


Capítulo 3 - Resoluções 359

Resolução:
Sejam BE = x, ZBFE = a e ZFBE = p , o que revela que a + p = 903 e,

portanto, ZFEQ = p , o que implica que ZDEA = a e ZEDA = p , revelando

então que os triângulos BEF e DAE são semelhantes (pois possuem ângulos

dois a dois congruentes), como ilustra a figura abaixo:

1 x
D C F

a ri

1
1
Q,

,□
A B
0 Cl
E
1 x

Dessa semelhança temos a seguinte proporção:

, 75-1
X ” 2
BE EF x 1
x2 + x-1 = 0
AD AE 7 1+ X -75-1
x" =
2

75-1
Ora, como BE=x>0, segue que x = —-— . Assim,

AE = 1 + x = 1 + ^1=>AE = ^Í1
2 2
360 Tópicos de Matemática - Olimpíadas - IME - ITA

113) Em um quadrado ABCD, AC e BD se interceptam em E. O ponto F sobre BC


é tal que os ângulos ZCAF e ZFAB são iguais, se AF intersecta BD em G e
se EG = 24, determine CF.

Resolução:
45°
Seja CF = x. Note que a = -^- = 22,5°. Por E trace uma paralela ao

segmento BC, que encontra o segmento AF no ponto H. Note que


ZBCD - 0 = 45°, pois BD é diagonal e portento bissetriz do ângulo ZABC do
quadrado ABCD. Assim, olhando para o triângulo ABF, segue que
ZAFB = <p = 90° - a = 90° - 22,5° = 67,5°

O que revela que ZDGF = 180° -45° -67,5° = 67,5° = <p e, portanto o
triângulo BGF é isósceles de base GF. Ora, como EH//BC, segue que
ZHEG = ZBCD = 0 = 45° e ZGHE = ZGFB - <p = 67,5° e.
e, portanto

ZHGE = ZBGF = <p = 67,5° , o que revela que o triângulo EGH também é

isósceles de base GH Assim, EH = EG = 24, conforme ilustra a figura a


seguir:
Capítulo 3 - Resoluções 361

A D

Por fim, como EH//CF, segue que os triângulos AHE e AFC são semelhante,
(pois possuem ângulos dois a dois congruentes), o que nos permite
estabelecer a seguinte proporção:
CF AE x AE x
— = 2 => X = 48
HE “ AC 24 “ 2.AE 24
Note que usamos o fato do ponto E ser o ponto médio da diagonal AC do
quadrado ABCD, pois o ponto E é o ponto de interseção das diagonais AC e
BD do quadrado ABCD.

PITÁGORAS

114) Calcule a medida do raio do circulo que passa pelo vértice C e é tangente aos
lados AB e AD do quadrado ABCD cujo lado mede 1m.

A B
362 Tópicos de Matemática - Olimpíadas - IME - ITA

Resolução:
Seja O o centro da circunferência. Ligando o ponto O aos ponto P e Q, que
sâo, respectivamente, os pontos de tangência da circunferência com os lados
AD e A8 do quadrado e por fim traçando o segmento OS. segue que
OP = OQ = OC = R, OS = PS - PO = AB - PO = 1 - R
Conforme ilustra a figura abaixo:

Por fim, aplicando o teorema de Pitágoras no triângulo retângulo OCS, segue


que

R2 = (1 - R)2 + (1 - R)2 R = (1-R)V2=>R = 2-V2

115) Um trapézio isósceles possui diagonais perpendiculares e área medindo


98cm2. Qual a medida da sua altura?

Resolução:
Capitulo 3 - Resoluções 363

A área do trapézio acima corresponda a somas das áreas dos quatro


triângulos no qual ele está dividido pelas suas diagonais. Ora, como a área do
trapézio é, por hipótese. 98cm2, segue que
a a b b a b a b „„
—- +---- +------+------ = 98 =5
2 2 2 2
a2 + b2+2ab = 196=>

(a + b)2 = 196 => a + b = 14 , pois a + b > 0 .

Por outro lado olhando para os triângulos retângulos isósceles de catetos


iguais a a e também o outro de catetos iguais a b. segue, por Pitágoras. que
x = bV2 e y = a 75 . Por fim, a área do trapézio também pode ser calculad;

(x + y)h
por , onde h é a medida da sua altura.
2
Assim,
(x+y)h
98=>(bV2 + a>/2)h = 196=> (a +b)V2h = 196 => 1472h = 196 =>
2

= 7 72 cm.
" 14s/2

116)
a) Mostre que se um quadrilátero tem diagonais perpendiculares então a
soma dos quadrados das medidas dos lados opostos é constante.
b) O quadrilátero ABCD representado na figura abaixo tem diagonais
perpendiculares?
364 Tópicos de Matemática - Olimpíadas - IME - ITA

Resolução:
Seja ABCD um quadrilátero cujas diagonais são perpendiculares, conforme
ilustra a figura abaixo:

Ora. como as diagonais são perpendiculares, temos quatro triângulos


retângulos, a saber: ABE, BCE, CDE e ADE. Aplicando o teorema de
Pitágoras a cada um deles, segue que:
[âABE —> a2 = x2 + z2
a2+ c2 = x2 + y2+ z2 + w,2: (1)
óCDE c2 = y2 + w2

óBCE-» b2 = z2 + w2
b2+ d2 = x2 + y2+ z2 + w,2 (2)
AADE-» d2 = x2 + y2

Portanto de (1) e (2), segue que a2 +C2 = b2 + d2

a) Motivado pelo resultado do item anterior, naturalmente somos levados a


fazer a o teste se a soma dos quadrados dos lados opostos é constante,

ou seja, se 62 + 472 = 332 + 342, o que de fato é verdade, pois

62 + 472 =2245 e 332 + 342 = 2245 . Mas tome um cuidado: o


resultado do item (a) garante que se as diagonais são perpendiculares,
então a somas dos quadrados dos lados opostos é constante, mas não
garante que se a soma dos quadrados dos lados opostos for constante
então as diagonais são perpendiculares. Mas esse resultado também é
Capitulo 3 - Resoluções 365

verdadeiro (conforme revela o teorema abaixo), porém tem uma


demonstração um pouco mais difícil.
TEOREMA: Seja ABCD um quadrilátero cujos lados opostos a, c e b. d
e essas medidas cumprem a condição a2+c2 = b2+d2, então as
diagonais desse quadrilátero são perpendiculares.
Demonstração:
Pode ser encontrada no livro Mathematical Chestnuts From Around the
World - Ross Honsberber - MAA - págs 185/186.
Diante do exposto, segue que o quadrilátero cujos lados medem 6,
34,47 e 33. nessa ordem possui as suas diagonais perpendiculares.

117) Traçamos o circulo T de centro O circunscrito a um triângulo ABC, retângulo


em A de catetos 3 cm e 4 cm. Encontre o raio do circulo W de centro O',
tangente aos catetos de ABC e interiormente a T
Resolução:
O enunciado nos leva na seguinte figura:

E,

r—3/2
366 Tópicos de Matemática - Olimpíadas - IME - ITA

Antes de qualquer coisa, BC = ^32 + 42 = 5

Perceba que:
i) Traçando a partir de O a perpendicular ao lado AB, OM, segue que M é
o ponto médio do lado AB, pois os triângulos ABC e OBM são
semelhantes, com razão de semelhança BC/BO = 1/2 (lembre-se que o
ponto O é o centro da circunferência circunscrito ao triângulo ABC e
ponto médio da hipotenusa BC). Portanto OM = AC/2 = 3/2.
ü) Por O traça-se uma paralela ao lado AB, determinando o ponto D como
a interseção desta reta com o segmento O’F Agora analisando a figura
3
acima não fica difícil perceber que DO = FM = 2-r e que DO' = r-- e

que o triângulo DOO' é retângulo em D, visto que os ângulos O’DO e


O'FA são alternos internos e O'FA = 90°, uma vez que F é o ponto de
tangência da circunferência menor com o lado AB. (no ponto de
tangência o raio e a reta tangente são perpendiculares).
iv) Diante do que já foi dito, o triângulo OO'D é retângulo tem catetos
3
medindo 2-r e r-— e hipotenusa medindo x. Por Pitágoras,

2
x2 = (2-r)2 + [r-|]
Mas ocorre que OE'=r, OE=5/2 e OE=OO’+O'E. Assim,
2
OE = OO'+O'E=>- = x + r=>x = -- r=^x2=f--r'|
2 2 '.2 )

2
Como x2 = (2-r)2 + ,, segue que

z 3\2 ír = 0 (não convém!)


+ [r-—J => r2-2r = 0 => r(r-2) = 0
|r = 2
Capítulo 3 - Resoluções 367

118) (UK Intermediate Challenge) Uma folha de papel retangular de lados 1 e -J2

é dobrada conforme ilustra a figura abaixo:

Qual o valor da medida d?

Resolução:
Imagine a folha de papel antes de ser dobrada (linha pontilhada da figura
abaixo):

G v^2
D
r

1-d ’

A x/2

1-d
d

B E C
368 Tópicos de Matemática - Olimpíadas - 1ME - ITA

Perceba que o segmento ED é proveniente do segmento DG ao fazermos a

dobra do papel, portanto ED = DG = 7ã . De modo análogo, o segmento AE é

proveniente do segmento AG ao fazermos a dobra, o que revela que

AE = AG = 1-d . Por fim, aplicando o teorema de Pitágoras no triângulo CDE

temos que'

(V2)2 = CE2 +12 => CE = 1

Assim. BE = BC-CE = 75-1.

Aplicando novamente o teorema de Pitágoras, mas agora no triângulo ABE,


segue que:

(l-d)2 = d2 + p2-l)2=>1-2d + d2=d2 + 2-2>/2 + 1=> d = 75-1

119) A figura abaixo mostra um retângulo ABCD com AB=12cm e BC=2cm, e uma
circunferência passando pelos pontos C e D e tangente ao lado AB
Determine o raio dessa circunferência.
Capítulo 3 - Resoluções 369

Resolução:
Seja O o centro da circunferência. Trace os raios OC e OE e seja F o ponto
de interseção do raio OE com o segmento CD. Ora. Como EF = BC = 2 e F é
o ponto médio do segmento CD, segue que DF = FC = 6, conforme ilustra a
figura abaixo:

R
R —2
F
D 'C
6 6
2
A B
E

Finalmente, aplicando Pitágoras ao triângulo OFC, segue que

R2 = (R-2)2+62 =>R2 = R2-4R + 4 + 36 =>4r = 40=>R = 10 cm.

120) (CG) No triângulo ACB representado na figura abaixo as medidas x, y, z e w

cumprem a condição x2 + z2 = y2 + w2. Mostre que ZBDC = 90“.


370 Tópicos de Matemática - Olimpíadas - IME - ITA

Resolução:
De fato, temos que

x2 + z2 = y2 + w2 => z2 - y2 = w2 - x2

Sendo Z.BDC = P , segue que ZADC = 180° — P . Assim aplicando a lei

dos cossenos aos triângulos ADC e BDC, segue que:

AADC w2 = x2 + AD2 - 2 ■ x • AD • cos (180° - p)


=-cosp
w2 = x2 + AD2 +2 x ADcosp

ABDC -> z2 = y2 + AD2 -2 y -ADcosp

Agora podemos reescrever as expressões acima como

[w2-x2 = AD2 + 2-x-ADcosp


z2-y2 = AD2-2-y-ADcosp

Subtraindo membro a membro, e usando o fato de que z2-y2 = w2-x2,

segue que:

2-x-AD-cosp + 2-2-AD-cosp =0=> 2-AD-cosp-(x + y) = 0

Ora, como 2*O,AD#0ex + y = AB*0, segue que

2-ADcosp(x + y) = 0=>cosp = 0

Por fim, como 0 < p < 180°, se cosp = 0 , segue que p = 90°. Assim,

ZBDC = p = 90°.
Capitulo 3 - Resoluções 371

OBSERVAÇÃO: A recíproca desse resultado também é verdadeira e é


bastante simples de ser demonstrada, ou seja, se ZBDC = 90°, então

x2 + z2 = y2+w2. Para ver isso, basta aplicar o teorema de Pitágoras aos

triângulos ADC e BDC. Vejamos:

No triângulo ACD, w2 = CD2 + x2 => CD2 = w2 - x2 (1)

No triângulo BDC, z2 = CD2 + y2 => CD2 = z2 -y2 (2)


Igualando-se as expressões (1) e (2), segue que:
w2 - x2 = z2 - y2 => x2 + z2 = y2 + w2

como queríamos demonstrar.

121) Pé um ponto no interior do retângulo ABCD tal que PA = 3, PC = 5 e PD = 4,


determine a medida do segmento PB.
D c
■1
5

P,
3

A B
372 Tópicos de Matemática - Olimpíadas - IME - ITA

Resolução:
Por P trace os segmentos DE e GF paralelos aos lados do retângulo ABCD,
conforme ilustra a figura abaixo:

b c
D H C

4 5
a a

E F
3 P~'~—
d d
A G c B
b

Aplicando o teorema de Pitágoras aos triângulos DPH, CHP, AGP e BGP,


segue que:

ADPH->a2+b2 = 42
a2+b2 + c2 + d2 = PB2 + 42 (1)
ABGP -> c2 + d2 = PB2

AAGP -> b2 + d2 - 32 a2 + b2 + c2 + d2 = 32 + 52 (2)


ACHP —> a2 + c2 = 52

Assim, de (1) e (2), segue que:

PB2 +42 =32+52 =.PB = 7Í8 =372 .


Capitulo 3 - Resoluções 373

122) P é um ponto interior ao quadrado ABCD tal que PA = 1, JPB = 2 e PC = 3.


Qual a medida do ângulo ZAPB ?

Resolução:
Fazendo uma rotação de 90° do quadrado ABCD em torno do ponto B (no
sentido anti-horário), obtemos a seguinte figura:

Com o movimento de rotação seja P' a nova posição do ponto P. Note que
BP' = BP = 2, pois o segmento BP' é o segmento BP girado de 90° no sentido
374 Tópicos de Matemática - Olimpíadas - IME - ITA

anti-horário em torno do ponto B, Ora, como o triângulo BPP’ é retângulo e


isósceles, segue que

PP'2 = 22+22 =>PP'2 = 8 e ZBPP'= ZBP’P = 45°

Por outro lado, perceba que, com o movimento de rotação o vértice C irá para
a posição do vértice A e portanto o segmento PC se transformará no
segmento P'A (pois o ponto P irá para a posição do ponto P’). Assim, P'A =
PC = 3. Por fim, perceba que:

P'A2 = 32 = 9

P'P2 + PA2 =8 + 1 = 9

O que revela que P'A2 = P'P2 + PA2 e portanto, pelo recíproco do teorema
de Pitágoras, segue que o triângulo P'PA é retângulo em P. Assim,
ZAPB = ZAPP'+ ZP'PB = 90° + 45° = 135°

123) Num quadrante de circunferência cujo raio mede 1. está inscrito um quadrado
e circunferência menor, que tangencia o quadrado ABCD e a circunferência
maior Qual a medida do raio da circunferência menor?
Capitulo 3 - Resoluções 375

Resolução:
Trace o segmento AT, onde T é o ponto de tangència da circunferência menor

com o quadrante maior, Sendo AB = a, segue que a72 = AC = 1 => a = — .


72

Sendo O o centro da circunferência menor, segue que os pontos A O e T são


colineares, pois as circunferências são tangentes internas, conforme ilustra a
figura abaixo:

72
Para finalizar perceba que AO = 1-r, AB = — + r e OQ = r. Aplicando

Pitágoras ao triângulo retângulo AOQ, segue que:

, \2 <72 r2 + (272 + 4)r-1 = 0 =>


(1-0 =^ + rj + r2

728 + 1672 -272 -4


r" 2
376 Tópicos de Matemática - Olimpíadas - IME - ITA

124) Dado que PQRS é um quadrado de lado a e que ABS é um triângulo


equilátero, determine, em função de a, a medida do lado do triângulo
equilátero ABS.

Resolução:
Sejam a e b as medidas dos lados do quadrado PQRS e do triângulo
equilátero ABS. Note que os triângulos retângulos SBR e SPA são
congruentes pois são triângulos retângulos com hipotenusas de mesmo
tamanho b (SB = AS - b) e além disso, também apresentam dois catetos
congruentes, pois SP = SR = a. Dessa congruência RB = PA = x, como ilustra
a figura abaixo'

—x

S a R
Capitulo 3 - Resoluções 377

Ora, como PQ = a e PA = x, segue que AQ = a - x e de modo completamente

análogo, QB = a - x. Aplicando Pitágoras ao triângulo ABQ, segue que

b2 = (a - x)2 + (a - x)2 => x = 2a 2b^

Assim, no triângulo SBR, segue que

b2 = a2 + x2=>

í2a-b>/2'l2 __
b2 = a2
l 2 J
4b2 = 4a2 + 4a2 - 4ab72 + 2b2 =>

2b2 + 4a>/2 b-8a2 = 0

Que pode ser vista como uma equação quadrática na variável b. Assim,

2b2 + 4aV2b-8a2 = 0=>b = í>/6->/2)a

125) (OBM)Em uma folha quadriculada em que cada quadrado tem lado 2cm, são
desenhados dois círculos como na figura ao lado. A distância minima entre os
dois circulos mede:
378 Tópicos de Matemática - Olimpíadas - IME - ITA

Resolução:
Inicialmente ligue os centros das circunferências. Sejam A e B os pontos em
que o segmento que une os centros das circunferências intersecta as
circunferências, conforme ilustra a figura abaixo:

2 aj / 1

2 1

Note que os pontos A e B são os pontos mais próximos das duas


circunferências e portanto a distância mínima entre as duas circunferências é
x = d(A,B). Aplicando Pitágoras ao triângulo hachurado da figura acima,

segue que:

(2 + X + 1)2 = 12 + 32 =>(3 + x)2 = 10=> 3 + x = VÍÕ=>x = VÍÕ-3

126) Um octógono regular ABCDEFGH está inscrito numa circunferência de raio 1,


e P é um ponto arbitrário dessa circunferência. Calcule o valor de

pa2+pb2+...+ph2.
Capitulo 3 - Resoluções 379

Resolução:
Como ABCDEFGH é um octógono regular, cada uma das diagonais AE, BF,
CG e DH é um diâmetro da circunferência, o que revela que os triângulos
APE, BPF, CPG e DPH são retângulos (em R), conforme ilustra a figura
abaixo:

Assim, pelo Teorema de Pitágoras,

PA2+PE2 = 22
PB2 +PF2 = 22
PC2 + PG2 = 22
PD2 +PH2 = 22

Adicionando, membro a membro as igualdades acima, segue que:

PA2 + PB2+... + PH2 = 16


380 Tópicos de Matemática - Olimpíadas - IME - ITA

127) (OPM) No triângulo ABC as medianas dos lados AB e AC são


perpendiculares. Sabendo-se que AB = 6 e AC = 8, determine a medida do
segmento BC.

Resolução:
Seja G o ponto de encontro das medianas BN e CM (o ponto G é o baricentro
do triângulo ABC). Se GM = u, segue que CG = 2.GM = 2u e se GN = v,
segue que BG = 2.GN = 2v, conforme ilustra a figura abaixo:

Aplicando Pitágoras nos triângulos retângulos BGM, CGN e BCG, segue que

aBGM u2 + (2v)2 = 32 => u2 + 4v2 = 9

ACGN -> v2 + (2u)2 = 42 => v2 + 4u2 = 16

ABCG -> (2v)Z + (2u)2 = x2 => 4(v2 +u2) = x2


Capitulo 3 - Resoluções 381

Adicionando, membro a membro, as duas primeiras igualdades, segue que


5(u2 + v2) = 25=>u2 + v2 = 5

Ora, como 4(v2 + u2) = x2 , segue que x2 = 4 5 => x = 2/5 .

LEI DOS SENOS E COSSENOS

128) Os lados de um triângulo medem a, b e c centímetros. Se forem satisfeitas as


relações 3a = 7c e 3b = 8c, qual o valor em graus, do ângulo oposto ao lado
que mede a centímetros.

Resolução:
Sejam A, B e C as medidas dos ângulos opostos aos lados de medidas a. b e

3a
c, respectivamente. Ora, como 3a = 7c e 3b = 8c, segue que c = —
7

8 8 3a 8a
b = -c = -— = — . Assim, aplicando a lei dos cossenos, segue que:

a2 = b2 +c2 - 2bccosA =>

2 8a 3a .
a2
■o “2———cosA =>

48 24 1
— cos A = — => cos A = — => A = 60° , pois 0o < A <180° .
49 49 2
382 Tópicos de Matemática - Olimpíadas - IME - ITA

129) ABCD é um quadrado e ABE é um triângulo equilátero de lado AB = 6.


Determine as medidas x e y dos segmentos DE e CF.

Resolução:
Ora. como ABCD é um quadrado, segue que ZBAD = 90° e como ABE é um
triângulo equilátero, segue que ZBAE = 60°, o que revela que
ZDAE = 90° - 60° = 30° . Por outro lado, AD = AE = AB = 6 Assim, aplicando
a lei dos cossenos ao triângulo ADE, segue que:

x2 = 62 + 62-2-6-6 cos30°=>

x2 = 72-36^3 =>

x = ^36(2-73)

x = 6x/2-73

Agora observando o triângulo CBF, segue que ZACB = 45° (pois AC é uma
diagonal do quadrado ABCD e portanto bissetriz do ângulo do vértice C). Da
mesma forma que fizemos no inicio da resolução para o vértice A, podemos
ver que ZCBF = 30°. Assim, no triângulo CDF,
ZBFC = 180°-45°-30° = 105°
Capitulo 3 - Resoluções 383

A figura abaixo ilustra todas essas informações:

o c
E
II 45°

6 105°

3o¥Z □0'
-7^60°

A 6 B

Por fim aplicando lei dos senos ao triângulo CBF, segue que
BC 6 , y 3
sen105°
y
sen30°
sen30° sen75° 1 4 y-
sen75°
2
Mas,
J6 +J2
sen75° = sen (45° + 30°) = sen45°cos30° + sen30°cos45° =
4

Portanto,
3 3 = 3(n/6->/2)
y=
sen75° J6 + J2
4

OBSERVAÇÃO: Apenas por curiosidade os valores de x e t são iguais, pois


pela fórmula de radicais duplos, segue que:

_ sf V3 1 'j
lV2~72j

= 3(x/6->/2) = y
384 Tópicos de Matemática - Olimpíadas - IME - ITA

130) ABCD é um quadrado e BCE é um triângulo equilátero. Se AB=6, determine a


medida x do segmento DF

Resolução:
Ora, como ABCD é um quadrado e BCE é um triângulo equilátero, segue que

ZABC = 90° e ZCBE = 60°, o que revela que

ZABE = ZABE + ZCBE = 90° +60° = 150“. Por outro lado AB=BE (pois o

quadrado ABCD e o triângulo equilátero BCE têm lados congruentes, visto

que possuem o lado BC em comum). Assim, o triângulo ABE é isósceles de

180°-150°
base AE, o que revela que ZBAE = ZBEA = = 15°. Ora, como
2

ZBAD = 90“. segue que:

ZDAF = ZBAD - ZBAE = 90° -15° = 75°


Capítulo 3 - Resoluções 385

A figura abaixo ilustra todas essas informações:

Por fim ZABD = 45°, pois BD é uma das diagonais do quadrado ABCD e
portanto bissetriz do ângulo interno do vértice B do quadrado ABCD Note que
o ângulo ZAFD é o ângulo externo do vértice F do triângulo ABF. o que
revela que ZAFD = ZBAE + ZABD = 15° + 45° = 60°. Agora aplicando a lei
dos senos ao triângulo ADF, segue que:

-A2- = => = =. x = 3 72 + 7ê
sen60° sen75° Vã V6 + V2
T 4

131) Seja ABC um triângulo qualquer e AD a bissetriz do ângulo interno do vértice


A, conforme ilustra a figura abaixo:
386 Tópicos de Matemática - Olimpíadas - IME - ITA

,. m n
Mostre que — =
c b
Resolução:
Se ZADB = p, segue que zADC = 180°-p. Assim, aplicando a lei dos
senos aos triângulos ABD e ACD, segue que:
m m sena
AABD -> ——
sena c senp
(D
senp
n n sena
>sen(180°-p) (2)
sena b senp
=senp

n
De(1) e (2), segue que —
c b

OBSERVAÇÃO: Há uma outra demonstração bastante elegante que é a


seguinte:
Nos triângulos ABD e ACD trace as alturas relativas aos lados AB e AC,
respectivamente. Note que essas alturas são de mesmo comprimento, pois o
ponto D pertence a bissetriz do ângulo A e como sabemos qualquer ponto
pertencente a bissetriz de um ângulo é equidistante dos lados do ângulo.
Seja h a medida de cada uma dessas duas alturas, conforme ilustra a figura
abaixo:
Capitulo 3 - Resoluções 387

Perceba que os triângulos ABD e ACD têm a mesma altura vinda do vértice A.
Assim a razão entre as medidas das áreas desses dois triângulos é igual a
razão entre as medidas das suas bases (a razão das medidas das áreas de
dois triângulos que possuem a mesma altura é igual à razão entre as suas
respectivas bases). Assim,
m (ABD)
n " (ACD)

c•h b h
mas ocorre que (ABD) = -^~ e (ACD) =-----. Assim.
2
ch
m (ABD) _ 2 c m n
n " (ACD) ” bT b c b
2

132) Seja ABC um triângulo qualquer e AE a bissetriz do ângulo interno do vértice


A, conforme ilustra a figura abaixo:

n
Mostre que —
c b
388 Tópicos de Matemática - Olimpíadas - IME - ITA

Resolução:
Ora. como ZBAE = 180°-a e definindo zAEC = p . aplicando a lei dos
senos nos triângulos ABE e ACE, segue que:
m c m senp
áABE (1)
sen (180°-a) senp c sena
=sena

b n sena
aACE -> —— (2)
sena senp b senp
... m n
De (1) e (2), segue que — = - , como queríamos demonstrar.
c b
133)
a) Na figura abaixo o triângulo ABC é isósceles de base BC = 1. Se

ZBAC = 36°, mostre que AB = AC = q> = 1.

b) A partir do item anterior, conclua que:

cos 36° = sen54° = —m


2

cos 72° = sen18° = —


2<p
Capítulo 3 - Resoluções 389

Resolução:
a) Ora, como o triângulo ABC é isósceles de base BC. segue que
180°-36°
ZABC = ZACB = = 72°
2
Traçando a bissetriz BD, do ângulo interno do vértice B, segue que
72°
ZCBD = ZABD =-----= 36° . No triângulo BCD. temos
2
ZBDC = 180°-72°-36° = 72°. o que revela que os triângulos BCD e ABD
são isósceles de bases CD e AB, respectivamente. Assim, BD = BC = 1. AD =
DB = 1. Sendo AB = x, segue que CD = x - 1, conforme ilustra a figura
abaixo:

Note que os triângulos ABC e BCD são semelhantes (pois apresentam


ângulos dois a dois congruentes). Assim, dessa semelhança temos a seguinte
proporção:

. 75 + 1
X =----------
BC AB 1 x
x2-x-1 = 0=> 2
CD BC x —1 1 -Võ-1
x" =
2

p ______ _ ____ n ____ __ 75+1 .....................


Ora, como AB=x>0, segue que AB = x =-----— = <p e pelo fato do triângulo

ABC ser isósceles de base BC, segue que AC = AB = <p . como queríamos

demonstrar
390 Tópicos de Matemática - Olimpíadas - IME - ITA

b) Aplicando a lei dos senos ao triângulo ABC, segue que


1 <p
9 1 ______ <P______ => cos 36° = —1 (p
sen36° sen72° sen36° 2sen36°cos36° 2
Ora. como 54' e 36° são medidas complementares. segue que
1
sen54° = cos 36° = — <p .

Aplicando a lei dos cossenos ao triângulo ABC, segue que

1 -r cp2 -(p
<p2=12+<p2-2 1<p cos 72° => cos 72° = (1)
2tp

x/5 + 1
Mas ocorre que o número é uma das raízes da equação

x2 - x -1 = 0 oque implica que <p2 — <p —1 = 0 <p2 = <p +1. Substituindo essa

informação em (1), segue que

COS72°=1 + 'p2~q> = 1 + (‘,> + 1)-<l> 1


=> cos 72° = —
2<p 2<p 2<p
’or fim, como as medidas 18° e 72° são complementares, segue que
1
sen18° = cos 72° = —
2cp

134) No Triângulo ABC da figura abaixo o ponto P pode deslizar livremente sobre o
lado BC. A partir de P são traçados os segmentos PM e PN que são
perpendiculares aos lados AB e AC, respectivamente. Determine a posição do
ponto P para que o comprimento do segmento MN seja o menor possível.
c

B
Capitulo 3 - Resoluções 391

Resolução:
Inicialmente note que o quadrilátero AMPN é circunscntivel pois os ângulos
AMP e ANP sâo retos e portanto AMP + ANP = 180°. o que implica que MAN
+ MPN = 180°, visto que num quadrilátero a soma das medias dos ângulos
internos deve ser sempre 360°. Na figura abaixo desenhamos a
circunferência que passa pelos pontos A, M, P e N.

Note que AP é o diâmetro desta circunferência (o triângulo APM é retângulo


em Me portanto está inscrito na semicircunferència. A figura abaixo mostra a
referida circunferência com o seu diâmetro AP
392 Tópicos de Matemática - Olimpíadas - IME - ITA

Aplicando a lei dos senos no AAMN segue que

= 2R => MN = 2R • senA => MN = AP ■ senA


senA
Como senA é fixo pois o ângulo A é fixo , segue que MN será mínimo quando
AP for mínimo Ora, mas AP é a distância do vértice A do triângulo ABC ao
lado BC e assim essa distância será minima quando AP for perpendicular ao
lado BC. Noutras palavras, MN será minimo quando P for o pé da altura
traçada do vértice A ao lado BC

135) (USA) Na figura abaixo, calcule a medida do lado do triângulo equilátero ABC,
sabendo que AD = AE = J7cm , BD = CE = 2cm e DE = 1cm .

Resolução:
Com as informações do enunciado podemos montar a figura a seguir:
Capitulo 3 - Resoluções 393

Aplicando a lei dos cossenos no triângulo ADE, segue que

12 = ('/zj2 + - 2^7-77 cosip => cos<p =

2
V27
E portanto, sen<p = -J1 - cos2 <p =
l-ÊP 14

Por outro lado, 2a + <p = 60° => 2a = 60° - ip , o que implica que

cos (2a) = cos (60° — <p) ==> 2 cos2 a = cos 60° cos <p + sen60°sen<p

Além disso, cos(2a) = 2cos2 a -1. Assim,

2 cos2 a -1 = cos 60° cos <p + sen60°sen<p

2cos2a-i = 112 + ^^7


214 2 14

2„ cos2 a = .1 + —
11
=> cos a = —5
14 277

Finalmente, aplicando a lei dos cossenos no triângulo ABD, segue que:

22 = a2 + (V?)2 -2a77 cosa => 4 = a2 + 7-2aV?=>

5 + VÍ3
a =---------- ou
a2 -5a + 3 = 0 => 2
5-VÍ3
a - -----------
2

5-7Í3 1 e por outro lado a > 1, pois pelo teorema da


Note porém que
2
desigualdade triangular aplicado ao triângulo ABD, segue que a > 77 + 2 > 1.

Assim, a única solução da equação quadrâtica a2 - 5a+ 3 = 0 que convém ao

5 + 7Í3
problema é a =
2
394 Tópicos de Matemática - Olimpíadas - IME - ITA

136) (CRUX-MATHEMATICORUM) Um quadrado cujo lado mede s está


simetricamente inscrito num setor circular de 60°. cujo raio mede r, conforme
ilustra a figura abaixo:

Calcule r/s.

Resolução:
Ora, como ABCD é um quadrado de lado s, segue que AB = s, o que revela que o
triângulo OAB é isósceles de base AO e ZAOB = 60°. Ora, como os ângulos da
base de um triângulo isósceles são congruentes, segue que ZOAB = 60°. Por
fim, como a soma das medidas dos ângulos internos do triângulo OAB é 180°,
segue que ZAOB + ZOAB + ZOBA = 60° => ZOBA = 60° . (então na verdade o
triângulo OAB é equilátero). Diante do exposto, temos que:
ZOBC = ZOBA + ZABC = 60° + 90° = 150°
Finalmente, aplicando a lei dos cossenos no triângulo OBC, segue que:

r2 = s2 +s2-2s s cos150° => r2 = 2s2 -2s2 => r2 = s2 (2 + 73) => - = 72+73

137) Inscreve-se um polígono convexo de 12 lados num circulo de modo que ele
possui, em alguma ordem, seis lados de comprimento 72 e seis de

comprimento 724 . Qual a medida do raio dessa circunferência? !


|

í
Capítulo 3 - Resoluções 395

Resolução:

Sejam AB e BC um par de lados adjacentes cujos comprimentos sejam

diferentes (isto é sempre possível!!! concorda? Pense um pouquinho e...).

Assim 0 arco (AB) +arco (BC) = 360°/6 = 60°. Agora perceba que o triângulo

OAC é equilátero e daí AC = R (medida do raio da circunferência) e que

ZABC = 150°, poisZABC é a metade da medida de um ângulo inscrito

correspondente a um arco de 300° (ângulo inscrito). Agora aplicando a lei dos

cossenos no aABC temos:

R2 = (V2)2 + (>/24)2 - 2 ■ ■ 724 COS150”


Dai, como cos! 50° = —— , concluímos que R = V38 .
396 Tópicos de Matemática - Olimpíadas - IME - ITA

138) (OLIMPÍADA IBEROAMERICANA) Um ponto P dista 5, 7 e 8 dos vértices de


um triângulo equilátero. Qual a medida do lado desse triângulo?

Resolução:
Ora. com o triângulo ABC é equilátero supondo que AB = BC = CA = a e
fazendo uma rotação de 60° (no sentido horário) do triângulo APC em torno
do vértice A ficamos com a seguinte figura:
Capitulo 3 - Resoluções 397

Como A = 60° (pois o triângulo ABC é equilátero), fazendo ZBAP = a . segue


que ZP'AB = ZPAC = 60°-a (pois os triângulos P'AB e PAC são
congruentes). Assim.
ZP' AP = ZP' AB + ZB AP = (60° - a) + a = 60°

Assim o triângulo isósceles APP' tem um ângulo de 60°, sendo pois


equilátero, o que revela que PP' = 5.

60° - a
A

5
a 60° — a
P’. 60° / 5
" '■/ 60° a
/ \]P
8
8
7

B a C

Agora aplicando a lei dos cossenos no triângulo PP’B segue que

72 = 82 + 52 -2-8 5 cos <p => cos <p = — => <p = 60°

Mais uma vez usando a lei dos cossenos, mas agora no triângulo AP'B, segue
que

a2 = 52 + 82 - 2 ■ 8 ■ 5cos120° a2 = 89-8o(-^j => a = vÍ29

139) Num triângulo de lados A B e C temos (a + b-c)(a + b + c) = 3ab . determine o

ângulo interno C .
398 Tópicos de Matemática - Olimpíadas - IME - ITA

Resolução:
Desenvolvendo, (a + b-c)(a + b +c) = 3ab obtemos.

[(a + b) - c][(a + b) + c] = 3ab => (a + b)2 -c2 = 3ab =>

a2 + 2ab +b2 -c2 = 3ab


Por outro lado, aplicando a lei dos cossenos ao triângulo ABC. segue que:
c2 = a2 + b2 -2abcosC
Assim.

a2 + 2ab + b2 - c2 = 3ab

a2 + 2ab + b2 - (a2 + b2 - 2abcosC) = 3ab =>

1
2ab cos C = ab => cos C = - =■ C = 60°
2

140) Determine a medida do lado AB do triângulo ABC representado na figura


abaixo:

135°

.3 30°"~
B C
Capitulo 3 - Resoluções 399

Resolução:
Aplicando a lei dos senos no triângulo APC, segue que:
x ^~^=>x = 2>/3-2
sen135° sen30°
Ora, como o triângulo ABC é retângulo, segue que

AB ■Jl AB
sen60° = =.AB = 3->/3
2-75-2 2 “ 2(^-1)

141) Na figura, ABCD é um quadrado, ZABE = 15° , EF = AF . Mostre que o BDE


é isósceles.

Resolução:
Seja a a medida do lado do quadrado ABCD. Assim BD = aV2 , por ser uma
das diagonais do trabalho. No triângulo retângulo AEF, temos que
ZFAE = ZFEA = 45° . Como o ângulo do vértice A do quadrado ABCD mede
90°. segue que ZBAE = 90°+ 45° = 135°. No triângulo ABE, perceba que
ZAEB = 180° -135° -15° = 30° , conforme ilustra figura abaixo:
400 Tópicos de Matemática - Olimpíadas - IME - ITA

(l
A B
O
30^ x
E F
L
a

D a. C

Agora aplicando a lei dos senos ao triângulo ABE, segue que:


a 72
X
——— => x = x = aV2
sen135° sen30° 2
2
O que revela que o triângulo BDE é isósceles de base DE, pois
BD = BE = a%/2.

142) Dois círculos se intersectam nos ponto A e B. PQ é um segmento que passa


por A e termina em P e Q conforme a figura:

BP
Prove que — é constante.
Capitulo 3 - Resoluções 401

Resolução:
Sejam 2a e 2cp as medidas dos arcos AB e BA quando vistos na
circunferência menor e maior, respectivamente Como os ângulos
ZAPB e ZAQB são ângulos inscritos nas circunferências, segue que as suas
medidas sâo, em graus, metade das medidas dos arcos correspondentes, ou
seja, ZAPB = a e ZAQB = <p , conforme ilustra a figura seguir:

Aplicando a lei dos senos nos triângulos ABP e ABO e percebendo que
y + 0 = 180° => seny = sen0 , segue que:
BP AB ABseny
aABP ------- =• BP = (1)
sen? sena sena

AABQ -> -BQ. = => BQ = Alsen2 ABseny


(2)
sen0 sencp sencp sencp

Dividindo membro a membro as igualdades (1) e (2), segue que:


ABseny
BP sena sencp
BQ ABseny sena
sencp
Ora, como os ângulos a e q> são constantes, pois a corda AB (que é comum

as duas circunferências) tem tamanho constante independente da posição


. . „ BP sencp . , .
dos pontos P e Q, segue que — =----- — e constante, pois sena esencp
BQ sena
são constantes pelo fato dos ângulos a e <p serem constantes.
402 Tópicos de Matemática - Olimpíadas - IME - ITA

143) Prove que em um paralelogramo. a soma dos quadrados dos quatro lados é
igual à soma dos quadrados das diagonais.

Resolução:
Consideremos o paralelogramo ABCD da figura abaixo:

C
a
D

d
b
b
D i0° - a

A a B
Aplicando a lei dos cossenos aos triângulos ABC e ABD, segue que:

AABC d2 = a2 + b2 -2abcosa

AABD D2 = a2 + b2 - 2abcos(180°-a) = a2 + b2 +2abcosra


=-cosa
Adicionando membro a membro as duas últimas igualdades, segue que
D2 + d2=2(a2+b2)

144) No triângulo ABC da figura abaixo, determine a medida do lado BC.


Capitulo 3 - Resoluções 403

Resolução:
Aplicando a lei dos cossenos no triângulo ADE, temos:

122 = 82+ 102-2-8-10-cos A => cosA =—


8
Agora, aplicando a lei dos cossenos no triângulo ABC, segue que:
BC2 = 252 + 202 - 2 • 25 ■ 20 ■ cos A
= 625 + 400-1000 -
8
= 900

Ou seja, BC2 = 900 => BC = 30 .

OBSERVAÇÃO: Uma outra saida seria perceber que. nesse caso, há uma
proporcionalidade entre os lados dos triângulos ADE e ABC, o que revela que
eles são triângulos semelhantes. Vejamos:
AD 8 _2 AE 10 2
AC “ 20 " 5 e AB
6 25 5
Como o vértice A é comum aos dois triângulos, segue que eles são
semelhantes com razão de semelhança 2/5. Assim,
DE 2 12 2 „„ „„
— = — =>---- = — =— BC = 30.
BC 5 BC 5

145) Um corredor A está sobre uma reta (r) e corre sobre ela no sentido Ax. Um
corredor B não está em (r) e, correndo em linha reta, pretende alcançar A
conforme ilustra a figura abaixo:

(rl
404 Tópicos de Matemática - Olimpíadas - IME - ITA

Considerando que BÂX = 120°, VA = 8,0m/s e Vb = 873 m/s (módulos das


velocidades dos corredores A e B respectivamente). Determine a medida do
ângulo a que a trajetória de B deve fazer em uma reta para que o encontro
seja possivel, supondo que A e B partem simultaneamente.

Resolução:
Antes de qualquer coisa lembre-se que a distância percorrida num tempo t
caminhando com uma velocidade constante v é d=v.t. Assim, t segundos após
a partida simultânea de A e B as distâncias por eles percorridas são,
respectivamente, dA = 8t e dB=8>/3t (metros). Para que ambos cheguem

simultaneamente ao ponto X, como partiram num mesmo instante eles devem


levar 0 mesmo tempo T. Assim as distâncias percorridas por A e B são , em
metros, dA = 8T e dB = 8\/3T , formando então o triângulo ilustrado na figura

abaixo:

(’’) dA = 8.T X
A

120°

dD = 8V3.T
a

Usando a lei dos senos no triângulo ABX, segue que:


8T 8>/3T 1
---------- => sena - —
sena sen120°------------ 2

Ora, como nesse caso, 0° < a < 90° , segue que a = 30°.
Capítulo 3 - Resoluções 405

146) (OBM) As alturas de um triângulo medem 12, 15 e 20. Quanto mede o maior
ângulo interno desse triângulo?

Resolução:
Sejam a,b, e c as medidas dos lados do referido triângulo e 12, 15 e 20 as
medidas das suas alturas, respectivas as esses lados. Ora,
r. basex altura
como (ABC) = S =-------- --------- . sugue que

S
a=—
6
15 b 20 c 2S
b=—
2 2 2 15
S
c=—
10
Além disso, o maior ângulo de um triângulo sempre fica oposto ao maior lado,
que por sua vez é o lado que corresponde a menor altura (pois o produto dos
comprimentos do lado pela sua respectiva altura é constante e igual a 2S).
Assim, nesse caso, o maior lado do triângulo mede a, pois estamos supondo
que este lado é correspondente a menor altura, no caso 12. Diante do
exposto, aplicando a lei dos cossenos ao triângulo ABC. segue que:

a2 = b2 +c2 -2bccosA =>

rsV _(2sA22 r_s_Y|2_2.2S.S_cosa^


leJ “lisJ A10J 15 10

4S2 s2 2S2 .
-------- cosA =>
36 225 100 75
_1_ 4 1 2
------ cos A =>
36 225 100 75
_1_ 1 2 cos A =>
----------
36 36 75
2
—cos A = 0 => cos A = 0 =;■ A = 90°
75
406 Tópicos de Matemática - Olimpíadas - IME - ITA

147) Se A e C são ângulos agudos do triângulo ABC se A = 30° e a altura traçada


de A tem a mesma medida da mediana traçada de C, determine o ângulo
interno C.

Resolução:
Por M trace o segmento MN//AD. Ora, como M é, por hipótese, o ponto médio

do segmento AB (pois CM é a mediana partindo de C). segue pelo teorema

da base média de um triângulo (aplicado ao triângulo ABD) que MN = ^AD.

Sendo ZBCM = a , no triângulo BMN, segue que

MN ^AD jCM 1
sena = =— =— = — => a = 30°
CM CM CM 2
Capitulo 3 - Resoluções 407

Conforme ilustra a figura a seguir:

,30a

e
M

a
D N B C

Note que os triângulos ABC e MBC são semelhantes pois os seus ângulos
são dois a dois congruentes (o ângulo ZMBC é comum aos dois triângulos.
ZBAC = ZMCB = 30° e portanto o terceiro par de ângulos também é
constituído de ângulos congruentes, pois nos dois triângulos a soma das
medidas dos três ângulos é constante e igual a 180°). Assim se ZACD = <p .

então BMC = ZACD = <p .

AD
Por outro lado, no triângulo retângulo ACD. segue que sen<p = —. Como

AD
AD=CM, segue que sen<p = . Definindo ZAMC = o e aplicando a lei

dos senos ao triângulo ACM, segue que:


1
AC CM CM sen30° CM 2
senfl sen30° AC senfl AC senfl
408 Tópicos de Matemática - Olimpíadas - IME - ITA

AD
Ora, como sencp = —, segue que:
AC '
1
sencp = —-2— => sencp =-------- (D
sen0 2sen0
Novamente olhando para a figura acima podemos perceber que
o + <P = 180° => senO = sencp , o que revela que (1) pode ser reescrita como

1 1 2 1 V2
V2
sencp =-------- => senp =--------- =• sen cp = — => sencp = ± —
2sen0 2sencp 2 2

Ora, como 0° < cp < 90°. segue que sencp > 0, o que revela que

sencp = — => <p = 45°, que á medida do ângulo interno do vértice C do

triângulo ABC.

148) Uma circunferência inscrita num triângulo ABC toca AB no ponto D de modo
que AD = 5 e DB = 3. Encontre a medida de BC, se à = 60°.

Resolução:
Sejam D, E e F os pontos de tangência da circunferência com os lados do
triângulo ABC. Ora, como, por hipótese, AD = d e DB = 3, supondo que
CF = x, segue pelo teorema de Pitot, que AF = 5, BE = 3 e CE = x, como
ilustra a figura abaixo:
Capítulo 3 - Resoluções 409

Por fim, aplicando a lei dos cossenos ao triângulo ABC. segue que

(3 + x)2 = 82 + (5 + x)2-2 8 (5 +x) cos60° =>

9 + 6x + x2 = 64 + 25 + 10x + x2 -16 (5 + x) —

4x = 40 =s> X = 10

Portanto a medida do lado BC é 3 + 10 = 13.

149) Sobre os lados de um triângulo ABC retângulo de lados AC = 6cm.


AB = 6^3 cm e BC = 12cm construimos três quadrados externos. Calcule a
medida dos lados do triângulo determinado pelos centros desses quadrados

F G
410 Tópicos de Matemática - Olimpíadas - IME - ITA

Resolução:
Sejam JK = x, KL = y e LJ = z. Trace os segmentos CJ, CK, BK e BL.
conforme ilustra a figura abaixo:
E

D
K
H C

X
J >
i .
V < i /

z ~7T B
I

F G
Antes de qualquer coisa, olhando para o triângulo ABC, note que

„ 6x/3 Vã
senC -------= — => C = 60'
12 2

senB = — = - => B = 30°


12 2
Ora, como AC = 6, segue que a medida da diagonal do quadrado ACHI é
6j2 . Note que J sendo, por hipótese, o centro do quadrado ACHI, segue que

JC = JA = = 3 72 , pois JC e JA correspondem a metade da diagonal

desse quadrado. Analogamente, sendo AB = 6^3 , segue que a medida da


Capitulo 3 - Resoluções 411

diagonal do quadrado ABGF é 6-j3\Í2 = 6>/6 . Assim, LA = LB = = 3^6 .

pois LA e LB correspondem a metade da diagonal desse quadrado, visto que


o ponto L é , por hipótese, o centro desse quadrado. Por fim, sendo BC = 12.
segue que a medida da diagonal do quadrado BCED é 12V2 . o que revela

que KC = KB = = 6V2 . pois o ponto K é, por hipótese, o centro desse

quadrado.

Diante do exposto, segue que


z = JA + LA = 3%/2 + 3>/6 = 3 (J2 + >/ê)

Olhando para o triângulo CJK, temos que CJ = 3</2, CK = 6^2


e ZJCA = ZKCB = 45°, o que revela que ZJCK = 45° +60° + 45° = 150°
Mas,
x/3
cos150° = -cos30° =------
2
Assim, aplicando a lei dos cossenos ao triângulo CJK, segue que

x2 = (3s/2)2 + (6s/2)2 - 2■ 372 6V2 - J => x = 3>/l0* 4x^3

De modo completamente análogo podemos determinar a medida y. De fato.


BK = 6x/2 . BL = 3\/6e ZKBC = ZABL = 45°. o que revela que
ZJCK = 45° + 30° + 45° = 120°. Finalmente, aplicando a lei dos cossenos ao
triângulo KBL, segue que:

y2=(6j2)2 + (376)2- 2-6V2-3'/e(--} =. y = 3-Jl4 + 4s/3

150) Uma circunferência inscrita num triângulo ABC toca AB no ponto D de modo
que AD = 5 e DB = 3. Encontre a medida de BC. se  = 60°.
412 Tópicos de Matemática - Olimpíadas - IME - ITA

Resolução:
Sejam D, E e F os pontos de tangência da circunferência com os lados do
triângulo ABC. Ora, como, por hipótese, AD = d e DB = 3, supondo que
CF = x, segue pelo teorema de Pitot, que AF = 5, BE = 3 e CE = x, como
ilustra a figura abaixo:

Por fim. aplicando a lei dos cossenos ao triângulo ABC, segue que

(3 + x)2 = 82 + (5 + x)2 - 2 ■ 8 ■ (5 + x) ■ cos 60° =>

9 + 6x + x2 = 64 + 25 + 10x + x2-16-(5 + x)-^=>

4x = 40 => x = 10

Portanto a medida do lado BC é 3 + 10 = 13.


Capitulo 3 - Resoluções 413

151) No triângulo ABC (obtusângulo) da figura abaixo, determine a medida b do


lado AC.

Resolução:
Aplicando a lei dos senos no triângulo ABC, segue que
48 27
9sen(3ci) = 16sena
sen (3a) se na

Por outro lado, sen (3a) = 3sena -4sen3a , Assim,

9sen(3a) = 16sena => 9^3sena - 4sen3aj = 16sena =>

36sen3a -11sena = 0 => sena (36sen2a -11l) = 0

O que revela que sena = 0 ou 36sen2a -11 = 0.

Se sena = 0 => a = 0° ou a = 180°. o que não convém ao problema, pois


esses ângulos não podem ser ângulos internos de um triângulo.
, 1
Então 36sen^a -11 = 0 sena =----- . o que revela que
6

cosa = ±Vl-sen2a = ±11-^ = + 5


V 36 ~6
Vamos analisar então os dois casos:
414 Tópicos de Matemática - Olimpíadas - IME - ITA

5
0 COSa = -
6
Nesse caso aplicando a lei dos cossenos ao triângulo ABC, segue que:

272 = b2 +482-2 b-48 — 729 = b2 +2304-80b =>


6

b' = 45
b2 - 80b + 1575 = 0 =>
b" = 35
Vamos verificar se esses valores são compatíveis com o problema. Ora, O
triângulo ABC tem lados 48, 27 e b Assim, se b = 45, temos que 48 < 27 +
45. o que é verdadeiro e portanto a desigualdade triangular é satisfeita. Mas,

por outro lado, 482 < 452 + 272, o que revela que nesse caso o triângulo ABC
não é obtusãngulo.
Se b = 35, temos 48 < 27 + 35, o que é verdadeiro e portanto a desigualdade
triangular é satisfeita. Nesse caso, 482 >352 + 272, o que revela que o
triângulo ABC é obtusãngulo, o que satisfaz o enunciado.
Portanto, b = 35.
5
COS a =-----
6
Nesse caso aplicando a lei dos cossenos aos triângulo ABC, segue que:

272 = b2 + 482 - 2 b • 48 f- —| => 729 = b2 + 2304 + 80b


V 67

b' = -45
b2 + 80b +1575 = 0 =>
b" = -35
Que são valores incompatíveis com o problema pois b>0, uma vez que b
representa a medida do lado de um triângulo.

152) Depois de uma pequena discussão, Paulinho Carlos e Ari seguiram cada um
o seu caminho, em direções de 120° uma com a outra. Suas velocidades
estavam na razão 1:2:4. Prove que, em qualquer instante, suas posições são
os vértices de um triângulo retângulo.
Capítulo 3 - Resoluções 415

Resolução:
Sejam k, 2k e 4k as velocidades de Carlos (C). Paulinho(P) e Ari(A),
respectivamente Após um tempo t, supondo que os três partiram
simultaneamente de um mesmo ponto P, as distâncias por eles percorridas
serão d = kt, 2d = 2kt e 4d = 4kt, respectivamente (distância = velocidade x
tempo), conforme ilustra a figura abaixo:

Aplicando a lei dos cossenos aos triângulos , APQ, ACQ e CPQ, segue que:

AP2 = (2d)2 + (4d)2 -2-2d-4d cos120° => AP2 = 28d2

AC2 = (4d)2 + (d)2-2 4ddcos120°=> AC2 = 21d2

CP2 = (d)2 + (2d)2 - 2 ■ d ■ 2d ■ cos 120° => AC2 = 7d2

Agora perceba que AP2 = AC2 + CP2, o que revela que o triângulo ACP é
retângulo em C.
416 Tópicos de Matemática - Olimpíadas - IME - ITA

153) (Teorema de Napoleão) Dado um triângulo ABC, construímos sobre cada um


dos seus lados um triângulo equilátero conforme ilustra a figura abaixo:

I
Mostre que o triângulo cujos vértices são os centros dos triângulos equiláteros
que foram construídos sobre os lados do triângulo original também é um
triângulo equilátero.

Resolução:
Seja ABC um triângulo qualquer. Fazendo as construções do enunciado e
lembrando que o centro de cada triângulo construído é o seu baricentro,
teremos a situação abaixo:
Capitulo 3 - Resoluções 417

Aplicando a Lei dos Cossenos ao triângulo ABC para o ângulo cc, teremos

b2 + c2-a2
a2 = b2 + c2 - 2b c cosa => cos a =
2bc
Por outro lado,

(ABC) = ^bcsena => sena = 2(ABC)


bc
Assim,
cos(60° + a) = cos 60° ■ cosa - sen60°sena
= 1 f b2 + c2-a2'l 75 2(ABC)
“ 2\. 2bc ) 2 bc
[b2 4-c2 - a2) - 4\/3 (ABC)
4bc

h /q
Note que AM = , pois tal segmento vale 2/3 da altura do triângulo

equilátero de lado b, ou seja,


2 2 bV3 bj3
AM - —h = —
3 3 2 ” 3

c73
Analogamente, temos que AN = ——

Agora aplicaremos a Lei dos Cossenos ao triângulo AMN, segue que

MN2 = AM2 +AN2-2-AMANcos(60° + a) =>

_2 bV3 cj3 (b2+c2-a2)-4^(ABC)


3 3 4bc

!a2+b2 + c2 + 4 73 (ABC)
MN =
6
418 Tópicos de Matemática - Olimpíadas - IME - ITA

Procedendo de modo completamente análogo para os segmentos MP e PN,


pode-se mostrar que

'a2 + b2 + c2 + 4^3 (ABC)


MP =
6

Ia2 + b2 + c2 + 4x/3 (ABC)


PN =
6

o que revela que MN = MP = PN e, portanto, o triângulo MPN é equilátero,


como queríamos demonstrar.

QUADRILÁTEROS INSCRITÍVEIS

154) Na figura abaixo determine a medida do ângulo a sabendo que ABC é um


triângulo retângulo em A, AM é bissetriz interna do ângulo do vértice A e MN é
perpendicular à hipotenusa.

Resolução:
Ora. como AM é bissetriz interna do ângulo do vértice A do triângulo ABC,
90°
segue que ZBAM = ZMAN = = 45°. Por outro lado perceba que o

quarilátero ABMN é inscritivel pois a soma dos ângulos internos dos vértices
A e M é 180°, pois cada um desses ângulos mede 90°, conforme ilustra a
figura a seguir:
Capitulo 3- Resoluções 419

„'M c

Por fim, perceba que na circunferência que foi traçada pelos vértices do
quadrilátero ABMN, arco MN é correspondente aos ângulos inscritos ZMBN
e ZMAN = 45° . Portanto,
a = ZMBN = ZMAN = 45°

155) Mostre que um trapézio é inscritivel em uma circunferência se. e somente se.
ele é isósceles.

Resolução:
(=>) Imagine um trapézio ABCD, de bases AB e CD e que é inscritivel. Se

ZABD = a, segue pelo fato do quadrilátero ABCD ser cíclico que

ZDCE =a . Ora, como CD//AB'(pois o quadrilátero ABCD é um trapézio de

bases AB e CD), segue que ZCBA = ZDCE = a , o que revela que o trapézio

ABCD é isósceles, pois apresenta os ângulos da base congruentes, como

ilustra a figura a seguir:


420 Tópicos de Matemática - Olimpíadas - IME - ITA

(<=) Suponhamos agora que ABCD seja um trapézio isósceles de bases AB e

CD, conforme ilustra a figura abaixo:

D C

a a
A B
Ora, como CD//AB, segue que ZADC = ZBCD = 180“-a , o que revela que
os ângulos opostos (dos vértices A e C e dos vértices B e D) do quadrilátero
ABCD são suplementares e portanto o quadrilátero ABCD é ciclico
(inscritivel).
Capitulo 3- Resoluções 421

156) Prove que todo paralelogramo inscritivel em uma circunferência é uma


circunferência é um retângulo.

Resolução:
De fato, num paralelogramo os ângulos de vértices opostos são iguais,
digamos a a. por exemplo. Ora, se o paralelogramo ABCD for inscritivel a
soma dos ângulos opostos é 180°. Assim, 2a = 180° => a = 90° . O mesmo
ocorrerá para os dois outros ângulos opostos e portanto os quatro ângulos
serão retos. Ora, um paralelogramo com os quadro ângulos retos é. de fato
um retângulo.

157) Na figura, o ponto C divide o arco ACB ao meio. Mostre que o quadrilátero
DEFG é inscritivel em uma circunferência.

Resolução:
Sejam 2a a medida, em graus, dos arcos AC e CB, 2p a medida, em graus,
do arco AD e por fim 20 a medida, em graus do arco BE. Ora, como a

i circunferência completa possui 360°, segue que o


360°- 2[! - 20 -4a , conforme ilustra a figura a seguir
arco DE mede
422 Tópicos de Matemática - Olimpíadas - IME - ITA

C
2a 2a

A F/
G
B

20
23
a+0
E

360° - 20 - 20 - 4a

Como o ângulo ZCED é um ângulo inscrito na circunferência, segue que a

sua medida, em graus, é a metade da medida, em graus, do arco

correspondente. Assim,

ZCED = ^±2P=£W)=a + p
2 2

Por fim como o ângulo ZDFG é um ângulo excêntrico interno, segue que

= = (360°-2P-29 - 4g-r 29) + 2g


= 180°-(a + p)
2

O que revela que os ângulos internos dos vértices E e F do quadrilátero

DEGF são suplementares e portanto o quadrilátero DEGF é um quadrilátero

cíclico (inscritivel). como queríamos demonstrar.

!
i
Capitulo 3- Resoluções 423

158) Os lados BC e AD de um quadrilátero ABCD são paralelos. Um circulo


encontra o lado AB em E e o lado CD em F. Prove que o quadrilátero AEFD é
cíclico.

Resolução:
De fato, como o quadrilátero BCFE é cíclico, segue que ZBCF = ZAEF = a .

Por outro lado como BC//AD, segue que os ângulos internos dos vértices C e

D do trapézio ABCD são suplementares, o que revela que

ZBCF = ZADF = 180°-ZBCF = 180° -a . Assim, olhando para o quadrilátero

ADFE os ângulos internos dos vértices D e E são suplementares, pois

ZADF = a e ZADF = 180°-a, o que revela que o quadrilátero ADFE é

cíclico, pois como sabemos um quadrilátero é cíclico se, e somente se os

seus ângulos opostos são suplementares.


424 Tópicos de Matemática - Olimpíadas - IME - ITA

159) Seja P o centro do quadrado construído sobre a hipotenusa AC do triângulo


retângulo ABC Prove que BP é bissetriz do ângulo ZABC.
A E

Resolução:
De fato, como ACDE é um quadrado e AD e CE são suas diagonais, segue
que ZAPC = 90°, visto que as diagonais de um quadrado são sempre
perpendiculares. Assim, o quadrilátero ABCP é inscritível pois tem dois
ângulos opostos suplementares, traçando a circunferência circunscrita ao
quadrilátero ABCP, conforme ilustra a figura abaixo

Ora, como ZPAC = ZPCA = 45° (pois as diagonais de um quadrado são

bissetrizes dos ângulos dos seus vértices), segue que os arcos AP e PC


Capítulo 3 - Resoluções 425

medem cada um 90°, pois sendo os ângulos ZPAC e ZPCA inscritos na

circunferência as medidas dos arcos correspondentes são, em graus, o dobro

da medida do ângulo. Por fim, perceba que os ângulos ZABP e ZCBP

também são ângulos inscritos na circunferência que “enxergam" os arcos AP

90°
e PC, respectivamente. Assim, ZABP = ZCBP = = 45° (mais uma vez

estamos usando o fato de que a medida de um ângulo inscrito numa

circunferência é, em graus, a metade da medida do arco correspondente).

Ora, se ZABP = ZCBP = 45°. segue que Prove que BP é bissetriz do

ângulo zABC , como queriamos demonstrar.

160) Na figura abaixo, calcule a medida do ângulo □, sabendo que ABCDE é um


pentágono onde B = D = 90°, AB = BC = CD = DE e que M é o ponto médio
do lado AE.
426 Tópicos de Matemática - Olimpíadas - IME - ITA

Resolução:
Suponhamos que AB = BC = CD = DE = a, tracemos as diagonais CE e CA
do pentágono ABCDE, assim como o segmento MC.

Como os triângulos ABC e CDE são retângulos, pelo teorema de Pitágoras


aplicado a esses triângulos segue que AC = CE = aV2 , o que revela que o
triângulo ACE é um triângulo isósceles de base AE. Ora, como o ponto M é,
por hipótese, ponto médio do lado AE, segue que o segmento MC é mediana
e portanto altura referente à base AE (num triângulo isósceles a medida e a
altura relativa a base coincidem). Diante do exposto temos que
ZEMC = ZAMC = 90°, o que revela que os quadriláteros ABCM e CDEM são
inscritíveis, pois cada um deles possui um par de ângulos opostos que
somam 180°. Note também que os triângulos ABC e CDE são triângulos
retângulos isósceles, o que revela que cada um dos seus ângulos agudos são
de 45° e portanto os arcos BC e CD medem 90° cada um pois os ângulos
ZBAC e ZCED são ângulos inscritos nas circunferências ( e como sabemos
a medida do arco é, em graus, o dobro da medida do correspondente ângulo
inscrito). Por fim perceba que nas duas circunferências os ângulos ZBMC e
ZCMD são ângulos inscritos que "olham” respectivamente para os arcos BC
e CD de 90° cada um, que revela que ZBMC = ZCMD = 45°, visto que a
Capítulo 3 - Resoluções 427

medida de um ângulo inscrito é a metade da medida do arco que ele


"enxerga".
Todas essas informações estão resumidas na figura abaixo:

45°
45°
í 90°

Por fim, perceba que a = ZBMD = 45° + 45° = 90° .

161) Uma variante bem mais difícil do problema anterior é o seguinte: Na figura
abaixo, calcule a medida do ângulo a, sabendo que ABCDE é um pentágono
onde B = D = 90°, AB = BC, CD = DE e que M éo ponto médio do lado AE.
428 Tópicos de Matemática - Olimpíadas - IME - ITA

Resolução:
Na verdade esse problema é um “disfarce” de um famoso teorema chamado
de "Teorema de Von Aubel” cujo enunciado é o seguinte:
Dado um triângulo qualquer ABC, se sobre os lados AB e AC construirmos
quadrados de lados AB e AC, sendo P e Q são os centros desses quadrados
e M o ponto médio do lado BC do triângulo ABC, então os segmentos MP e
MQ são congruentes e ortogonais, conforme ilustra a figura abaixo:

Demonstração (do teorema):


Apenas para facilitar a escrita sejam 2a, 2b e 2c as medidas dos lados BC.
AC e AB respectivamente. Inicialmente marque os pontos N e R. pontos
médios dos segmentos AB e AC respectivamente. Agora perceba que
PN = NA = c e que QR = RA = b. conforme ilustra a figura abaixo:
Capitulo 3 - Resoluções 429

Agora perceba que MR é paralelo a AB e MR = c, pois MR é a base média do


triângulo ABC em relação â base AB, que mede 2c. De modo completamente
análogo, perceba que MN é paralelo a AC e MN = b. pois MN é a base
média do triângulo ABC em relação a base AC, que mede 2b. Diante do
exposto o quadrilátero ANMR é um paralelogramo, Sendo ZBAC = a , segue
pelo paralelismo dos lados do quadrilátero ANMR que ZBNM = zCRM = a .
conforme ilustra a figura abaixo:

Para finalizar perceba que os triângulos PNM e QRM são congruentes pelo

caso LAL (PN = RN, ZPNM = ZQRM = 90° + a e NM = QR) e portanto da

congruência dos triângulos PMN e QRM segue que MP = MQ e além disso

QR _L MN e RM ± PN , o que implica que MP ± MQ , ou seja, ZPMQ = 90°.

Visto o teorema de Aubel, voltemos à questão original...


430 Tópicos de Matemática - Olimpíadas - IME - ITA

No pentágono dado no enunciado da questão tecemos as diagonais AC e EC.


conforme ilustra a figura abaixo:

Agora perceba que sendo AB=BC e ZABC = 90°, podemos imaginar o ponto
B como o centro de um quadrado construído sobre o lado AC do triângulo
ACE e com lado AC e de modo completamente análogo, sendo ED = DC e
ZEDC = 90°, podemos imaginar o ponto D como o centro de um quadrado
construído sobre o lado CE do triângulo ACE e com lado CE, conforme ilustra
a figura abaixo:
G

A c

11

E
D

Portanto pelo teorema de Aubel, segue que MB = MD e MP J. MQ => a = 90’.


Capitulo 3 - Resoluções 431

162) ABCD é um quadrado cujas diagonais cortam-se no ponto 1. Constrói-se


exteriormente um triângulo equilátero ABM. Calcule a medida do ângulo ZAIJ,
sabendo que J é o ponto médio do lado AM.

Resolução:
Inicialmente tracemos o segmento BJ conforme ilustra a figura abaixo:

C a
B

a a M
a
a/2
J
a/2
D A
a

Seja a e medida do lado do quadrado ABCD e do triângulo equilátero ABM.


Ora, como o triângulo ABM é equilátero e o segmento BJ é mediana, segue
que o segmento BJ é bissetriz do ângulo interno do vértice B do triângulo
ABM. Assim, ZABJ = 30° . Por outro lado, como o triângulo ABM é equilátero
o segmento BJ também é altura e portanto, ZAJB = 90° . Como ZAIB = 90°
(medida do ângulo formado pelas diagonais de um quadrado), segue que o
432 Tópicos de Matemática - Olimpíadas - IME - ITA

quadrilátero AJBI é inscritível visto que ZAJB-ZAIB = 180°, conforme ilustra


a figura abaixo:

C a
B

30\ a

a M

a/2

D A
a
Diante do exposto, podemos perceber que os ângulos ZABJ e ZAIJ
“enxergam" o mesmo arco na circunferência exibida na figura acima. Assim,
a = ZAIJ = ZABJ = 30° .

163) (IME)Quatro retas intersectam-se formando quatro triângulos conforme a


figura abaixo. Prove que as circunferências circunscritas aos quatro triângulos
possuem um ponto em comum.
Capítulo 3 - Resoluções 433

Resolução:
Sejam O, e tyas circunferências circunscritas aos triângulos ABF e DEF,

respectivamente. Ora, como Q, e Qj "passam" pelo ponto F, será que F é o


único ponto em comum a essas duas circunferências? Vamos mostrar que
nâo. De fato, se F fosse o único ponto em comum as circunferências n, e

isso implicaria que e íl3 seriam circunferências tangentes em F e


nesse caso AF e FE seriam os diâmetros dessas dias circunferências e
portanto os triângulos ABF e DEF seriam triângulos retângulos tais que
ZABF = 90° e ZDEF = 90° o que não é possível, pois se assim fosse
teriamos zBEC - 90° e então, no triângulo BCE teriamos,
ZBEÇ + ZDEF + ZBCE = 180° => ZBCE = 0°
.90“ =90"

o que seria um absurdo, pois o ângulo ZBCE é um dos ângulos internos o<_
triângulo BCE e portanto não pode ser nulo. Assim o ponto F nâo pode ser o
único ponto em comum das circunferências n, e . Seja P outro ponto de

interseção de Q, e , como ilustra a figura abaixo:

A ",
434 Tópicos de Matemática - Olimpíadas - IME - ITA

Agora perceba que o quadrilátero ABFP é inscritível e portanto


ZBAF = ZBPF = a. pois esses dois ângulos "enxergam" o mesmo arco 8F
em Q,, conforme ilustra a figura abaixo:

Por outro lado o quadrilátero DEPF também é inscritível. Assim, se


ZEDF = <p, segue que:

ZEPF = 180° - ZEDF = 180° - <p = ZADC

Assim,

ZBPE = ZBPF + ZEPF = a + (180° - <p) = 180° + a - <p

Por outro lado no triângulo ACD, temos que


a + (180° - <p) + ZACD = 180° => ZACD = <p - a

Agora perceba que o quadrilátero BCEP é inscritível, pois


ZBPE + ZBCD = (180° + cr - <p) + (<p - a) = 180°
Capítulo 3 - Resoluções 435

A figura abaixo resume o que foi exposto até esse momento:

Ora, como o quadrilátero BCEP é inscritivel, segue que o ponto P pertence a


circunferência circunscrita ao triângulo BCE, como ilustra a figura abaixo:

Resta mostrar que o ponto P também pertence a circunferência Q,.


circunscrita ao triângulo ACD e para isso vamos mostrar que o quadrilátero
ACDP também é inscritivel. De fato, olhando para o quadrilátero inscritivel
ABFP, temos que
ZAPF = 180° - ZABF = ZBCF = 0
Por outro lado no quadrilátero inscritivel DEPF, se ZEPF = (t. segue que

ZDEF = ZEPF = p . pois esses dois ângulos "enxergam" o mesmo arco DF

em Q? , conforme ilustra a figura a seguir:


436 Tópicos de Matemática - Olimpíadas - IME - ITA

Assim,
ZAPD = ZAPF + ZDPF = 0 + p

Por outro lado, no triângulo BCE temos que 0 + p + ZBCE = 180°, o que
revela que os ângulos opostos dos vértices P e C do quadrilátero APDC são
suplementares, pois ZAPD=6+p e ZBCE são tais que

0+p+ZBCE = 180°. Assim, o ponto P pertence a circunferência circunscrita


ao triângulo ACD.
Diante do exposto, o ponto P pertence as circunferências circunscritas aos
triângulos ABF, DEF, BCD e ACD, sendo pois, o ponto comum a essas quatro
circunferências como ilustra a figura abaixo:

' ’< ÍÍ3


Capítulo 3 - Resoluções 437

164) Quatro circunferências de mesmo raio possuem um ponto O em comum,


conforme ilustra a figura abaixo:

Sabendo de os segmentos AB, BC, CD e DA são tangentes a cada duas


circunferências (conforme a figura acima), mostre que o quadrilátero ABCD é
cíclico (inscritível).

Resolução:
Marque os centros A'. B', C e D’ das quatro circunferências, pontos de
tangência das circunferências com os lados AB, BC, CD e DA do quadrilátero
ABCD e os respectivos raios nos pontos de tangência, como ilustra a figura
abaixo

Ora, como o ponto O é um ponto comum as quatro circunferências, segue


que OA' = OB’ = OC = OD' = r, o que revela que os pontos A'. B’. C e D'
438 Tópicos de Matemática - Olimpíadas - IME - ITA

pertencem a uma mesma circunferência, o que equivale a dizer que o


quadrilátero A'B'C'D' é um quadrilátero cíclico. Por outro lado perceba que
B'C7/BC pois as distâncias dos pontos B' e C ao segmento BC são iguais a r.
o que revela que ZABC = ZA'B'C = a e ZDCB = ZD'C'B'= p.
Analogamente, A’D7/AD. ZBAD = ZB'A'D’= 0 e ZCDA = ZC'D'A'=<p.
Ora. como o quadrilátero A'B’C’D’ é cíclico, segue que os ângulos internos
dos vértices A' e C são suplementares, assim como os ângulos internos dos
vértices B' e D' ou seja,
ZB’A'D,+ ZD'C'B, = e + p = 180°
ZA'B'C'+ ZC'D'A' = a + <p = 180°
Assim.
ZBAD + ZDCB = ZB'A'D'+ZD'C'B' = 0 + p = 180°
ZABC+ZCDA = ZA'B’C'+ZC'D'A' = a + <p = 180°
O que revela que o quadrilátero ABCD é cíclico (inscritível).

165) Um quadrilátero convexo ABCD é formado pela justaposição de cinco


quadriláteros convexos menores conforme ilustra a figura abaixo:

Mostre que, se os cinco quadriláteros menores são cíclicos, isto é, cada um


deles pode ser inscrito num círculo, então o quadrilátero ABCD também é
cíclico.
Capitulo 3 - Resoluções 439

Resolução:
Para começar vamos nomear as medidas de todos os ângulos dos cinco
quadriláteros cíclicos, conforme ilustra a figura abaixo:

Lembrando que a condição necessária e suficiente para que um quadrilátero


seja cíclico é que a soma das medidas dos ângulos opostos seja 180°,.
Assim, para mostramos que o quadrilátero ABCD é cíclico basta mostrar que
(m + n) + (d + e) = 180° e (a+ p) + (h-M) = 180°

De fato, como os cinco quadriláteros menores são, por hipótese, cíclicos, p o


demos afirmar que:
a + b = 180°, c+d = 180°, e + g = 180°, f + h = 180°, m + j = 180’

l + i = 180°, n + q = 180°, p + o = 180°. u+s = 180°, t = 180°

Assim,
ím + j = 180° , , .
j ' => (m + n) + j + q = 360°
|n + q = 180° ' ’

/C + d = 180° =,(d + e) + c + g = 360°


|e + g = 180° ' ’
440 Tópicos de Matemática - Olimpíadas - IME - 1TA

Adicionando, membro a membro as duas últimas igualdades acima, segue


que
[(m + n) + j + q = 360°
(m+ n) + (d + e) +j + q + c+g = 720° (1)
|(d + e) + c + g = 360°

Por outro lado, olhando para os vértices do quadrilátero central, segue que:
j + s + g = 360°. t + b + f = 360°, q + u + c = 360° e l + o + r = 360°

Assim,
fj + s + g = 360°
(j + s + g) + (q + u + c) = 720°
q + u + c = 360°

(j + c + q + g) + (s + u) = 720° =>(j + c + q + g) = 540° (2)


=180°

Substituindo (2) em (1), segue que:


(m + n) + (d + e) + j+ q + c + g = 720° => (m -r n) + (d + e) = 180°
=540°

Combinando de modo análogo as equações que ainda não foram usadas,


demonstra-se que:
(a + p) + (h + i) = 180°

o que demonstra que o quadrilátero ABCD é cíclico.

166) Um quadrilátero ABCD é dito bicêntrico quando este é inscritivel e


circunscritivel. Prove que a área máxima de um quadrilátero bicêntrico com
perímetro fixo ocorre quando as duas circunferências são concêntricas.

Resolução:
De fato, como estamos supondo, por hipótese, que o quadrilátero ABCD é
inscritivel, temos que

(ABCD) = ,J(p-a)(p-b)(p-c)(p-d)
Capítulo 3 - Resoluções 441

Por outro lado, também, por hipótese, o quadrilátero ABCD é circunscritivel.o


que revela que
p-a = c
p -b = d
a + c = b + d = p=>
p-c = a
p-d = b

Logo, a medida da área do quadrilátero ABCD pode ser calculado por:


(ABCD) = ^(p-a) (p- b)(p- c)(p- d) = Vãbcd

Além disso, pela desigualdade entre as médias para quatro números reais
positivos, temos que:
a+b-c+d
Vabcd<
4

Ora, como 2p = a + b + c + d segue que

a+b+c+d
____ s
í/abcd 4
------------ =>_ 'A/ãbcd ABCD) *< y
V(ABCD) s | =• ((ABCD)
O que releva que a valor máximo da medida da área do quadrilátero ABCD é
2
— e além disso, a área máxima ocorre se. e somente se, a = b = c - d.
4

Logo, o quadrilátero que queremos é um quadrado. Num quadrado ambas


circunferências, inscrita e a circunscnta, tem centro no ponto encontro de
suas diagonais, que completa a resolução do problema.

167) Sejam ABC um triângulo e M o ponto médio do lado SC. Se D. E são os pés
das alturas relativas aos lados AC, AB. respectivamente, prove que o
quadrilátero BCDE é cíclico e que M é o centro a circunferência circunscrita a
esse quadrilátero.

Resolução:
De fato, observando a figura abaixo pode-se perceber que o segmento BC
"enxerga” dois ângulos retos; um no vértice E (pois CE é altura relativa ao
442 Tópicos de Matemática - Olimpíadas - IME - ITA

lado AB do triângulo ABC) e outro no vértice D (pois BD è altura do triângulo


ABC em relação ao lado AC), o que revela que o quadrilátero BCDE é cíclico.

Para provarmos que o ponto M é o centro da circunferência circunscrita ao


quadrilátero BCDE, o fato primordial aqui é lembrar que num triângulo
retângulo a mediana relativa a hipotenusa tem a metade do tamanho da
hipotenusa. Na figura acima perceba que ME é mediana relativa à hipotenusa
do triângulo retângulo BEC (o ângulo do vértice E é reto pois CE é altura
relativa ao lado AB). Assim,
ME = BM = MC (1)

Analogamente, perceba que MD é a mediana relativa a hipotenusa do


triângulo retângulo BCD, o que revela que:
MD = BM = MC (2)

De (1) e (2) podemos então concluir que ME = MD, o que revela que MD =
ME = MB = MC e portanto os pontos B,C, D e E estão a uma mesma distância
de M e pertencem a uma mesma circunferência (pois, como já provamos o
quadrilátero BCDE é cíclico), o que nos permite concluir que o ponto M é o
centro dessa circunferência.
Capitulo 3 - Resoluções 443

ÂNGULOS NA CIRCUNFERÊNCIA

168) Considere um triângulo ABC e pontos X. Y e Z nos lados BC. AC e AB.


respectivamente tais que A, B, C, X, Y e Z são todos distintos entre si. Mostre
que as circunferências circunscritas aos triângulos AYZ, BZX e CXY têm um
ponto M em comum (esse ponto é chamado de ponto de Miquel do triângulo
ABC).

Resolução:
Seja Q o ponto de interseção das circunferências circunscritas aos triângulos
AYZ e BXZ, conforme ilustra a figura abaixo:
444 Tópicos de Matemática - Olimpíadas - IME - ITA

Vamos provar que a circunferência circunscrita ao triângulo CXY também


passa pelo ponto Q e portanto o ponto Q é um ponto comum as três
circunferências. De fato, note que os quadriláteros AZQY e BXQZ são
inscritiveis, portanto ZQYC = ZAZQ ZQXC = zBZQ = p conforme
ilustra a figura abaixo:

Como ct + p = 180°, segue que o quadrilátero XCYQ é inscritível e portanto a


circunferência circunscrita ao triângulo CXY também passa pelo ponto Q, que
é portanto um ponto comum as circunscritas aos triângulos AYZ, BZX e CXY.
(O ponto Q é chamado de ponto de Miquel do triângulo ABC)

169) Na figura abaixo a reta r é tangente à circunferência e é perpendicular a reta s


no ponto P. determine a medida do ângulo ZPBC sabendo que o ângulo
ZACB mede 40°

p c (r)
Capítulo 3 - Resoluções 445

Resolução:
Como a reta (r) é tangente á circunferência no ponto C o raio da
circunferência e a reta (r) são perpendiculares no ponto c. Traçando-se o
diâmetro CE, segue do fato de que zBPC = ZPCE = 90°, que BP//CE e
portanto os ângulos ZECB e ZABC são alternos internos, o que revela que
ZECB = ZABC = a. Além disso o ângulo ZACP é um ângulo de segmento
que "enxerga" o mesmo arco AC que o ângulo . Como ZABC = n . segue
que o arco AC mede 2a e portanto ZACP = a, conforme ilustra a figura
abaixo:

2a
_ZL
p C (r)

Finalmente, como ZECP = 90°, segue que

a + 40° + a = 90° =5- 2a = 50° => a = 25° .

170) Demonstre o teorema de Hiparco.


Para qualquer quadrilátero inscritivel, a razão entre as diagonais é igual a
razão da soma dos produtos dos lados que concorrem com as respectivas
diagonais".
446 Tópicos de Matemática - Olimpíadas - IME - ITA

.. r . m ad + bc
Na figura abaixo, — =
ab + cd

Demonstração:
Sejam M e N os pontos sobre a circunferência circunscrita ao quadrilátero
ABCD tais que os arcos AM e CD sejam congruentes e os arcos DN e AB
sejam congruentes. Conforme ilustra a figura abaixo:

B b c

m
ai
n

N
Capitulo 3 - Resoluções 447

Dessa forma.

AM = CD, pois os arcos AM e CD são iguais e arcos iguais implicam


segmentos iguais.
DN = AB, pois os arcos AM e CD são iguais e arcos iguais implicam
segmentos iguais.
BM = CN, pois temos a igualdade dos arcos BM = BA + AM = ND + CD
= NC = CN.
MC = BN = AD, pois temos a igualdade dos arcos MC e BN.
Agora aplicando o teorema de Ptolomeu aos quadriláteros MABC e
NBCD, segue que

BMm = MCa +AMb


CN-n = BN-c + ND-b

Dividindo as duas últimas igualdades membro a membro, segue que:


BM m MC-a +AM b m da -cb m ad + bc
CN-n BN c + ND b n dc + ab n ab + cd
448 Tópicos de Matemática - Olimpíadas - IME - ITA

171) Determine os comprimentos das diagonais de um quadrilátero inscritivel


ABCD cujos lados medem a, b, c e d.

Resolução:
Sejam m en os comprimentos das diagonais, conforme ilustra a figura abaixo:

Pelo teorema de Ptolomeu temos que:


mn = ac + bd
e pelo teorema de Hiparco, temos que:
m ad + bc
n ab + cd
Assim,

□ m i + bd)• ( ----
ad + bc' j(ac + bd)(ad + bc)
m = mn- — = (ac
n ' ; lab + cd. V ab + cd

n2 = mn- — = (ac+bd).f^É') /(ac+ bd) (ab + cd)


m ' ' l ad + bcJ V ad + bc

172) No interior de um polígono regular de n lados de comprimento L, estão


situados n círculos , todos de raio r.cada um tangente a dois círculos vizinhos
e a dois lados consecutivos do polígono, Calcule, em função de L e e de n. o
valor de r
Capitulo 3 - Resoluções 449

Resolução:
O enunciado nos permite montar a seguinte figura: (Note que o ângulo AOB

mede — e portanto MOB mede — )


n n

No triângulo OPN, no triângulo MOB.

taK=
n
L/2
r+x

Assim,
í rsen .'l

t9fâ = ^tg
L
2 L KL ••(3
r+x

SenÈ)
2
senK, n

ÊL a
r rsen í
L=2
• sen!
I
=>L = 2r
senK) + ?
senK, cosS cos0 .
450 Tópicos de Matemática - Olimpíadas - IME - ITA

173) (CRUX-MATHEMATICORUM) No interior de um circulo de raio R são


construídos dois círculos menores de raios a e b, conforme ilustra a figura
abaixo:

Se os dois círculos menores são tangentes ao círculo maior nos pontos P e Q


e os dois círculos menores intersectem-se nos pontos S e T e além disso os
pontos P, S e Q são colineares, mostre que R = a + b.

Resolução:
Sejam O o centro do círculo maior cujo raio é igual a R, Oi o centro do circulo

menor de raio a que tangencia o circulo maior no ponto PeOjo centro do


circulo menor de raio a que tangencia o circulo maior no ponto Q. Como os
círculos centrados em O, e em O2 são tangentes ao circulo centrado no ponto

O, segue que os pontos O, O-, e P são colineares assim como os pontos O,

O2 e Q também são colineares.

Note que o triângulo OPQ é isósceles pois OP = OQ = R. o triângulo C^PS é


isósceles, pois OtP = O2S = a, assim como o triângulo O2SQ também é
isósceles, pois O2S = O2Q = b. Assim,
ã
No triângulo OPQ, ZOPQ = ZOQP = a
No triângulo O<PS, ZO,SP = ZO,PS = ZOPS = a

No triângulo O2PS, ZO2SQ = ZO2QS = ZOQS = a


Capitulo 3 - Resoluções 451

Conforme ilustra a figura abaixo:

Por fim, perceba igualdades ZO,SP = ZOQS = a e

ZO2SQ = ZOPS = a revela que o quadrilátero 00,302 é um paralelogramo

e portanto seus lados opostos sâo congruentes. Assim,


OO, = O2S => R — a — b => R - a + b

174) Seja M o centro da semicircunferência abaixo, determine a .

Resolução:
Inicialmente trace os segmentos MC e DC. Perceba que MA = MC = R (raio),
o que revela que o triângulo ACM é isósceles de base AC e portanto.
ZACM = ZMAC = 10°. Note também que o arco BC mede 20°, pois o arco
BC corresponde ao ângulo inscrito ZBAC = 10° e, como sabemos a medida
452 Tópicos de Matemática - Olimpíadas - IME - ITA

de um arco, em graus, é o dobro da medida, em graus, do ângulo inscrito


correspondente. Note que o ângulo ZCDB é um ângulo inscrito que
20°
“enxerga” o arco BC de 20°, o que revela que ZCDB = — = 10°, pois a

medida de um ângulo inscrito, em graus, é a metade da medida, em graus do


arco correspondente. Além disso, no triângulo AMF, temos
ZAMD = 180°-10°-100° = 70°, o que revela que o arco AD também mede
70°, pois, como sabemos, a medida de um arco, em graus, é igual a mediada,
em graus, do ângulo correspondente. Assim o arco CD mede
180°-70°-20° = 90° Ora, como o triângulo CDM é retângulo e isósceles
segue que os seus ângulos agudos mede cada um 45°. Ora, como
ZACM = 10° e ZCDB = 10°, segue que ZBDM= ZDCF = 45°-10° = 35°,
conforme ilustra a figura abaixo:

90°
D

--
35°
70°

35°’ C
G
F
20°

100°^-"
70°,
A M B

Para finalizar, perceba que a é a medida de um ângulo externo do triângulo


CDG, o que nos permite afirmar, pelo teorema do ângulo externo, que:
a = 35° +10° = 45°
Capitulo 3 - Resoluções 453

175) Sejam AB, BC dois lados adjacentes de um polígono regular de 9 lados


inscrito num circulo de centro O, conforme ilustra a figura abaixo.

Sejam M o ponto médio de AB e N o ponto médio do raio OT perpendicular a


BC. Determine a medida, em graus, do ângulo ZOMN = a .

Resolução:
Na figura abaixo perceba que O arco AB mede 360°/9 = 40°. e que o arco BT
mede 40°/2 = 20°, visto que o ponto T é o ponto médio do arco BC. Portanto
ZAOT = 60° (pois é a soma das medidas dos arcos AB e BT). Como AO =
OT (raio da circunferência), segue que o triângulo AOT é isóscels e como
possui um ângulo de 60° é, na verdade equilátero, pois
180°-60°
ZOAT = ZOTA = = 60°
2

Por fim trace o segmento NA, que é mediana e portanto altura do triângulo
equilátero AOT, conforme ilustra a figura a seguir:
454 Tópicos de Matemática - Olimpíadas - IME - ITA

Diante destas informações concluímos que A, M. N e O são vértices de um


quadrilátero cíclico (mscritível numa circunferência), visto que os triângulos
ANO e AMO são retângulos de hipotenusa AO (que é o diâmetro da
circunferência na qual o quadrilátero está inscrito), conforme ilustramos a
seguir:

Assim, ZOMN = ZOAN = 30°, pois subtendem na referida circunferência o


mesmo arco ON.
Capitulo 3 - Resoluções 455

176) Seja P um ponto sobre o arco BC do circulo circunscrito ao triângulo


equilátero ABC. Prove que PA = PB + PC.

Resolução:
De fato, observando a figura abaixo

Podemos perceber que o quadrilátero ABPC é inscritivel. Assim, pelo teorema

de Ptolomeu (num quadrilátero inscritivel, o produto das diagonais é igual a

soma dos produtos dos lados opostos), temos que:

PABC = PBAC + PCAB

Assim, sendo a a medida do lado do triângulo equilátero ABC, segue que:

PA-a = PB a + PC-a => PA-a = (PB+ PC) a => PA = PB +PC


456 Tópicos de Matemática - Olimpíadas - IME - ITA

177) (OMERJ) Na figura, a reta t è tangente ao circulo e paralela ao segmento DE.


SE AD = 6, AE = 5 e CE = 7, qual o valor da medida do segmento BD = x?

Resolução:
Seja ZABC = a . Como o ângulo ZABC é um ângulo inscrito segue que o
arco AC mede 2a (a medida do arco corresponde, em graus, ao dobro da
medida do ângulo correspondente). Além disso, o ângulo ZCAF é um
ângulo de segmento que "enxerga" o arco AC de medida 2a , o que revela
que ZCAF = a (a medida, em graus, de um ângulo de segmento é igual a
metade da medida do arco que ele corresponde). Como DE//(t). segue que os
ângulos ZAED e ZCAF são ângulos alternos internos, o que revela que
ZAED = ZCAF = a . Além disso, o ângulo ZBAC = p é um ângulo que é
comum aos triângulos ABC e ADE. Ora, como os triângulos ABC e ADE já
possuem um par de ângulos em comum, a saber, a e p , segue que o
terceiro ângulo de cada um desses dois triângulos também tem a mesma
medida <p = 180°-a-p.
Capitulo 3 - Resoluções 457

O que, por fim revela que os triângulos ABC e ADE são semelhantes (pois
têm ângulos congruentes) e portando seus lados são proporcionais, o que nos
permite escrever:
AB AC 6+x 12
— => x = 4
AE AD 5 6

178) Considere três circunferências n^,n2 e n3 tais Que os seus centros O-i, Oa e

O3 não estão alinhados. Sejam r, s e t os eixos radicais de ,

Q, e Q3 e Q2 e Q3 , respectivamente. Prove que r, set são concorrentes

em um ponto chamado centro radical.

Resolução:
Antes de qualquer coisa lembre-se que o eixo radical de duas circunferência é
o lugar geométrico dos pontos do plano que contêm as suas circunferências
tal que todos os pontos possuem a mesma potência em relação a cada uma
das duas circunferências.
Seja Perns (esse ponto existe pois os centros das três circunferências não
estão, por hipótese, alinhados). Ora, como r é o eixo radial das
circunferências fi, e fi2e s é 0 eixo radical das circunferências d, e n3 ,
458 Tópicos de Matemática - Olimpíadas - IME - ITA

segue que Pot(P,Q,) = Pot(P,n2) e PotJP.Q,) = Pot(P,Q3), o que implica

que Pot(P,n2) = Pot(P,n3), o que revela que o ponto P também pertence a

reta t, que é o eixo radical das circunferências fi2 e íl3 . conforme ilustra a
figura abaixo:

179) (OCM) Duas tangentes OA e OB são traçadas a um circulo de um ponto


externo O. Uma corda AC é construída paralela a OB e uma secante OC é
desenhada intersectando o círculo em E. Se K é o ponto de interseção de OB
com o prolongamento de AE, prove que OK = KB.
Capítulo 3 - Resoluções 459

Resolução:
Defina ZACE = a. Ora, como o ângulo ZACE é um ângulo inscrita na
circunferência, a medida do arco correspondente. AE é, em graus, o dobro da
medida do ângulo, ou seja, 2a . Note que o ângulo ZOAE é um ângulo de
segmento que "enxerga" o arco AE da circunferência, o que revela que
ZOAK = a (pois a medida, em graus, de um ângulo de segmento é igual a
medida do arco correspondente). Além disso, temos por hipótese, que
AC//OB, o que revela que os ângulos ZACEe ZEOK sâo alternos Internos e
portanto possuem a mesma medida, ou seja. ZEOK = ZACE = a, como
ilustra a figura abaixo:

definido ZOKE = tp, segue que os triângulos OKE e AKO são semelhantes,

pois possuem dois ângulos em comum (se possuem dois ângulos em comum,
na verdade possuem os três ângulos em comum, visto que a soma das
medidas dos três ângulos é fixa e igual a 180°). Dessa semelhança temos a
seguinte proporção:

— = — => OK2 = KAKE (1)


=>OK
KA OK
Finalmente, pela potência do ponto K em relação a circunferência, segue que:

KA KE = KB2 (2)

Assim, de (1) e (2), segue que OK2 = KB2 =. OK = KB , como queríamos


demonstrar.
460 Tópicos de Matemática - Olimpíadas - IME - ITA

180) Seja P um ponto no interior de um círculo tal que existem três cordas que
passam
por P e tem o mesmo comprimento. Prove que P é o centro do circulo.

Resolução:
Sejam AB, CD e EF três cordas de uma circunferência Q que se intersectam
no ponto P, tais que AB = CD = EF = d . conforme ilustra a figura abaixo:

Pela potência de P em relação a n , segue que ax = by = cz . Por outro lado,

como AB = CD = EF = d , segue que x + a = y + b = c + z = d. Chamando do


produto constante ax = by = cz de D, temos o seguinte sistema de equações:

ax = by = cz = D
x+a=y+b=c+z=d

x +a = d
Por exemplo, se olharmos para as equações , segue que x e a são
xa = D

as raízes da equação quadrática Z2 - dZ + D = 0 e além disso, como

íy + b = d
, segue que y e d também são as raizes da equação quadrática
]yb = D

z+c= d
Z2 - dZ + D = 0 e por fim, como , segue que z e c também são as
ZC = D

raizes da equação quadrática X2 - dX + D = 0 . Ora. se imaginarmos que a e


Capítulo 3 - Resoluções 461

p são as raízes da equação quadrática À2-dÀ + D = 0, como existem seis


distâncias envolvidas nas três cordas, segue, pelo princípio da cada dos
pombos (ou princípio de Dirichelet), que pelo menos três dessas distâncias
são iguais. Ora, se tomarmos três pontos sobre a circunferência Q cujas
distâncias iguais até o ponto P são iguais esses três pontos serão os vértices
de um triângulo inscrito à circunferência Q e portanto o ponto P é o centro
dessa circunferência.

181) (SHORTLIST - IMO-85) Um circulo de centro O e raio R passa pelos pontos


A e C e intersecta os lados AB e BC do triângulo ABC nos pontos K e N,
respectivamente. Os círculos circunscritos aos triângulos ABC e KBN
intersectam-se em dois pontos distintos B e M. Mostre que ZOMB = 90°.

Resolução:
Na figura abaixo as retas BM, NK e AC são os eixos radicais das três
circunferências e P é o ponto de interseção desses três eixos radicais (P é o
centro radical):
B

M
Ü3

'k

,0

c ZA

Seja R a medida do raio da circunferência Oj. Olhando para o ponto B e

calculando a sua potência em relação a circunferência Q-j, por um lado,

Pot(B, O|) = BO2-R2 e por outroPotfB.Q,) = BN-BC, o que revela que

BN BC = BO2-R2.
462 Tópicos de Matemática - Olimpíadas - IME - ITA

Por outro lado. olhando para a circunferência Q>. temos que

BM • BP = BN ■ BC. Ora, como BN ■ BC = BO2 - R2, segue que:

BMBP = BNBC = BO2-R2 (1)


Ocorre que a potência do ponto P em relação a circunferência Qj é

PotfP.íij) = PO2-R2. e por outro PotfP.Qj) = PNPK , o que revela que

PN PK = PO2-R2.

Por outro lado olhando para a circunferência temos que

PM PB = PN ■ PK . Ora, como PN ■ PK = PO2 - R2, segue que:

PM PB = PN PK = PO2 - R2 (2)
Por fim, subtraindo, membro a membro as igualdades (1) e (2), segue que:
ÍBMBP = BO2-R2
=> BO2 - PO2 = BP (BM - PM)
|pmbp = po2-r2
Mas ocorre que BP = PM + BM , assim,

PO2 - BO2 = BP(PM - BM) = (PM + BM) (PM - BM) = PM2 - BM2

Assim, podemos reescrever:

PO2 - BO2 = PM2 - BM2 => PO2 - PM2 = BO2 - BM2


Finalmente, como o segmento OM é um lado comum aos triângulos OBM e
OPM, segue do exercício 120 que ZOMB = 90° , como ilustra figura abaixo.
B

M
n.3

'k
n.
N

P C /A
Capitulo 3 - Resoluções 463

182) Na figura abaixo, estão desenhados dois círculos de raios 8cm e 6cm. cujos
centros estão situados a uma distância 12 cm um do outro. Por P, um dos
pontos de interseção dos círculos, passa um segmento de reta QR tal que as
cordas QP e PR possuem o mesmo comprimento. Ache o quadrado do
comprimento de QP.

Resolução:
Inicialmente complete a figura conforme fizemos abaixo: (perceba que o ponto
a 3a
R está na reta que liga os centros das duas circunferências, pois - — . o
6 18
que garante que os triângulos cujos lados medem a, c e 6 e 3a. b e 18 são
semelhantes).
464 Tópicos de Matemática - Olimpíadas - IME - ITA

Aplicando o teorema de Pitágoras no triângulo sombreado acima temos:


(2a)2 + (b-c)2 = 122 (1)
Por outro lado, nos outros dois triângulos retângulos, temos que:

b = 764 - a2 e c = 736-a2 (2)


Substituindo (2) em (1), segue que:
, /------------- / --\2
4.a2 + V64-a2 -V36-a2 = 144 => 4a2 = 130

Portanto, (PQ)2 = (2a)2 = 130.

183) Mostre, na figura abaixo, que GE = HF.

Resolução:
Seja P o ponto de interseção das retas AB e CD (que nâo aparece na figura).
Pelo teorema de Pitot, segue que PA = PC e PB = PD. Assim,
AB = PB - PA = PD - PC = CD
Por outro lado, mais uma vez pelo teorema de Pitot, segue que AG = GE, HF
= HD. GB = GF e HE = HC. Assim.
AB = AG + BG
= GE + GF
= GE + (GE + EF)
=2GE+EF
Capitulo 3 - Resoluções 465

Analogamente.
CD = DH + HC
= HF + HE
= HF + (HF + FE)
= 2HF-EF
Ora, como AB = CD , segue que
AB = CD =■ 2GE + EF = 2HF +EF GE = HF
como queríamos demonstrar.

184) Na figura abaixo o circulo de raio R e o círculo de raio r são tangentes entre si
e também são tangentes a reta (s). O circulo menor tem raio r, e também é
tangente aos dois primeiros círculos e á reta (s).

Mostre que = +

Resolução:
Antes de resolver o problema propriamente dito, consideremos uma situação
mais simples: imagine que duas circunferências de centros O e O' e de raios
466 Tópicos de Matemática - Olimpíadas - IME - ITA

R e r são tangentes entre si e a uma reta (t), nos pontos P e Q, conforme


ilustra a figura a seguir:

p Q

Qual a distância entre os pontos P e Q?

Ligue os centros das duas circunferências e trace o segmento O’T//(t),


conforme ilustra a figura abaixo:

P Q

Aplicando Pitágoras no triângulo OO'T, segue que:

(R + r)2 = O'T2+(R-r)2 O'T = 2VRr


Capitulo 3 - Resoluções 467

Agora vamos resolver o nosso problema: chamemos de P. Q e T os pontos de


tangència das circunferências com a reta (t). conforme ilustra figura abaixo:

(t)
P Q T
Utilizando o fato que acabamos de provar, se olharmos para as
circunferências de raios R e r, segue que PT = 2>/Rr. Por outro lado, se

olharmos para as circunferências de raios R e r,, segue que PQ - 2xRr, e

por fim, se olharmos para as circunferências de raios r e rt. segue que


QT = 2x/rq . Mas ocorre que PT = PQ +QT. Assim,

PT = PQ + QT =■ 2x/Rr =2X/Rq +2^ =>

-.'R r = X/Rq + /rr,

Dividindo todos os membros por ^/'Rrr, , segue que

x/Rr, x/riÇ
7Rr _ x/Rq Jrq J___ 1_ 1
X/Rrr, x/Rrq x/RnÇ ,/q x/r x/R

Como queríamos demonstrar.


468 Tópicos de Matemática - Olimpíadas - IME - ITA

185) (IBERO) Dado o triângulo escaleno ABC, sejam D,E e F os pontos de


tangência dos seus lados com a circunferência inscrita no triângulo ABC. Se
G e o ponto de Interseção do segmento AD com a circunferência inscrita no
triângulo ABCD. M é o ponto médio do segmento EF e I é o incentro do
triângulo ABCD, mostre que os pontos D, G, M e I são conciclicos, isto, é são
vértices de um quadrilátero inscritível.

Resolução:
Calculando a potência do ponto A em relação a circunferência, segue que
AG AD = AE2, pois o segmento AE é tangente a circunferência. Por outro
lado note que o triângulo AIE é retângulo em E, pois o segmento IE é o raio
no ponto de tangência da circunferência inscrita com o lado AC, que é
justamente o ponto, com o ilustra a figura abaixo:
Capítulo 3 - Resoluções 469

Ora, como triângulo AIE é retângulo em E, segue por uma das relações
métricas dos triângulos retângulos, que AE2=AMAI. Por outro lado, já

sabemos que AG ■ AD = AE2 , o que revela que AG■AD = AM ■ Al e portanto


os pontos D,G,M e I pertencem a uma mesma circunferência, pois como
sabemos,

PA-PB = PC PD A, B, C e D pertencem a uma mesma circunferência


Assim, existe uma circunferência passando pelos pontos D. G. M e I, como
ilustra a figura abaixo:
470 Tópicos de Matemática - Olimpíadas - IME - ITA

186) Considere um triângulo ABC, a bissetnz interna AD do ângulo do vértice A e a


mediana AM. Considere ainda a circunferência n , definida pelos pontos A D
e M, conforme ilustra a figura abaixo:

Sendo E e F os pontos de interseção da circunferência n com os lados AB e


AC do triângulo ABC, mostre que BE = CF.

Resolução:
Sejam AB = c, BC = a e CA = b. as medidas dos lados do triângulo ABC. Ora,

como M é o ponto médio do lado BC, segue que BM = MC = |. Por outro

BD c
lado, pelo teorema das bissetrizes internas, segue que — . Além disso,
b
BD + DC = a. Assim,

BD + DC = a
BD c =>BD = -55_e DC = -2L
b + cb + c
DC "b

Por fim, calculando a potência do ponto B em relação â circunferência, segue


que:

BE ■ BA = BD BM => BE ■ c = -55- .5 => BE = a2


b+c 2 2(b + c)
Capitulo 3 - Resoluções 471

e de modo completamente análogo, calculando a potência do ponto C em


relação à circunferência, segue que:

a2
CFCA = CMCDaCF.b = -—=>CF =
2 b+c 2(b + c)

o que revela que BE=CF, como queríamos demonstrar.

187) Na figura abaixo a reta (t) é o eixo radical das circunferências Q, e Se o


segmento AB é tangente as duas circunferências e M é a interseção de AB
com a reta (t), mostre que o ponto M é o ponto médio do segmento AB.

(í)

Resolução:
De fato, como os segmentos AM e MB são tangentes às circunferências
n, e n2 , respectivamente, segue que

AM2 =MPMQ e MB2 = MP-MQ

Assim, AM2 = MB2 => AM = MB , o que revela que o ponto M é o ponto médio
do segmento AB.
472 Tópicos de Matemática - Olimpíadas - IME - ITA

188) (O teorema da corda quebrada - Arquimedes) O ponto M é o ponto médio


do arco AB da circunferência Q , mostrada na figura abaixo:

Se o ponto C é um ponto arbitrário do arco MB e D é o pé da perpendicular de


M À corda AC, mostre que o ponto D bissecta a linha poligonal ACB, ou seja,
AD = DC + CB

Resolução:
Ora, como o ponto M é o ponto médio do arco AB, segue que as cordas AM e
MB são congruentes. Além disso, note que ZMAC = ZMBC = a , pois esses
dois ângulos “olham" para o mesmo arco CM da circunferência n . conforme
ilustra a figura abaixo:

Fazendo uma rotação em torno do ponto M de modo a fazer com que MB


coincida com AM (o que é possível, pois AM = MB) o ponto C irá ocupar uma
nova posição C' sobre a corda AC, e, evidentemente, AC = BC, pois o lado
Capítulo 3 - Resoluções 473

BC do triângulo BCM irá ocupar a posição AC', após o movimento de rotação,


conforme ilustra a figura abaixo;

Por outro lado o movimento de rotação leva o segmento MC no segmento


MC, o que revela que MC = MC e, portanto, o triângulo MCC é isósceles de
base CC. Ora, como MD é a altura relativa a base C'C desse triângulo, segue
que CD = DC. Assim,
AD = AC'+ C' D = BC + CD = DC + CB
como queríamos demonstrar.

189) Na figura abaixo o triângulo ABC é equilátero e está inscrito no círculo O S<.
M e N são pontos médios dos lados AB e AC e P é a interseção do
prolongamento do segmento MN com o circulo Q, mostre que o ponto N
divide o segmento MP em razão áurea.
474 Tópicos de Matemática - Olimpíadas - IME - ITA

Resolução:
Seja 2a a medida do lado do triângulo equilátero ABC. Ora, como M e N são,
por hipótese, pontos médios dos lados AB e AC, respectivamente, segue que
AM = MA = NA = NA = a e MN = a, pois MN é a base média do triângulo ABC.
Sendo NP = x. segue pela simetria da figura, que EM = x, conforme ilustra a
figura abaixo:

Utilizando a potência do ponto N em relação a circunferência circunscrita ao


triângulo ABC, segue que

NPNE = AN-NC => x.(a + x) = aa => —= ——


a a+x
que é justamente a definição de um ponto dividir um segmento em ração
áurea (a parte menor está para a parte maior, assim como a parte maior está
para o todo).
O problema termina aqui!
Apenas para ser mais explicito,

-a + aV5 r 75-1'1
x a => x2 -r ax -a2 = 0 =
x' =
= Ja
a a -r x -a-aVõ (-Võ -l'!
x" =
Capítulo 3 - Resoluções 475

como NP = x > 0, segue que NP = x = ~ja• 0 ?ue implica que

X -J5 - 1 . j r • - -• 75-1-1
- = —-— que e, por definição a razao aurea. (O numero de ouro<p = —-—

é por definição, o inverso da razão áurea).

190) (OMRN) Uma formiga tem de caminhar do ponto A ao ponto B. veja figura a
seguir.

Existem duas possibilidades: caminhar a partir de A ao longo do semicírculo C


ou ir ao longo dos semicírculos desenhados no diâmetro, AB, do circulo
maior. Sabendo que o comprimento de uma circunferência é proporcional ao
seu raio, qual é o caminho que a formiga deve escolher, se ela pretende
percorrer o menor percurso?

Resolução:
A resposta é que a formiga pode escolher qualquer um dos caminhos, pois os
dois caminhos tem o mesmo comprimento. Vamos provar que os caminhos
tem o mesmo comprimento ainda que tenhamos k semicírculos desenhados
ao longo do diâmetro AB. Se o círculo tem raio r. o caminho C mede
2nr
---- = nr .
2
Agora, vamos calcular o comprimento do outro caminho. Suponha que ao
longo do diâmetro AB existam k semicírculos, cada um deles com raio
medindoq.tj rk, respectivamente. O comprimento do percurso ao longo

dos k semicírculos será igual a:


nr, + nr2 + ...+ nrk = n(r, + r2 + ... + rk) (1)
476 Tópicos de Matemática - Olimpíadas - IME - ITA

Por outro lado,


2r, + 2r2 + ... + 2rk = AB = 2r =>

r, + r2 +... + rk - r (2)
Substituindo (2) em (1). segue que:
nc, + nr2 + .. +rtrk = z(r, + r2 + .. + rk) = nr

o que nos leva a concluir que os dois caminhos possuem o mesmo


comprimento.

191) (OMRN) Na figura abaixo os dois círculos com centros P e Q são tangentes
externamente no ponto A.

Sabendo que o segmento BC é tangente a ambos os círculos. Determine a


medida do ângulo ZBAC

Resolução:
Trace os raios PB e QC e o segmento PQ, formando o quadrilátero BCQP.
Observe a figura abaixo:
Capítulo 3 - Resoluções 477

Como o segmento BC é uma tangente comum aos dois círculos, segue que
ZPBC = zQCB = 90° (no ponto de tangência o raio e a reta tangente são
perpendiculares). Definindo ZAPB = 2a e ZAQC = 2$ , segue que as

medidas (em graus) os arcos AB e AC são, respectivamente, 2a e 2* (a


medida de um arco, em graus, é igual a medida em graus do ângulo central
correspondente). Ora, como as medidas dos arcos AB e AC sâo,
respectivamente. 2a e 2<j>, segue que as medidas dos ângulos semi-

inscritos ZABC e ZACB são a e 4> , respectivamente (a medida em graus


de um ângulo semi-inscrito, em graus, é a metade da medida, em graus, do
arco correspondente). Lembrando que a soma das medidas dos ângulos
internos do quadrilátero BCQP é 360°, segue que
90° + 2a + 2<j> + 90° = 360° => 2 (a + *) = 180° => a + <f> = 90°

Finalmente, no triângulo ABC, tem-se:


ZBAC + (a + (t>) = 180° => ZBAC + 90° = 180° =• ZBAC = 90°

Como queríamos demonstrar.

192) Sejam a, b e c as medidas dos lados de um triângulo ABC e R a medida do


raio da circunferência circunscrita a esse triângulo. Mostre que:
1 1 _1_£ 1
ab ac bc r2

Resolução:
De fato,
_1_ _1_ a+b+c
(1)
ab ac bc abc
I
i
Por outro lado, sabemos que
(ABC) = ^=>abc = 4R(ABC) (2)
478 Tópicos de Matemática - Olimpíadas - IME - ITA

e também
a + b + c' 2 (ABC)
(ABC) = p r — ■ r=ja + b + c = (3)
2 r
Substituindo (2) e (3) em (1), segue que:
1 1 1 a+b+c
----- i----- + — -- ------------
ab ac bc abc
2 (ABC)
r
4R(ABC)
1
“ 2rR

Mas ocorre que em qualquer triângulo.


R > 2r =.R2 > 2rR
ri ■. n- . r->2 ~ n-n
IA t £.1 —S IA C. £.1 I A —r* ------ JT
— =■ — È 4+
R2 2rR 2rR R2

Portanto,
11111 1111
--- H----- +---- =------ >----- =>---- +---- +---- £-----
ab ac bc 2rR r2 ab ac bc r2

como queríamos demonstrar.

192) (RPM)Seja ABC um triângulo acutângulo de circuncentro O e sejam D, E e F


as interseções das semirretas AO, BO e CO com os lados BC. AC e AB.
respectivamente. Sendo R o circunraio do triângulo ABC, mostre que.
1116
OD + OE + OFâR

Resolução:
Sejam hlF h2 e h3 as medidas das alturas do triângulo ABC relativas aos lados
AB, BC e CA, respectivamente e z,x e y as distâncias do circuncentro 0 aos
lados AB, BC e Ca. respectivamente, conforme ilustra a figura a seguir:
Capítulo 3 - Resoluções 479

°i
■y!

Assim,
2 (ABC)
a=
h,
(ABC) = ^1 bh2 chg 2 (ABC)
~2~~~2~=> b = (1)
h2
2 (ABC)
c=
h3

Como o triângulo ABC é, por hipótese, acutãngulo, segue que

(2)
2
Substituindo (1) em (2), segue que:
2(ABC) 2 (ABC) 2(ABC)
X
h1 , h2
y ^3
y
(ABC) =
2 2 2

(ABC) = (ABC) + +
[_ni n2 **3

A + X + _L = 1
h-j ^2 ^3
480 Tópicos de Matemática - Olimpíadas - IME - ITA

Por outro lado, sabemos que a média harmônica de trés números reais
positivos u.vewé menor do que ou igual a média aritmética desses mesmos
trés números reais, ou seja,

3 u+v+w w)[- + - + —1 á9
<--------- (□
2 2 3 ^u V w>
u V w
x y z . . , h-.
Ora, como — + —+ — = 1, tomando u = —, v = — ew = — e aplicando a
h2 h3 x y z
desigualdade acima, segue que

ÍÍ!l + !k + Í13.1 fjL 4. + >9=> —+ — +^2->9 (3)


Ix y zJlh, h2 h3 J x y z

Por outro lado, note que ocorrem as seguintes semelhanças de triângulos.


AI AD h, AO + OD
AADJ - AODL => — OD = x ~ OD
Üi = A+1
OL x OD
BE Ü2 = BO + OE h2 = —R + <1
ABEG-AOIE=> — —
OI OE y OE y OE

ACFH ~ AOFK => — = — ha = CO + OF Íi = A+i


OK OF z OF z OF
Adicionando, membro a membro as três últimas igualdades acima, segue que
h1 . ^2 . h3 p 1 + —1 + 3
x y z OE ofJ

Mas, por (3), temos que — + — + —> 9 . Portanto,


x y z
^1 . ^2 , ^3
>9=s r[— + 1 + 3>9=>
x y z lOD OE

1 1 1 ' 1 + A>Ê
R
OD OE OF. a6^õ5 + OE OF R

como queríamos demonstrar.


Capitulo 3 - Resoluções 481

ÁREAS

193) Usando as figuras abaixo demonstre o teorema de Pitágoras.

b b

1
b

1
b
b b
a

c b
b

Resolução:
As duas figuras sâo quadrados congruentes e portanto possuem ares iguais.
Olhando para a figura da esquerda, a sua área é a2 + 4 - ^ = az + 2bc . Já na

figura da direita a medida área éb2 +4 — + c2 = b2 + 2bc + c2. Ora, como as

áreas sâo iguais, segue que


a2 + 2bc = b2 + 2bc + c2 a2 = b2 + c2

194) Observe a figura a seguir. Por um ponto da diagonal do retângulo foram


traçadas paralelas a seus lados. Mostre que as áreas dos retângulos
sombreados sâo iguais.
482 Tópicos de Matemática - Olimpíadas - IME - ITA

Resolução:
Nomeie os vértices dos retângulos e sejam x, y, z e w as medidas das áreas
das regiões indicadas como ilustrado na figura abaixo:
A E D

F H

B G c
Assim,
(ABD) = (BCD) =>x + w + z = w + y + z=>x = y

195) (PROFMAT-2011)Na figura abaixo, as retas r e s são paralelas a uma


distância 2 uma da outra, AB é um segmento unitário contido em s, X é um
ponto de r com AX = 5ePéopéda perpendicular baixada de B sobre AX.

(?•)
X

Determine o comprimento de BP.

Resolução:
Note que da distância entre as retas paralelas r e s corresponde a medida da
altura do triângulo ABX em relação a base AB. Já o segmento BP
corresponde a latura do triângulo ABX em relação ao lado AX. pois P é o pé
Capitulo 3 - Resoluções 483

da perpendicular traçada a partir de B ao lado AX. Assim, a medida da área


do triângulo ABX pode ser calculada de duas maneiras distintas, a saber:

(ABX) = ^M = ^ = 1
AX BP 5BP
(ABX) =
2 2
6. RP 2
Portanto, = 1 => BP =

196) (PROFMAT-2011)Se espremermos um círculo de raio 10cm entre duas retas


paralelas que distam entre si 10cm, obteremos uma figura de área menor,
mas mesmo perímetro que o círculo original. Se as partes curvas desta figura
obtida são semicircunferências,

IOct»

Determine a razão da área da figura espremida pela área do círculo inicial

Resolução:
Como o circulo inicial tinha raio de 10cm a sua área era de

Ao = n 102 = 100ncm2 . Após ser espremido, o circulo original transformou-

se na figura dada, que pode ser decomposta num retângulo de base a e altura
10cm mais dois semi-círculos de raio 5cm. Ora. por hipótese, temos que o
perímetro da figura final é o mesmo que do circulo original, o que releva que
2a + n-5 + n-5 = 2-it-10=>2a = 10n=>a = 57t
Portanto a medida da área da figura que corresponde ao circulo original
espremido corresponde a soma das medidas das áreas de um retângulo de
lados a = 5n cm e altura 10cm mais dois semicírculos de raio 5cm. Assim,

A = 5n-10 +2-— n52 = 75ncm2


2
A 75n 3
Finalmente a razão pedida é — =
Ao 100n 4
484 Tópicos de Matemática - Olimpíadas - IME - ITA

197) (OBM) Em algum momento, na primeira metade do século passado, uma


pessoa chamada Afrânio tinha um valioso terreno desocupado, perto do
centro da cidade do Rio de Janeiro. Com a urbanização da cidade, ruas novas
foram abertas e o terreno de Afrânio ficou reduzido a um triângulo ABC,
retângulo em 8, ainda de grande valor, pois o lado AB media 156 metros Pois
bem, Afrânio morreu e em seu testamento os advogados encontraram as
instruções para dividir o terreno "igualmente" entre seus dois filhos. Era assim
“um muro deve ser construído perpendicularmente ao lado AB. de forma que
os dois terrenos resultantes da divisão tenham mesmo valor; o que tem a
forma de um trapézio será do meu filho mais velho e o outro será do mais
novo". Os advogados concluíram que os terrenos deviam ter mesma área,
pois o testamento dizia que deveríam ter mesmo valor. Mas não foram
capazes de decidir em que posição deveria ficar o muro. Conta meu avô que
o episódio ganhou as páginas dos jornais por vários dias, com leitores
opinando de diversas maneiras sobre a posição correta do muro. Ele falava e
se divertia muito com as opiniões absurdas mas, ao mesmo tempo, me
instigava a resolver o problema. E o problema retorna para vocês. Em que
posição, relativamente ao lado AB do terreno, o muro deve ser construído?
c

A x M B
156
b 4
Resolução:
Note que os triângulos ABC e AMN são semelhantes. Assim, a razão entre as
medidas das suas áreas é igual a razão de semelhança entre eles, ou seja,

2S í-]2
= kl56> iTã
156
= -7= => x = "56 „ 11 o, 3m
42 42
Capitulo 3 - Resoluções 485

198) (TREINAMENTO OBMEP) A figura a seguir mostra um trapézio com bases


medindo 20 cm e 14 cm e com os outros dois lados medindo 5 cm cada um.
Duas circunferências com centros A e B sâo tangentes âs bases, uma ao lado
esquerdo e outra ao lado direito. Pergunta-se qual é o comprimento do
segmento AB.

14

20

Resolução:
Inicialmente nomeie os vértices da figura e desenhe os raios das
circunferências nos pontos de tangênci J, D E e I , como ilustra a figura
abaixo:
L J

K
5 /f

C M
3 -+- 14 3.
P
Ora, como o trapézio CFHL é isósceles segue que
CF-LH 20-14
CM = FN = = 3 e DE = 14. Usando Pitágoras no triângulo
2 2

CML, segue que 52 = 32 + (2r)2 => r = 2.

Por outro lado note que


(CFHL) = (CFBA) + (HLAB) + (CAL) + (HBF)

(20 + 14) 4 (20 + AB)-2 (14 + ABJ-2 5-2 5-2


2 “ 2 + 2 + ~2~ 2
136 = 40 + 2-AB +28+2-AB+ 10 + 10 => 4-AB = 48 => AB =12
486 Tópicos de Matemática - Olimpíadas - IME - ITA

199) (OMRN)Na figura abaixo, suponha que todas os segmentos verticais são
paralelas, que todas os segmentos horizontais são igualmente espaçados e
que todos os ângulos são retos. Nessas condições, que fração da figura toda
representa a parte hachurada?

Resolução:
Transferindo as áreas sombreadas para a parte inferior da figura conforme
ilustra a figura abaixo temos que a medida da área sombreada corresponde a
1
- da área total do retângulo, visto que os segmentos horizontais estão
4
igualmente espaçados, veja a figura abaixo:

200) (OMRN)Na figura abaixo, todas as medidas mostradas são em centímetros.


Qual é a área da região hachurada?
4 3 6 3 2

■1 4

3 3

6 6

3 3
2 2
2 3 G 3 4
Capítulo 3 - Resoluções 487

Resolução:
Transferindo as partes sombreadas para o retângulo central conforme ilustra a
figura abaixo temos que a soma das medidas das áreas sombreadas é (2 + 3
+ 6 + 3 + 4) 6 = 108cm2.

Observe a figura abaixo:

4 3 6 3 2

4 4

3 3

6 G

3 3
2 2
2 3 6 3 4

201) (UFPE)O retângulo ABCD ilustrado a seguir está dividido em seis quadradc
e tem perímetro 21. Qual a medida da sua área?
D C

,n
A B

Resolução:
Se x é a medida do lado do menor quadrado que aparece na dissecção do
retângulo, temos que os lados dos outros quadrados medem 2x, 3x. 5x e 8x.
O perímetro do retângulo ABCD é 2(x + x + 3x + 8x + 8x) = 21, e dai x =1/2. A
área do retângulo mede (13x).(8x) = 13.8/4 = 26.
188 Tópicos de Matemática - Olimpíadas - IME - ITA

>02) Se o quadrilátero ADCD da figura abaixo é um trapézio de bases AB e CD,


mostre que as medidas Si e S2 das áreas dos triângulos hachurados são
iguais.

Resolução:
De fato, os triângulos ABC e ABD têm a mesma área pois tem a mesma base
AB e a mesma altura (que é a distância entre as bases AB e CD). Assim.
(ABC) = (ABD) =. (ABE) + S2 = (ABE) + S, => S, = S2

203) Um trapézio ABCD é dividido em quatro triângulos conforme ilustra a figura


abaixo:

Sendo S, e S2 as medidas das áreas dos triângulos hachurados, determine,


em função de S, e S2, a medida da área do trapézio ABCD
Capitulo 3 - Resoluções 489

Resolução:
Pelo exercício anterior, sabemos que (ADE) = (BCE) = S . Sejam AE = u e

EC = v, então como os triângulos ADE e CDE tem a mesma altura partindo do

vértice D, segue que a razão entre as medidas das suas áreas é igual a razão

entre as medidas das suas bases, ou seja,

(ADE) u s u
(CDE) V S1 V
(D

De modo completamente análogo, os triângulos ABE e BCE têm a mesma

altura partindo co vértice B e, portanto, a razão entre as medidas das suas

áreas é igual a razão entre as medidas das suas bases, ou seja.

(ABE) u S2 u
(2)
(BCE) v S v

de (1) e (2), segue que:

A= => s2 = s,s2 =■ s = 7s,s2

Diante do exposto, segue que


(ABCD) = (ABE) + (ADE) + (BCE) + (CDE)
= S-, + S + S + S2
= S-| + 2S + S2
= S-, + 2^S^2 + S2
490 Tópicos de Matemática - Olimpíadas - IME - ITA

204) As bases do AB e CD do trapézio isósceles ABCD representado na figura


abaixo medem a e b e a sua altura mede h. Determine, em função de a, b e
h, a medida da área do triângulo hachurado

Resolução:
Sejam x e y as alturas (partindo do vértice E) dos triângulos CDE e ABE,
conforme ilustra a figura abaixo:

como os triângulos ABE e CDE são semelhantes (pois possuem ângulos dois

a dois congruentes), segue que — — . Por outro lado, x + y = h. Assim,


a

x b
bh ah
7=ã => x =------ e y =--------
a+b a+b
x+y = h
Capítulo 3 - Resoluções 491

Portanto,

ah
(ABE) = |ay = —1 a------ a2h
=
2 a + b 2(a + b)

bh b2h
(CDE) = —bx = —b------ =
[ J 2 i + b~ 2(a + b)
2 a+b

Mas, pelo exercício anterior, sabemos que (ADE) = ^(ABE)(CDE) . Assim.

' a2h b2h abh


(ADE) =
2(a + b) 2(a + b) 2(a-b)

205) (Olimpíada de Maio) Um retângulo de papel 3cm x 9cm é dobrado ao longo


de uma reta, fazendo coincidir dois vértices opostos Deste modo se forma um
pentágono. Calcular a medida da área desse pentágono.

Resolução:
Após ocorrer a dobra na folha de papel, perceba que o segmento CD
transforma-se no segmento AD', o que revela que AD' = CD = 3. Da mesma
forma o segmento DG transforma-se no segmento GD' e. portanto, se DG = x.
segue que GD' = DG = x. Por fim, como AD = 9 e GD = x. segue que AG = 9 -
x. Além disso, com a dobra do papel o ângulo do vértice C do retângulo ABCD
492 Tópicos de Matemática - Olimpíadas - IME - ITA

transforma-se no ângulo ZAD'G, o que revela que ZAD'G = ZBCD = 90°.


como ilustra a figura abaixo:

D'

3 a;

A 9 — a; G
x D
- -1

y
3 i 3

B 9-y F
___ n c
V
9

Aplicando Pitágoras ao triângulo AD'G, segue que

(9-x)2 = x2 + 32 => 18x = 72 => x = 4

Analogamente, ao fazer a dobra do papel o segmento CF transforma-se no


segmento FA, portanto se CF = y, segue que FA = CF = y e BF = BC - FC = 9
- y. Assim aplicando Pitágoras ao triângulo ABF, segue que

y2 = 32 + (9 - y)2 => 18y = 90 =■ y = 5

Assim BF = 9- y = 9- 5 = 4, o que revela que


(ABFGD') = (ABFG) + (AGD1)
(AG + BF)-AB AD'D'G
" 2 +’ 2

(5 + 4)-3 34
2 + 2
= 13,5 + 6
= 19,5cm2
Capítulo 3 - Resoluções 493

206) (OBM) Um terreno quadrangular foi dividido em quatro lotes menores por
duas cercas retas unindo os pontos médios dos lados do terreno As áreas de
três dos lotes estão indicadas em metros quadrados no mapa a seguir.

D
G

A
250

H
E
200 210

B F C

Qual é a área do quarto lote, representado pela região escura no mapa?

Resolução:
Note que o quadrilátero EFHG é um paralelogramo. pois se traçarmos as
diagonais AC e BD do quadrilátero ABCD. teremos pelo teorema da base
média de um triângulo que EF//AC, AF = AC/2, GH//AC, GH = AC/2, EG//BD.
EG = BD/2, FH//BD e FH = BD/2, conforme ilustra a figura abaixo:
494 Tópicos de Matemática - Olimpíadas - IME - ITA

Note que (EGI) = (GHI) = (HFI) = (EFI) = x , pois as diagonais de um

paralelogramo o dividem em quatro triângulos de mesma área. Sejam a. b, c


e d as medidas das áreas indicadas abaixo:

D
G

A X- ' b
a X ' '
x
H
E x
x
c

B F C

Ora, como os triângulos AEG e ABD são semelhantes com razão de


semelhança 1/2, segue que a razão entre as suas áreas é 1/4 (quadrado da
-j
razão de semelhança), ou seja, a = —(ABD). De modo completamente

análogo. c = ^(BCD), d = -l(ABC) e b = ^-(ACD) .

mas ocorre que

(ABD) + (CBD) = (ABCD) => 4a + 4c = (ABCD) => a + c = i(ABCD)

(ABC) + (ADC) = (ABCD) => 4d + 4b = (ABCD) => b + d = -l(ABCD)

o que revela que a + c - b + d .

Assim,
a + c = b + d => (a + x) + (c + x) = (b + x) + (d + x)
Capitulo 3 - Resoluções 495

Olhando para a figura

podemos ver que


a + x = S, b + x = 250, c + x = 210 e d + x = 200

portanto,
(a + x) + (c + x) = (b + x) + (d + x) => S + 210 = 250 + 200 => S = 240

OBSERVAÇÃO: Uma outra solução seria a seguinte:


Ora como os pontos E, F, H e G são pontos médios dos lados do quadrilátero
ABCD, ligando-se através de segmentos, o ponto I aos vértices A, B. C e D.
formamos quatro pares de triângulos com áreas iguais em cada par, conforme
ilustra a figura abaixo:

250'.

21Q

C
496 Tópicos de Matemática - Olimpíadas - IME - ITA

Por exemplo, note que os triângulos AEI e BEI têm a mesma área, pois têm
bases de mesma medida (pois o ponto E é ponto médio do lado AB) e têm a
mesma altura vinda do vértice I. De acordo com a figura acima, temos que:
S =x+y
=> S + 210 = x + y + z +w
210 = w + z

250 = y + z
=> 450 = x + y + z + w
200 = x + w
Assim,
S + 210 = 450 => S = 240

207) Na figura abaixo temos um quadrado dois semicírculos e um quarto de

circulo. Determine a razão - , entre as medidas das áreas hachuradas.


b

Resolução:
Indique por c a medida da área de cada uma das partes que não estão
hachuradas nos dois semicírculos, como ilustra a figura abaixo:

c b
t
Capitulo 3 - Resoluções 497

Note que a soma das medidas a e c corresponde a medida da área de um

semicírculo de raio -- , enquanto que duas vezes a medida c mais as medidas

a e b correspondem a um quarto da medida da área de um círculo de raio r.


Diante do exposto, segue que:

1 PY2 =>a + c =-----


a + c = —n —
rP ní2
=> c =-------- a (D
2 I2J 8 8

2c + a + b = — n t2 (2)
4
substituindo (1) em (2), segue que:

f >P aKa-h . ’tf2 o K :P


2
lT-aJ+a+b=——’2a+a+b=~
—a + b = O=>a = b=^- = 1
b

208) Na figura abaixo a medida da área do quadrado maior é 1.

Sabendo que o quadrado maior foi subdividido em quadrados menores, qual é


a medida da área do quadrado pequeno hachurado?

Resolução:
Ora, como lado do quadrado original mede 1 e o seu lado foi dividido em três
partes iguais, segue que cada um dos quadrados seguintes têm lado 1/3.
498 Tópicos de Matemática - Olimpíadas - IME - ITA

Olhando para o centro da figura perceba que um desses quadrados de lado


1/3 teve o seu lado dividido ao meio, produzindo quatro quadrados de lado 1/6
(a metade de 1/3). Por fim, um desses quadrados de lado 1/6 teve o seus
lados divididos em 5 partes iguais, o que revela que cada um dos quadrados
menores tem lado 1/30 (pois é 1/5 de 1/6). Assim a medida do lado do
quadradinho menor em preto é 1/30, o que revela que a medida da sua área é

—■

130/ 900

209) (UFPE) O retângulo ao lado foi dividido em nove quadrados. Se o quadrado


menor (hachurado) tem área 1, qual a medida da área do retângulo?

Resolução:
A partir do quadrado hachurado de lado 1, chamando de x a medida da lado
do quadrado do canto inferior esquerdo a figura podemos completas a figura
da seguinte forma:

D x F9 C

x (t - 2) - 4 = x - 6
r+5
x+9
x+ 1 4
^{x. + 9) - (x — G) = 15
(x + 1) - (x - 3) - 4 ------ 4
x+1 (x - 2) — 1 = x — 3
1
x-2
TWjl___
lz-2 15
x Z— l X-1

A ■x — 1 2x - 3 B
Capitulo 3- Resoluções 499

Por fim, olhando para o quadrado do canto inferior direito podemos afirmar
que 2x-3 = 15=>x = 9. Assim,

AB = x + (x-1) + (2x - 3)
= 4x -4
= 4-9-4
= 32

AD = x + (x +1) + (x + 5)
= 3x + 6
= 3-9 + 6
= 33

o que nos permite concluir que (ABCD) = 32 ■ 33 = 1056 u a.

210) (PROFMAT-2011) Considere um triângulo retângulo isósceles ABC com


hipotenusa BC. Tomando o ponto A como centro e AB como raio,
consideramos o arco de circunferência delimitado pela corda BC
Consideremos ainda a semicircunferência de diâmetro BC, conforme a figura
abaixo:

Designando por T a área da região triangular ABC e por S e L as áreas das


outras duas regiões.
Prove que L = T.
00 Tópicos de Matemática - Olimpíadas - IME - ITA

Resolução:
Suponha que AB = AC = a. Por Pitágoras, BC = a>/2. Note que T + S
corresponde a medida da área de um quarto de círculo de raio a, enquanto

que S + L corresponde a medida da área de um semicírculo de raio .

Assim,

T + S = —rta2
4

L + S„ = -^
1
—J V
1 a2

O que revela que T + S = L + S=>T = I_

211) Na figura abaixo ABCD é um quadrado inscrito num círculo de lado 1. Sobre
cada um dos lados do quadrado foram construídos semicírculos cujos
diâmetros são os lados do quadrado, conforme ilustra a figura abaixo:

Determine a medida da área da região hachurada.


Capitulo 3 - Resoluções 501

Resolução:
Trace as diagonais AC e BD do quadrado. Pelo resultado do exercício anterior
cada um dos 4 triângulos formados apresentam a mesma área a de cada
uma das “lúnulas" conforme ilustra a figura abaixo:

Diante do exposto, a soma das [áreas das 4 ‘lúnulas’ corresponde a medida


da área do quadrado. Ora, como o circulo no qual o quadrado está inscrito
tem raio 1, sendo ( e medida do lado do quadrado, segue que AC= (42.

Mas por outro lado, AC = 2R = 2 1 = 2 . Assim,

íx/2= 2 =>< = -?- = 42


42

Portanto a medida da área hachurada é S = (2 = (v2)2 = 2u.a


502 Tópicos de Matemática - Olimpíadas - IME - ITA

212) Calcule a área do triângulo ABC abaixo, sendo dados BD = 4. DE = 2, EC = 6,


BF = FC = 3.

Resolução:
Inicialmente indique as distâncias fornecida pelo enunciado, conforme ilustra
a figura abaixo:

Ora, como BD = 2 DE e F é o ponto médio do segmento BC, segue que o


ponto D é o baricentro do triângulo ABC e o segmento BE é uma media do
triângulo ABC. Assim, (ABE) = (BCE), pois BE sendo mediana do triângulo

ABC, implica que o ponto E é ponto médio do lado AC e portanto dos


triângulos ABE e BCE têm bases AE e EC de mesma medida e uma mesma
Capitulo 3 - Resoluções 503

altura partindo do vértice B Por fim perceba que o triângulo BCE é equilãtero
de lado 6. Ora, como a medida da áreas de um triângulo equilátero de lado a

. a2V5
e —-— , segue que

(ABE) = (BCE) = ---i = 9 75

O que revela que (ABC) = (ABE) + (BCE) = 973+ 9^3 = 1875 u.a.

213) No quadrado ABCD da figura abaixo foram traçados quadro arcos de circulo
com centro nos vértices do quadrado e cujo raio coincide com o lado a do
quadrado ABCD. Determine a medida da área da região hachurada.
a D
A

/I
a a

B a C

Resolução:
Sejam x, y e z as medidas das áreas indicadas na figura abaixo:
, a D
A
y

a a
y y

y
B a c
504 Tópicos de Matemática - Olimpíadas - IME - ITA

Claramente, x + 4y + 4z = a2, pois a soma de todas as áreas dá a área do

1 ■>
quadrado. Por outro lado, x + 2y + 3z = —rra , pois x + 2y + 3z corresponde a
4
medida da área de um quadrante de circulo de raio a, centrado em B, por
exemplo. Para finalizar precisamos achar mais uma equação envolvendo as
variáveis x, y e z para podermos resolver o sistema e então determinar o valor
do x, que corresponde a medida da área desejada. Para isso, observe a figura
abaixo

Note que u corresponde a medida da área de um triângulo equilátero de lado


a2V3
a, ou seja, u =------- e v corresponde a diferença entre a área de um setor
4
circular de 60° (e raio a) e a área u, do triângulo equilátero, ou seja,

na2 a 2 Vã
v = —na2 - u =----------------
6 6 4

Assim,
j

u + 2v = +2
fna2 a2j3^' , na2 a2 Vã
l 6 " 4 J. 3 4

i
Capitulo 3 - Resoluções 505

Por outro lado olhando para a primeira figura do inicio da nossa resolução,
na2 a2V5 _______ „
segue que x + 2z + y = u + 2v e, portanto, x + 2z + y = —-------------
. , o que
, nos

permite montar o seguinte sistema de equações lineares:

x + 4y + 4z = a2
(n + 3 - 3>/3)a2
x + 2y + 3z = ^rra2 => x =
3
na2 a2V3
x + 2z + y = —--------- —■

214) Seja ABC um triângulo tal que os lados BC e AC medem respectivamente a e


b. Sabendo que a área deste triângulo é dada por ^-(a2+b2j, calcule a

medida do ângulo ACB.

Resolução:
1
Como a área de um triângulo é dada pela fórmula (ABC) = -absenC , segue
2
que:

■i(a2 + b2) = ^-absenC => senC = a2 + b2


2ab

Portanto,
(a-b)2
senC-1 =
2a b 2ab

(a-b)'I2 2
Por outro lado, - à 0 , pois (a-b) â 0,a > 0 e b > 0 e além disso,
2ab
senC < 1 => senC-1 <0
506 Tópicos de Matemática - Olimpíadas - IME - ITA

Portanto,

(a~b)'i2
0< — < 0 => 0 < SenC -1 < 0 => senC -1 = 0 => senC = 1 => C = 90°
2ab
=SenC—1

Além disso, note que acarreta a = b. Assim, o triângulo é isósceles. Os


ângulos são A = B = 45° e C = 90°.

215) Prove a desigualdade de Weintzenbock: Num triângulo ABC,


(ABC)£^(a2+b2+c2)

Ocorrendo a igualdade se, e somente se o triângulo for eqüilátero.

Resolução:

Como
(ABC) = ^-ab-SenC

c2 = a2 + b2 -2abcosC

Demonstrar a desigualdade
(ABC)<^|(a2+b2 + c2)

É equivalemte a demonstrar

-ab-SenCS— Í2a2 + 2b2 -2abcosc) •=. absenC < -y-(a2 + b2 -abcosc)


2 12 l

ou ainda,
3absenC < 73 (a2 + b2 j - 75ab cosC o 3absenC + 73abcosC < 73 (a2 + b2)

que é equivalente a
a2 + b2 > abfTÕsenC + cosC)
Capitulo 3 - Resoluções 507

Mas
VãsenC + cosC = 2Í—senC + -<
cosCj = 2(cos30°senC + sen30°cosC) = 2sen(C +• 30°) £ -2
l 2 2

portanto,
a2 +b2 > ab(x/3senC + cosCj a2 + b2 > ab (-2) = -2ab o a2 + b2 > -2ab

Ou ainda,
a2 + b2 > -2ab «• a2 + 2ab + b2 > 0 <=■ (a + b)2 > 0

Que é evidententente verdadeira, portanto a desigualdade


a2 + b2 > ab(>/3senC + cosC)

Também é verdadeira e por conseguinte a desigualdade original

íABC)slf(a2+b2+c2)
Também é verdadeira, visto que todos os passos efetuados são todos

reversíveis, o que permite que você saía da desigualdade

(a + b)2 àO

Que é obviamente verdadeira e chegue a desigualdade desejada

(ABC)Sf(a2+b2 + c2)

, . a2 73
No caso do triângulo ser equilátero, a = b = c => (ABC) =—-—. que e

justamente o valor que se obtém quando substituímos a=b=c na

desigualdade (ABC) < ^(a2 +b2 + c2j. Veja:

(a2+a2 + c2) = -^3a2=^.


—(a2+b2 + c2) =—
12* I 12
508 Tópicos de Matemática - Olimpíadas - IME - ITA

Portanto ocorre a igualdade no caso do triângulo equilátero. Essa sugestão do


livro, me parece que não resolve a recíproca, ou seja.
seja, se

(ABC) = —(a2+ b2+c2j, então o triângulo é equilátero.

COMENTÁRIO FINAL:
Assim, para provar a reciproca , ou seja, se a igualdade

(ABC) =—(a2+b2 + c2j ocorre então o triângulo é eqüilátero podemos

pensar assim:
Note que

(ABC) = ^/p(p - a)(p-b) (p-c)


l(a + b + c) (b + c-a) (a + c-b) (a + b-c)
”V 2 2 2 2
= 1^2 (a2b2 + a2c2 + b2c2) - (a4 + b4 + c4 j

Por outro lado,

(a2-b2)2 + (b2-c2)2 + (c2- a2]2 sOo

2(a4 + b4 + c4) -2(a2b2 + a2c2 + b2c2) > 0 c=>

2(a4+b4+c4)>2(a2b2+a2c2 + b2c2)«

4 (a4 +b4 +c4j > 4^a2b2 + a2c2 + b2c2) <=>

4(a4+b4+c4) 4(a2b2 +a2c2 +b2c2)


3 “ 3 ~

(a4 +b4 +c4) + 3(a4 +b4 +c4) 4(a2b2 +a2c2 + b2c2)


3 2 3 ~
Kr-b4 +c4) + 2(a2b2 +a2c2 +b2c2) 6(a2b2 + a2c2 +b2c2)-3(a4 +b4 +c4)

3 " 3 °
Capitulo 3 - Resoluções 509

(a2+b2+c2)2
a 2 (a2b2 + a2c2 + b2c2) - (a4 + b4 + c4) «
3
=(ABC)'

(ABC)<^|(a2+b2 + c2)

Note que a igualdade ocorre se e somente se


(a2 - b2)2 + (b2 -c2)2 + (c2 - a2)2 = 0 « a = b = c

216) São dados no plano dois quadrados de lado 1 tais que o centro de um deles
coincide com um vértice do outro. Determine os valores possíveis para a área
da porção comum aos dois quadrados.

Resolução:
Na verdade a medida da área da região comum aos dois quadrados
corresponde a 1/4. A explicação é a seguinte: desenhe os dois quadrados
numa posição genérica tais que o centro de um deles coincide com um vértice
do outro, conforme ilustra a figura abaixo:

A região comum aos dois quadrados é o polígono EOBM. Note que os


triângulos EOL e EMN são congruentes (caso ALA, pois EM = EL, ELO =
EMN = 90° e OEL = MEN = 90° - a). Assim,
510 Tópicos de Matemática - Olimpíadas - IME - ITA

( )= área
(EOBM) = (EOL) + (ELMN)
= (EMN) + (ELMN)
= (ELMB)
(ABCD)
4

~4
217) (OBMEP-2007) A figura abaixo foi feita com quatro quadrados de 10cm de
lada cada um. Os vértices A, B e C também são centros dos quadrados
correspondentes. Qual a medida da área da região sombreada?

Resolução:
De acordo com a resolução do problema anterior a região de interseção entre
dois quadrados vizinhos corresponde a 'A da superfície de cada quadrado.
Ora. como cada quadrado tem lado 10cm, segue que a medida da área de
cada um deles é de 100cm2, segue que cada região em comum de quadrados

vizinhos tem área 'A x 100cm2 = 25cm2 Ora, como os quadrados de cima e

de baixo não se intersectam podemos então computas para esses dois uma
área de 200cm2. Restando então adicionar a parte do quadrado de centro B

que ainda não foi computada, ou seja, 75cm2 (pois 25cm2 desse quadrado já

foram computados quando consideramos a área do quadrado de centro A) e


por fim a área do quadrado de centro C que ainda não foi computada, ou seja,
50cm (pois 25cm desse quadrado já foram computados quando
Capitulo 3 - Resoluções 511

consideramos o quadrado de centro B e mais 25cm2 da sua área também já


foram computados quando consideramos o quadrado inferior no início da
nossa solução). Diante do exposto a medida da área da região sombreada é
de 200cm2 + 75cm2 + 50cm2 = 325cm2.

Note que uma outra alternativa seria pensar assim: os quatro quadrados
juntos somariam 400cm2 de área. Como há três regiões de interseção, com
área % x 100cm2 = 25cm2 cada uma, segue que a medida da área sombreada

é de 400cm2 - 3 x 25cm2 = 325cm2.

218) (OBMEP-2013) Dois quadrados de papel se sobrepõem como na figura. A


região não sobreposta do quadrado menor corresponde a 52% de sua área e
a região não sobreposta do quadrado maior corresponde a 73% de sua área.
Qual é a razão entre o lado do quadrado menor e o lado do quadrado maior?

Resolução:
Sejam a e b as medidas dos lados do quadrado menor e do quadrado maior
respectivamente. Portanto as medidas das suas áreas são a2 e b2

respectivamente Quando olhamos para o quadrado menor a região comum


aos dois quadrados corresponde a 100% - 52% = 48% da sua área, ou seja.

0,48a2. Quando olhamos para o quadrado maior a região comum aos dois

quadrados corresponde a 100% - 73% = 27% da sua área, ou seja. 0,27b2.

Ora, como a região de interseção é uma única região, essas duas "contagens"
são iguais, ou seja,

_9_ a 3
0,48a2 = 0,27b2
Í-Í = -48
IbJ 16 b 4
512 Tópicos de Matemática - Olimpíadas - IME - ITA

219) Demonstre a fórmula de Heron: A medida da área de um triângulo de lados


a,í> e cé dada por:
(ABC) = 7p(p-a)(p-b)(p-c)

Resolução:
Sabemos que:
(ABC) =-^b-c-senA (*)

Por outro lado, a2 = b2 +c2 -2bccos A => cos A = ^^a2 -(b2 + c2))

|2
SenA = 7l-cos2 A = Lí—(a2-(b2 + c2))|
L2bc\ ' //J
4b2c2-(a2 ~(b2 + c2j)

=v 4b2c2

-25Z^(2bc)2-(a2-(b2+c2))'.2
= 2^^2bC + a' ~b2 ' c2)(2bc- a2 + b2 + c2) (*’)

=ã^4a2 - (b2 ■2bc+c2)>2+2bc+c2) -a2]


=ié2-(b-c)2][(b+c)2-a2J
= .J(a + b - c) (a - b + c) (b + c + a) (b + c - a)

Por outro lado,


a + b - c = (a + b + c) - 2c = 2p - 2c = 2 (p - c)
2p = a + b + c=> a - b + c = (a + b + c) - 2b = 2p - 2b = 2 (p - b)
b + c-a = (a + b + c)-2a = 2p- 2a = 2(p- a)

Substituindo esses resultados em (**) obtemos:

senA = V2P2(p-c)2(p-b)2(P_a) = ê- >/p(p-a)(p-b)(p-c)


z_LJU UU
Capitulo 3 - Resoluções 513

Então substituindo esse resultado em (*), segue que


-i 12 i______ —---------------------------
(ABC) = —b c senA = —bc —^p(p-a)(p-b)(p-c)

Portanto,

(ABC) = ^/p(p-a)(p-b)(p-c)

220) Para que valores de n é possível particionar um triângulo equilátero de lado n


em trapézios de lados medindo 1, 1, 1 e 2?

Resolução:
Para que segmentos de comprimentos 1, 1, 1 e 2 formem um trapézio é
preciso que o de medida 2 seja a base maior, dois de medida 1 sejam os
lados não paralelos e o outro de medida 1 seja a base menor, conforme
ilustra a figura abaixo:

1
h.

1 1/2

Por Pitágoras, l2=h2+(j) => h = ~ . Assim a medida da área deste

trapézio é

(2 + 1)4 3^3
S = ---------- 4- =------ u.a.
2 4
Por outro lado a medida da área de um triângulo equilátero de lado n é

_n2V3 - .. . ......... .
------- . Supondo que esse triângulo possa ser dividido em k trapézios cujos
4 " '
lados medem 1, 1, 1 e 2, segue que a soma das medidas das áreas desses
trapézios deve ser igual a medida da área do referido triângulo.
514 Tópicos de Matemática - Olimpíadas - IME - ITA

Asstm,

3^3 . n2V3 2 ,,
-------k =-------- => n = 3k
4 4
O que revela que n2 é um múltiplo de 3 e portanto n também tem de ser
múltiplo de 3 (o quadrado de um número só é múltiplo de 3 se o próprio
número for). Assim n = 3s para Algum s inteiro. Portanto, sempre que n for
um múltiplo de 3 será possível particionar o triângulo equilátero de lado n em
trapézios de lados 1, 1, 1 e 2, pois sendo n um múltiplo de 3, em particular
n = 3s , podemos dividir cada lado do triângulo em s partes de comprimento
3 cada uma e subdividirmos o triângulo original em triângulo equiláteros
menores de lado 3, conforme ilustra a figura abaixo no caso particular em que
=4

Agora perceba particularmente que cada triângulo de lado 3 pode ser


subdividido em três trapézios com lados 1, 1, 1 e 2 com o seguinte
procedimento:
0 Marque o baricentro G do triângulo ABC de lado 3.
ü) Tome pontos D. E e F nos lados AC, AB e BC respectivamente tais que
CD = 1, AE = 1 eBF = 1.
iii) Trace os segmentos GD. GE e GF.
Capitulo 3 - Resoluções 515

Conforme ilustra a figura abaixo:

Agora perceba que GD = GE = GF = 1.


Para ver isso vamos mostrar, por exemplo, que GF = 1 (os outros dois casos
são análogos!)

De fato, traçando a mediana CM os triângulos CFG e CBM são semelhantes.

1
D

2
G

A 1 E M B

3
a GF MB x 2
Assim, — — = —=>x = 1.
FC BC 2 3
516 Tópicos de Matemática - Olimpíadas - IME - ITA

Fazendo o mesmo para GD e GE chegamos a ilustra a figura abaixo:

A 1 e 2 B

O que revele que todo triângulo equilátero de lado 3 pode ser particionado em
3 trapézios de lados 1, 1, 1 e 2. Assim como todo triângulo equilátero de lado
n, múltiplo de 3 pode ser particionado em triângulos equiláteros menores de
lado 3, segue que todo triângulo equilátero cuja medida do lado é um múltiplo
de 3 . pode ser subdividido em trapézios de lados 1, 1, 1 e 2.
Note que se a medida do lado for n = 3s , podemos particionar esse triângulo

equilátero em s2 triângulos menores, equiláteros de lado 3, e cada um destes


menores em 3 trapézios de lados 1, 1, 1 e 2. Portanto um triângulo da lado

n = 3s pode ser subdividido em 3s2 trapézios de lados 1, 1 ,1 e 2.

221) ABCD é um trapézio de bases BC e AD e lados não paralelos AB e CD. Seja


E o ponto médio do lado CD e suponha que a área do triângulo AE8 seja
igual a 360cm2 Calcule a área do trapézio.

Resolução:
c B

D A
Capitulo 3 - Resoluções 517

Marque o ponto F, médio de AB.

C B

D A

Note que os triângulos BEF e AEF têm a mesma área, pois FB = FA e eles
Têm a mesma altura a partir do vértice E. Assim,

(BEF) = (AEF) = = 180cm2.

Por outro lado, os triângulos AEF e BEF tem o lado EF em comum. Tomando
esse lado como base, segue que as alturas desses dois triângulos, partindo e
A e de B são iguais (digamos h), visto que os dois referidos triângulos
possuem a mesma área. Assim,
(BEF) =-^EF-h => 180 = ^EF-h =s EF-h = 360

Diante do exposto, a medida da altura do trapézio ABDC é 2h e lembrando-se


AD -r BC
que = EF (teorema da base média de um trapézio) segue então
2
que:

(abcd) = La2(-bc) ■ (2h) = EF ■ 2h = 2 (EF) • h = 2 x 360 = 720cm2

222) Por um ponto P no interior de um triângulo ABC traçamos retas paralelas aos
lados de ABC. Tais retas particionam ABC em três triângulos e três
paralelogramos. Se as áreas dos triângulos são iguais a 1, 4 e 9. calcule a
área de ABC.
518 Tópicos de Matemática - Olimpíadas - IME - ITA

Resolução:

Como GJ, HE e Fl são paralelas aos lados AC. BC e AB respectivamente,


segue que os triângulos DEF, DGH e DJI são semelhantes ao triângulo ABC
(e semelhantes ente si) Assim,
( )= área
2
(DEF) FD 1
(ABC) AB, (ABC) IabJ

(DGH)_ ÍGH) 2 4 f GH)2


(ABC) IabJ => (ABC) - IãbJ
(DJI) =“f2L'|
IabJ
2=> 9
(ABC) (ABC) IabJ
Notando que FD=AG e que DI=HB, pois AGDF e HBID sâo paralelogramos e
a dicionando membro a membro as igualdades


“I—abV
1 FD 1
(ABC) J AB /(ABC)
4 GHV2
f------- GH 2
segue que:
(abc) “IabJ = AB /(ABC) '
2
Dl 3
9 21
(ABC) AB AB /(ABC)

1 2 3__ FD + GH + DI AG+GH+HB AB „
=---- = 1 =>
/(ABC) /(ABC) '(ABC) AB AB AB

^(ABC) = 1 + 2 + 3 => (ABC) = (1 + 2 + 3)2 => (ABC) = 36 U.a,

Este fato é geral, ou seja, se os três triângulos menores tivessem área


Capitulo 3 - Resoluções 519

S,. S2 e S3, teríamos que (ABC) = (y/S? + y/s? + y/s^)2 e a demonstração é

exatamente a que fizemos ao resolver o problema acima.

223) Um hexágono convexo tem 3 lados consecutivos medindo a e os outros 3


medindo b. Sabendo que tal hexágono está inscrito em um circulo de raio R.
pede-se:
a) Calcular R em função de a e b
b) Calcular a área do hexágono em função de a e b

Resolução:
a) Sejam AF e FE um par de lados adjacentes cujos comprimentos sejam
diferentes (isto é sempre possível!!! concorda? Pense um pouquinho
e...). Assim o arco (AF) +arco (FE) = 360°/3 = 120°. o que revela que o
ângulo central AOB é de 120° ( a medida de um ângulo central é igual a
medida, em graus, do arco que ele determina na circunferência Além
disso perceba que o ângulo AFE mede 120°, pois é um ângulo inscrito
numa circunferência que “olha" para o arco AE de medida 240° (a
medida de um ângulo inscrito é a metade da medida do arco
correspondente na circunferência) Veja a ilustração abaixo:

Agora aplicando a lei dos cossenos nos triângulos AEF e AOE. temos:
ÍAE2 = R2 + R2 -2RRcos120° AE2 = 3R2
[AE2 = a2 + b2 - 2ab cos 120° AE2 = a2-b2.ab

3R2 = a2+b2 + ab => R =


520 Tópicos de Matemática - Olimpíadas - IME - ITA

b) Note que o hexágono pode ser dividido nos quatro triângulos destacados
na figura abaixo:

Os três triângulos ABC. AFE e CDE são congruentes e o triângulo ACE é


equilátero de lado AE = r73 (veja a primeira lei dos cossenos que aplicamos
acima!). Assim a medida da área dos hexágono pode ser calculada da
seguinte forma:

(ABCDEF) = ^absen120° + ^absen120° + ^absen120° + (rV3)2 Vã


4
= 2ab^ + ^3R2
2 2 4
3 .75 375 (a2+b2 + ab)
= 2abT+—li}
975ab + 375 (a2 + b2) + 375ab
12
373 (a2 +b2 + 4ab)
= 12
73pa + b)2 + 2ab]
.. _
Capitulo 3 - Resoluções 521

224) Seja ABCD um trapézio de área 18 cm,22 e a soma das diagonais 12 cm.
Mostre que as diagonais são perpendiculares e iguais.

Resolução:
Imagine que a medida de um dos ângulos formados pelas diagonais (no seu
ponto de interseção no interior do trapézio) seja a e o seu suplemento p.
conforme ilustra a figura a seguir:

Sejam: ( ) = área. AC = x + y = D e BD = z + w = d.
Note que (ABCD) = (ABE)+ (BCE) + (CDE) +(ADE) e que senp = senct pois

a + P = 180°.
Assim,
1111
18 = -xzsena + -zysenp + -ywsena + -xwsenp

36 = xzsena + zysena + ywsena + xwsena

36 = (xz + zy + yw + xwjsenn

36 = [x(z + w) + y (z + w) Isena =>

36 = (z + w) (x + y) sena => 36 = D.d.sena

Por outro lado D + d = 12. Por fim usando a desigualdade das médias
aritmética e geométrica, temos:

VÕd < = — = 6=>x/Õd<6=>Dd<36


2 2
522 Tópicos de Matemática - Olimpíadas - IME - ITA

36
Mas 36 = D d ■ sena => Dd =--- e então,
sena
Dd < 36 => - 3?— < 36 => sena > 1
sena
Mas por outro lado sabemos que sena < 1 para todo a real. Assim,

sena< 1 . n
=> sena = 1 => a = 90°, pois 0o < a < 180°.
sena £ 1
Portanto as diagonais são perpendiculares. Além disso, note que sendo
u = 90°, temos que:
Ddsena -- 36 => Ddsen90° = 36 Dd = 36

Neste caso ocorre a igualdade entre as médias aritmética e geométrica,


„ D+d 12 „
MA =—
a = -------= —=6
2r~ 2
Mg = VDd = V36 = 6

E como sabemos, as médias aritmética e geométrica são iguais se, e


somente se, os números são iguais, ou seja, D = d.

225) Sendo r o raio do circulo inscrito a um triângulo de lados medindo a.b e c e


seja X,Y e Z as distâncias do incentro aos vértices A,B e C respectivamente,
. . XYZ abc
demonstrarque:
demonstrar que: ------ =------ ; onde p é o semiperimetro.
r p

Resolução:
Capítulo 3 - Resoluções 523

Seja ( ) = área
Note que
u
senp = L, cosp = —

„ r „ w
senO = — , cos0 = —
Y Y
r v
sena = — , cos a = —
Z Z
Por outro lado, 2a + 2p + 28 = 180° => a + p + 0 = 90° => a + p = 90° - 0
Portanto.
sen(a + [5) = sen(90°-0) => senacosp + senpcosa = cosfl

Substituindo os respectivos valores, segue que


r u r v w
sena cos p + senpcosa = cosO => —— + (1)
XZ Y
Analogamente,

a + 0 = 90° - p sen(a + 0) = sen(90°-p)


cc + p + 0 = 90° =>
P + 0 =90°-a sen(p + 0) = sen(90°-a)

r w r v u_
senacos0 + sen0cosa = cosp ZY X
(2)
senpcos0 + senOcosp = cosa r w r u v
x Y Yx Z
Portanto de (1) e (2) segue que'

r u r v w r(u-rv) w rb w
Z X XZ Y ZX = Y zx Y
r w r v u r(w + v) u ra u
zY Yz X ZY ~ X^ zY X
r w r u v r(w + u) v rc V

XY YX Z XY " Z xY = ^Z
multiplicando membro a membro as três últimas igualdades acima, segue que

r3abc wuv r3abc


------- = wuv C)
(XYZ)2 XYZ XYZ
524 Tópicos de Matemática - Olimpíadas - IME - ITA

Para finalizar lembra de duas coisas, a saber:

|./p(p-a)(p-b)(p-c)
(ABC) p (p — a) (p — b) (p — c) = p2r2 (p-a)(p-b)(p-c) = pr2

E que (não é difícil provar) w = (p-b), u = (p-a) ev = (p-c). Assim,

substituindo essas informações em (*), segue que

r3abc r3abc , , r3abc 2 rabc abc XYZ


-^Z = wuv^~XYz=(p~b)(p7a)(p-c^3õ<Z = pr =>xyz=p=>~p~ r
-pr1

226) Qual a medida da área do triângulo cujos lados são -J3. 2 e Jõ ?

Resolução:
Seja a a medida do ângulo oposto ao lado cuja medida é -J5 . Aplicando a
lei dos cossenos, segue que

(Vs) = (>/3) + 22 -2 \/3 -2cosa => cosa =

Ora. como sen2a + cos2a = 1, segue que

sen2“+ÍTj . -733
= 1 => senct =------
6

Lembrando que a medida da área de um triângulo é % do produto dos


comprimentos de dois lados multiplicado pelo seno da medida do ângulo
formado por estes dois lados, segue que
AT=1^.2.sena = lV3.2.4^ = ^ua-
'2 2 6 2
Capitulo 3 - Resoluções 525

227) (Austrália) Na figura abaixo, a distância do centro O do circulo C ás cordas AB


e CD são 4 cm e 3 cm, respectivamente. Sendo S-p S2. S3 e S4 as medidas

das áreas das regiões determinadas pelos segmentos e pela linha da


circunferência, verifique que:
(S1 +S3)-(S2 + S4)=48.

Resolução:
Como as cordas AB e CD estão a uma distância de 4 cm e 3 cm do centro O.
respectivamente, vamos traçar segmentos EF e GH situados a 4 cm e 3 cm
do centro O, respectivamente , conforme ilustra a figura abaixo. Observe

Na figura anterior x, y, z e 48 representam as medidas das áreas das figuras


em que se encontram (perceba que calculamos 48 como sendo 8.6=48. pois
526 Tópicos de Matemática - Olimpíadas - IME - ITA

o retângulo central tem dimensões 8 cm e 6 cm), assim de acordo com a


figura anterior temos que: S, = x , S2=x + y, S3 = x + y+ z +48 e S4 = x + z

dai segue que;


(S -I + s3) - (S2 + s4) = [x + (x + y + z + 48)] - [x + y + (x + z)J = 48

228) O quadrilátero ABCD foi seccionado em quatro triângulos conforme ilustra a


figura abaixo:

Se os números que aparecem no interior de cada um dos triângulos


representam as respectivas áreas dos triângulos, determine a medida da área
indicada por x.

Resolução:
Quando dos triângulos tem a mesma altura partindo de um mesmo vértice, a
razão entre as medidas das suas áreas corresponde a razão entre as
medidas das suas respectivas bases. Assim,
x DE
(os triângulos BCE e DEC tem a mesma altura partindo de C)
120 EB
300
= (os triângulos ABE e ADE tem a mesma altura partindo de A)
200
„ . . x 300
Portanto, ----- = 200x = 300 x 120 => x = 180 u.a..
120 200
Capitulo 3 - Resoluções 527

229) (OMRN) Na figura ao lado, ABCDE é um pentágono cujo lado DC está sobre
a reta ®. Traçamos EF//AD e BG//AC. Verifique que o triângulo AGF e o
pentágono ABCDE têm a mesma área.

Resolução:
Ligue o ponto A ao D e A ao C. Com isso, formamos os triângulos que têm a
mesma área, pois têm a mesma base AD e a mesma altura, já que EF//AD.
por hipótese. De modo análogo, têm a mesma área, pois têm a mesma base
AC e a mesma altura, já que BG//AC. Agora, olhamos para a área.
S(ABCDE), do pentágono:
S(ABCDE) = S(AADE) + S(AACD) + S(óADF)
= S( AACD) + S(AACG) = S( AAFG)
Como queríamos provar

230) (OMRN)Determine a medida da maior área que pode possuir um pentágono


eqüilátero de lado 1cm e que possui um dos seus ângulos internos retos

D E
528 Tópicos de Matemática - Olimpíadas - IME - ITA

Resolução:
B
1
1
■■■ \
C
A

1
A \y
1

D 1 E
( ) = Área
1-1 1
Na figura acima (ABE) = — = — .

Agora observemos o quadrilátero EBCD de lados 72,1, 1 e 1. Segue que

(EBCD) = (BCD) + (EBD) = ^1 -1 • senC + ^1 ■ V2senE =>

1 15
(EBCD) = ^senC + ^y-senE

Aplicando a lei dos cossenos nos triângulos BCD e EBD,

BD2 =2-2 cos C = 3 - 2V2 cos E => 2 cos C - 2-72 cos E = -1 =>

cos C - J5. cos E = - —


2

Mas podemos reescrever cos C-71 cos E = como

1 _ -J2 c 1
cos C - 72 cos E = - — — cosC------ cosE = —
2 2 2 4
Assim,

r1 72 - 1
- cos C------ cos E = — — = — cos2 C - — cos C cos E + — cos2 E
2 2 4 16 4 2 2
1 J5 (EBCD)2 = ^sen2C + yhenCsenE + ^sen2E
(EBCD) = — senC-r-^-senE

Adicionando as duas expressões acima

1 cos (C + E) +1 =. (EBCD) = ^-^cos(C + E)


(EBCD)2
16 16 2
Capítulo 3 - Resoluções 529

Assim a medida da área (EBCD) será máxima quando cos(C + E) = -1 e

'j'2.
valerá (EBCD) =,— + — . Assim a máxima medida da área do pentágono
' ’ v16 2

m 42 1
será (ABCDE) = ------1- .
16--- 2 2

231) AB é um segmento de tamanho 7 . P é um ponto tal que a distância de P ã


linha suporte do segmento AB é 3. Nestas condições determine o menor valor
possível para AP BP

P P' J3

a I

A 7 B E

Resolução:
Note que o triângulo ABP tem uma base de comprimento fixo igual a 7 e uma
altura fixa igual a 3, independente da posição do ponto P sobre a reta definida
pelos pontos P e D. Ora, se o triângulo ABP tem uma base de comprimento
fixo e uma altura de comprimento fixo, então a medida da sua área é fixa,
visto que a medida da área de um triângulo é a metade do produto entre os
comprimentos a sua base e da sua altura relativa a essa base. Assim.
1 21
(ABP) = —7-3 = —

Por outro lado, (ABP) = IaBBP- sena .Assim,

1 21 21
— AB BP sena = — => AP BP =-------
2 2 sena
530 Tópicos de Matemática - Olimpíadas - IME - ITA

Portanto o produto AB BP será mínimo quando o valor do sena for máximo,


ou seja, quando sena = 1 (que é o máximo valor que o seno de um número
real assume). Diante do exposto, segue que o valor mínimo assumido pelo
produto ABBP é 21.

232) Se um triângulo ABC tem área S, mostre que a medida da área do triângulo
3
formado pelas medianas do triângulo ABC é — S .

Resolução:
Notação: ( ) = área

Sejam AE = ma, BF = mbeCD = mc as medidas das medianas do triângulo

ABC. Lembrando que num triângulo qualquer a medida do segmento que vai
de um dos vértices ao baricentro do triângulo corresponde a 2/3 do
2 1
comprimento da respectiva mediana, segue que BG = -mb, GE = -ma.

Agora a partir do ponto E prolongue o segmento GE até o ponto H tal que


1
EH = GE = -ma e construa o segmento BH, formando então o triângulo BGH,

conforme ilustra a figura a seguir


Capítulo 3 - Resoluções 531

2 112
Note que BG = -mb e GH = GE + EH =-ma+ -ma =-ma. Vamos provar

2
agora que BH = -mc. Para isso basta observar que BGCH é um

paralelogramo, pois as suas diagonais GH e BC se cruzam ao meio, visto que


BE=EC (pois E é ponto médio de BC) e por construção GE = EH. Ora. se
BGCH é um paralelogramo, segue que os seus lados opostos BH e GC tem a
2
mesma medida e daí BH = GC = -mc. Assim o os lados do triângulo BGH

2 2 2 S S
Sa° 3ma’ 3mb 6 3m<: e além disso a sua área é 2 — pois
6 " 3 '

S
(BEH) = (CEG) = (BGE) = — (as medianas de um triângulo o dividem em 6
6
triângulos de mesma área) e além disso os triângulos BEH e CEG sâo
congruentes (caso LLL). Assim,
s s
(BGH) = (BGE) + (BEH) = — + — = 2—=
s S
3
532 Tópicos de Matemática - Olimpíadas - IME - ITA

Finalmente perceba que o triângulo BGH cujos lados medem


2 2 2
-ma, -mb e -mc é semelhante ao triângulo XYZ cujos lados são

2 A

ma, mb e mc com razao de semelhança -. Assim,

s
(BGH) 3
4 =>(XYZ) = ^S
3
(XYZ) (XYZ) 9

233) (CRUX-MATHEMATICORUM) Os segmentos DE, CE, BF e CF dividem o


retângulo ABC em regiões menores conforme ilustra a figura ao lado. Quatro
destas regiões, dois triângulos e dois quadriláteros estão hachuradas na
figura ao lado. As áreas destas quatro regiões hachuradas são 9, 35, 6 e x.
Determine o valor de x.
e c
9

E
x

35
y 6
A F D

Resolução:
Além da variável x, que já vem na figura do enunciado, sejam y, z e w as
medidas das áreas das regiões indicadas na figura abaixo:
B C

E y X
w

35
z
6
A F D
Capitulo 3 - Resoluções 533

Note que:

ADCD (ABCD)
(ABCD) = ADCD. (CDE) = , o que implica que (CDE) =
2 2
Por outro lado,
BC BA (ABCD)
(ABCD) = BC BA , (BCF) = . O que implica que (BCF) =
2 2
Além disso,
(ABF) + (CDF) + (BCF) = (ABCD) =>

(ABCD)
(ABF) + (CDF) = (ABCD)- (BCF) =
2
=(ABC0)/2

Portanto, (ABF) + (CDF) = (CDE).

Mas ocorre que:


(ABF) + (CDF) = (9 + y + 35) +(6 +w) = 50 +y + w e (CDE) = y + x + w

Assim,
(ABF) + (CDF) = (CDE) =>50 + y + w = y + x + w=>x = 50

234) (OBM) Na figura, todas as circunferências menores têm o mesmo raio


r e os centros das circunferências que tocam a circunferência maior são
vértices de um quadrado. Sejam a e b as áreas cinzas indicadas na figura.
Então qual a razão a/b ?

Resolução:
Claramente a área a corresponde a área de uma circulo de raio r. ou seja,

a = nr2. Por outro lado, traçando-se um diâmetro da circunferência maior é

fácil ver que o seu diâmetro mede 3-2r = 6r, portanto o raio de circunferência
maior mede 3r. Assim a área b corresponde a 1/4 da área do circulo maior
menos a área de cinco vezes a área de % do circulo menor de raio r, ou seja.
534 Tópicos de Matemática - Olimpíadas - IME - ITA

r2 _ 9nr2 - 5nr2 4nr2


b = — n (3r)2 — 5 — tii = --------- = 71tr
( 2
4 ' ' 4 ~ 4 4
i a nr2
O que revela que - = —= 1.
b nr2

235) (CRUX-MATHEMATICORUM) A figura abaixo é constituída por seis


quadrados numerados por I, II, III, IV, V e VI. Mostre que a soma das medidas
das áreas dos quadrados I, II e III corresponde a três vezes a somas das
medidas das áreas dos quadrados IV, V e VI.

Resolução:
Antes de qualquer coisa vamos nomear os vértices e os comprimentos dos
lados conforme ilustra a figura abaixo:
Capitulo 3 - Resoluções 535

Além disso, consideremos os ângulos ZMBN = a , ZLAU = 0 . ZVCY = y ,


ZBAC = ip , ZACB = 0 e ZABC = X.

Olhando para os vértices A, B e C, lembrando que o ângulo total AM cada um


desses vértices é de 360° e que os ângulos internos dos polígonos IV, V e VI
sâo de 90° (pois essas figuras são quadrados) podemos escrever:
Vértice A: 0 + 90° + 90° + <p = 360° => 0 + <p = 180°

Vértice B: a + 90° + 90° + X = 360° => a + X = 180°


Vértice C: y + 90° + 90° + 0 = 360° => y + 0 = 180°

Aplicando a lei dos cossenos nos triângulos MBN, CVY e LAU, obtemos:

AMBN —> a2 = e2 + f2 -2ef cosa

ACVY c2 = d2 -t-f2 -2dfcosy (1)

ALAU -» b2 = d2 + e2 - 2de cos 0

Mais uma vez aplicando a lei dos cossenos, mas agora no triângulo ABC,
temos:

d2 = e2 + f2 -2ef cosá

e2 = d2 + f2 -2df cos0 (2)


f2 = d2 + e2 -2decos<p

Lembrando que
0 + <p = 180° =5 <p = 180° - 0 => cos <p = - cos 0

a + X = 180° => X = 180° - a => cos X = - cos a (3)


y + 0 = 180° => 0 = 180°-y => COS0 = -cosy

Agora, substituindo (3) em (2), segue que:


d2 = e2 + f2 - 2ef cos X==>d2=e2 + f2+ 2ef cos a

e2 = d2 + f2 -2df cosO =>e2 = d2 +f2 + 2df cosy (4)


536 Tópicos de Matemática - Olimpíadas - IME - ITA

f2 = d2 + e2 -2decos<p => f2 = d2 + e2 + 2decos(5

Agora, adicionando membro a membro as igualdades de (1), obtemos:


a2 + b2 + c2 = 2 (d2 + e2 + f2 j - 2ef cos a - 2df cos y - 2de cos |l

Adicionando membro a membro as igualdades de (4), obtemos:

d2 +e2 + f2 = 2^d2 + e2 + f2) + 2ef cosa + 2dfcosy + 2decos(t =>

-[d2 + e2 + f2) = 2ef cosa + 2df cosy + 2decos(í

Por fim. temos que:

a2 + b2 + c2 = 2 (d2 + e2 + f2)-2ef cosa - 2df cosy -2decosp

-[d2 + e2 + f2j = 2ef cos a + 2df cos y + 2decos0

Adicionando membro a membro essas duas últimas igualdades, segue que:


a2+b2 + c2 = 3(d2 + e2 + f2)

Observando que
área(l) = a2.área(II) = c2,área(III) = b2,área(IV) = d2,área(V) = e2 e área(VI) = f2,

segue que:
área(l)-r área (II) + área (III) = 3 [área (IV) + área (V) +área (VI)]

236) Num triângulo ABC ra,rb,rcer são, respectivamente, os raios das

circunferências ex-inscritas e da circunferência inscrita. Mostre que a área

(ABC) desse triângulo pode ser determinada por (ABC) = ^ra rb rc r .


Capítulo 3 - Resoluções 537

Resolução:
De fato, sendo a,b e c as medidas dos lados do triângulo ABC e p o seu semi-
perimetro sabemos que

Pr
(P~a)ra
(ABC) =
(P-b)rb
(P~c)rc

Multiplicando membro a membro essas quatro igualdades, obtemos:

(ABC)4 = p(p-a)(p-b)(p-c)r ra rb-rc

Por outro lado, pela fórmula de Heron temos

(ABC)2 =p(p-a)(p-b)(p-c)

Assim,

(ABC)4 =p(p-a)(p-b)(p-c)rrarb-rc =>

(ABC)4 = (ABC)2rrarbrc=.

(ABC) = rrarbrc => (ABC) — ^ra -rb -rc ■ r

237) Num triângulo ABC ra.rb,rc e r são, respectivamente, os raios das

circunferências ex-inscritas e da circunferência inscrita. Mostre que


1 1 2 2
«■'■•a
rb fc

Resolução:
De fato, sendo a, b e c as medidas dos lados do triângulo ABC e p o seu
semi-perimetro sabemos que

pr
(P“a)ra 1 p 1 p-a 1 p-b 1 p-c
(ABC) = (p-b)rb r ” (ABC)’ra “ (ABC)'rb " (ABC) ® rc “ (ABC)

(p-c)rc
538 Tópicos de Matemática - Olimpíadas - IME - ITA

Adicionando membro a membro as três últimas igualdades obtemos:


1 1 1_p-ap-b p-c_ 3p-2p _ p _ 1
r“ + + “ (ABC) " (ABC) + (ABC) " (ABC) ” (ABC) " r

,11
O que revela que - = — +
r ra ib tc

238) Mostre que as medianas de um triângulo dividem o triângulo em seis


triângulos menores de mesma área.

Resolução:
Seja ABC um triângulo com medianas AM, BN e CP, conforme ilustra a figura
abaixo:

Usando o fato de que dois triângulos que possuem a mesma base e a mesma
altura tem áreas iguais, segue que: (BGM) = (CGM) = x , (CGN) = (AGN) = y

e (AGP) = (BGP) = z , conforme ilustra a figura abaixo:


Capitulo 3 - Resoluções 539

Por fim, mais uma vez usando o note que usando o fato de que dois
triângulos que possuem a mesma base e a mesma altura tem áreas iguais,
segue que:
(ABM) = (ACM) =>2z + x = 2y-i-x=>z = y

(BCN) = (ABN) =>2x + y = 2z + y=>x = z

Portanto x = y = z.

239) (HUNGRIA) S é um ponto no interior do aABC tal que as áreas dos triângulos
ABS, BCS, CAS são todas iguais. Prove que S é o baricentro de ABC.

Resolução:
Inicialmente prolongue os segmentos AS, BS e CS até que atinjam os lados
BC, AC e AB nos pontos M, N e P, respectivamente, conforme ilustra da
figura abaixo-
540 Tópicos de Matemática - Olimpíadas - IME - ITA

Usando o fato de que para triângulos que tem a mesma altura a razão das
suas áreas é igual a razão das suas bases, segue que:
Se (ABS) = (ACS) = (BCS) = T , temos:

BM (ABM) (SBM) (ABM)-(SBM) (ABS) T 1


MC “ (ACM) " (SCM) ~ (ACM) - (SCM) " (ACS) ” T "

O que revela que o ponto M é ponto médio do segmento BC. Analogamente,

CN (BCN) (ANS) (BCN)-(ANS) (ABS) T 1


NA (ABN) ” (CNS) “ (ABN) - (CNS) " (BCS) " T ”

O que revela que o ponto N é ponto médio do segmento AC. Por fim,
AP (AGP) (APS) (ACP)-(APS) (ACS) T
PB “ (BCP) " (BPS) “ (BCP)-(BPS) “ (BCS) “ T “

O que revela que o ponto P é ponto médio do segmento AB. Ora, como M, N
e P sâo, respectivamente, os pontos médios dos lados BC, AC e AB do
triângulo ABC, segue que os segmentos AM, BN e CP são suas medianas e
portanto o seu ponto de concorrência, o ponto S, é pois, o baricentro do
triângulo ABC.

240) (IME/1990 - AIME/1985) Seja Pum ponto no interior de um triângulo ABC,


dividindo-o em seis triângulos, quatro dos quais têm áreas 40, 30. 35 e 84.
como mostra a figura. Calcule a área do triângulo ABC.
A

84 E
F,
.P
35

40 30

B D C
Capítulo 3 - Resoluções 541

Resolução:
Sejam x e y as medidas das áreas dos triângulos BFP e AEF,
espectivamente e BD = a, DC = b, CE = c, EA = d, AF = e. FB = g, conforme
ilustra a figura abaixo:

e d

84 y E
F,
P
35 c
x
f
40 30

B a D b C

Usando o fato de que para triângulos que tem a mesma altura a razão das
suas áreas é igual a razão das suas bases, segue que:
40 a 84 + x + 40
3Õ“ b ” y + 35 + 30

40 + 30 + 35 c 35
x+84+y d y
35 + y + 84 e 84
30+40+x 7 x
Assim temos o seguinte sistema de equações:
40 84 + X + 40
30 y + 35 + 30
40 + 35 + 30 35
x + 84 + y y
35 + y + 84 84
30 + 40 + x x
Ora, como só temos duas variáveis, basta pegarmos duas quaisquer dessas
três equações para resolver o sistema. Tomemos, por exemplo, as duas
primeiras:
542 Tópicos de Matemática - Olimpíadas - IME - ITA

4 = 84 + x + 40
3 y + 35 + 30 3x +372 = 4y + 260 ’3x-4y = -112
40 + 35 + 30 35 35x + 35y + 2940 = 105y x-2y = -84
x + 84 + y y
'3x-4y = -112
Multiplicando a segunda equação por -3, segue que
-3x + 6y = 252

Adicionando membro a membro, temos 2y = 140 => y = 70 . Por fim,


substituindo y por 70, por exemplo, na primeira equação segue que
3x-4-70 = -112 => 3x = 168 => x = 56. Portando a medida da área do
triângulo ABC é
(ABC) = 84 + 56 + 40 + 30 + 35 + 70 = 315 I

241) (TREINAMENTO DA OBMEP) Com os dados da figura abaixo, calcule a


razão entre as áreas x e y.

Resolução:
Lembrando que a medida da área de um triângulo é % do produto de dois
lados multiplicado pelo seno do ângulo entre eles, segue que:

x = -10-9senA = 45senA
2
Capítulo 3 - Resoluções 543

x + y = 112 ■ 15 • senA = 90senA => y = 90senA - x = 90senA - 45senA = 45senA

45senA
Assim, — = = 1.
y 45senA

242) (OMRN) O professor Paulinho deixou desatentamente sobre uma mesa dois
exemplares do livro “OLIMPÍADAS DE MATEMÁTICA DO ESTADO DO RN”
! de modo que um dos exemplares cobre parcialmente a capa do outro
conforme ilustra a figura da esquerda representada a seguir.

7b\

Considerando que os exemplares possuem o formato perfeitamente


retangular, descubra, justificando a sua resposta, se a medida da área da
região comum as capas, que está representada sombreada na figura da
direita, é maior, menor ou igual a metade da medida da área total da capa de
cada um dos exemplares.

Resolução:
Sendo b eh os comprimentos dos dois lados do retângulo que representa a
capa de um dos livros segue que a medida da área de uma das capas é
S = bh. Na figura a seguir a medida da área da região comum as duas
capas é A + B.
544 Tópicos de Matemática - Olimpíadas - IME - ITA

bh S „ „ „ . . „ S . „ S
Note que A = . Como B > 0, segue que A + B = — + B> —. Assim a
2 “ 2 2 2

medida da área comum as duas capas é MAIOR que a metade da área total
da capa de um dos exemplares do livro.

1
243) Mostre que o perímetro de um triângulo de área — é maior que 2.
n

Resolução:
Sabemos que a medida da área de um triângulo ABC pode ser calculada por
(ABC) = pr, onde p é o seu semi-perímetro e r é a medida do raio da

circunferência inscrita nesse triângulo. Ora, como estamos supondo que o


1 1 1
referido triângulo tem área — , segue que — = pr => p = — . Por outro lado, a
n n nr
área do circulo inscrito num triângulo é menor que a área do próprio triângulo.
Assim,

nr2 2 (nr)2 < 1 => nr < 1


n
1 2
Ora, como p = — , segue que o perímetro 2p é dado por 2p = — e então:
nr nr

nr <1 => — >1 — > 2 => 2p > 2


nr nr
Capitulo 3 - Resoluções 545

244) (PERU) Determine a razão entre as medidas das áreas hachuradas no


triângulo ABC da figura abaixo.

Resolução:
Indique as medidas das áreas hachuradas por x e y, assim como as medidas
das áreas não hachuradas por u e v. Além disso trace as alturas do triângulo
ABC em relação aos lados BC e AC e indique as medidas das suas alturas
por h e H, respectivamente, conforme ilustra a figura abaixo:

Note que

X + u = (ABD) =
= 1 => 2x - 3y = 3v - 2u
y+v 2
y + v = (ACD) = y = h
546 Tópicos de Matemática - Olimpíadas - IME - ITA

2H
x + v = (ABE) = y = H
=> = 3 => 3x- 2y = 2u- 3v
y + u = (CBE) = ^ y+u 3

Assim.
2x-3y = 3v-2u
3x-2y = 2u-3v

Adicionando membro a membro as duas últimas equações acima, segue que


_ x ,
5x-5y = 0=>x = y=> — = 1

245) (Teorema de Ceva para áreas) Mostre que se num triângulo ABC três
cevianas AE, BG e CD são concorrentes, então o produto alternado das áreas
dos seis triângulos menores formados é constante, ou seja, sendo x, y, z, u, v
e w as medidas das áreas dos seis triângulos menores, então x yz = uv-w.

Resolução:
De fato, o Teorema de Ceva para os lados do triângulo ABC afirma que

CG-AD-BE = GA-DB-EC o que é equivalente a -SÉ = 1. Por outro


GA DB EC
Capítulo 3 - Resoluções 547

lado, usando o fato de que para dois triângulos de mesma altura a razão das
suas áreas é igual a razão das suas bases, segue que
x CG
u " GA
y AD
v ~ DB
_z = BE
w “ EC

Multiplicando membro a membro essas três últimas igualdades, segue que:


x y z ----- x
AD BE = 1=>xyz
CG ----------- = u-vw
U V w GA DB EC

246) (PERU) O triângulo ABC da figura abaixo é retângulo e isósceles.

Se AS é um arco de circunferência de centro B e raio BA e QT é uma

semicircunferência de centro O. Sendo x, y e z as medidas das áreas das

regiões hachuradas, mostre que z = x + y.

Resolução:
De fato, sejam AB = AC = a e u a medida da área da região central da figura,
conforme ilustra a figura a seguir
548 Tópicos de Matemática - Olimpíadas - IME - ITA

Ora. como o triângulo ABC é isósceles segue que. Assim x + y + u


corresponde a área de um setor de 45° de um circulo de raio BA = a, ou seja,

45° 2 na2
x+y+u= na => x + y + u =
' 360°

Note que a medida do raio do semi-circulo inscrito no triângulo ABC é OM.


Além disso perceba que OM = MB = a/2, pois o triângulo OBM é um triângulo
retângulo isósceles, visto que ZOBM = 45° e MB é perpendicular a OM (no
ponto de tangência o raio e a reta tangente são perpendiculares). Por fim
perceba que u+z corresponde a medida da área do semicírculo inscrito no
triângulo ABC. Assim,

1 (a^2 na2
U + Z = —7t
2 \2) 8

Diante do exposto temos que:

na2
x+y+u=—
y + u = u + z=>z = x + y
na2
u + z =-----
8
Capítula 3 - Resoluções 549

247) Na figura abaixo ABCD é um quadrado de lado 1. Sabendo que o ponto E é


ponto médio do lado BC, determine a medida da área do triângulo BGE

Resolução:
Traçando-se a diagonal AC, determinamos o ponto O, centro do quadrado
ABCD. Ora, como E é o ponto médio do segmento BC e O é o ponto médio
do segmento AC (pois as diagonais de um quadrado cruzam-se ao meio),
segue que os segmentos AE e BO são medianas do triângulo ABC. o que
revela que o ponto G é o baricentro do triângulo ABC. Por fim traçamos
também a terceira e última mediana do triângulo ABC. o segmento CF Ora.
como sabemos as medianas de um triângulo dividem o triângulo em seis
triângulos menores de mesma área, conforme ilustra a figura abaixo:
550 Tópicos de Matemática - Olimpíadas - IME - ITA

Como a medida da área do triângulo ABC é a metade da área do quadrado


1 1
ABCD, que tem lado 1, segue que (ABC) = --12 = —. Assim, pelo que foi

exposto, segue que


(BEG) = l(ABC)U.l
12

248) O triângulo ABC da figura abaixo é retângulo e tem catetos de medidas 9 e


12. As duas circunferências são tangentes entre si e a dois lados do triângulo
ABC. Sabendo que as duas circunferências têm o mesmo raio r, determine o
valor de r.

A 12 C

Resolução:
Antes de mais nada, por Pitágoras, temos que

BC = -Js2 + 122 = -J225 = 15


Por outro lado, podemos dividir o triângulo ABC nos triângulos ABD, ACD e
BCD, conforme ilustra a figura abaixo:
Capítulo 3 - Resoluções 551

Assim,
(ABC) = (ABD) + (ACD) - (BCD) =.

912 9-3r 12 r
+ —=> 108 = 54r =>r = 2
2 2 2 2

249) O triângulo ABC representado na figura abaixo é retângulo em A. O diâmetro


AE da semicircunferência se encontra sobre o lado AB do tnângulo ABC Os
lados AC e BC tangenciam a semicircunferência em A e D, respectivamente
Se AC = 5 cm e BC = 12 cm. Determine a medida do raio da
semicircunferência inscrita no triângulo ABC.

Resolução:

Inicialmente, por Pitágoras, BC2 =52 + 122 =>BC = 13. Trace o raio OD da
semicircunferência assim como o segmento OC. Ora, como o ponto C é
externo à circunferência e os pontos A e D são pontos de tangència. segue
que CD = CA = 5 (teorema de Pitot). Além disso, perceba também que AO =
r, o que implica que 08 = 12-r, conforme ilustra a figura abaixo:
552 Tópicos de Matemática - Olimpíadas - IME - ITA

Diante do exposto podemos escrever:


(ABC) = (AOC) + (OCD) + (OBD) =>

12-5 5r
— + — => 18 ■ r = 60 => r = — => r = — cm.
2 2 2 2 18 3

250) (OBM) A circunferência abaixo tem raio 1, o arco AB mede 70° e o arco BC
mede 40°. A área da região limitada pelas cordas AB e AC e pelo arco BC
mede:

Resolução:
Inicialmente trace os raios AO, OB e OC assim como a corda BC Ora, como
o ângulo ZAOB é central e o arco AB mede 70°, segue que ZAOB = 70°,
pois a medida de um ângulo central, coincide com a medida, em graus, do
arco correspondente. Por outro lado como o arco BC mede 40° e o ângulo
ZBAC é um ângulo inscrito, segue que ZBAC = 20° , pois, como sabemos a
medida de um ângulo inscrito numa circunferência corresponde a metade da
medida do arco correspondente a ele. Alêm disso o ângulo ZBOC é um
ângulo central e portanto ZBOC = 40° , pois o arco BC mede 40° e como já
dissemos, a medida de um ângulo central, coincide com a medida, em graus,
do arco correspondente. Note também que o triângulo BOC é isósceles de
Capitulo 3 - Resoluções 553

180° — 40°
base BC, o que releva que ZOBC=ZOCB= ----- = 70° Esses fatos

estão ilustrados na figura abaixo:

Por fim, perceba que ZAOB = ZCBO = 70°,o que revela que AO//BC. Assim
os triângulos ABC e OBC têm a mesma área pois tem a mesma base BC e a
mesma altura, pois AO//BC e a altura de cada um deles é. pois, a distância
entre essas retas paralelas Assim, supondo que a medida da área do
segmento circular situado entre o segmento BC e a linha da circunferência
seja x, temos que
(Região ABC) = (ABC) = (OBC)^x = (setor BOC)

Mas ocorre que OBC é um setor de 40° num circulo de raio 1. Portanto a sua
área é:
40°
(setor BOC) = —nl:R2=—n':12 = —
9 9

Portanto, a medida da área da região limitada pelas cordas AB e AC e pelo

arco BC é — .
9
554 Tópicos de Matemática - Olimpíadas - IME - ITA

251) (Teorema de Viviane) Demonstre que a soma das distâncias de um ponto


qualquer do interior de um triângulo equilátero aos lados do referido triângulo
é sempre igual a medida da altura do triângulo.

Resolução:
Seja P um ponto qualquer no interior do triângulo equilátero ABC de lado a, e
x.y e z as distâncias do ponto P aos lados BC, CA e AB, respectivamente,
conforme ilustra a figura abaixo:

Trançando segmentos que ligam o ponto P a cada um dos vértices do


triângulo ABC formamos três triângulos menores cuja soma das áreas
corresponde a área do triângulo ABC, conforme ilustra a figura a seguir:
Capítulo 3 - Resoluções 555

Assim,
(ABC) = (BCP) + (ACP) + (ABP)

ah ax ay az ah a(x + y + z)
~2+~2+~2 x+y+z=h
2 T 2

Como queríamos demonstrar.

252) São dados: quadrado de lado a e um triângulo equilátero de lado a. como na


figura abaixo. Calcule a área hachurada.

Resolução:
Seja h a medida da altura do triângulo CGH, relativa â base CH. Observe as
seguintes semelhanças de triângulos:

A a D
F

a
H

ax/oX
2 '

K E
B a C h J a—h
556 Tópicos de Matemática - Olimpíadas - IME - ITA

CH CE CH
ACHE-BAE — =>CH = -
BA BE a 2a 2

JE GJ a-h „ , a-h
li) AGJE- AABE=> — =------ => GJ =--------
AB BE a 2a 2

GJ CJ GJ h
ü) acjg-ackf => GJ =
FK CK a Vã 'a
2 2

hV3=^ 2x/3 -1
Assim, h = ---------- a . Portanto, a área do triângulo CGH
11
será

(CGH) = --CH-h = -----^—U (2^-1) a2


(CGH) =
' ; 2 2 2 11 44

OBSERVAÇÃO: Há uma outra saida clássica para essa questão que é a


seguinte:
CH CE CH
ACHE-BAE = —=>CH = -
BA BE a 2a 2

Além disso. ZHCG = 180°-90°-60° = 30°. Por outro lado,


(HCE) = (HCG) + (CGE)

1 1 1
-•CECH = -CH-CGsen30° +—CGCE-sen60’=>
2 2 2
1 a 1 a 1
— a ■ — =-------CG - sen30° + - CG ■ asen60° =>
2 2 2 2 2

cG_2a(273-l)
11
Capitulo 3 - Resoluções 557

Assim,
(CGH) = IcH CG■ sen30’

_1 £ 2a (2^3-l) i
-2 2 11 2
(2V3-l)a2
44

253) Na figura abaixo três quadrados foram construídos sobre os lados de um


triângulo retângulo ABC. Sendo x, y e z as medidas das áreas dos tnângulos
hachurados, mostre que x = y = z.

Resolução:
Sejam a, b e c as medidas dos lados do triângulo retângulo ABC, conforme
ilustra a figura a seguir:
558 Tópicos de Matemática - Olimpíadas - IME - ITA

F c G

c b
No triângulo retângulo ABC, temos que sen9 = — e senp = -
a a
Além disso, olhando para os vértices B e C do triângulo ABC, segue que:
c
Vértice C: 0 + <p + 90° + 90° = 360° => 0 + <p = 180° => sen<p = sen0 = —
a
b
Vértice B: p + a + 90° + 90° = 360° => p + a = 180° => sena = senp = -
a

Portanto,
1 1 c 1
x = -absemp = -ab- = -bc
2 2 a 2

y = — bc
2
1 1 b
z = — acsena = — ac — = — bc
r
2 2 a 2

O que revela que x = y = z, como queríamos demonstrar.


Capítulo 3 - Resoluções 559

254) Seja ABCD um quadrado de lado a . Calcule a medida da área da figura


hachurada, sabendo-se que M e N sâo pontos médios dos lados AD e CD,
respectivamente.

Resolução:
Inicialmente trace a diagonal BD. sejam BF = x, FD = y e BG = z. além disso
nomeie os demais vértices da figura da seguinte forma:

Perceba que os triângulos CDM e BCN são congruentes pelo caso LLL
Assim se ZCBN=a e ZBNC = <p então ZDCM = a e ZCM D = . Olhando

para o triângulo retângulo BCM temos que a + <p = 90°. Por outro lado no

triângulo CGN, o ângulo externo do vértice G é ZCGB = a + q> = 90°. o que

revela que o triângulo BGF é retângulo em G.


560 Tópicos de Matemática - Olimpíadas - IME - ITA

Por outro lado, aplicando Pitágoras ao triângulo NBC, segue que

BN= h)2 +
a2 = —
2
Assim, no triângulo BCN temos que:
a 2
cos a =--- — = —r=
a75 75
2
Agora no triângulo BGF (que é retângulo em G), temos.
BG 2 z 2a
BC 75 a "75

Por outro lado, como BD é uma das diagonais do quadrado ABCD de lado a,
segue que BD = a72. Perceba também que os triângulos DNF e ABF são
semelhantes por apresentarem ângulos dois a dois congruentes, donde
podemos estabelecer a seguinte proporção.

=> X = 2y
a a
2
Assim,

' aV2

x + y = aV2 => 2y + y = aV2 3y = a-Jl => y~


2a 72
X =---------
3
Por fim aplicando 0 teorema de Pitágoras ao triângulo BGF, segue que
2
BF2 = GF2 + BG2
r2a^2 2a
=>GF =
3n/5

Ora. como o triângulo BGF é retângulo (em G) a medida da sua área pode ser
calculada por:
(BGF) = -BG GF = -~ -?^ = —a2
' ' 2 2 75 3^5 15
Capitulo 3 - Resoluções 561

Ora, como os triângulos BEF e BGF são congruentes (caso LLL), segue que a
medida da área do quadrilátero BEFG corresponde ao dobro da medida da
área do triângulo BGF Assim,

4a2
(BEFG) = 2(BGF) = 2=Í-
?5

255) (OPM) Na figura seguinte, AD mede um terço do comprimento de AB e os


pontos E e F dividem o lado BC em três partes iguais. Sabendo que a área do
triângulo ABC é 9cm2, qual é a área da região sombreada ADEF?

Resolução:
Se AD = x, como AD é 1/3 de AB, segue que DB = 2x. Por outro lado, como
os pontos E e F dividem o lado BC em três partes congruentes, sejam BE =
EF = FC = y, conforme ilustra a figura abaixo:
562 Tópicos de Matemática - Olimpíadas - IME - ITA

Note que o triângulo ABF tem a mesma altura do triângulo ABC (partindo do
vértice A). Mas, a base BF = 2y do triângulo ABF é 2/3 da base BC = 3y do
triângulo ABC. Assim, a medida da área do triângulo ABF é 2/3 da medida da
área do triângulo ABC, ou seja,

(ABF) = -(ABC) = - • 9 = 6cm2

Traçando-se o segmento DF, note que os triângulos ADF e ABF tem a mesma
altura partindo do vértice F. Ora, como AD = x é 1/3 de AB = 3x, segue que a
medida da área do triângulo ABF.

Assim,

(ADF) = l(ABF) = 1 -6 = 2cm2

Portanto, (BDF) = (ABF)- (ADF) = 6- 2 = 4cm2 . Por fim, perceba que o ponto

E é o ponto médio da base do triângulo BDF, o que revela que


(DEF) = (BDF) = j = 2cm2 . Assim,

(ADEF) = (ADF) + (DEF) = 2 + 2 = 4cm2


Capitulo 3 - Resoluções 563

256) (OMRN) Um retângulo pequeno está inteiramente contido num retângulo


maior ABCD conforme ilustra a figura abaixo:
D C

A B

Usando somente linhas retas, construa justificando a sua construção), uma


linha que divide a região hachurada em duas partes de mesma área.

Resolução:
O fato primordial para a solução desse problema é perceber que toda reta que

“passa" através do centro de um retângulo o divide em dois polígonos de

mesma área. Então, para solucionarmos o problema traçamos inicialmente as

duas diagonais do retângulo maior, o que determina o seu centro O, conforme

ilustra a figura a seguir:

D C

A B
564 Tópicos de Matemática - Olimpíadas - IME - ITA

Agora tracemos também as diagonais do retângulo menor, determinando o


seu centro P, conforme ilustra a figura abaixo:
D C

A B
Assim a reta (t) definida pelos ponto O e P divide cada um dos retângulos em
dois polígonos de mesma área e portanto divide a região hachurada em duas
regiões de mesma área, conforme ilustra a figura a seguir:

A B

Para ver isso de forma mais clara veja das figuras a seguir:

D C

S
O
(*)
s

A B
Capítulo 3 - Resoluções 565

D C

A B

onde representamos por S (na figura de cima) a medida da área de cada um


dos polígonos em que o retângulo ABCD fica dividido pela reta (t) e por u a
medida da área de cada um dos polígonos em que o retângulo interno fica
dividido pela reta (t) (figura de baixo). Diante do exposto, as medidas das
áreas das regiões sombreadas que ficam acima e abaixo da reta (t) quando
excluímos o retângulo menor no interior do retângulo maior são iguais a S - u.

257) Um retângulo é dividido em 9 retângulos menores conforme ilustra a figura


abaixo:
D C

12 4 6

A B

Os números indicados no interior dos retângulos menores correspondem as


suas respectivas áreas. Qual a medida da área do retângulo maior?
566 Tópicos de Matemática - Olimpíadas - IME - ITA

Resolução:
Antes da resolução propriamente dita, vamos demonstrar a seguinte relação:
se um retângulo ABCD é subdividido em quatro retângulos menores de áreas
x, y, z w e w, conforme ilustra a figura abaixo:

a b
D E

c X y C
G
F H

d z w
d

A a * b B

Então xw = yz.

De fato, se a, b, c e d representam as medidas dos lados dos retângulos


menores (como ilustra a figura acima),
segue que:
x = ac
=> xw = abcd (1)
w = bd

íz = ad
=> yz = abcd (2)
|y = bc

De (1) e (2), segue que xw = yz, como queríamos demonstrar.


Voltando ao problema proposto, se chamarmos de x,y,z e w as medidas das
áreas dos retângulos menores cujas áreas são desconhecidas, temos:

D E C

x 6 y
JM O H G

12 4 6
J H
Q F
z 8 w

A L M B
Capitulo 3 - Resoluções 567

Portanto, utilizando a propriedade que demonstramos no início da resolução,


segue que
Retângulo DEQJ -> x-4 = 12 6 => x = 18
Retângulo ECHQ -> 6 -6 = 4-y = y = 9

Retângulo NGBL 4-w = 6-8 => w = 12


Retângulo INLA —> 12-8 = 4-z =>z = 24
Ora, como a medida da área do retângulo maior corresponde a soma das
medidas das áreas dos retângulos menores, segue que
(ABCD) = 24 + 8 + 12 + 6 + 4 + 12 + 9 + 6 + 18 = 99 u.a.

258) (OBMEP-Adaptada) A figura mostra um retângulo de área 42 cm2 com os


pontos médios dos lados em destaque. Qual é a área, em cm2, da região
cinza?

Resolução:
Trace as duas diagonais do retângulo e nomeie os seus vértices como ilustra
a figura abaixo:
568 Tópicos de Matemática - Olimpíadas - IME - ITA

Ora. como M e N são, por hipótese, pontos médios dos lados AB e BC.
respectivamente, segue que AN e CM são medianas dos triângulo ABC e
portanto o ponto G é o seu baricentro. Traçando-se a terceira mediana BO,
desse triângulo, segue que o triângulo ABC fica dividido em seis triângulos
menores de mesma área (as medianas de um triângulo o dividem em seis
(ABCD) 42
triângulos de mesma área). Ora, como (ABC) = i—-—- = — = 21, segue

que

(AOG) = (COG) = ^(ABC) = 1 ■ 21 = |

Assim,

(ACG) = (AOG) + (COG) = | 1 = 7

Raciocinando de modo completamente análogo com o triângulo ACD,


podemos concluir que

(ACG') = (AOG') + (COG’) = | +1 = 7

Portanto,

(AG ’ CG) = (ACG') + (ACG) = 7 + 7 = 14 cm2

259) Dada uma folha retangular ABCD de lados a e b , com a>b, é dobrada ao
longo da sua diagonal , determine a medida da área da região que fica
sobreposta, indicada pela parte hachurada na figura abaixo.

D
a

A a
Capítulo 3 - Resoluções 569

Resolução:
Após a dobra do papel ao longo da diagonal AC, seja E a interseção dos
segmentos AB e CD. Sendo CE = x, como CD = a, segue que DE = a - x.
sendo ZCEB = a, segue que ZDEA = ZCEB = a , pois são ângulos opostos
pelo vértice. Sendo ZBCE = <p , segue que <p = 90°-a, o que revela que

ZDAE = 90° - a = <p , conforme ilustra a figura a seguir

a—x

Note que os triângulos BCE e DAE sâo congruentes (caso ALA. pois BC = AD
= b, ZEBC = ZEDA = 90° e ZBCE = ZDAE = <p), o que relava que

BE = ED = a - x e AE = CE = x. Aplicando Pitágoras ao triângulo BCE,


segue que
2 , \2 .2 a2+b2
x =(a-x) +b =>x = —------

Por fim,
>1
(ACE) = lx ■ xsen (180° - a) = — x2senct

Mas, no triângulo BCE, segue que sena = - . Assim,


x
(ACE) = ix2sena = -x2 •— = -bx
v ' 2 2x2
570 Tópicos de Matemática - Olimpíadas - 1ME - 1TA

a2 + b2
Ora, como x = —- -----, segue que:

(ACE)=
k
ibf^l=±M
1 2 < 2a J 4a

260) O pentágono ABCDE consiste de um quadrado ACDE cujo lado mede 8cm, e
um triângulo isósceles ABC, tal que AB=BC. Se a área do pentágono mede
90cm2 determine a medida da área do triângulo BEC.

Resolução:
Ora, como ACDE é um quadrado de lado 8cm, traçando-se AC. segue que
AC = 8cm CE = 8^2cm e ZACE = 45° (pois CE é uma diagonal do
quadrado ACDE e portanto bissetriz do ângulo interno do vértice C), Sejam
AB = BC = x e ZBCA = a, conforme ilustra a figura a seguir:
Capitulo 3 - Resoluções 571

(X C
A
8 45°

8 8
8\/2

E D
8

Como (ABCDE) = (ACDE) + (ABC), segue que:

o 1 13
90 = 8 + — ■ 8 • xsena => xsena = —
2 2
Por outro lado, como o triângulo ABE é isósceles, traçando a sua altura AH
relativa ao lado AC, segue que AH = HC = 4 (pois num triângulo isósceles a
altura relativa a base também é mediana da base), conforme ilustra a figura
abaixo:
B

x X

a
A
4
□1
H 45’
C

8 8
8>/2

E D
8
572 Tópicos de Matemática - Olimpíadas - IME - ITA

4
No triângulo BCH, temos que cosa = — => xcosa = 4 . Mas ocorre que

(BCE) = ^BC-CE-sen(45° + a)

Por outro lado,


sen (45° +a) = sen45°cosa + senacos45°
72 72
= —cosa + — sena
2 2
= -^-(cosa + sena)

Assim,
1
(BCE) = BC • CE ■ sen (45° + a)
2
2 •x-872 72
— (cosa + sena)
2
= 4(xcosa + xsena)

= 42cm2

261) Um certo número de quadrados unitários são dispostos conforme a figura ao


lado. Sendo O o vértice do canto esquerdo inferior do primeiro quadrado, P e
Q os vértices dos cantos superiores direitos dos quadrados 2011 e 2012
desta sequência de quadrados, determine a medida da área do triângulo
OXY.

P Q

O
Capitulo 3 - Resoluções 573

Resolução:
p o

O T R S

Na figura acima note que os triângulos OXT e POR são semelhantes (pois
apresentam ângulos dois a dois congruentes). A razão de semelhança desses
. . OT __ 1
dois triângulos é----- o que revela que
~' OR ~ 201 r

Í2ÍI1 = [—!—] (OXT) = —!—(OPR)


(OPR) I2011J ' ' 20112

Mas ocorre que (OPR) = ^OR-RP = ^2011-1 = , o que nos permite

concluir que
(OXT) = —(OPR) = 122°11 1
t > 20112 20112 2 4022
Analogamente, perceba que os triângulos OYT e OQS também si
semelhantes (pois também apresentam ângulos dois a dois congruentes). A
OT 1
razão de semelhança desses dois triângulos e
é —— = ■ , o que revela que
ZU IZ

(OYT) . f —2 => (OYT) = —Ly (OQS)


(OQS) <20122 ' ' 20112

1 1 2012
Mas ocorre que (OQS) = ^OSSQ =-2012-1 =, o que nos permite

concluir que
1 2012 1
(0YT) = ^?(°QS) = 20122 2 4024
Assim,
(OXY) = (OXT)-(OYT) = ^ 1 1
4024 2x2011x2012
574 Tópicos de Matemática - Olimpíadas - IME - ITA

262) A primeira quadratura rigorosa de uma área curvilinea, conhecida como


Lúnulas de Hipócrates, feita por volta de 430 a.c. pelo filósofo e geômetra
de mesmo nome, está descrita na construção abaixo.

Os arcos ACB, BFC e AEC são semicircunferências.


Se as medidas dos diâmetros AC e BC são. respectivamente, iguais a 3cm e
4cm , calcule a soma das áreas das regiões sombreadas.

Resolução:
Sejam AB = c, BC = a e AC = b as medidas dos lados do triângulo ABC e T, x,
y, z e w as medidas das áreas indicadas na figura abaixo:

Note que z+w+T corresponde a medida da área de um semicírculo de raio


c/2, ou seja,
2
t + z + w = —1 rt M
T —
2 UJ (1)
Capítulo 3 - Resoluções 575

z + x corresponde a medida da área de um semicírculo de raio b/2. ou seja.


2

z+x=r(í) (2)

e, por fim, y + w corresponde a medida da área de um semicírculo de raio a/2.


ou seja,
2

w+y=r® (3)

Fazendo (1) - (2) - (3), obtemos:


(T + z + w)-(z + x)-(w + y) = \ (c2 -b2 -a2|

Por outro lado, como o triângulo ABC é retângulo. segue que

c2 = b2 + a2 => c2 - b2 - a2 = 0 . e, portanto.

(T + z + w) - (z + x) - (w + y) = - n (c2 - b2 - a2) T-x-y = O=>x + y = T



ou seja, a soma das medidas das áreas das lúnulas é igual a área do
triângulo ABC. Ora, como o triângulo AVC é retângulo de catetos 3cm e 4cm.

segue que T = 3-1 = 6cm2 , o que revela que

x + y = 6cm2.

263) Na figura abaixo o triângulo ABE é isósceles de base AB, ZBAC - 30° e
ZACB = ZAFC = 90° . Determine a razão entre as áreas do triângulo ESC e
do triângulo ABC.

A B
576 Tópicos de Matemática - Olimpíadas - IME - ITA

Resolução:
Seja AB = 2a. Como o triângulo ABC é retângulo em C, segue que nesse
triângulo:
BC ^ = BC = a
sen30° = — =>
AB 2 2a
73 AC
cos 30° = —=> AC = 3n/3
AB 2 " 2a
Assim. a medida da área do triângulo ABC é

AC-CB ■a a 2 73
(ABC) =
2 2 2
Por outro lado, como o triângulo ABE é, por hipótese, isósceles, segue que
ZEBA = ZEAB = 30°. Olhando para o triângulo retângulo ACF,segue que
ZACF = 180°- 90° - 30° = 60°
Ora. como ZACB = 90° , segue que
ZFBC = ZACB-ZACF = 90°-60° = 30°. Assim no triângulo CBF, segue
que ZCBF - 180°-90° - 30° = 60°. Ora, como ZEBA = 30°, segue que
ZCBE = ZCBF - ZEBA = 60° - 30° = 30° , como ilustra a figura abaixo:

No triângulo retângulo CBE,


73 .CE^ ax/3
tg30° = — =>
CB 3 a 3
Capitulo 3 - Resoluções 577

No mesmo triângulo CBE, ZCEB = 180°-90°-30’ = 60’. Olhando para o


triângulo ESC.
ZCSE = 180’ - 60° - 60° = 60’
o que releva que o triângulo ESC é eqüilátero pois possui três ângulos
congruentes. Assim,

CE2-73
M 2.
l, 3 J •73 a2j3
(ESC) =
4 ~ 4 12
Portanto a razão entre as medidas das áreas dos triângulos ESC e ABC é

a2V5
(ESC) 2
(ABC) a2V3 6
2

264) Très quadrados ABGH, DEJI e BCDF são arranjados conforme ilustra a figura
abaixo:

E
A

Mostre que os dois triângulos hachurados possuem áreas iguais

Resolução:
Seja a a medida do lado do quadrado BCDF. Sendo zFDI = « . segue que
ZIFD = 90’-a = <p
578 Tópicos de Matemática - Olimpíadas - IME - ITA

Ora, como ZIFD = 90°-a = ip , segue que ZBFG = 90°-<p = a e portanto

ZGBF = 90°-a = <p , No triângulo FDI, temos Dl = asemp . Como DEJI é, por

hipótese, um quadrado, segue que DE = Dl = asemp .

Analogamente no triângulo BFG, BG = asena. Assim, como ABGH é, por


hipótese, um quadrado, segue que AB = BG = asena .
A figura abaixo ilustra todas essas informações:

c
180° - a
180° - ç> a
(1
p E
' ( &. ti a.semp
Cl
a..$enal_ a
■P a a.sentp
a. seita
a
H G F J

Diante do exposto, temos:

(ABC) = AB ■ BC ■ sen (180° - <p)

= ^-(asena)asen<p

= ^a2senasen<p

Analogamente,

(DCE) = 1 DE ■ CD ■ sen (180° - a)

= ^(asen(p)a -sena

= ^a2sen<psena

O que, finalmente, revela que (ABC) = (DCE), como queríamos demonstrar,


Capitulo 3 - Resoluções 579

265) Qual a medida da área de um pentágono regular de lada a?

Resolução:
360°
A medida do ângulo central num pentágono regular é —— = 72°. Note que o

pentágono regular ABCDE pode ser dividiso em 5 triângulos isósceles de


lados x, x e a, conforme ilustra a figura abaixo:

Ora, como o triângulo ODE é isósceles de base DE, segue que


180°-72°
ZOED = ZODE = = 54°
2
Assim, aplicando a lei dos senos no triângulo ODE. segue que
x a sen54°
(1)
sen54° sen72° sen72°

Por outro lado, sabemos que sen54° = e cos72° = ~. onde ç> = —

(número de ouro).

Assim, sen72° = v1-cos2 72° = 1- -^ = ^^-1


4<p2 2<p
580 Tópicos de Matemática - Olimpíadas - IME - ITA

1
sen54° 2'P <p2
o que revela que e, portanto, por (1),
sen72° _1_ 74<p2-1
2<p
X2-i
segue que:
sen54° tp2
x = a------- — => x = a ,
sen72° ^4lp2-1

Por fim,
(ABCDE) = 5-(ODE)
= 5 — x x sen72° = — x2 — Jlw2 -1
2 2 2(pv

5f
— a
<p2
4 K2 - <p

5 <p3 = a2
4
V4|p2 - 1
. 75 + 1 • •
onde <p = —-— e numero de ouro

266) Dado um hexágono regular ABCDEF de lado 2, em que AD e CE intersectam-


se em G, determine a medida da área do quadrilátero ABCG.
Capítulo 3 - Resoluções 581

Resolução:
Trace as diagonais BE e CF do hexágono ABCDEF, conforme ilustra a figura
abaixo:

É um fato conhecido que todo hexágono regular fica dividido por três das suas
diagonais em seis triângulos equiláteros cujo lado tem a mesma medida do
lado do hexágono. Assim, na figura acima, os triângulos ABO. BCO e CDO
são equiláteros de lado 2. Além disso CG é mediana do tnângulo CDO. pois
os triângulos CGO e CGD são congruentes (pelo caso LAL, pois CG é lado
comum aos dois triângulos ZOCG = ZGCD = 30° e CO = CD = 2). o que
revela que o triângulo CGO é a metade de um triângulo equilàtero
Lembrando que a medida da área de um tnângulo equilátero de lado a é

a2 73
4 '

segue que
(ABCG) = (ABO) + (BCO) + (CGO)
2275 22V3 1 22V3
=------- +-------- +------------
4 4 2 4
5 73
=------ u.a.
2
582 Tópicos de Matemática - Olimpíadas - IME - ITA

267) Um dodecágono regular é um polígono regular é um polígono regular com 12


lados Se a distância entre dois vértices opostos de um dodecágono é de 4
cm. determine a medida da área deste dodecágono em cm2.

Resolução:
Ora, como o dodecágono é regular, ele pode ser subdividido em 12 triângulos
isósceles conforme ilustra a figura abaixo:

Note que a medida do ângulo ZAOB é 333 = 30°. Assim, a medida da área
*

do dodecágono é S = 12-(AOB) = 12 ■—-2-2-sen30° = 12 cm2


2 "TvT"
Capítulo 3- Resoluções 583

268) No diagrama ao lado os segmentos AB, CE e FH são paralelos. Além disso


os segmentos AF e BG são perpendiculares a AB. Suponha que a área do
retângulo ABCD é x, a área do retângulo CDGF é y, e área do triângulo BDE
é z. Determine a área de DEHG em termos de x, y e z.
A B

c D E

F G H

Resolução:
Seja w a medida da área do quadrilátero DEHG, conforme ilustra a figura
abaixo:
A B

C D

y w

F G H

Note que
(ABCD) BD CD BD x BD
(CDGF) ~ DG-CD " DG y DG

y DG y+ x DG + BD y+x BG BD x
x " BD x BD x BD BG x+y
584 Tópicos de Matemática - Olimpíadas - IME - ITA

Por outro lado, os triângulos BDE e BGH são semelhantes (pois apresentam
BD
ângulos dois a dois congruentes), como razão de semelhança — . Assim,
BG

(BDE)
(BGH) = fóJ^BGH’ = Z'
Mas ocorre que (BGH) = (BDE) + (DEHG). Portanto,

2
(BGH) = (BDE) + (DEHG) => z- = Z + (DEHG)

'f \2
(OEHG)-.. (iil) -1

269) (UFG) A "árvore pitagórica fundamental" é uma forma estudada pela


Geometria Fractal e sua aparência característica pode representar o formato
'os galhos de uma árvore, de uma couve-flor ou de um brócolis, dependendo
sua variação. A árvore pitagórica abaixo foi construída a partir de um
angulo retângulo, ABC, de lados AB = 3, AC = 4 e CB = 5, e de quadrados
onstruidos sobre seus lados. A figura ramifica-se em quadrados e triângulos
retângulos menores, semelhantes aos iniciais, sendo que os ângulos C, F e I
são congruentes, seguindo um processo iterativo que pode se estender
infinitivamente.
Capítulo 3- Resoluções 585

Com base nessas informações, calcule a área do triângulo GHI, integrante


dessa árvore pitagórica.

Resolução:
O fato dos ângulos dos vértices C, F e I serem congruentes juntamente com o
fato de que os ângulos dos vértices de quadrados são sempre de 90’. revela
que
Z8CA = ZEFD = ZHIG = a
O que revela que ZCBA = ZFED = ZlHG = 90°-a e. portanto, que os
triângulos ABC, DEF e GHI sâo semelhantes (pois possuem ângulos internos
! dois a dois congruentes).
Assim, olhando para os triângulos semelhantes DEF e ABC podemos
escrever a seguinte proporção’
DF EF DF AC DF 4 16
— => DF = —
AC “ BC AC “ BC 4 5 5
16
Ora, como o quadrilátero DFIH é um quadrado, segue que Hl = DF = —.

Assim a razão de semelhança dos triângulos GHI e ABC é

16
k = 2ÍL = Z = J6 . Ora, como a razão entra as medidas das áreas de duas
BC 5 25
figuras semelhantes é igual ao quadrado da razão de semelhança, segue que

(GHI) 2 (G-HA = f2®']2 =. (GHI) = — (ABC)


(ABC) (ABC) I25J ' ' 625 '

3,4
Mas ocorre que (ABC) = -y = 6 . Assim,

(GHI) = ||| (ABC) => (GHI) = ||| • 6 (GHI) = u a.


586 Tópicos de Matemática - Olimpíadas - IME - ITA

270) O triângulo ABC da figura a seguir tem área igual a 1. Cada um de seus lados
foi dividido em três partes iguais. Calcule a área do triângulo sombreado.

Resolução:
Inicialmente nomeie os pontos da seguinte forma:

Trace o segmento CL Note que os triângulos BFL e CFL tem a mesma altura
partindo do vértice L. Como CF = 2FB, segue que a medida da área do
triângulo CFL é o dobro da medida da área do triângulo FBL. Assim, se
(FBL) = x => (CFL) = 2x e de modo completamente análogo, se (CHL) = y

então (AHL) = 2y , conforme ilustra a figura a seguir:


Capitulo 3 - Resoluções 587

Por outro lado, olhando AC como base do triângulo ABC, segue que o
triângulo BCH tem a mesma altura (partindo do vértice B) do triângulo ABC.
mas tem 1/3 da sua base AC, o que revela que (BCH) = - (ABC) = 1-1 = 1

Mas ocorre que (BCH) = 2x + y , o que nos permite concluir que 2x - y = 1

Por outro lado, olhando BC como base do triângulo ABC, os triângulos ACF
ABC tem a mesma altura (partindo de A), mas a base do triângulo ACF é 2/3
2 2 2
da base BC do triângulo ABC, o que revela que (ACF) = -(ABC) = --1 = -

Mas ocorre que (ACF) = 3y + 2x , o que nos permite concluir que

„ 2
3y + 2x = -,

Diante do exposto, temos o seguinte sistema de equações lineares:

2x + y = -
' 3 1 4
=>x = — ey = —
, 2 21 21
3y + 2x = -
588 Tópicos de Matemática - Olimpíadas - IME - ITA

Por fim, podemos dividir a figura originai da seguinte forma:

x G
H

2x 2x
vF

?/- x l> x '


M i/ - x
-~"x 2:i:
A J E B
2
Ora. como (ACF) = —, segue que

2
(ACF) = (KLM) + (ACM) + (CKLF) => - = (KLM) + 2y + 3x=>

| = (klm)+A^=>(Klm) = 1

271) (OMPLP-2012) Um quadrilátero ABCD está inscrito numa circunferência de


centro O. Sabe-se que as diagonais AC e BD são perpendiculares. Sobre
cada um dos lados do quadrilátero ABCD construimos semicírculos,
externamente, como mostra a figura.

a) Mostre que os triângulos AOB e COD têm a mesma área.


b) Se AC = 8 cm e BD = 6 cm, determine a área da região sombreada.
Capitulo 3 - Resoluções 589

Resolução:
a) Trace os raios AO, OB, OC e OD, determinando os ângulos centrais
ZCOD = a e ZAOB = . Note que os arcos CD e AB medem a e rp ,
respectivamente, pois a medida, em graus, de um arco é igual a medida,
em graus do ângulo central correspondente, conforme ilustra a figura
abaixo:

Por outro lado o ângulo excêntrico AO'B é reto. Assim,

00°„ med(AB) + med(CD)


=>9Q° = ÍLÍíí.=.a + cp = 180°
2
Ora, como a + <p = 180° , temos que sena = senip . Portanto,
1 1
(AOB) = - R • R • senrp = — R2sena

1 1
(COD) = - R ■ Rsenct = -R2sena

o que revela que (AOB) = (COD), como queríamos demonstrar.

b) A medida da área de um quadrilátero ABCD em função das diagonais

AC e BD é (ABCD) = AC-BD-sena onde a é a medida do ângulo

entre as diagonais. Assim, no caso do quadrilátero ABCD dessa questão


temos: AC = 8cm, BD = 6cm e a = 90° , o que revela que
590 Tópicos de Matemática - Olimpíadas - IME - ITA

(ABCD) = iACBDsena = ^8-6sen90° = 24cm2

Sejam AB = 2x, BC = 2y, CD = 2z e DA = 2w as medidas dos lados do


quadrilátero ABCD, conforme ilustra a figura abaixo:

Lembrando que a + <p = 180° => cosíp = - cosa e aplicando a lei dos

cossenos aos triângulos ABO e CDO, segue que:

AABO —» (2x)2 = R2+R2-2■ R-R■ coscp


=-cosa

4x2 =2R2+2R2 cosa

ABCO (2z)2 = R2 + R2-2RRcosa


4z2 =2R2-2R2cosa

Adicionando, membro a membro a duas últimas igualdades acima, segue que]


4(x2 + z2j = 4R2 => x2 + z2 = R2

Trabalhando de modo completamente análogo com os triângulos ADO e BCO

podemos verificar que y2 + w2 = R2 .


Capitulo 3 — Resoluções 591

Agora, sejam P, Q, T, S, p, q, t e s as medidas das áreas indicadas na figura


abaixo:

Assim,

S + s = - nx2
2
1 2
Q + q = 2nz
=> (adicionando membro a membro)
T +1 = — y2
2n
P + p = — rtw2
2

(S + Q + T + P) + (s + q + t + p) = i n (x2 + z2j + ^y2 + w2j =>

(S + Q + T + P) + (s + q + t + p) = ln(2R2)=.

(S + Q + T + P) + (s + q + t + p) = nl:R2 (1)

Mas,
(ABCD) + (s + q + t + p) = nR2 (2)
592 Tópicos de Matemática - Olimpíadas - IME - ITA

Por fim de (1) e (2), segue que


(S + Q + T + P) + (s + q + t + p) = (ABCD) + (s + q +1 + p) =>

S + Q + T + P = (ABCD) = 24cm2

que é justamente a soma das medidas das áreas hachuradas.

272) Determine as posições dos pontos E e F sobre os lados AB e BC do retângulo


ABCD para que os triângulos ADE, BEF e CDF possuam áreas iguais.

Resolução:
Considere as medidas indicadas na figura a seguir:

b-y
Capitulo 3 - Resoluções 593

Assim,
xb (a-x)y bx
(ADE) = (EBF) =>y =------ (1)
2 ~ 2 a-x

xb a(b-y)
(ADE) = (CDF) => bx = ab - ay (2)
T" 2

Substituindo (1) em (2), segue que:


. bx ax 2 o 2 „
bx = ab - a------ => x = a--------- => x - 3ax + a = 0 =>
a-x a-x

(3 + 75)a (3-V5)a
x= OU X “ 2
2

Note que
3 + 75 ;i1^x_(3 + ^)a > a, o que não é possível. Assim,
2 2

(3-7s)a
x= , o que revela que
2
(3-75) a (75-1)
a-x = a- 2 = 2 3

(3-75)a
bx b'
Ora, como y =------ , segue que y = —
a-x
a-
2
(3-75' ^y=mb
2
Assim, os pontos E e F dividem os lados AB e BC em razão áurea.

273) Dado um quadrilátero convexo ABCD em que as suas diagonais AC e BD


formam um ângulo a. Mostre que (ABCD) =-^AC-BD-sena .
594 Tópicos de Matemática - Olimpíadas - IME - ITA

Resolução:
De fato, seja E o ponto de encontro as diagonais AC e BD como ilustra a
figura abaixo:

(ABCD) = (ABE) + (BCE) + (CDE) + (ADE)


1 11 1
= — xwsen (180° - a) + — ywsena + — yzsen (180° - a) + — xzsena

1111
= — xwsena + — ywsena + — yzsena + — xzsena
2 2 2 2
= x (w + z) sena + y (w + z)

= (w + z) (x + y) sena
=BD =AC

= —ACBD-sena
2

274) (OBM) Uma sala quadrada com 81 m2 de área tem o seu piso inteiramente
coberto por dois tapetes retangulares A e B, que não se superpõem, conforme
mostrado na figura (1) abaixo. Em certo momento, o tapete B é deslocado, o
tapete A é girado de 90° e colocado sobre o tapete B, conforme indicado na
figura (2).
(1) (2)

B B
A
>
1
Capitulo 3 - Resoluções 595

Sabendo que a área do tapete B é o dobro da área do tapete A, e que as


medidas dos seus lados são dadas por números inteiros, calcule a área da
parte do piso que ficou descoberta.

Resolução:

O) _ (2)
B B i
A
1

Na figura 1 chamamos a área de A de x e a de 8 de 2x. Teremos então 3x =


81 m2 e x = 27 m2, então a área de A - 27 m2 e seus lados são: 3 e 9; área

de 8 = 54m2 e seus lados 6 e 9.

Assim a região de interseção dos dois tapetes na figura (2) é um retângulo de


dimensões 3 por 6, ou seja, é uma região de área 3x6 = 18m2. Ora. como os
dois tapetes são capazes de cobrir exatamente a sala, quando não
sobrepostos, segue que a região não coberta pelos dois tapetes na figura (2)
é equivalente (mesma área) a região descoberta, o que revela que a parte do
piso que ficou descoberta na figura (2) é de 18m2.

275) (IME)Considere o quadrado XYZW lado a. Dividindo-se cada ângulo desse


quadrado em quatro partes iguais, obtém-se o octógono regular representado
na figura abaixo. Determine o lado e a área desse octógono em função de a.
As respostas finais não devem conter expressões trigonométricas.
596 Tópicos de Matemática - Olimpíadas - IME - ITA

Resolução:
Trace a diagonal XZ, o segmento AO, onde O é o centro do quadrado (e do
octógono) e A é o ponto médio do lado XY, conforme ilustra a figura abaixo:

«75

XY a
Ora, como o lado do quadrado XYZW mede a, segue que XA =
2 2 '

XW a e OX = ^ = ^
OA =----- Assim, na figura acima temos que
2 2 2 2
a
x + y = — e pelo teorema das bissetrizes internas de um triângulo, aplicado
2
x y
ao triângulo XAO, temos —= — => x = -l2y . Temos então o seguinte
aV2
2 2

sistema de equações lineares:


a (2->/2)a (s/2 - l)a
x+y=—
2 =>x “ 2
e y = -------- —
2
x = x/2y
Capitulo 3- Resoluções 597

360°
Agora observando o triângulo CBO, segue que ZBOC =------ = 45°.
8
a
X 2 A Y
TcT

aV2 ■^Ti
2
X X

W z
Seja z a medida do lado do octógono (que é regular pela simetria da própria
construção do octógono). Aplicando a lei dos cossenos ao triângulo BOC,
segue que:

z2 = X2 + x2 -2xxcos45° =>

z2 = 2x2^1-^J => z = x^/2-72 => z = X-J2-V2

(2-V2)a
Mas ocorre que x = . Assim a medida z do lado do octógono é:
2

(2-^)a ^2-^ ~ 2-
=>z
z
2 2
Finalmente a medida da área S do octógono corresponde a oito vezes a
medida da área do triângulo BOC. Assim, S = 8 (BOC).

Mas ocorre que

(BOC) = — x-xsen45° = — x2^- 72|~(2-^)a' 2 , x V2(2-^)2a2


=» (BOC) = -A__J----
' > 2 2 2 4 2
598 Tópicos de Matemática - Olimpíadas - IME - ITA

Portanto.

, x ^2(2-^fa2
S = 8 • (BOC) = 8----- i- '-----

x^(2-j2)2a2 -J2 (6-4v/2)a2 275(3-2^) a2


S ” 2 “ 2 2

S = ^(3-272) a2 =(3^-4) a2

276) (IME) Sejam p o semi-perimetro de um triângulo, S sua área, r e R os raios de


suas circunferências inscrita e circunscrita, respectivamente. Demonstre que
vale a seguinte desigualdade:

^S.r-R^
9 27

Resolução:

Inicialmente vamos mostrar que r-R<Sy. De fato, sendo a,b e c as

medidas dos lados do triângulo, segue que a,b e c são números reais não
negativos e portanto, pela desigualdade entre as médias aritmética e
geométrica, temos que:

a +b+c (a + b + c)3
Mg £Ma => abc < (1)
3 27

Por outro lado, sabemos que 2p=a + b + c, S = p-reS = ^. Substituindo

essas informações em (1), segue que:

abc s : 4RS £ => 4Rpr < 8pi =>rR<


27 ' 27 27 27
que é justamente a desigualdade desejada.
Capítulo 3- Resoluções 599

O-/3
Agora vamos demonstrar a outra desigualdade, ou seja, —Ssr-R. De

fato, como S = pr segue que

■yS<rRayp.r<r-R«yp<R

a+b+c
Por outro lado,
P = ~2~ e a = 2RsenA, b = 2RsenB e c = 2RsenC .
Assim,

2J3 2>/3f a + b + c <r 2 73 í 2R(senA + senB + senC)^


——p < R <=>------
9 9 2

í^-(senA + senB + senC) < 1 o senA + senB + senC <

9 73
senA + senB+senC < o senA + senB + senC s
273 73 2

Mas isso de fato ocorre, pois A + B + C = 180° e


senA + senB + senC A+B+C
------------- z------------- s sen (’), o que revela que:
3 3

senA + senB+ senC ( A+B+C


--------------------------- < sen -
3 l
senA + senB + senC < sen 180°
-------------------
3 3 .
sen A + sen B +senC
< sen60° =>
3

senA + senB + senC 73


3

373
senA + senB + senC <--
3
Como queríamos demonstrar.
600 Tópicos de Matemática - Olimpíadas - IME - ITA

OBSERVAÇÃO: Na solução acima foi crucial o uso da desigualdade (*) Há


algumas maneiras de estabelecê-la. Há um resultado mais geral, não apenas
para a função seno que é o seguinte:

Teorema: (Desigualdade de Jensen) Sejam I um intervalo da reta ef.! R

uma função. Se x,,.. ,xnel e t, tne[0,1], comt,+... + tn = 1, então

tlx1+... + tnxn el e

i) f convexa =>f (t1x1 ‘nXn)á‘if(xi) + -+tnf(xnJ

Ü) f côncava ^ff^x.. fnxn) ^»1f(xl) ‘nf(xn)

Uma demonstração para esse teorema pode ser vista no livro Análise Real,
volume 1 - Elon Lages Lima - SBM - Coleção Matemática Universitária.

No caso da nossa questão a função f:[O,rt]—>R dada por f(t) = sent é uma

função côncava (Se para todos a < b em / o gráfico de f entre as retas x = a e

x = b estiver acima da reta que passa por (a, f(a)), (b, f(b)), então fé côncava,

•i
e reciprocamente). Assim tomando t-, = t2 = t3 = - e x, = A, x2 = B e x3 = C,

segue que

t,f (X1) + t2f (x2) + t3f (x3) < f (tlXl + t2x2 + t3x3) =.

1 11 11 ("1 1 1 'l
-senA + -senB + -senC < sen -A + -B + -C =>
3 33 33 \3 3 3 /

senA + senB + senC A-BrÇ


---------------------------- < sen
3 3
como queríamos demonstrar.
Capitulo 3 - Resoluções 601

277) Na figura abaixo temos um triângulo ABC e a sua circunferência ex-inscrita


tangente ao lado BC. Sendo P, Q e R os pontos de tangência a circunferência
com as retas suportes dos lados AB, BC e CA, respectivamente, sendo p o
semi-perímetro do triângulo ABC, mostre que:

a) AT = AQ = p - c e CQ = CP = p - a .

b) BP = BT = p .

c) (ABC) = (p-b)-rb.

d) (ABC) = 7p(p-a)(p-b)(p-c) (FÓRMULA DE HERON).

Resolução:
a) Se AT = x, segue pelo teorema de Pitot, que AQ = AT = x.
Analogamente, se CQ = y, segue pelo teorema de Pitot, que CP = CQ =
y, o que revela que x + y = b. Por outro lado, mais uma vez pelo
teorema de Pitot, temos que BT=BP, o que implica que:
BT = BP=>c + x = a + y=>x-y = a- c
Temos então o seguinte sistema de equações lineares
x+y=b b+a-c b+c-a
=> x =----------- e y =------------
x-y = a-c 2 2
602 Tópicos de Matemática - Olimpíadas - IME - ITA

Mas ocorre que 2p = a + b + c . o que revela que:

b + a- c = b + a + c-2c = 2p-2c = 2(p-c)


b + c- a = b + c + a-2a = 2p-2a = 2(p-a)

Assim,
b + a-c 2(P~C)
x =----------- = p-c
2 2
b + c-a = ^ = P-a
y =----------
2

b) De fato, por Pitot, temos que BP = BT. Mas,


BT = BA + AT = c + (p - c) = p

Assim, BP = BT = p.

c) Trace os segmentos OA, OB e OC, conforme ilustra a figura abaixo:

perceba que
(ABC) = (AOB) + (BOC) - (AOC) =>

(ABC) = ^ arb b-rb (c + a-b)rb


2 2 2
Capitulo 3 - Resoluções 603

mas ocorre que c + a- b = c + a + b-2b = 2p-2b = 2(p-b). Portanto.

+ a-b)rb
(ABC) =—
2
(ABC) = 2(P 2b) rb =b (ABC) = (p - b) • rb

OBSERVAÇÃO: Apesar de não ser pedido na questão, vale a pena comentar


que de modo completamente análogo pode-se demonstrar que
(ABC) = (p - a) ra e (ABC) = (p-c)-rc.

d) Consideremos o triângulo ABC, sua circunferência inscrita e também


uma das suas circunferências ex-inscritas, como ilustra a figura abaixo.

Ora, como CF e CO são as bissetrizes dos ângulos interno e externo do


vértice C do triângulo ABC, segue que elas são perpendiculares, o que
implica que os triângulos BFD e BOP são semelhantes (pois possuem
ângulos dois a dois congruentes). Dessa semelhança, temos a seguinte
proporção:
DF BD P-b, Pr
P b (P-b) (1)
PO BP tb
604 Tópicos de Matemática - Olimpíadas - IME - ITA

OBERVAÇÃO: Essa mesma relação pode ser obtida igualando-se as


seguintes fórmulas para o cálculo da área do triângulo ABC : (ABC) = pr e

(ABC) = (p-b)rb.

Por outro lado, os triângulos CDF e COP também são semelhantes, pois
apresentam ângulos dois a dois congruentes. Dessa semelhança temos a
seguinte proporção
DF DC p-c
rrb =(p-a)(p-c) (2)
CP " PO p-a rrb

Finalmente substituindo (1) em (2), segue que:

rrb = (p-a)(p-c) pr = (p-a)(p-c)=>


(p-b)

pr2 = (p — a) (p — b) (p — c)

p2r2 = p(p-a)(p-b)(p-c) =>

(pr)2 = p(p-a)(p-b)(p-c)

mas ocorre que (ABC) = pr, portanto,

(pr)2 = p(p-a)(p-b)(p-c)=»

[(ABC)]2 = p (p - a) (p - b) (p - c) =>

(ABC) = 7p(p-a)(p-b)(p-c)

como queríamos demonstrar.

278) Prove que a área de um triângulo nunca excede 1/6 da soma dos quadrados
das medidas dos seus lados, isto é para um triângulo ABC com lados a, b e c

AABC < ^[a2 + b2 +c2j


Capitulo 3 - Resoluções 605

Resolução:
Sabemos que:
111
(ABC) = -absenC = -bcsenA = -acsenB

Ora. como senC < 1 => < 1 => ab 2 2(ABC) . De modo completamente
ab
análogo, temos que bc>2(ABC) e ac>2(ABC). Adicionando membro a

membro essas três últimas desigualdades, segue que:


ab + bc +ac > 6(ABC) (1)

Por outro lado, já é uma velha conhecida nossa a desigualdade

a2 +b2 + c2 > ab + bc-i-ac (2)


Assim, de (1) e (2), segue que
a2 +b2 + c2 à ab + bc + ac > 6 (ABC) => (ABC) < ^(a2 + b2 + c2)

279) (OBM)Um quadrado de lado 12 foi dividido em sete regiões retangulares que
não se sobrepõem, conforme a figura. Uma delas é um quadrado de vértice
C, cuja área é metade da área de cada um dos dois retângulos vizinhos: outra
é um quadrado de vértice A, cuja área é metade da área de cada um dos do1
retângulos vizinhos.

A B

C D
606 Tópicos de Matemática - Olimpíadas - IME - ITA

a) Mostre que o quadrilátero destacado é um quadrado.


b) Calcule a área do quadrado destacado.

Resolução:
a) Seja x a medida do lado do quadrado de vértice C, e y a medida do
lado do quadrado de vértice A. Ora, por hipótese, os retângulos que
estão vizinhos a esses quadrados têm o dobro da área deles, então eles
irão ter a largura com a mesma medida dos quadrados e comprimento,
igual ao dobro do lado do quadrado, conforme ilustra a figura a seguir:

A y 2y x B

2y
T
2x- y
2x

1
|-2x— y
x x

D 2x x C

Diante do exposto, segue que a medida do lado do quadrado maior mede 3x.
Para calcularmos o lado do quadrilátero central, basta retirarmos o que não
pertence a ele. Logo, retiraremos x + y. Mas isso ocorrerá dos dois lados,
então os dois lados do quadrilátero destacado são iguais a 3x - (x + y) = 2x -
y. o que revela que o quadrilátero central é um quadrado de lado 2x - y. (note
que todos os seus ângulos são retos pois os quadriláteros adjacentes a ele
são retângulos, por hipótese).
b) Pelo que já vimos no item anterior, sabemos que a medida do lado do
quadrado maior é 3x e por outro lado, pelo enunciado é igual a 12, o que
revela que 3x = 12, e, portanto x = 4. Por outro lado, olhando para o lado
de cima do quadrado maior, segue que 3y + x = 3x, ou seja 3y = 3x
„ _ 8
Como x = 4, então y = -. Por fim, como o lado do quadrado destacado
Capitulo 3 - Resoluções 607

é 2x - y, então sua área é (2x - y)2 Substituindo os valores de x e y.

segue que
2
(2x-y)2 = 2-4-8 256
= ■----- u.a.
3. 9
é a medida da área do quadrado hachurado.

280) (OBMEP-TREINAMENTO)A figura abaixo mostra um triângulo de altura 1


dividido por duas retas paralelas á sua base em três partes de mesma área.
Qual é a altura do trapézio central?

1 x S

A 8

Resolução:
Inicialmente vamos nomear os vértices da figura da seguinte forma:

Ora, como DE//AB, segue que os triângulos CDE e CAB são semelhantes
(pois apresentam ângulos dois a dois congruentes). Assim, a razão entre as
608 Tópicos de Matemática - Olimpíadas - IME - ITA

medidas das suas áreas é igual ao quadrado da razão de semelhança entre


esses dois triângulos, ou seja,

(CDE) _í(1-x-y) 2S m . í2
(CAB) l'
(CAB ( 1

De modo completamente análogo, os triângulos CFG e CAB também sâo


semelhantes, o que nos permite escrever:

(CFG) = (1-X-y)2 => 1-X-y =


(CAB)

Í2
Ora, como y = 1- J- , segue que:

■O
281) Dado um quadrilátero ABCD, prove que os pontos médios M, N, P, Q dos
lados AB, BC, CD. DA formam um paralelogramo cuja área é a metade da
área do quadrilátero ABCD.

Resolução:

Traçando as diagonais AC e BD formamos os triângulos ABC, ACD, ABD e


BCD. Ora, como M,N, P e Q são pontos médios dos segmentos AB. BC, CD
e DA, respectivamente, segue pelo teorema da base média de um triângulo,
Capítulo 3- Resoluções 609

que MN//AC, MN = AC/2 . QP//AC, QP = AC/2, MQ//BD. MQ = BD/2 e


NP//BD. NP = BD/2. Assim,
MN//AC eQP//ACoque implica que MN//QP (1)
MQ//BD e ZP//BD o que implica que MQ//NP (2)

de (1) e (2), segue que o quadrilátero MNPQ é um paralelogramo (pois tem


lados opostos paralelos)
Por outro lado, pelo fato de que MN = AC/2. QP = AC/2, MQ/ = BD/2 e NP =
BD/2 segue que temos quadro pares de triângulos semelhantes
(aabc-amnb, aacd-apqd, aabd-aamq e abcd-ancp) cuja

razão de semelhança, em todos os casos, é k = e portanto

(MNB) _ 2 (MNB) = ^-(ABC)


(ABC)

(PQD) ,,2 (PQD) =^(ACD)


(ACD)

(AMQ) = k2 => (AMQ) = i(ABD)


(ABD)

(NCP) ,,2
(NCP) = j (BCD)
(BCD)

Mas,
(ABCD) = (ABC) + (ACD) = 4(MNB) + 4(PQD) =

(MNB) +(PQD) =—(ABCD) (3)

Analogamente.
(ABCD) = (ABD) + (BCD) = 4(AMQ) + 4(NCP) =>

(AMQ) + (NCP) = j (ABCD)


(4)

Mas ocorre que


(ABCD) = (MNPQ) + (MNB) + (PQD) + (AMQ) + (NCP) (5)
610 Tópicos de Matemática - Olimpíadas - IME - ITA

substituindo (3) e (4), em (5). segue que:

(ABCD) = (MNPQ) + (MNB) + (PQD) + (AMQ) + (NCP)


= i(ABCD) l(ABCO)

(ABCD) = (MNPQ) + ^(ABCD)

(MNPQ) =-(ABCD)

282) Determine a medida da área do paralelogramo AEFG, sabendo-se que o


paralelogramo ABCD tem área 2.

Resolução:
Trace o segmento BE formando o triângulo ABE, conforme ilustra a figura
abaixo:
Capítulo 3 - Resoluções 611

Note que o triângulo ABE tem a mesma base AB e a mesma altura h


(distância entre as retas paralelas AB e CD) do paralelogramo ABCD Assim.
ABh (ABCD) ___ 2 1
2 = (ABCD) = ABh e (ABE) =
2 2 2

Por outro lado o mesmo triângulo ABE tem a mesma base AE e a mesma
altura H (distância entre as retas paralelas AE e GF) do paralelogramo AEFG,
o que revela que
AF H
1 = (ABE) = —=> AE ■ H = 2 => (AEFG) = 2

283) Considere um triângulo genérico ABC de área S. Divida cada lado em três
partes congruentes, através de dois pontos. Uma esses pontos ao vértice
oposto, obtendo um hexágono, conforme ilustra a figura abaixo:

i Determine a medida da área do hexágono, em função de S.

Resolução:
Inicialmente observe os triângulos hachurados na figura abaixo:
612 Tópicos de Matemática - Olimpíadas - IME - ITA

Traçando o segmento CT podemos construir a seguinte figura auxiliar:

Note que os triângulos ABC e BCI tem a mesma altura h (partindo do vértice

2 2
B). Como Cl = - CA, segue que (BCI) =-(ABC). Analogamente, os

triângulos ACF e ABC tem a mesma altura partindo do vértice A. Como

2 2
CF = —CB, segue que (ACF) = - (ABC). (Note que aqui estamos utilizando o

fato de que a razão das medidas das áreas de dois triângulos que possuem a

mesma altura é igual a razão entre as medidas das suas bases). Ora, como

2 2
(BCI) = -(ABC) e (ACF) = -(ABC), segue que (BCI) = (ACF).

Por outro lado, como Cl = 2 IA , segue que o triângulo CTI tem o dobro da
área do triângulo ATI. Assim, se (ATI) = x, segue que (CTI) = 2x.

Analogamente, como CF = 2-FB, segue que o triângulo CFT tem o dobro da


área do triângulo BFT. Assim se (BTF) = x', segue que (CTF) = 2x',

conforme ilustra a figura a seguir:


Capitulo 3 - Resoluções 613

Assim,
(BCI) = (ACF) => 2x + 3x' = 3x+ 2x' => x' = x

2
Ora, como por hipótese, (ABC) = S e (ACF) = - (ABC), segue que

2 2 2S 2S 2S
(ACF) = — (ABC) => 3x + 2x' = — S => 3x'+ 2x‘ = => 5x' = —- => x' = ——
3 3 3 3 15

De modo completamente análogo podemos mostrar que os três triângulos


2S
—•..
hachurados mais claros da figura abaixo tem a mesma área —
614 Tópicos de Matemática - Olimpíadas - IME - ITA

De modo completamente análogo, traçando o segmento AU podemos


construir a figura a seguir:

Ora, como os triângulos ABC e ACD tem a mesma altura partindo do vértice C
1 1
e AD = — AB, segue que (ACD) =-(ABC). Analogamente, como os

triângulos ABC e BCI tem a mesma altura partindo do vértice Be Al = - AC,

segue que (ABI) = ^(ABC) (mais uma vez estamos utilizando o fato de que a

razão das medidas das áreas de dois triângulos que possuem a mesma altura
1
é igual a razão entre as medidas das suas bases). Ora, (ACD) = — (ABC) e

(ABI) = ^(ABC), segue que (ACD) = (ABI).

Por outro lado, como BD = 2 DA , segue que o triângulo BDU tem o dobro da
área do triângulo ADU. Assim, se (ADU) = y‘, segue que (BDU) = 2y‘.

Analogamente, como Cl = 2 IA, segue que o triângulo CIU tem o dobro da


área do triângulo AIU. Assim se (AIU) = y , segue que (CIU) = 2y , conforme

ilustra a figura a seguir:


Capitulo 3 - Resoluções 615

23/
U

'V 1 2-</'
A D B

Assim,
(ACD) = (ABI) => 3y + y' = 3y'+ y => y’ = y
-j
Ora, como por hipótese, (ABC) = S e (ACD) = — (ABC), segue que

(ACD) = ^(ABC)=>3y+y' = ^S=>3y'+y' = |=>4y' = |=>2y' = -

De modo completamente análogo podemos mostrar que os três triângulos


... S
hachurados mais escuros da figura abaixo tem a mesma área —.
6 '
616 Tópicos de Matemática - Olimpíadas - IME - ITA

Finalmente a medida da área do hexágono é a diferença entre a medida S da


área do triângulo ABC e as áreas dos triângulos hachurados, ou seja,
,,, . 2S „ S S
(Hexágono) = S- 3------- 3 — = —
' 1 15 6 10

284) Mostre que para qualquer polígono convexo de área 1, existe um


paralelogramo de área menor que 2 que contém o polígono.

Resolução:
Seja P um polígono convexo de área 1. Esse polígono possui dois pontos A e
B que estão o mais distante possível. Seja d(A,B) = diam(P), o diâmetro de

P. sejam (r) uma reta perpendicular ao segmento AB passando por A e (s)


uma reta perpendicular ao segmento AB passando por B. Dessa forma a faixa
localizada ente as retas (r) e (s) contém o polígono P, conforme ilustra a figura
abaixo:

(>■)

Agora considere as retas (t) e (u), perpendiculares a (r) e (s) que "passam"
pelos vértices de P mais a esquerda (C) e mais a direita (D), respectivamente,
como ilustra a figura a seguir:
Capítulo 3 - Resoluções 617

F
(0 A (’■)
K
J L

B J («)
(«)
Sendo F, K, J e H os pontos de interseção das retas (r), (s), (t) e (u) duas a
duas, segue que o quadrilátero FKJH é um retângulo (e portanto um
paralelogramo) que contém o polígono P. Para finalizar, vamos mostrar que
(FKJH) = 2 e ai teremos então um paralelogramo que qualquer polígono

convexo de área 1 está contido num paralelogramo de área 2. De fato,


traçando-se os segmentos AD, DB AC e CB, formamos os triângulos ABC e
ABD, ambos de base AB e alturas de medidas x e y em relação a essa base
AB, conforme ilustra a figura abaixo:

(t)
618 Tópicos de Matemática - Olimpíadas - IME - ITA

Como os triângulos ABC e ABD estão propriamente contidos no polígono P


de área 1 e eles tem apenas o segmento AB em comum, segue que a soma
das duas áreas é menor que a área do polígono P, ou seja,
(ABC) + (ABD) <1.

Mas.
(ABC) + (ABD) < 1

AB x AB y .
------- • +------- < 1 => AB (x + y) < 2
2 2 V ’
Por fim, note que AB(x + y) é justamente a medida a área do retângulo

FKJH, pois AB = HF e x + y = HJ, o que releva que:


(FKJH) = HJ-HF = AB ■ (x + y) < 2 => (FKJH) < 2

o que demonstra o resultado desejado.

284) (CANADÁ-2013)Um quadrado WXYZ de lado 6 é desenhado no interior de


um quadrado EFGH de lado 10, de modo que os seus lados não se
intersectam, mas são dois a dois paralelos, como ilustra a figura abaixo:

E F

Z Y

H G

Mostre que a soma das áreas dos trapézios EFXW e GHZY independe da
posição do quadrado WXYZ no interior do quadrado EFGH, desde que os
lados de um não intersectem os lados do outro e sejam dois a dois paralelos.
Capitulo 3 - Resoluções 619

Resolução:
Suponha que os segmentos paralelos EF e WX estejam separados por uma
distância x. Isto significa que a altura do trapézio EFXW é x. Ora, como WX
mede 6 e EF mede 10, segue que a medida da área do trapézio EFXW é
(EFXW) = (1,° ?6)x = 8x

Seja y a distancia entre os segmento paralelos YZ e GH, como ilustra a figura


abaixo:

E F

x
w X

6 10

Z Y

H G

Assim, 10 = x + 6 + y=>y = 4-x. Portanto a medida da área do trapézio

HGYZé

(GHZY) = (1O*6)(4-*L32-8X
X

Portanto,
(EFXW) + (GHZY) = 8x + (32 -8x) = 32 u.a.

Independente da posição relativa dos dois quadrados, desde que seus lados
sejam dois a dois paralelos e não se intersectem.

OBSERVAÇÃO: Há um problema relacionado com esse, mas bem mais


difícil, que deixamos como desafio:
É colocado no interior de um quadrado ABCD um outro quadrado PQRS de
620 Tópicos de Matemática - Olimpíadas - IME - ITA

modo que os lados desses dois quadrados não se intersectam. Os segmentos


AP, BQ, CR e DS são construídos dividindo a região entre os quadrados em
quatro quadriláteros convexos como mostra a figura abaixo:

Prove que a soma das medidas das áreas dos quadriláteros APSD e BCRQ é
igual a soma das medidas das áreas dos quadriláteros ABQP e CDSR.

285) (OMRN-2015) Na Figura a seguir, temos quatro quadrados, com lados


medindo 11, 9, 7 e 5, respectivamente.

Quanto mede a área das regiões cinzas menos a área das regiões pretas?

Resolução:
Olhando na Figura da esquerda para a direita, sejam C, e C2 as áreas cinzas,
Pi e P2 as áreas pretas e B,, B2 e B3 as áreas brancas.
Capitulo 3 - Resoluções 621

Queremos encontrar:
(C, + C2)-(P,+P2)

Agora, observe que

C, + B, =112
B, + P, + B2 = 92
B2 + C2+B3 = 72
B3 + P2 = 52

Adicionando a primeira e a terceira equações, segue que


C, + B, + B2+C2 + B3 = 121 + 49 =>(0,4-02) + B,+ B2 + B3 = 170

e adicionando a segunda e quarta equações, segue que


B, + P, + B2 + B3 + P2 = 81 + 25 => (P, + P2) +B, + B2 + B3 =106

Portanto,

(C, + C2) + B, + B2 + B3 = 170


(P, + P2) + B,+B2 + B3 = 106

Subtraindo membro a membro essas duas equações, segue que


(C, + C2) - (P, + P2) = 170 -106 => (C, + C2) - (P, + P2) = 64cm2

286) Sendo ABCD um quadrado e n um circulo inscrito nesse quadrado. Se


S, + S2 + S3 = 8 cm2, determine a medida da área S.

D C

A B
622 Tópicos de Matemática - Olimpíadas - IME - ITA

Resolução:
Represente por A a medida da área da parte branca do circulo inscrito no

quadrado ABCD, e por a a medida do lado desse quadrado, conforme ilustra

a figura abaixo:

D
a
C

St
S

a
a

s.
A B
a
Dessa forma a medida do raio do círculo inscrito na quadrado ABCD é .

Assim,
2
A + S = n(f) A
a
+o
S =-----
4
1 o s, + s2+s3 + a = ^-
Si + S2 + S3 + A = — na2

O que revela que


A + S-S1 + S2+S3+A S = S1 + S2+S3=>S = 8cm2
Capítulo 3 - Resoluções 623

287) (OMRN-2015) Na figura a seguir, temos um semicirculo de raio R = 10 cm. Os


pontos B e C dividem o semicirculo AD em três arcos de comprimentos iguais.

Calcular a área sombreada do triângulo curvilíneo ABC.

Resolução:
Ora, como os pontos B e C dividem o arco AD em três partes iguais, segue
que cada um desses arcos e os ângulos centrais correspondentes medem
60°. Além disso, o segmento BC e paralelo ao diâmetro AD. Sendo O o centro
da semi-circunferência, trançando-se os raios OB e OC, segue que os
triângulos ABC e OBC possuem a mesma área, pois possuem a mesma base
BC e a mesma altura relativa a essa lado, conforme ilustra a figura abaixo:

B C

R\
A /R
\ 60a/
60o/V7\60°
A o D

Diante do exposto, a medida S da área hachurada é dada por:


624 Tópicos de Matemática - Olimpíadas - IME - ITA

S — (ABC) + Aseamenl0 arcuiar gc


” (OBC) + Aseflment0 orcuiar ac
~ AseIor circular da 60*

= -7XlO2
6
50n 2
=----- cm
3

288) (CANADÁ-2014)Na figura abaixo ABCD é um retângulo de lados AB=100 e


BC=20. Os triângulos AFB e ODE são retângulos em E e F , respectivamente.

Determine e medida da área da região hachurada.

Resolução:
Trace o segmento EF como ilustra a figura abaixo:
Capitulo 3 - Resoluções 625

Note que ao traçar o segmento EF o retângulo ABCD fica dividido em dois


retângulos menores AEFD e BCFE. Além disso AF, DE, BF e CE sâo as
diagonais desses retângulos menores. Quando traçamos as diagonais de um
retângulo as diagonais cruzam-se ao meio e portanto dividem retângulo em
quatro triângulos de mesma área. Assim,
Ir-r-oX "I
(EFG) = l(AEFD)e(EFH) = l(BCFE)

Portanto,
(GFHE) = (EFG) + (EFH)
= 2(AEFD) + 1(BCFE)

= 1[(AEFD) + (BCFE)]

= 1(ABCD)

= -100.20
4
= 500

289) (PERU)As medidas dos raios das três circunferências apresentadas na figura
abaixo são 6cm, 5cm e 4cm. Se A é a medida da área da região hachurada e
B é a medida da área da região branca, determine a razão A/B.
626 Tópicos de Matemática - Olimpíadas - IME - ITA

Resolução:
Note que a medida A da área hachurada corresponde a diferença entre as
medidas das áreas dos círculos de raios 5cm e 4cm, ou seja,
A = n52 —n42 = 9-cm2. Por outro lado a medida B da área branca
corresponde a medida da área do circulo menor mais a diferença entre as
medidas das áreas dos círculos de raios 6cm e 5cm, ou seja,

B = it42 + (n62 - n52) = 27ncm2

Portanto,
A 9n
B 27Í 3

290) (PERU)Na figura abaixo os círculos e Q3 têm raios 8cm, 4cm e

6cm respectivamente, determine a medida da área da região hachurada.

Resolução:
Inicialmente vamos nomear por x, y, z.w e u as medidas das áreas das
regiões assinaladas na figura a seguir
Capitulo 3 - Resoluções 627

Assim, podemos escrever:

x + y + z = n82 x + y + z = 64n
y + z + w = n42 =>• y + z + w = 16rt
u + w + z = n62 u + w + z = 36rt

Por outro lado da medida da área hachurada é A = x + z + u , adicionando


membro a membro as três equações acima, segue que
(x + z + u) + 2(y + z + w) = 116n=>A + 2.n42 = 166n => A = 84ncm2

291) (PERU)Na figura abaixo o ponto B é o ponto médio do arco AC. Calcule a
medida da área da região hachurada, sabendo que AD=4cm e DC=3cm.
628 Tópicos de Matemática - Olimpíadas - IME - ITA

Resolução:
Trace o segmento AC e perceba que o triângulo ACD é retângulo. Por

Pitágoras,

AC2 = AD2 + DC2 => AC = \/42 + 32 = 5

Por outro lado o quadrilátero ABCD é inscritivel, o que revela que

ZADÇ + ZABC = 180° => ZABC = 90°


= 90’

Além disso, como o ponto B é, por hipótese, o ponto médio do arco AC, segue

que as cordas AB e BC são congruentes. Sendo AB=BC=a, mais uma vez por

Pitágoras, agora no triângulo ABC, segue que

AC2 = AB2 + BC2 => 52 = a2 + a2 a2 = —


2

Por fim, a medida S da área hachurada é a diferença entre as medidas das

áreas do círculo e do quadrilátero, ABCD (que por sua vez corresponde a

somas das medidas das áreas dos triângulos ABC e ADC), ou seja.

S = Acirculo - (ABC)-(ADC)

= nR2--AB.BC--AD.CD
2'—2
2

=n(i)
25n-49
1Í5-14.3
2 2 2

cm2
4
Capitulo 3 - Resoluções 629

292) (PERU)O maior circulo da figura abaixo tem raio 10cm. Qual a medida do raio
do circulo menor da figura abaixo, de modo que a medida da sua área seja
numericamente igual a medida da área hachurada?

Resolução:
Seja r a medida do raio do circulo menor. Ora, como por hipótese a medida
da área do circulo menor corresponde a medida da área hachurada. e a
medida da área hachurada corresponde a diferença das medidas das áreas
dos dois círculos, segue que
- OA
‘menor — ^maior ~ Amenor
^maior * ^-^menor Amaior

Portanto,
2irr2 = rt102 => r = 5>/2cm

293) (PERU)Calcule a medida da área da região hachurada, sabendo-se que


AB//CD. AB=8, CD=6 e que o arco BD mede 90°.
630 Tópicos de Matemática - Olimpíadas - IME - ITA

Resolução:
Ligue o centro O do círculo com os pontos A,B C e D, conforme ilustra a figura
abaixo:

Note também que


a + p + 2.90° = 360° =0 a + p - 180° => cosp = - cosa

Aplicando a lei dos cossenos nos triângulos OAB e OCD (onde O é o centro
do circulo) e lembrando que, por hipótese, AB = 8 e CD = 6, segue que

82 = R2+R2-2.RRcosa 64 = 2R2 - 2R2 cos a


62 = R2 + R2 -2RR cosp =*
36 = 2R2+2R2 cos a
= -cosa

Adicionando as duas últimas equações, segue que 100 = 4R2 => R = 5.

Retornando a equação 64 = 2R2 -2R2cosa . Obtemos,

64 = 2.52 - 2.52cosa => COS a = —


25
Ora, como sen2a + cos2 a = 1 e nesse caso sena>0 (pois 0° < a <180°).
segue que
2
24
sen ?a + í —
7 r = 1 => sena = — . Como a + p = 180° temos também que
125J
24
senp - sena = —.
25
Capítulo 3 - Resoluções 631

Ora. como a área hachurada (de medida S) corresponde a dois setores de


90° e dois triângulos isósceles, segue que
S = 2.A5e,orae90. + (OAB) + (OCD)

= 2. —nR2 + — .R.R.sena + — .R.R.senB


4 2 2
- 2. —n52 + — 52 — + —52—
4 2 25 2 25
25n + 48
=------------ u.a.
2

294) Sobre cada um dos lados de um triângulo retângulo ABC foram construídos
semicírculos conforme ilustra afigura abaixo:

Se S> e S2 são as medidas da áreas hachuradas, mostre que a medida da

área do triângulo ABC é igual a (ABC) = S, - S2.

Resolução:
Inicialmente denote por x, y, z e w as medidas das áreas das regiões brancas
assinaladas na figura a seguir:
632 Tópicos de Matemática - Olimpíadas - IME - ITA

Assim, as medidas das áreas dos três semicírculos são x + y + S,, z + w + S,

e y + w + S2. Por outro lado, pelo Teorema de Pitágoras generalizado (veja a

observação da página 26), temos que a medida da área do semicírculo

construído sobre a hipotenusa é igual a soma das medidas das áreas dos

semicírculos construídos sobre os catetos. Assim,

y + w + S2 = (x + y + S,) + (z+ w + S^ => S2 = (x + z + S^ + S,

Por fim, note que a medida da área do triângulo ABC é (ABC) = x+ z +S,.

Assim,

S2 - (x S,) + S, => S2 = (ABC) + S1 =>(ABC) = S2-S,


Capítulo 3 - Resoluções 633

295) Determine da medida da área hachurada, em função da área S do triângulo


ABC, sabendo-se que os pontos assinalados dividem cada lado em partes
iguais.

Resolução:
Inicialmente trace o segmento CD, conforme ilustra a figura abaixo

Note que o triângulo BCD tem a mesma altura (partindo do vértice C) que o
triângulo ABC, mas sua base BD é 1/3 da base AB. Logo a sua área é 1/3 da

área do triângulo ABC, ou seja, (BCD) = -l(ABC) = jS . Assim,

1 2
(ACD) = (ABC) - (BCD) = S - - S = | S
634 Tópicos de Matemática - Olimpíadas - IME - ITA

Agora, olhando para os triângulos CDE e ACD segue que eles possuem a
mesma altura (partindo do vértice D) e a base CE do primeiro corresponde a
1/4 da base AC do segundo. Assim,
1 12 1
(CDE^fACDj^-S^S

Mas ocorre que


2 11
(ADE) = (ACD) - (CDE) = -S -- S = - S

Agora analisando a figura

Podemos escrever
1
(ADE) = — (3x)(4y)sena = 6xysena

(AFG) = jxysena

Portanto,

(AEG) ^xysena
— => (AFG) = —-S = —S
(ADE) 6xysena 12 1 7 12 2 24
Capitulo 3 - Resoluções 635

Finalmente,
(DEFG) = (ADE)- (AFG) = 1S -S => (DEFG) = S

296) Sobre os lados AB e AC de um triângulo retângulo foram construídos os


quadrados ACDE e ABFG, conforme ilustra a figura abaixo.

A
n j
B

G H F

Mostre que as medidas da áreas do quadrado ACDE e do retângulo AGHJ

são iguais.

Resolução:

Sejam AC = b, AB = AG = c e ZBAC = a , conforme ilustra a figura a seguir:


636 Tópicos de Matemática - Olimpíadas - IME - ITA

b
E
,ct
A
ZL B
j
c

c c

G H c F

Assim, (ACDE) = b2 e (AGHJ) = AG.AJ = c.AJ. Mas ocorre que no triângulo

AJ
retângulo ACJ temos que cosa = —=> AJ = bcosa . Por outro lado, no

AC
triângulo ABC temos que cosa = — —. O que nos permite concluir que
c

b2 .
AJ = bcosa = b —
Alt- t
— . Finalmente, temos que:
c c

(AGHJ) = AG.AJ = C.AJ = Cy = b2

O que revela que o quadrado ACDE e o retângulo AGHJ tèm áreas iguais.
Capitulo 3 - Resoluções 637

Segunda solução:
Trace os segmentos BE e CG, conforme ilustra a figura abaixo;

b i
E

A
4—-U-
/
n'~'-
j
B
c
i
i
i
i
i
c i c.
i
i
i
i

G H F
c

Note que a medida da área do triângulo ABE corresponde a metade da


medida da área do quadrado ACDE pois eles possuem uma mesma base AE
e uma mesma altura ED, ou seja,
(ABE) = 1 (ACDE)

Analogamente, a medida da área do triângulo ACG corresponde a metade da


medida da área do retângulo AGHJ pois eles possuem a mesma base AG e a
mesma altura GH, ou seja,

(ACG) =—(AGHJ)
638 Tópicos de Matemática - Olimpíadas - IME - ITA

Para finalizar, perceba que os triângulos ABE e ACG sâo congruentes (caso

LAL. pois AE = AC, AB = AG e ZBAE = ZCAG = a + 90°) e portanto têm

áreas iguais. Assim,

(ABE) = (ACG) => i(ACDE) = ^(AGHJ) => (ACDE) = (AGHJ)

297) (OBM-2015) João cortou os quatro cantos de uma folha retangular e obteve o
um octógono equiângulo ABCDEFGH, como mostra a figura abaixo. Sabendo
que AB = 2\/2 , BC = 7,CD = 3^2, DE = 3,EF = 4 72 e GH = 5^2 , determine
a área desse octógono.
G F

4\/2
5'/2

H
3

D
A
3\/2
2\/2
B C
7

Resolução:
As medidas dos ângulos internos de um octógono equiângulo são iguais a
(n-2)180° (8-2)180°
ai = = 135°
n 8
Capitulo 3 - Resoluções 639

Assim, prolongando os lados AH. BC, DE e FG, obtemos um retângulo


subtraído de quatro triângulos retângulos isósceles de lados x,y,z e w,
conforme ilustra a figura abaixo:

W G F
z P
Q

z
w 4\/2
572
E

H
3

D
A
2x/2 3v/2 :v
X

M X B C y N
7

Note que os triângulos ABM, CDN, EFP e GHQ sâo triângulos retângulos
isósceles pois cada ângulo interno do octógono equiângulo medindo 135°
segue que cada ângulo agudo de cada um desses triângulos mede 180°-
135°=45°. Assim, aplicando Pitágoras nos triângulos ABM. CDN. EFP e GHQ,
segue que

x2 + x2 =(272)2
x=2
y2 + y2 = (3>/2)2
y=3
z2+z2 = (472)2 z=4
w=5
w2 + w2 = (572)2

O que revela que o retângulo MNPQ tem lados de medidas


MN = x+ 7 + y = 2 +7 +3 = 12 e NP = y-i-3 + z = 3 + 3 + 4 = 10
640 Tópicos de Matemática - Olimpíadas - IME - ITA

Por fim. a medida da área do octógono ABCDEFGH é igual a medida da área

do retângulo MNPQ menos as medidas das áreas dos triângulos ABM, CDN,

EFP e GHQ, ou seja,

(ABCDEFGH) = (MNPQ) - (ABM) - (CDN) - (EFP) - (GHQ)


1111
= 12.10-—2.2—3.3-—4.4-—5.5
2 2 2 2
= 93 u.a.

Os lados do retângulo são 4S- + BC + -££ = 2 + 7 + 3 = 12 e


72 72

+ DE + ^7 = 3 + 3 + 4 = 10. Com isso, a área do octógono ABCDEFGH é


72 72 “

22 32 42 52
10-12 = 93 .
2 2 2 2

298) (OBM-2014) O retângulo da figura foi repartido por meio de três segmentos
em várias regiões, algumas retangulares e outras triangulares. A linha não
paralela aos lados é uma diagonal e os números indicam as áreas em m2 das
regiões brancas em que se encontram. Qual é a do retângulo original?

24
18

8
Capitulo 3 - Resoluções 641

Resolução:
Sejam x, y e z as medidas das áreas assinaladas na figura a seguir:

Ora, como uma diagonal de um retângulo o divide em dois triângulos


congruentes, segue que x = 18ey = 8. Mas ocorre que há uma
proporcionalidade entre as medidas das áreas dos quatro retângulos menores
(como no problema 257). Assim,

24.z = (y + 8)(x + 18) => 24.Z = 16.36 => z = 24

Como todas as partem que compõe a figura dada eram do retângulo original,
segue que a medida S da área do retângulo original corresponde a soma das
medidas das áreas dessas partes, ou seja,
S = 24+ 36+ 16+ 24 = 100 cm2

299) (CANADÁ-2013) No diagrama abaixo ABC é um quarto de uma pizza circular


com centro A e raio 20cm. Esse pedaço de pizza foi colocado sobre um prato
circular, conforme ilustra a figura abaixo
642 Tópicos de Matemática - Olimpíadas - IME - ITA

Qual a fração da área do prato está sendo coberta pela fatia de pizza?

Resolução:
Traçando o segmento BC, note que o triângulo ABC é retângulo e está
inscrito na circunferência (que é a borda do prato circular).

Assim BC é o diâmetro desse circunferência. Sendo BC=2R, segue, por


Pitágoras, que

BC2 = AC2 + AB2 (2R)2 = 202 + 202 => R = 10J2 cm

Portanto, a medida da área S da região do prato é

- Aprato = n(l0>/2)2 = 200n .

Já a medida da área S' da região coberta pela pizza é

S - AseIordeS0, - — n202 - 100n , o que nos permite concluir que

S' 100n 2
S ” 200n 2
Capitulo 3 - Resoluções 643

300) (OBM-2014) figura ao lado, temos AF = 12 cm, AE = 16 cm. Os vértices do


quadrado EFGH pertencem aos lados do quadrado ABCD e os pontos I. J. K.
L são pontos médios dos lados de EFGH.

a) Qual é a área do quadrado ABCD?


b) Qual é a área do quadrado EFGH?
c) Qual é a área do quadrado cinza no interior do quadrado EFGH?

Resolução:
a) Ora, como, por hipótese, o quadrilátero ABCD é um quadrado, segue
que os ângulos dos vértices A, B, C e D são retos e portanto os
triângulos AEF, BFG, CGH e DEH são todos congruentes pelo caso
LAL. Assim podemos concluir que DE = AF = 12cm e portanto,
AD = AE + ED = 16 + 12 = 28cm

Assim o quadrado ABCD tem lado medindo 28cm, o que implica que a
medida da sua área é (ABCD) = 282 = 784cm2 .
644 Tópicos de Matemática - Olimpíadas - IME - ITA

b) Ora, como o quadrilátero ABCD é, por hipótese, um quadrado, segue


que zEAF = 90° e portanto o triângulo AEF é retângulo. Por Pitágoras,
temos que

EF2 = AE2 + AF2 => EF = Vl22 + 162 => EF = 20cm

Portanto o quadrado EFGH tem lado medindo 20cm, o que revela que a
medida da sua área é (EFGH) = 202 = 400cm2 .

c) Seja x a medida do lado do quadrado MNPQ conforme ilustra a figura


abaixo:

Pelo fato do quadrilátero MNPQ ser um quadrado em cada um dos seus


vértices há um ângulo reto, o que implica que
ZGMK = ZHNL = 90°

O que nos permite concluir que MK é paralelo a NH e portanto que os


triângulos GMK e GNH são semelhantes. Além disso, a razão de
GK GK _1
semelhança desses triângulos é k = . Assim,
GH “2GK“2

GM 1 GM GM
= — => GM = x
GN 2 GM + MN 2 GM + x 2
Capitulo 3 - Resoluções 645

De modo completamente análogo podemos mostrar que


NH = EP = FQ = x , conforme ilustra a figura abaixo

Por fim aplicando Pitágoras no triângulo GHN, segue que

GH2 =NH2+NG2 202 = x2 +(2x)2 5x2 = 202 => x2 = 80

Assim a medida da área do quadrado MNPQ é (MNPQ) = x2 = 80cm2.


Apêndices
648 Tópicos de Matemática - Olimpíadas - IME - ITA

APÊNDICE 1

PITÁGORAS TRIGONOMÉTRICO

Carlos A.Gomes
UFRN- Natal/RN

É tão bela quanto antiga a demonstração do teorema de Pitágoras que se


encontra no livro dos elementos de Euclides. Neste pequeno artigo daremos uma
prova que usa a mesma idéia da demonstração que se encontra nos elementos,
mas com um sabor trigonométrico, o que torna a conclusão de que a medida da
área do quadrado construído sobre a hipotenusa de um triângulo retângulo
corresponde a soma das medidas das áreas dos quadrados construídos sobre os
catetos deste mesmo triângulo bem mais rápida.

c
B
v
Xa
«_____ 3
b.cosa c.cosí3

X Y

Na figura acima temos que A = X + Y . Assim basta mostrarmos que X = B e que


Y = C . De fato, observe que
X = a ■ b cos a e Y = a • c cos p
b c
Por outro lado olhando para o triângulo retângulo temos cosa = — e cosp = —.
a a
Assim podemos concluir que
A = X +Y = abcosa + a-ccosB = ab-- + ac- = b2+c2=>A = b2+c2
a a
Como B = b2 e C = c2 segue que A = b2 + c2=>A = B + C
Apêndices 649

APÊNDICE 2

VOCÊ SABIA QUE LEI DOS COSSENOS VALE PARA OS SENOS?


Carlos A. Gomes
UFRN- Natal/RN
(Publicado na RPM)

Como é? É isso mesmo! Veja: é fato bastante conhecido que num triângulo ABC
qualquer é válida a lei dos cossenos, a saber:

a2 = b2 + c2 - 2bc cos a
■ b2 = a2 + c2 - 2ac cos 0
c2 = a2 + c2 - 2accos y

Vamos mostrar que essa relação é preservada para os senos dos ângulos internos
desse triângulo, ou seja,
sen2a = sen2p + sen2y - 2sen0seny cosa
■ sen2p = sen2a + sen2y - 2senaseny cos p
sen2y = sen2a + sen2p-2senasenpcosy

Com efeito, usando a também conhecida lei dos senos no triângulo ABC, segue
que:
a = 2Rsena
a b c
= 2R =• • b = 2Rsenp
sena senp seny
c = 2Rseny

Onde R é a medida do raio da circunferência circunscrita ao triângulo ABC. Assim,


por exemplo,
a2 = b2 + c2 -2bccosa =>
(2Rsena)2 = (2Rsen0)2 +(2Rseny)2 -2(2Rsenp)(2Rseny)cosa =>
650 Tópicos de Matemática - Olimpíadas - IME - ITA

4R2sen2a = 4R2 (sen2p + sen2-/ - 2senpseny cos a) =>

sen2a = sen2p +sen2-/ -2senpseny cosa

As demais igualdades podem ser demonstradas de modo completamente análogo.

OBSERVAÇÃO: As igualdades obtidas para os senos são consequência da


semelhança entre os triângulos abaixo, decorrente da lei dos senos:

APÊNDICE 3
LEI DOS SENOS PARA OS COSSENOS
Carlos A Gomes
UFRN - Natal/RN
(Publicado na RPM)

Na RPM 42 escrevi um breve artigo que tinha como titulo: “você sabia que a
lei dos cossenos vale para os senos?". Agora este pequeno artigo tem como
objetivo exibir um fato curioso: É bem conhecida de todos a lei dos senos, que
relaciona as medidas a,b, e c dos lados de um triângulo qualquer com os senos
das medidas dos seus respectivos ângulos internos (opostos) A, B e C e a medida
R do raio da circunferência circunscrita ao triângulo ABC. a saber
abc
----- - =----- - =----- — = 2R . O fato curioso é o seguinte: Se denotarmos por a, b, e
senA senB senC r
c as medidas das distâncias de cada um dos vértices de um triângulo ABC
(acutângulo) ao ortocentro é válida a bela relação—-— = —-— = —-— = 2R , que
cos A cosB cosC
costumo chama-la de "lei dos senos para os cossenos". (conhecí este fato no
meio de uma questão que o discuti com o Ralph na obm-l). Os Matemáticos se
encantam e vivem procurando simetrias e padrões esse , na minha opinião é um
belo exemplo da simetria e da beleza da Matemática!.
Para atingirmos o resultado acima mencionado vamos usar uma propriedade
do ortocentro que é a seguinte:
Apêndices 651

"Se H é o ortocentro de um triângulo ABC então o ponto simétrico de H com


relação a qualquer dos lados do triângulo ABC pertence a circunferência
circunscrita ao triângulo ABC".

A demonstração deste fato está no final do artigo!


Admitindo como verdadeiro o resultado acima, observe que o triângulo CDH
(na figura abaixo) é congruente ao triângulo CDH' (LAL), visto que possuem dois
lados em comum; DH e DH' (pois estamos supondo que o ponto H' é o simétrico
de H em relação ao lado BC); o lado comum CD ezCDH = ZCDH'= 90° . Assim
podemos concluir que CH' = c. Agora aplicando a lei dos senos no triângulo ACH'
temos —— = 2R (note que os triângulos ABC e ACH' possuem a mesma
sena
circunferência circunscrita). Por outro lado temos que a + C = 90” o que
c c
implica que sena = cosC. Assim, concluímos que ------- = 2R=>------ — = 2R. De
sena cos C

modo completamente análogo podemos concluir que —— = 2R e que


cos A
b = 2R . Diante do exposto temos que é válida a seguinte igualdade;

a b c
= 2R
cos A cosB cosC

É bastante claro que esta propriedade não é válida no caso em que o


triângulo ABC for retângulo, visto que neste caso cosA = 0 e também não é
válida no caso em que o triângulo ABC for obtusângulo, visto que neste caso
cosA < 0 e assim —-— < 0 e portanto não poderia ser igual a 2R > 0.
cos A
652 Tópicos de Matemática - Olimpíadas - IME - ITA

Demonstração da propriedade do Ortocentro:


Apesar de necessitarmos da propriedade de que o ponto simétrico do
ortocentro pertencer á circunferência circunscrita ao triângulo apenas no caso do
triângulo acutângulo, vamos mostrar que essa propriedade é válida em geral, isto,
é para um triângulo qualquer Para isso analisaremos os três casos, a saber

Caso 1: O triângulo ABC é retângulo

Neste caso o ortocentro H coincide com o vértice A. Assim o simétrico de H


com relação a AB ou a AC é o próprio A, que trivialmente é um ponto da
circunferência circunscrita ao triângulo ABC. Agora vamos verificar que o simétrico
do ponto H (que coincide com o vértice A, nesse caso) com relação ao lado BC
pertence a circunferência circunscrita.
De fato, por A trace um segmento perpendicular a BC. Este segmento
encontrará a circunferência circunscrita ao triângulo ABC no ponto E. Vamos
mostrar que E é o simétrico de A com relação a BC.

Note que os triângulos ABD e BDE são congruentes (ALA) pois


ZADB = ZBDE = 90°, o lado BD é comum e ZABD = ZDBE = p , visto que ambos
“enxergam" arcos de mesma medida 2p. Assim, por essa congruência de
Apêndices 653

triângulos, temos que AD = DE e então E é o simétrico de H (que coincide com A)


em relação ao lado BC.

Caso 2: O triângulo ABC é acutângulo

Neste caso vamos prolongar AD até que encontre a circunferência


circunscrita ao triângulo ABC em E. Vamos mostrar que E é o simétrico de H com
relação a BC.

De fato,

Note que os triângulos BDH e BDE sâo congruentes (ALA), visto que têm o
lado BD em comum ZBDH = ZBDE = 90° e ZDBH = ZDBE pois ZDBH = a e
ZDBE = ZCAE = a ( pois são ângulos inscritos que enxergam o mesmo arco CE).
Como triângulos BDH e BDE são congruentes segue que DH = DE e portanto o
ponto E (que pertence a circunferência circunscrita ao triângulo ABC) é o simétrico
de H com relação ao segmento BC. De modo análogo mostrariamos que o
simétrico de H com relação a AC e o simétrico de H com relação a AB pertencem a
circunferência circunscrita ao triângulo ABC.
654 Tópicos de Matemática - Olimpíadas - IME - ITA

Caso 3: O triângulo é obtusãngulo

..ios prolongar AD até encontrar a circunferência circunscrita no ponto E.


vamos mostrar que E é o simétrico de H com relação a BC.
De fato, na figura abaixo supomos que os arcos BF e FA medem
respectivamente 20 e 2a o que implica que ZACB = a + p (pois a medida de um
ângulo inscrito é a meda da medida, em graus do arco correspondente). Agora
perceba que ZBHD = ZACB = a + p , visto que são ângulos de lados
perpendiculares. Finalmente perceba que ZBED = a + p, uma vez que o ângulo
ZBED "enxerga" na circunferência o arco BA cuja medida é 2(a + p). Dessa
forma os triângulos BDH e BDE são congruentes (ALA) pois têm o lado BD em
comum, ZBDH = ZBDE = 90° e ZBHD = ZBED = a + p . Dessa congruência
segue que DH = DE e que portanto o ponto E (que pertence a circunferência
circunscrita ao triângulo ABC) é o simétrico de H com relação ao lado BC.
Apêndices 655

Para finalizarmos vamos mostrar que o ponto simétrico de H com relação a


aos lados AC e AB pertencem a circunferência circunscrita ao triângulo ABC. Na
figura abaixo note que a altura BH encontra a circunferência no ponto F. Vamos
mostrar que F é o simétrico de H com relação a AC. Com efeito, considere o
triângulo AFH. Note que ZAFH é a medida de um ângulo externo do triângulo
ABF e portanto ZAFH = ZABF+ZBAF = a+ p , visto que ZABF = a pois este

ângulo enxerga o arco FA de medida 2a e ZBAF = p pois este ângulo enxerga


um arco de medida 2p. Assim podemos concluir que os triângulos AGH e AGF
sâo congruentes (ALA) visto que apresentam o lado AG em comum,
ZAGH = zAGF = 90° e ZAHG = ZAFG = a+ p . Dessa congruência podemos
concluir que GH = GF e portanto que o ponto F (que pertence a circunferência
circunscrita ao triângulo ABC) é o simétrico de H com relação ao lado AC. Por um
raciocínio completamente análogo mostra-se que o simétrico de H com relação ao
lado AB pertence a circunferência circunscrita ao triângulo ABC, o que finaliza a
demonstração.

— a + (i

'A

2/>v
F//
a + fi C

BI
f O

a+0 E
656 Tópicos de Matemática - Olimpíadas - IME - ITA

APÊNDICE 4

UMA BELA DEMONSTRAÇÃO (SEM PALAVRAS) DA FÓRMULA DE HERON

Carlos A. Gomes
UFRN - Natal / RN
(Publicado na RPM e no livro
Geometria em Sala de Aula - SBM)

A conhecida fórmula de Heron S = ^/p(p — a)(p — b)(p-c) , para o cálculo da


medida da área , S , de um triângulo cujos lados medem a, b e c e semi-
perimetrop = C ,pode ser provada de diversas maneiras. O propósito deste

pequeno artigo é exibir uma belademonstração dessa fórmula que foi publicada no
THE COLLEGE MATHEMATICS JOURNAL, vol 32, N°4, september 2001
(Heron's formula via proofs without words). de autoria de Roger B.Nelsen,
Lewis & Clark College, Portland, OR.
Seja ABC um triângulo com lados medindo a, b e c conforme ilustramos na
figura (1) abaixo. Nesta figura também representamos as bissetrizes dos ângulos
internos do triângulo ABC assim como a sua circunferência inscrita. Na figura (2)
mostramos os raios, de medida r, nos pontos de tangência da circunferência
senta com os lados do triângulo ABC. Observe:

Figura 1

Figura 2
Apêndices 657

Perceba que o semi-perimetro do DABC satisfaz as seguintes relações:


x = p-a
p = x + y + z = x + a = y + b = y + c=' y = p-b
z - p-c
Agora vamos mostrar dois lemas (sem palavras) dos quais segue-se de modo
imediato a fórmula de Heron.

Lema 1. A medida da área S de um triângulo é igual ao produto da medida do raio


da sua circunferência inscrita pela medida do seu semi-perímetro.

Demonstração:

Note que podemos decompor o triângulo da figura acima, formando o seguir


retângulo:

X V z
Observando as figuras acima nâo é dificil perceber que:
S = r(x + y + z) = p.r

Lema 2. Se <x, p e y são medidas positivas de três ângulos tais


que a + p + y = 90°, então
tgatgp + tgatgy + tgptgy =1
658 Tópicos de Matemática - Olimpíadas - IME - ITA

Demonstração:
De fato,
a + P + y = 90° => a + p = 90° - y =>
tg(a + p) = tg(90°-y)=> tga^tgP = — =>
1 1 k 1 1-tgatgp tgy
tgatgy + tgptgy = 1-tgatgp=> tga ■ tgp + tga ■ tgy + tgp-tgy =1

Teorema (Fórmula de Heron). A medida da área de um triângulo com lados


medindo a. b e cé dada porS = ^/p(p — a)(p—b)(p-c) , ondep = a + +

Demonstração:

No triângulo ABC da figura acima, temos 2a + 2p + 2y = 180° => a + p + y = 90°.


Além disso.
r . „ r . r
tga = —, tgp = - e tgy = -
x y z

Assim, aplicando o lema 2, segue que


tga ■ tgp + tga ■ tgy + tgp • tgy = 1 =>
r r r r r r .
---------- 1------------1---------- = 1
x y x z y z
r2 (x + y + z) r2p r2p2 S2
1=
xyz xyz pxyz pxyz
S = 7pxyz
Por outro lado, como já vimos no início, x = p-a,y = p-b e z = p-c . Assim,
S = ,,/pxyz => S = ^/p(p-a)(p-b)(p-c)

como queríamos demonstrar.


Apêndices 659

APÊNDICE 5

HERON PARA QUADRILÁTEROS...BRAHMAGUPTA!

Augusto J Macêdo e Carlos A. Gomes


UFRN - Natal/RN
(Publicado na RPM)

É bastante conhecida a fórmula de Heron para determinar a pedida da área


de um triângulo quando são conhecidas as medidas dos seus lados,
(ABC) = Jp(p-a)(p-b)(p-c), onde onde p = a + ^ + c g
é o 0 semi-perímetro.

Entretanto è pouco conhecida e divulgada no ensino médio a generalização dessa


fórmula para um quadrilátero inscritivel (fórmula de Brahmagupta) cujos lados
medem a, b, c e d, que é dada pela
expressão (ABCD) = J(p - a) (p - b) (p - c) (p - d) . Ainda menos conhecida e
divulgada è a generalização dessa fórmula para um quadrilátero qualquer
(inscritivel ou não) cujos lados medem a, b, c e d e sendo também conhecido o, a
média aritmética de dois ângulos opostos do quadrilétero
(ABCD) = J(p - a) ■ (p - b) ■ (p - c) ■ (p - d) - a ■ b ■ c ■ d ■ cos2 0
Esse pequeno artigo foi escrito pelo meu amigo prof Augusto Macedo há 10
anos e só agora resolví ressussitá-lo e divulgá-lo para todos os amantes da
geometria I
1. QUADRILÁTERO INSCRITIVEL
Iniciemos analisando o quadrilátero inscritivel abaixo:
660 Tópicos de Matemática - Olimpíadas - IME - ITA

I. Acompanhe os seguintes passos:


sena = senp
i) a + p = 180°=>
cosa = -cosp
Na figura acima é fácil ver que: ( ) = medida da área
1
AABC => (ABC) = --ab-sena

AACD=> (ACD) = -|-c-d-senp

1 1
Logo (ABCD) = (ABC) + (ACD) = -.a.b.sena + --c■ d■ senp . Mas ocorre

quea + p = 180°, visto que o quadrilátero ABCD acima é inscritível. Assim segue
que sena = senp e portanto,
1111
(ABCD) = (ABC) + (ACD) = --a b sena + -■ c-d-senp = -■a-b-sena +-■ c-d-sena

ou seja, (ABCD) = --(ab + cd) sena .


iii) Aplicando a lei dos cossenos nos triângulos ABC e ACD
[x2 = a2 + b2 - 2ab cos a
obtemos < . Mas ocorre que cosp = - cosa , visto que
[x2 = c2 + d2 - 2cd cos p
a + p = 180°. Diante disso, segue que
a2+b2-c2-d2
a2 + b2 - 2ab cos a = c2 + d2 + 2cd cos a => cos a =
2 ■ (ab + cd)

Comosena = Vi - cos2 a , temos entãosena =


L [a2 + b2-c2-d2f
( 2-(ab + cd) J

II. Cálculo da área


-|
Temos de (iii) que (ABCD) = — (ab + cd)-sena , então

r a2+b2 _ c2 _ d2 )2
(ABCD) = ~(ab+ cd)- 1-
l 2(ab + cd) J

(ab + cd)2 4-(ab + cd)2 - (a2 + b2 -c2 - d2)


(ABCD) =
4 4-(ab + cd)2

= ■ [(2ab + 2cd)2 - (a2 + b2 - c2 - d2) }


(ABCD)
Apêndices 661

(ABCD) = [(2ab + 2cd + a2 +b2 -c2 - d2)'(2ab + 2cd-a2 -b2 +c2 + d2|1 =>

(ABCD) = ■ £(a2 + 2ab + b2 j - (c2 - 2cd + d2)] • Qc2 + 2cd + d2) - (a2 - 2ab + b2)]

(ABCD) = J-L.[(a + b)2-(c-d)2].[(c + d)2-(a-b)2] =»

(ABCD) = ^-L.(a + b + c-d)(a + b-


d)(-a + b + c + d) (a-b + c + d)

(ABCD) = ^-2 (p-d)-2 (p - c) • 2 • (p - a) • 2 • (p - b) =>

(ABCD) = j(p-a)(p-b)(p-c)(p-d)

2. QUADRILÁTERO QUALQUER.
Consideremos um quadrilátero qualquer conforme ilustra a figura abaixo:

I. Acompanhe os seguintes passos:


i) Fazendo 0 = temos então 29 = a + p
ü) Na figura acima é fácil ver que :
AABC => (ABC) = - a b-sena

AACD (ACD) = i-c d senp


662 Tópicos de Matemática - Olimpíadas - IME - ITA

Logo.
(ABCD) = (ABC)+(ACD) = ~a b sena + - c d senp=s2 (ABCD) = a-b-sena+ c d senp=>

[2 (ABCD)]2 = (a b sena + c d senp)2 =>

4- (ABCD)2 = a2 b2 sen2a + 2a-bcd-senasenp + c2 - d2 ■ sen2p (*)


iii) Apli cando a lei dos cossenos nos triângulos ABC e ACD
x2 = a2 + b2 - 2ab cos a
obtemos Donde segue que:
x2 = c2 + d2 - 2cd cos p
2abcosa-2cdcosp = a2 + b2 -c2 - d2 =>
a2+b2-c2-d2
abcosa-cdcosp =------------------ —

(a2 +b2 -c2 -


(ab cos a-cd cos p)2
' 4
(a2 + b2 -c2 -d2)
..'b2cos2a-2a b c d cosa cosp + c2d2cos2p = ----------------------- — (**)
4

II. Cálculo do área


Adicionando as igualdades (*) e (**) obtemos:
22 [a2+b2-c2-d2l _
a b -2a b c d• (cosa cosp - senasenp) + c2 - d2 = i----------- ---------- + 4 (ABCD)

Mas, coscc cosp-sena-senp = cos(a + p). Lembre que definimos anteriormente


2B = a + p Assimcos(a + p) = cos(2-0) = 2 cos20-1. Logo

(a2+b2-c2-d2)
a2 b2 — 2a b c d ■cos(a +p) +c -d2 =L----------- ---------- í- + 4 (ABCD) =>

(a2+b2-c2-d2)2
a2b2 — 2abc-d cos(2-0) + c2 d2 = a----------- ---------- '— + 4.(ABCD) =>

(a2+b2-c2-d2)2
a2 b2-2a.b c d.(2.cos20-l) + c2.d2
= A----------- ---------- L + 4. (ABCD)2 =>

a2 b2-2a b.c.d.(2.cos2B_1j + c2.d2 = (32+b2-c2-d2) +4 (abcd)2^


Apêndices 663

2 2 (a2 + b2-c2-d2\2
a2 • b2 - 4 • a • b ■ c • d ■ cos2 0 - 2a • b • c • d + c2•d2 = ----- ----- + 4 (ABCD)

, (a2 + b2-c2-d2)
(ab-cd) -4-a b-c d-cos20 = 1-------- -- L+4.(abCD) => - 4
4
(a2+b2_c2_d2)
(ab-cd)2
(ABCD)2 = ----------- 1 -ab-cd- cos2 0 =>
4 16

, (2ab-2cd)2 -(a2 + b2 - c2 - d2)


(ABCD) =--------- ' -ab-cd cos2 0 =>
veja a demonstração do
quadrilátero inscritivel

(ABCD)2 = (p - a) ■ (p - b) ■ (p - c) • (p - d) - a ■ b ■ c • d ■ cos2 0 =>

(ABCD) - -j(p - a) ■ (p - b) ■ (p - c) • (p - d) - a ■ b ■ c ■ d ■ cos2 0 .

APÊNDICE 6

UMA SOMA INCRIVELMENTE INVARIANTE

Carlos A.Gomes
UFRN- Natal/RN
(Publicado na Revista da
Olimpíada Ibero Americana de Matemática)

Um resultado altamente inesperado é encontrado no maravilhoso texto


"Advanced Euclidean Geometry” de Roger Johnson's publicado pela Dover
publications em 1960 (e republicado nos últimos anos). É um antigo teorema
japonês que pelo seu surpreendente conteúdo merece ser "ressuscitado" agora na
lingua portuguesa para que ele seja conhecido e apreciado com muito louvor pelos
amantes da harmonia da matemática e em especial pelos geômetras.

TEOREMA: Seja um polígono convexo, que possa ser inscrito numa


circunferência. Se triangularizarmos esse polígono a partir de um de seus vértices
(traçando diagonais, é claro!) e inscrevermos em cada um destes triângulos uma
circunferência a soma das medidas dos raios dessas circunferências permanece
constante independente da triangularização realizada com o polígono.
664 Tópicos de Matemática - Olimpíadas - IME - ITA

Veja um exemplo ilustrativo com um hexágono:

as duas figuras acima a soma das medidas dos raios das circunferências inscritas nos

triângulos é a mesma (apesar das medidas dos raios das várias circunferências serem

possivelmente diferentes).

Para verificarmos o que foi dito acima usaremos o

TEOREMA DE CARNOT: Num triângulo ABC a soma (algébrica) das distâncias do


circuncentro do triângulo ABC aos lados AB, AC e BC desse triângulo é igual a
R+r. onde R é a medida do raio da circunferência circunscrita ao triângulo ABC e r
a medida do raio da circunferência inscrita no triângulo ABC.
' A convenção de sinais será a seguinte:
• Quando pelo menos uma parte do segmento está no interior do triângulo.
(-) Quando o segmento está completamente fora do triângulo.
Observe as figuras:
Apêndices 665

Vamos considerar aqui o caso do triângulo acutângulo. Os outros dois casos


podem ser verificados de modo razoavelmente semelhantes.

HA

.. M
Hc/ o.

A
c
hb Me
b

Na figura acima, temos que:

2-(ABC) = (a + b + c) r, ( ) = Área
2(ABC) = aOMA + bOMB+cOMc =>
r (a + b + c) = aOMA+bOMB+cOMc (0)

Agora perceba que:


ZAOB = 2ZC; ZBOC = 2ZA; ZAOC = 2ZB (Pelo teorema do ângulo inscrito).
Assim podemos observar as seguintes semelhanças:
AHb AHc OMa
AABHb ~ aACHc - aBOMa => -—8=-^. = -^. (D

BHa BHc OMb


ABAHa - ABCHc - ACOMb => (2)
CHb CHa omc
ACBHb - ACAHa - A AOMC => (3)
a b R

Assim, de (1), (2) e (3) temos que


AHb _ AHC _ OMa . ahb+AHç OMa R(AHB + AHc) = (b + c).OMA
c - b R
(4)
bha BHC _ OMb bha+bhc _ omb
=> R(BHA+BHC) = (c + a).OMB (5)
c a " R c+a R
666 Tópicos de Matemática - Olimpíadas - IME - ITA

CHB CHa OMc CHB + CHa = OMg_ R(CHb + CHaj = (a + b).0Mc


a = b = R a+b R
(6)
Adicionando (4), (5) e (6) temos:
OMa(b + c) + OMB(a + c) + OMc (a + b) = R(a + b + c) (7)

finalmente fazendo (7) + (0) temos:


OMa (b + c) + OMb (a + c) + OMc(a + b) = R(a + b + c)

a-OMA + b-OMB +c-OMc = r (a + b + c)

(OM A + OMB + OMC ) (a + b + c) = (R + r) (a + b + c)

Finalmente, OMA + OMB + OMC = R + r.

OBSERVAÇÃO: Na convenção estabelecida para os sinais da medida algébrica


dos segmentos o sinal (-) é justificável para garantir a igualdade que relaciona as
medidas das áreas conforme ilustramos abaixo.

Como (ABC) = (OAC) + (OCB) - (OAB), segue que:


r(a + b + c) = a-OMA + b-OUB - c OMc

Bem. de posse do teorema de Carnot vamos agora provar a nossa jóia rara!
Vejamos: Primeiro perceba que qualquer triangularização de n-ágono com
diagonais desse polígono gera n-2 triângulos. Vamos assumir que os triângulos
são numerados de 1 a (n - 2). Seja ni a medida do raio da circunferência inscrita no
i-ésimo triângulo e para cada triângulo seja OOj = aj • O,MA + b| • OjMB + q • O,MCi e
Apêndices 667

dai pelo teorema de Carnot temos queii + R = OiMc . Calculando a soma desse
resultado aplicado a cada triângulo temos:
q +r2 +... + rn_2 = OCq + OO2 +... + OOn_2 -(n - 2) R

Perceba que a soma OCq + OO2 +... + OOn consiste na soma algébrica das
perpendiculares traçadas aos lados do n-ágono são contadas apenas uma vez (e
com sinal positivo) já as perpendiculares às diagonais são contadas duas vezes
(uma com sinal positivo e outra com sinal negativo) conforme ilustra a figura
abaixo:

Assim a soma OO, + OO2+... +OOn_2 corresponde a soma das distâncias de O


aos lados do poligono (que é constante) e dai concluímos que q+ r2+... + rn_2 é
constante pois q + r2 + ... + rn_2 = 00, +OO2 +OOn_2 + (n -2)R = constante.
668 Tópicos de Matemática - Olimpíadas - IME - ITA

REFERÊNCIAS BIBLIOGRÁFICAS

[1) Andreescu, Titu, Enesc, Bogdan. Mathematical Olympiad Treasures.


Birkhauser.
[2] Andreescu, Titu; Feng; Zuming. Geometria Problems on maxima and minima.
Birkhauser.
[3] Barbeau, Edward J, Klamkin, Murray S., Moser, William O. J. Five Hundred
Mathematical Challengs. The Matehamatical association of America. 1995
[4) Berzsenyi, George; Mauer. Stephen B. The Contest Problem Book V. The
Mathematical Association of America. 1997.
(5] Carneiro, Emanuel; Paiva, Francisco Antonio M. de; Campos, Onofre -
Olimpíadas Cearenses de Matemática: 1981-2005. Ensino Fundamental.
Realce Editora & Ind. Gráfica Ltda. 2006
(6] Carneiro, Emanuel; Paiva, Francisco Antonio M. de; Campos, Onofre -
Olimpíadas Cearenses de Matemática: 1981-2005. Ensino Médio. Realce
Editora & Ind Gráfica Ltda. 2006
(7] Chentzov, N.N.; Shklarsky, D.O.; Yaglom, I.M. The USSR Olympiad Problem
Book - Selected Problems and Theorems of Elementary Mathematics. W. H.
Freeman and Company. 1962
[8] Crux Mathematicorum. Canadian Mathematical Society.
[9] D'Angelo, John P.; West, Douglas B. Mathematical Thinking - Problem-
Solving and Proofs Prentice Hall.
[10] Edwards, Josephine D.; King, Declan, J,; 0'Halloran, Peter J.AII The Best
From The Australian Mathematical Competition. Australian Mathematics
Competition. 1986
[11] Fomin, Dimitri; Genkin, Sergey; Itenberg, llia. Mathematical Cirdes (Russian
Experience). American Mathematical Society. 1996
[12] Hahn, Liong-Shin. New México Mathematics Contest Problem Book.
University of México. 2005
[13] lezzi, Gelson - Fund. De Matemática elementar vol. 09. Atual Editora, 2013.
[14] Salkind, Charles T. The Contest Problem Book II. The Mathematical
Association of American. 1966
[15] Salkind, Charles T; Earl, James M. The Contest Problem Book III. The
Mathematical Association of American. 1973
[16] Schneider, Leo J. The Contest Problem Book VI. The Mathematical
Association of American. 2000
[17] Santos. David A. Junior Problem Seminar. Personal Home Page. 2004
Apêndices 669

[18] Reiter, Harold B The Contest Problem Book VII. The Mathematical
Association of American. 2006
[19] http://camel.math.ca/CMS/Olympiads/
[20] Honsberger, Ross. In Pólya's Footsteps. The Mathematical Association of
America. 1997
[21] Honsberger, Ross. Mathematical Chestnuts from Around the World. The
Mathematical Association of America. 2001
[22] Honsberger, Ross. Mathematical Delights. The Mathematical Association of
America.
[23] Honsberger, Ross. More Mathematical Morsels. Mathematical Association of
America. 1991
[24] Larson, Loren C. Problem-Solving Through Problems. Springer-Verlag.
[25] Larson, Loren; Guy, Richard; Vaderlind, Paul The Inquisitive Problem Solvr
The Mathematical Association of America. 2002

[26] Moreira, Carlos G. T. de A.; Wagner, Eduardo. 10 Olimpíadas Ibero-


americanas de Matemática. OEI. 1996
[27] Olimpíada de Matemática do Estado de São Paulo de 1977 a 1997. Questões
e Soluções. ACIESP No. 106. ACIESP. 1999
[28] Olimpíada de Matemática do Estado de São Paulo de 1977 a 1997: Questões
e Soluções. 2°. Grau ACIESP No. 106. 1999
[29] Olimpíada de Matemática do Estado de São Paulo de 1977 a 1997: Questões
e Soluções de 5a. a 8a. Séries do 1o. Grau. ACIESP No. 102. 1999
[30] Salkind, Charles T.; Earl, James M. The Contest Problem Book III. The
Mathematical Association of American. 1973
[31] Taylor, P J. Tournament of the Towns 1984-1989. Australian International
Center for Mathematics Enrichment. 1992
[32] Trigg, Charles W. Mathematical Quickies. Dover. 1985.
[33] www.obm.org.br
[34] www.oma.org.ar
[35] www.mat.uc.pt/~opm/
[36] Zeitz, Paul. The Art and Craft of Problem Solving. Second Edition. John
Wiley & Sons, INC. 2007
_ lll ' ■-
A Editora VestSeller tem o prazer de lançar no mercado brasileiro
, pais uma excelente coleção de Matemática para o segmento de
preparação para os vestibulares IME ITA, bem como para as Olimpíadas de
Ciências exatas nacionais e internacionais. \

. Com sua vasta experiência no ramcl, o professor Carlos Gomes


. presenteia os estudantes e professores brasi.léiros com uma obra prima que
' .permite a qualquer leitor obter um grande salto de conhecimento na
Matemática Elementar em um curto intervalo de tempo.
. ' KSUB
Com didática admirável e notável capacidade de sintese, a
I
presente obra fornece aos alunos uma grande quantidade de problemas
clássicos de Geometria Plana, complementados, ao final do livro, com as *
resoluções detalhadas de todos os problemas, o que é permite um estudo
eficaz e produtivo em especial para ôs leitores autodidatas.
~
- -------------------- A presente obra pode ser usada tanto por professores e estudantes
do segmento IME ÍTAe Olimpíadas, como por bacharelandos ou
mestrandros em Matemática ou até mesmo por aficcionados por Geometria
E~”*^Íí$a? Os problemas apresentados nessa obra são realmente de tirar o
fôlego! '
Com essa excVlénte obra, a VestSeller tem a certeza de estar mais
uma Veí dando üma notável contribuição para a melhoria do nível e da
qualidade dos livros de Matemática disponíveis para estudantes e
professores em todo Brasil.

ISBN B5bDbS3MS-7

?aa sto bS345U

Você também pode gostar